Das fünfte Welträtsel: Bewusstsein

Naturalismus, mentale Verursachung, Willensfreiheit. Ein inspirierender Gastbeitrag von Prof. Dr. Timm Grams.

Mein humanistisch gebildeter Freund neckt mich, indem er hin und wieder lateinische Brocken fallen lässt – da reicht meine Oberrealschulbildung nicht hin. Geist kontra Materie, ein alter philosophischer Streit schimmert da durch. Unterschiedliche Denkrahmen eröffnen unterschiedliche Sichtweisen. Dass unsere Ansichten dennoch verblüffend oft konvergieren, schafft eine gewisse Treffsicherheit im Urteil.

Ignorabimus?

Bei Gelegenheit höre oder lese ich von ihm das Zitat „Ignoramus et ignorabimus“. Ich schlage nach und erfahre, was das heißt: „Wir wissen es nicht und wir werden es niemals wissen.“ Dieser Ausspruch ist der Gipfelpunkt eines Vortrags eines gewissen Emil du Bois-Reymond von vor knapp eineinhalb Jahrhunderten. So erfahre ich von diesem großen Physiologen und Philosophen, der die elektrische Natur der Nervensignale aufdeckte und sie dadurch der Messung zugänglich machte.

Du Bois-Reymond formulierte damals, was er acht Jahre später in einer Reihe von sieben Welträtseln als das fünfte benannte: Das Entstehen der einfachen Sinnesempfindungen. Er schreibt (du Bois-Reymond, 1982, S. 35 ff.): „Ich fühle Schmerz, fühle Lust, schmecke Süßes, rieche Rosenduft, höre Orgelton, sehe Rot […] Es ist eben durchaus und für immer unbegreiflich, dass es einer Anzahl von Kohlenstoff-, Wasserstoff- Stickstoff-, Sauerstoff- usw. Atomen nicht sollte gleichgültig sein, wie sie liegen und sich bewegen, wie sie lagen und sich bewegten, wie sie liegen und sich bewegen werden. Es ist in keiner Weise einzusehen, wie aus ihrem Zusammenwirken Bewusstsein entstehen könne.“

Du Bois-Reymond sieht hier Grenzen der Naturerkenntnis und er beschließt seinen Vortrag mit dem markanten „Ignorabimus!“

So unmittelbar und unverfälscht uns der rote Hut, das zustimmende Nicken, die Skyline von Frankfurt, die pochenden Töne der Fünften Sinfonie Beethovens erscheinen, für mich spitzt sich alles auf die Frage zu, ob mein Nachbar die Farbe Rot genauso empfindet wie ich, oder ob sich sein Erleben derselben Situation von dem meinen unterscheidet.

Weshalb man sich heute scheut, hinter du Bois-Reymonds Ignorabimus ein Ausrufezeichen zu setzen und ein Fragezeichen vorzieht, liegt auch daran, dass man immer besser erfasst, wie die Erregungsmuster im Gehirn mit den vorgefundenen Situationen zusammenhängen. Dieses wachsende Wissen scheint mir aber eher eine Seitwärtsbewegung zu sein. Es sagt uns nach wie vor nichts über das Wesen des Erlebens. Die geistigen Vorgänge sind aus ihren materiellen Bedingungen nicht zu begreifen. Das Bewusstsein bleibt rätselhaft.

Für du Bois-Reymond ist unserer Naturerkenntnis eine harte Schranke gesetzt. Das verdeutlicht er mit diesem Bild (1882, S, 38): Selbst auf der höchsten denkbaren Stufe unseres eigenen Naturerkennens gleichen unsere „Anstrengungen, über diese Schranke sich fortzuheben, einem nach dem Monde trachtenden Luftschiffer“.

Der Luftschiffer hat wenigstens den Mond vor Augen. Peter Bieri meint, dass wir nicht einmal den Sehnsuchtsort kennen (1992, S. 56): „Für das Rätsel des Bewusstseins gilt etwas, was für sonstige Rätsel nicht gilt: Wir haben keine Vorstellung davon, was als Lösung, als Verstehen zählen würde.“

Sackgassen

Jegliche Metaphysik sorgt für eine Einschränkung des Denkens. Wer seinen Denkrahmen mutwillig einengt, kommt der Lösung des Welträtsels nicht näher. Einen solchermaßen verfehlten Versuch zur Lösung des Welträtsels unternimmt der Naturalist. Er scheitert schon auf der Ebene der Logik; Erfahrungen und Fakten sind für den Nachweis seines Missgeschicks nicht vonnöten.

Der Naturalist baut sein Wissen auf die metaphysischen Annahmen, dass es nur eine Welt gebe, dass diese unerschaffen sei und von Gesetzmäßigkeiten regiert. Vor allem soll diese Welt – die Realität – „außerhalb unseres Denkens“ angesiedelt sein (Mahner, 2018, S. 46); sie ist folglich „in ihrer Existenz und ihren Eigenschaften unabhängig von unserem Bewusstsein“ (Vollmer, 2013, S. 22).

Mir widerstrebt zu erklären, dass das nicht zusammenpasst. Es ist zu offensichtlich. Wenn es nur eine Welt gibt, die es zu erkennen gilt, wo finden die Gedanken über diese Welt dann ihren Platz? In der Welt kann ihr Platz nicht sein, denn die Welt soll ja unabhängig vom Bewusstsein und den darin aufgehobenen Gedanken existieren. Irgendwo anders ist auch kein Platz, da es ja nur diese eine Welt gibt. Daraus folgt Gedankenlosigkeit. Der Begriff vom „Bewusstsein“ ist damit hinfällig. Ohne Gedanken aber gibt es keine Philosophie. Der Naturalismus löscht sich selbst aus. Komisch, dass es diese Naturalisten gibt: Philosophen ohne Philosophie.

Da ich schon einmal bei meiner Lieblingsbeschäftigung bin, beim Zerpflücken des Naturalismus, komme ich auf ein weiteres Welträtsel zu sprechen, dessen Existenz der Naturalist schichtweg abstreitet, anstatt es zu lösen.

Das siebente Welträtsel betrifft die Frage nach der Willensfreiheit (du Bois-Reymond, 1882, S. 84). Der Naturalist hat auch hier eine einfache Lösung: Es gibt sie gar nicht, diese Willensfreiheit. Schmidt-Salomon schreibt: Das Konzept der Willensfreiheit sei die „Unterstellung, dass eine Person sich unter exakt denselben Bedingungen anders hätte entscheiden können, als sie sich de facto entschieden hat“ (S. 102). Er meint, dass identische Ursachen notwendigerweise identische Folgen haben.

Das mag wohl sein. Jedoch lässt sich keine Versuchsanordnung denken, mit der die Entscheidung einer Person durch Herstellen identischen Bedingungen überprüft werden könnte. Es müsste schon dieselbe Person sein. Aber nach dem ersten Durchlauf ist deren mentale Ausstattung bereits verändert, so dass sie für den folgenden Prüflauf nicht infrage kommen kann. Das Popper-Kriterium ist unerbittlich: Die Aussage des Naturalisten ist metaphysisch, unwissenschaftlich, Glaubenssache.

Für beide Welträtsel läuft die Lösung des Naturalisten auf Leugnung hinaus: Bewusstsein gibt es nicht, genauso wenig die Willensfreiheit. Das verstößt so eklatant gegen den gesunden Menschenverstand, dass dieser dem Naturalisten nicht folgen mag. Dass sich die Naturalisten rettungslos verheddern, wenn es um das Geistige geht, war in diesem Weblogbuch schon mehrfach Thema, unter anderem im Artikel Der Spiegel der Natur.

Dazu passt eine grundsätzliche Überlegung des Peter Bieri. Von ihm ist dieses Trilemma:

  1. Mentale Phänomene sind nichtphysikalische Phänomene.
  2. Mentale Phänomene sind im Bereich physikalischer Phänomene kausal wirksam.
  3. Der Bereich physikalischer Phänomene ist kausal geschlossen.

Jede dieser Aussagen scheint für sich gesehen vernünftig zu sein. Pech ist nur, dass sie nicht allesamt gültig sein können: 1&2 schließt 3 aus; 1&3 schließt 2 aus und 2&3 schließt 1 aus.

Der Naturaliste lehnt 2 ab; also bleiben 1 und 3. Da er nur eine Welt anerkennt, gibt es keine nichtphysikalischen Phänomene. Folglich kann es keine mentalen Phänomene geben. Die Aussage 1 wird damit bedeutungsleer. Das zeigt erneut: Bewusstsein und Willensfreiheit gibt es für den Naturalisten nicht; es handelt sich bestenfalls um Illusionen, die der Körper für uns bereithält (Schmidt-Salomon, 2006, S. 12). Damit lockt uns der Naturalist in eine Denkschleife: Eine Illusion, die niemand wahrnimmt, ist keine. Wir fragen nach dem Adressaten der Illusion und landen wieder beim Anfang unserer Suche, bei der Frage nach dem Wesen des Bewusstseins. So werden wir Opfer einer Wortakrobatik ohne tieferen Sinn.

Seitwärtsbewegung

Unter den Fesseln der Metaphysik bekommen wir für die Welträtsel bestenfalls Scheinlösungen. Machen wir uns davon frei. Die Wissenschaft kennt solche Beschränkungen nicht. Einzig die Widerspruchsfreiheit und die Übereinstimmung mit den Fakten stecken den Denkrahmen ab. Es gibt, so scheint es mir, ziemlich viele Wissenschaftler, die sich mit dem Welträtsel abmühen. Ich will hier nur auf zwei von ihnen zu sprechen kommen: Wolf Singer und Roger Penrose.

Das Nachdenken des Menschen über sein Nachdenken hat so seine Schwierigkeiten. Um die Verwirrung in Grenzen zu halten, habe ich die wichtigsten Begriffe in einer Tabelle gruppiert. Die linke Spalte bringt die Grobeinteilung zwischen Außenwelt und Innenwelt: Von der ersteren haben wir nur die „Bilder im Kopf“, die Erscheinungen. Sie bilden die Innenwelt. (Florey, 1997; Grams, 2020, S. 248ff.)

Die rechte Spalte beinhaltet den Weltausschnitt, der für das Studium des Bewusstseins relevant ist. Auch hier betrifft der obere, dunkel hinterlegte Teil die nicht direkt erkennbare Außenwelt und der untere die Innenwelt.

(Ich rechtfertige meinen Sprachgebrauch, „Diesseits“ und „Jenseits“ betreffend, damit, dass es von der unfasslichen Realität zu Gott nur ein kleiner gedanklicher Schritt ist. Die Kluft zwischen Diesseits und Jenseits ist demgegenüber riesig.)

Lassen wir Immanuel Kant zu Wort kommen. Er schreibt in seinem Werk „Kritik der reinen Vernunft“ (2011, „Anhang. Von der Amphibolie der Reflexionsbegriffe“, S. 283):„Wenn wir aber auch von Dingen an sich selbst etwas durch den reinen Verstand synthetisch sagen könnten (welches gleichwohl unmöglich ist), so würde dieses doch gar nicht auf Erscheinungen, welche nicht Dinge an sich selbst vorstellen, gezogen werden können. Ich werde also in diesem letzteren Falle in der transscendentalen Überlegung meine Begriffe jederzeit nur unter den Bedingungen der Sinnlichkeit vergleichen müssen, und so werden Raum und Zeit nicht Bestimmungen der Dinge an sich, sondern der Erscheinungen sein: was die Dinge an sich sein mögen, weiß ich nicht und brauche es auch nicht zu wissen, weil mir doch niemals ein Ding anders als in der Erscheinung vorkommen kann.“

Der weiße Pfeil spielt auf diesen Gedanken Kants an: Da wir nur die Erscheinungen erfahren, und nicht „die Dinge an sich“, gruppieren wir die vermeintlichen Dinge der Außenwelt nach Maßgabe der Erscheinungen. Wir „denken“ also in Pfeilrichtung.

Die Ursache-Wirkungsrichtung ist aber genau entgegengesetzt. Die Wissenschaft hat bei der Aufklärung dieser Kausalbeziehungen seit Mitte des 19. Jahrhunderts große Fortschritte gemacht. Die elektrischen Nervenimpulse infolge eines Umweltreizes lassen sich messen. Die ursächliche Abhängigkeit der Nervenimpulse von den Umweltreizen ist heutzutage kein großes Geheimnis mehr. Der sensorischen Input sorgt für spezifische Aktivitätsmuster der Nerven. Das liefert die gehirninternen Repräsentationen des Inputs.

Objekte der Umwelt werden nun aber nicht unmittelbar in ihrer Ganzheit erfasst. Das Gehirn ist arbeitsteilig organisiert. Ein vorüberfahrendes Taxi erregt die Nervenzellen für die Erfassung von Farbe, von Kontur und von Orientierung und Bewegung usw. Das geschieht in getrennten Bereichen des Gehirns. Wie aus diesen Teilrepräsentationen die Repräsentation des Objekts wird, ist eines der großen Rätsel der Neurowissenschaften (Ramachandran, Blakeslee, 1998, S. 80).

Auch wenn dieses Rätsel dereinst zur Zufriedenheit aller gelöst sein wird, fehlt doch immer noch der entscheidende Schritt. Genaugenommen ist man dadurch dem Ziel nicht näher gekommen. Es läuft auf eine Seitwärtsbewegung hinaus, wie die des Luftschiffers, der dem Monde einfach nicht näher kommt.

Eine solche Seitwärtsbewegung liefern Andreas Engel, Peter König und Wolf Singer (1993). Sie schreiben: „Die richtige Zuordnung verlangt nach einem Mechanismus der in der Vielzahl aktivierter Zellen selektiv diejenigen kennzeichnet, die auf ein und dasselbe Objekt antworten.“

Als Bindemittel sehen sie das oszillatorische Feuern der Nervenzellen. Synchronisiertes Feuern hebt die Zusammengehörigen von den übrigen ab – so lautet die Hypothese. Die Teilrepräsentationen können sich so zu einem Gesamtbild fügen. Das aber hebt die Repräsentation nicht über die physikalische und physiologische Ebene hinaus. Das Geschehen bleibt im Jenseits verortet.

Neue Ideen gefragt

Seit der Zeit des du Bois-Reymond hat sich in der Wissenschaft viel getan. Meilensteine sind die Relativitätstheorie und die Quantenmechanik. Einem Verständnis des Bewusstseins sind wir Menschen dennoch nicht viel näher gekommen. Wir sind immer noch „Luftschiffer, die nach dem Monde trachten“.

Der Nobelpreisträger dieses Jahres, Roger Penrose, schreibt im Vorwort zu „The Emperor’s New Mind“, dass die heutige Physik dem Phänomen des Bewusstseins nicht gerecht wird (2016, S. xv): “The conscious aspects of our minds are not explicable in computational terms and moreover that conscious minds can find no home within our present day scientific world view.” Und weiter (S. 580): „One can argue that a universe governed by laws that do not allow consciousness is no universe at all. I would even say that all the mathematical descriptions of a universe that have been given so far must fail this criterion. It is only the phenomenon of consciousness that can conjure a putative ‘theoretical’ universe into actual existence.”

Solange die von uns erkannten Gesetzmäßigkeiten der Welt ein bewusstes Denken nicht zulassen, kann auch das Universum nicht gedacht werden. Es existiert demnach für uns nicht. Die heutige Wissenschaft ist offenbar nicht reichhaltig genug, um das Geistige zu verstehen.

Timm Grams studierte bis 1972 Elektrotechnik an der TH Darmstadt (Dipl. – Ing.) und promovierte 1975 an der Universität Ulm (Dr. rer. nat). Simulation, Zuverlässigkeitstechnik und Prozessdatenübertragung waren seine Arbeitsschwerpunkte in der Industrie. Seit 1983 ist er Professor an der Hochschule Fulda mit den Lehrgebieten Nachrichtentechnik, Programmkonstruktion, Simulation und Problemlösen. Denkfallen und Regeln zu deren Vermeidung sind seit vierzig Jahren seine Leidenschaft. Darüber schreibt er auch in seinem Blog Hoppla!, wo dieser Beitrag bereits vorher erschienen ist. Die Titelabbildung stammt von qimono auf Pixabay.

Quellen

  • Bieri, Peter (1992): Was macht das Bewusstsein zu einem Rätsel? Spektrum der Wissenschaft 10/1992, 48-56.
  • du Bois-Reymond, Emil (1882): Über die Grenzen der Naturerkenntnis. Die sieben Welträtsel. Zwei Vorträge von 1872 und 1880. Leipzig: VON VEIT & COMP.
  • Engel, Andreas K.; König, Peter; Singer, Wolf (1993): Bildung repräsentationaler Zustände im Gehirn. Spektrum der Wissenschaft 9/1993, 42-47.
  • Florey, Ernst (1997): Das fünfte Welträtsel – Ignorabimus. Festschrift zum 100. Todestag von Emil du Bois-Reymond. Berichte und Abhandlungen / Berlin-Brandenburgische Akademie der Wissenschaften. 4/1997, 161-174.
  • Grams, Timm (2020): Klüger irren – Denkfallen vermeiden mit System. Heidelberg: Springer.
  • Kant, Immanuel (2011): Kritik der reinen Vernunft. Köln: Anaconda. (Nach der zweiten Auflage von 1787.)
  • Mahner, Martin (2018): Naturalismus. Aschaffenburg: Alibri.
  • Penrose, Roger (2016): The Emperor’s New Mind. Oxford: University Press. (Zuerst erschienen 1989)
  • Ramachandran, Vilayanur S., Blakeslee, Sandra (1998): Phantoms in the Brain. New York: HarperCollins
  • Schmidt-Salomon, Michael (2006): Manifest des evolutionären Humanismus. Aschaffenburg: Alibri.
  • Singer, Wolf (1997): Bewusstsein, etwas „Neues, bis dahin Unerhörtes“. Festschrift zum 100. Todestag von Emil du Bois-Reymond. Berichte und Abhandlungen / Berlin-Brandenburgische Akademie der Wissenschaften. 4/1997,175-190.
  • Vollmer, Gerhard (2013): Gretchenfragen an den Naturalisten. Aschaffenburg: Alibri.

Avatar-Foto

Die Diskussionen hier sind frei und werden grundsätzlich nicht moderiert. Gehen Sie respektvoll miteinander um, orientieren Sie sich am Thema der Blogbeiträge und vermeiden Sie Wiederholungen oder Monologe. Bei Zuwiderhandlung können Kommentare gekürzt, gelöscht und/oder die Diskussion gesperrt werden. Nähere Details finden Sie in "Über das Blog". Stephan Schleim ist studierter Philosoph und promovierter Kognitionswissenschaftler. Seit 2009 ist er an der Universität Groningen in den Niederlanden tätig, zurzeit als Assoziierter Professor für Theorie und Geschichte der Psychologie.

757 Kommentare

  1. Schön, dass ich auf den Beitrag aufmerksam wurde, als ich meinen eigenen Studierenden wieder das “Bieri-Trilemma” erklärte und das berühmte Zitat Emil du Bois-Reymonds präsentierte.

    Ulkig übrigens, dass du Bois-Reymond seinen zurückhaltenden Standpunkt als “männliche Entsagung” charakerisierte: Davon sollten sich die heutigen Angeber, Blender und Hochstapler in der Wissenschaft mal eine Scheibe abschneiden.

  2. Das ist ein schöner Beitrag, schön gedacht und schön geschrieben, danke dafür!

    Ich neige ja dazu – ich weiss, ich wiederhole mich ad nauseam – du Bois-Reymonds (Leib-Seele) Problem dadurch zu lösen, dass ich alles für beseelt halte, ja der “bewussten Seele” sogar das ontologische Primat über die Materie zuweisen würde, kurz, ich neige, eines überaus altmodischen monistischen Idealismus mich zu befleissigen — um’s auch grammatisch mal ganz altfränkisch und so sperrig wie Kant zu formulieren.

    Helfen tut’s mir aber aucn nicht allzuviel, denn dann krieg’ ich das Problem halt “andersherum”, und muss mich fragen lassen, wie es sich wohl anfühle, ein Stein zu sein. Immerhin hat der Ansatz den Vorteil, dass ich – von der Fülle MEINES Bewusstseins ausgehend – mich dem des Steines annähern kann, indem ich Inhalte WEGnehme: ich muss also nichts qualitativ ganz neues hinzusetzen, es unterbleibt der “Sprung”. Und gefragt, wie es sich wohl anfühle, ein Stein zu sein, würde ich sagen: “Vor allem mal schwer und zweitens hart.”

    Jaja, ich weiss, Spinoza, Schopenhauer, Panpsychismus. Reizvoll daran ist halt, dass nichts (auch kein “Ding an sich”) aussen vor bleibt.

  3. Stephan Schleim

    Zitat Wikipedia zum Bieri-Trilema:

    1. Mentale Phänomene sind nichtphysikalische Phänomene.
    2. Mentale Phänomene sind im Bereich physikalischer Phänomene kausal wirksam.
    3. Der Bereich physikalischer Phänomene ist kausal geschlossen.

    Was meinen Sie dazu, wenn man Punkt 1 des „Bieri-Trilemma“ (Mentale Phänomene sind nichtphysikalische Phänomene), er enthält meiner Meinung nach ??? einen typischen Kategorienfehler, „Abbildungen“ kann man nicht mit dem „Abzubildenden“ gleichsetzen, z.B. wie folgt anders formuliert:

    Mentale Phänomene können bei physikalischen Prozessen auftreten und wie auch andere mathematisch/informelle Abbildungen, z.B. in neuronalen Systemen verarbeitet werden.

    Wechselwirkungen zwischen Körper und Geist sind natürlich möglich….

  4. Ohne Wortakrobatik kommen wir nicht aus. Wir denken in Worten (Begriffen).
    Wir nehmen unsere Welt aber auch ohne Worte auf, durch Schweigen.
    Die Welt muss nicht erst gedacht werden, damit sie existiert. Sie ist der Zuchtmeister, der uns beherrscht. Viele Wortakrobaten haben wohl noch keinen Hunger gehabt oder Angst.

    Was schreibt da Kant ? „was die Dinge an sich sein mögen, weiß ich nicht und brauche es auch nicht zu wissen, weil mir doch niemals ein Ding anders als in der Erscheinung vorkommen kann.“

    Doch das geht heutzutage und auch schon früher. Der geschulte Physiker begreift die Vorgänge mithilfe der Mathematik, die dann in Formeln gegleitet werden.
    Die können sogar der Logik widersprechen.

    Das zum Anfang. Der Zuchtmeister ruft, mein Magen knurrt.

  5. @Elektroniker

    Was meinen Sie dazu, wenn man Punkt 1 des „Bieri-Trilemma“ […] z.B. wie folgt anders formuliert

    Meiner Erfahrung nach ist es in der Regel nicht hilfreich, etwas Verständliches durch etwas Unverständliches zu ersetzen.

    Für eine abschließende Beurteilung, ob nach der von ihnen vorgeschlagenen Ersetzung überhaupt das für das Trilemma Wesentliche noch gilt, dass 1,2 und 3 sich wechselseitig ausschließen, fehlt mir das intellektuelle Vermögen.

    Wenn Sie mentale Phänomene einer anderen Kategorie zuordnen als physikalische, dann können sie 1) auch einfach als richtig annehmen. Stimmt dann 2) oder 3) ihrer Meinung nach nicht?

  6. Womit ich Schwierigkeiten habe, ist die Widerspruchsfeiheit. Ist das Erkennen und Vorhandensein eines Paradoxons nicht die “Kehrseite der Logik” und somit Teil der Bewusstheit? Ohne Logik kein Paradoxon.

  7. @Welträtsel Bewusstsein

    „Dazu passt eine grundsätzliche Überlegung des Peter Bieri. Von ihm ist dieses Trilemma:

    1. Mentale Phänomene sind nichtphysikalische Phänomene.
    2. Mentale Phänomene sind im Bereich physikalischer Phänomene kausal wirksam.
    3. Der Bereich physikalischer Phänomene ist kausal geschlossen.“

    Für mich ist es recht einfach: Wir müssen 3. aufgeben.

    In der Quantenphysik gibt es die Wellenfunktion, die regelmäßig zusammenbricht, und dann eingeschränkt zufällige Ergebnisse produziert. Nur deren Wahrscheinlichkeit ist festgelegt, die konkreten Fakten bleiben in gewissem Rahmen variabel. Genau hier müsste die Geisteswelt eingreifen können, indem auf die Zukunft hin gezielt die Fakten gestaltet werden können. So würde der Gang der Dinge in gewissem Umfang geistesgesteuert laufen können, ohne dass hier irgendein Naturgesetz dabei verletzt werden müsste.

    Unser Bewusstsein selbst kann dann auch eine Synthese eines kosmischen Geistes mit unserem lokalen Gehirn sein, verknüpft durch diese Quantenzufälle, wenn sie gezielt sein können.

    @hwied 15.01. 12:52

    „Was schreibt da Kant ? „was die Dinge an sich sein mögen, weiß ich nicht und brauche es auch nicht zu wissen, weil mir doch niemals ein Ding anders als in der Erscheinung vorkommen kann.“

    Wenn ich ein umfassendes Computermodell, z.B. von der Faltung eines Proteins haben kann, dann habe ich schon auch ein Ding an Sich im Computer.

    Dieses Modellprinzip weitergedacht: wenn wir mal eine Komplettsimulation von einzelnen ganzen Zellen haben, dann haben wir hier was erfasst, dass ziemlich viel mit der Welt da draußen zu tun hat, ohne es selbst wahrnehmen zu müssen. Der Computer baut uns das einfach so zusammen, dass wir ein richtig gutes Bild davon bekommen.

    Wenn man das mit dem ganzen Gehirn macht, und auch die ganzen Zufallsprozesse mit abbildet, und diese nicht mit Pseudozufallszahlen, sondern mit richtigen physikalischen Zufallszahlen betreibt, dann hätten wir nicht nur ein Modell, sondern künstliches Bewusstsein in wirklich menschlicher Form. Und damit eigentlich sogar künstliches Leben.

    Und wenn wir die Prinzipien verstanden haben, die da ablaufen, dann können wir richtig loslegen, und künstliche „Seelen“ entwerfen und produzieren, und Sciencefiction, die man erstmal schreiben müsste, sogar in die Realität holen können. Jenseits der derzeitigen Seitwärtsbewegungen wäre nicht nur der Mond zu erreichen, sondern noch viel mehr, womöglich ein ganzer Kosmos zu erreichen. Als Perspektive für die nächsten Jahrtausende zumindest.

    Wir müssen wirklich „nur“ „3. Der Bereich physikalischer Phänomene ist kausal geschlossen.“ aufgeben.

  8. Mussi,
    eine sehr tief gehende Einsicht. Das Paradoxon zeigt die Grenzen der Logik auf.
    Das bedeutet, dass die Logik kein geschlossenes Denksystem ist und sich nicht aus sich selbst (durch sich selbst) erklären und beschreiben lässt.

    Wenn man diesen Gedanken auf den Menschen anwendet, dann kann sich der Mensch nicht selbst aus sich selbst vollständig erklären. Wenn er das doch tut, dann kommt er zu dem Paradoxon “warum lebe ich, wenn ich doch sterben muss”.

    Eine Sichtweise, mit der man aus der Hülle des Menschen gleiten kann, die nennt man Philosophie. Und wem das immer noch zu unbefriedigend ist, der greift zur Religion. Da wird gesagt, dass der Mensch teleologisch ist, also zielgerichtet. Dann erfährt man, dass der Mensch und das ganze Universum einen Ursprung hat, den die Bibel so formuliert: “Im Anfang war das Wort und das Wort war bei Gott und Gott war das Wort……”
    Um das so formulieren zu können, dazu braucht man Bewußtsein, also das , was hier unser Thema ist.

  9. Im Kino sehen wir bewegte Bilder – obwohl nur unbewegte Fotos gezeigt werden. D.h. die Wahrnehmung von Bewegung ist eine Illusion (die sich aus der Arbeitsweise des Gehirns ergibt).

    Im obigen Blog-Beitrag wird die Idee – dass ´Bewusstsein´ nur eine Illusion sein könnte – ausgeschlossen.

    Wenn aber eine mögliche Lösung von vorne herein ausgeschlossen ist – dann ist eine sinnvolle Lösung einer Fragestellung nicht möglich

  10. @ Jeckenburger

    Wenn man den Kollaps der Wellenfunktion, der einen Zustand festlegt, als Resultat der Messung ansieht (wie das die Kopenhagener Interpretation tat), dann ist man in der Tat nicht weit weg von Berkeley’s ur-idealistischem “esse est percipi” (“Sein heisst wahrgenommen werden”). Bloss daraus einen Freiheits- oder Willensakt (Zitat von oben: “indem auf die Zukunft hin gezielt die Fakten gestaltet werden können”) zu stricken, erscheint mir schwierig.

  11. KRichard,
    “die Wahrnehmung von Bewegung ist eine Illusion”.
    Das wäre nur richtig, wenn man die Zeit anhalten könnte. Nur dann und nur dann erstarrt die Bewegung zu einem Standbild.
    Die Zeit ist unerbittlich, die schreitet voran. Deshalb ist die Bewegung keine Illusion , die ist real und sie kann als Bewegungsenergie berechnet werden.

    Und wenn dich mal deine Ex verfolgt, weil du deine Unterhaltszahlungen nicht geleistet hast, dann ist das auch keine Illusion.

  12. @Helmut: Anatomie des Idealismus

    Wie interessant: Ein monistischer Idealist wird Anatom… aber so, wie ich die Position verstehe (als echten Monismus), geht das weiter als dem Bewusstsein nur das “ontologische Primat” einzuräumen: Letztlich ist dann alles eins, nämlich Bewusstsein.

    Ich muss mich da als Agnostiker nicht so festlegen und kann es mit Herbert Feigl (und vielleicht auch Immanuel Kant) halten: Was wir Körper und Geist nennen, das sind vielleicht nur unterschiedliche Zuschreibungen von einem Ding (an sich), das wir in letzter Konsequenz nicht erkennen können, weil wir es immer nur als Erscheinung, als Phänomen kennen.

    (Schrieb ich, während das niederländische Kabinett Rutte-III zurücktrat.)

  13. @Joker: Bieri-Trilemma

    Da würde ich dich ja gerne einmal in meine Vorlesung zum Leib-Seele-Problem einladen. Die nächste dazu steht aber wohl erst im Dezember wieder an.

    Lass es mich mal so sagen: Ob die drei einander ausschließen, liegt natürlich auch daran, wie man sie genau versteht. Oftmals ist es eben eine Frage der Definition.

    Darf ich auch dich noch einmal an Das kleine Einmaleins des Leib-Seele-Problems erinnern? Darin habe ich die Formulierung von Westphal aufgenommen, der es in vier Prämissen aufdröselt, um den logischen Widerspruch deutlicher zu machen.

  14. @ Schleim

    .. ja, ich denk’, so strikte monistisch meine ich das. Das, was ist, ist EINES, nämlich nur Erscheinung, nur Phänomen, “dahinter” ist kein unerkennbares Anderes, alles ist “Bewusstsein”. Das ist aber alles völlig unoriginell und nur Schopenhauer, der aber immerhin so klug war, für dies “eine”, das ist, nicht (nur) den Begriff “Bewusstsein” zu verwenden, sondern eben den der “Vorstellung” (um damit die Phänomene, die sich ihrer selbst nicht bewusst sind, mithin also das, was wir heut’, mit Freud, das “Unter-” oder “Unbewusste” nennen) mit hinein zu bekommen.

  15. @hwied
    Im Kino werden nachprüfbar nur Abfolgen von Einzelfotos gezeigt – das ist die Realität. Unsere Wahrnehmung von Bewegung ist daher eine Illusion – die nicht der Realität entspricht.

    ´Zeit´ hat keine Dauer – aus diesem Grund kann ´Bewusstsein´ auch keine Dauer haben.

    D.h. wenn man verstehen will, was ´Bewusstsein´ ist bzw. wie ´Bewusstsein´ entsteht – muss man dazu auch eine Erklärung für das Wesen von Zeit liefern (= was vergeht, wenn Zeit vergeht) und diskutieren. Sonst kommt man zu keinem sinnvollen Ergebnis.

  16. Wer sich nicht erklären kann wie Bewusstsein in diese Welt kommt muss doch auch Schwierigkeiten haben zu erklären wie Leben in eine unbelebte Welt kommt. Oder nicht?

  17. Zitat: „Es ist eben durchaus und für immer unbegreiflich, dass es einer Anzahl von Kohlenstoff-, Wasserstoff- Stickstoff-, Sauerstoff- usw. Atomen nicht sollte gleichgültig sein, wie sie liegen und sich bewegen, wie sie lagen und sich bewegten, wie sie liegen und sich bewegen werden. Es ist in keiner Weise einzusehen, wie aus ihrem Zusammenwirken Bewusstsein entstehen könne.“

    Es ist zwar nicht einzusehen, ist aber offensichtlich so, dass unter bestimmten Umständen (Druck, chem. Einfluss, ….) „Empfindungen“ (als Phänomen) entstehen. Offensichtlich scheint auch, dass dabei Ladungsträger, z.B. Elektronen, frei werden sollten, sonst könnten sie im neuronalen System nicht ausgewertet werden. Vermutlich spielt auch die Dynamik der Prozesse eine Rolle.

    Einschränkungen des Denkens sind offensichtlich nicht zweckmäßig.

    Z.B. scheint es so, dass Neuronen den UND Gatterschaltungen der Elektronik (Schaltalgebra) entsprechen und ähnlich wirken (frei nach W.McCulloch). Sie sind aber definitiv keine (strengen) UND Gatter. Damit wäre die Sache eigentlich „abgehakt“.

    Es scheint aber so, dass ein Neuron dann triggert, wenn auf möglichst vielen Dendriten möglichst gleichzeitig, elektrische Ladungsträger eintreffen. Das entspräche am ehesten einem „qualifizierten UND Gatter“.

    Zitat: „Wie aus diesen Teilrepräsentationen die Repräsentation des Objekts wird, ist eines der großen Rätsel der Neurowissenschaften“

    Eigentlich nicht mehr.

    Zitat: „Eine solche Seitwärtsbewegung liefern Andreas Engel, Peter König und Wolf Singer (1993). Sie schreiben: Die richtige Zuordnung verlangt nach einem Mechanismus der in der Vielzahl aktivierter Zellen selektiv diejenigen kennzeichnet, die auf ein und dasselbe Objekt antworten.“

    Meiner Meinung nach ist es auch so. Die Neuronen (qualifizierte UND Gatter) „können in der Hauptsache“ nur derartige Synchronisationen verknüpfen, auswerten und sozusagen „Muster“ erkennen.

    Es kommt jetzt noch darauf an, dass die Teilrepräsentationen korrekt zusammengeführt werden. So wie z.B. Bildpunkte auf der Netzhaut zu einem Bild emergieren. Die „Bildpunkte“ bekommen einen örtlichen und zeitlichen Zusammenhang.

    Der „böse Wolf“ im Märchen hat genau gewusst, was er alles „simulieren“ muss, um vom Rotkäppchen für deren Oma gehalten zu werden.

    In bestimmten Bereichen des Gehirns wird elektrisch und in „Neuronengattern aufbereitete“ Information, z.B. abstrakte örtliche Komponenten (Omas Hütte, …), bildhafte Komponenten (Omas Haube, …), verbale Komponenten (Omas Stimme) …. zusammengeführt und haben auf bestimmten flächigen Neuronenstrukturen im Gehirn des Rotkäppchen das Muster, oder Objekt „Oma“ (abstrakt, nicht optisch) „abgebildet“ (Neuronentriggerungen) und entsprechende Handlungen gesteuert.

    Die flächigen Neuronenstrukturen (Zwischenschichten, Endschichten…) könnte man sich vorstellen wie eine Mattscheibe oder einen Bildschirm (oder die Augennetzhaut).

    Bewusstsein ist, was auf den „internen Bildschirmen“ und an der „Empfindungssensorik“ abgebildet und allenfalls weiter verarbeitet werden kann.

  18. KRichard,
    dass wir Einzelbilder als “Film” wahrnehmen ist der Trägheit der Augen geschuldet. Ein Insekt mit Vielfachaugen, das sieht die einzelnen Bilder, weil sie jeweils einen anderen Bildwinkel haben.
    Die Zeit ist gequantelt, weil die Materie gequantelt ist. Unser Bewusstsein ist auch gequantelt, ganz zwangsläufig, weil unsere Neuronen in der Anzahl begrenzt ist.

    “Ich denke also bin Ich”, wer will hier päpstlicher sein als der Papst ?

  19. @KRichard

    Sie nehmen an, dass in meinem Blog-Beitrag die Idee, dass ´Bewusstsein´ nur eine Illusion sein könnte, ausgeschlossen wird.

    Ich aber habe geschrieben: Eine Illusion, die niemand wahrnimmt, ist keine. Wir fragen nach dem Adressaten der Illusion und landen wieder beim Anfang unserer Suche, bei der Frage nach dem Wesen des Bewusstseins.

  20. @Helmut Wicht 15.01. 15:02

    „Wenn man den Kollaps der Wellenfunktion, der einen Zustand festlegt, als Resultat der Messung ansieht… „

    So meine ich das nicht. Nicht der Physiker, der Messungen vornimmt, bestimmt die konkrete Ausgestaltung der teils zufälligen Prozesse. Der Physiker erzwingt mit der Messung nur den Zusammenbruch.

    Ich kann wohl davon ausgehen, dass in der ganzen Natur überall die Wellenfunktionen sich nur teilweise in Superpositionen verknüpfen, und eben regelmäßig auch ohne bewusste Messungen zusammenbrechen müssen. Die konkrete Ausgestaltung der Zufallsprozesse wäre dann einer Art kosmischem Geist zuzuordnen, nicht dem bewusstem Menschen selbst, und schon gar nicht irgendwelchen Physikern, die Experimente machen.

    Wenn ich eine Webcam betrachte, die mit viel zu wenig Licht hauptsächlich Pixelrauschen produziert, so würde ich genau diesen Datenstrom als Grundlage für echte Zufallszahlen verwenden, wenn ich z.B. bei der Produktion von Computerkunst Zufallszahlen einbinden will.

    @KRichard 15.01 15:27

    „D.h. wenn man verstehen will, was ´Bewusstsein´ ist bzw. wie ´Bewusstsein´ entsteht – muss man dazu auch eine Erklärung für das Wesen von Zeit liefern (= was vergeht, wenn Zeit vergeht) und diskutieren. Sonst kommt man zu keinem sinnvollen Ergebnis.“

    Der Zeitrahmen unserer Existenz ist wohl der eines ewigen Reisenden durch unsere Zeit. So wie man in einem Auto sitzt, und über die Autobahn fährt, so zieht die Welt ruhig und gleichmäßig an uns vorüber. Das ist sozusagen die ureigene Zeitstruktur eines Bewusstseins.

    Entsprechend genügen Aktualisierungen von über 36 Momentaufnahmen pro Sekunde, um diesen Zeitfluss übergangslos aufzunehmen. Als Menschenbewusstsein zumindest. Fliegen oder kleine Vögel haben da deutlich höhere Auflösungen. Aber Zeitreisende sind sie so wie wir.

  21. @Elektroniker 15.01. 15:54

    „Bewusstsein ist, was auf den „internen Bildschirmen“ und an der „Empfindungssensorik“ abgebildet und allenfalls weiter verarbeitet werden kann.“

    Diese Abbildung kann doch auch in einen Geistesraum stattfinden, und muss nicht ausschließlich neuronal konstruiert werden. Einerseits kennen wir diese neuronalen Strukturen eben derzeit überhaupt gar nicht, und andererseits kommt uns unser Erleben des aktuellen Raums, in dem wir uns befinden, doch auch so sehr echt vor, dass man meine könnte, unsere Erlebniswelt hätte direkten Kontakt zur erlebten Außenwelt.

  22. @ Joker 15.01.2021, 14:15 Uhr

    Zitat: „1. Mentale Phänomene sind nichtphysikalische Phänomene.“

    Diese Aussage scheint Grund für die Widersprüchlichkeit der Aussagen.

    Mentale Phänomene sind sehr wohl die „Abbildung“ physikalischer Phänomene.

    Es besteht offensichtlich (aber nur schwer beweisbar) ein Zusammenhang aber sicher keine Identität. Derartige Aussagen sind sozusagen nicht „erlaubt“, irgendwie vergleichbar damit, dass eine Zahl nicht durch 0 dividiert werden darf. (Sonst kommt man mit der Logik in Teufels Küche…)

    So wie ein Foto die Abbildung eines Hauses sein kann, aber kein Haus ist. Der Zusammenhang ist aber korrekt.

    Wechselwirkungen zwischen Körper und Geist sind natürlich möglich….

    Würde dies berücksichtigt, sollte das „Bieri-Trilemma“ aufgelöst werden können, alle 3 Aussagen scheinen gleichzeitig korrekt.

  23. @Elektroniker // 15.01.2021, 17:00 Uhr

    »Wechselwirkungen zwischen Körper und Geist sind natürlich möglich….«

    Um bei Ihrer Analogie zu bleiben: Wie wechselwirkt die Abbildung des Hauses mit dem abgebildeten Haus?

  24. @KRichard 15.01. 15:27

    „(= was vergeht, wenn Zeit vergeht).“

    Generell kosmisch betrachtet, haben wir den Übergang von der Wellenfunktion zu ihrem Zusammenbruch, der die konkreten physikalischen Fakten erst festlegt. Vergangenheit sind die längst zusammengebrochenen Wellenfunktionen, Gegenwart gerade die Wellenfunktion in ihrem „Schwebezustand“, und die Zukunft der Bereich, der von den sich durch die Zeit ziehenden Wellenfunktionen noch gar nicht erreicht worden sind.

    Bezüglich der Relativitätstheorie bedeutet dieses, dass das 4-Dimensionale „Blockuniversum“ nur für die Vergangenheit gültig ist. Und die Gegenwart bräuchte ein kosmisches Bezugssystem, wenn sie konsistent daran arbeiten will, wie alles Geschehen in diesem Kosmos sich per Zusammenbruch von Wellenfunktionen manifestieren will. Hier kann eigentlich nur das Bezugssystem herhalten, indem der Mikrowellenhintergrund des Universums isotrop ist.

    Hier wäre dann der ganze Kosmos vom Urknall ausgehend gemeinsam dabei, sich durch Realisation des Zusammenbruchs der Wellenfunktionen in der Zeit vorwärts zu arbeiten. Wir als biologische Bewusstseinswesen wären also alle gemeinsam dabei und da mittendrin unterwegs.

    @Martin Holzherr 15.01 15:45

    „Wer sich nicht erklären kann wie Bewusstsein in diese Welt kommt muss doch auch Schwierigkeiten haben zu erklären wie Leben in eine unbelebte Welt kommt. Oder nicht?“

    Oder andersrum: Wenn wir „3. Der Bereich physikalischer Phänomene ist kausal geschlossen.“ aufgeben, und die geistige Mitgestaltung von Zufallseffekten die Grundlage unseres Bewusstseins ist, dann kann ein kosmischer Geist auch in der Zellchemie mitspielen, die Evolution unterstützen und auch zur Entstehung des Lebens beigetragen haben.

  25. @Balanus 15.01. 17:25

    „Um bei Ihrer Analogie zu bleiben: Wie wechselwirkt die Abbildung des Hauses mit dem abgebildeten Haus?“

    Naja, erstmal brauchen wir ein Haus, damit das Licht des Hauses eine Abbildung in der Optik der Kamera erzeugen kann. Und wenn wir kein Haus fotografieren, sondern ein Passbild machen wollen, dann gibt sich der abgebildete Mensch größte Mühe, einen guten Eindruck zu machen, ohne dabei bei den biometrischen Vorgaben durchzufallen.

  26. Bewusstsein ist genau so ein verschwommener imaginärer Begriff wie Materie, Seele oder GEIST.( MATERIE ist ein grober Überbegriff für unzählige Elementarteilchen mit und ohne Masse, GEIST kann jeder für sich-je nach Bildungsstand interpretieren; SEELE ist immer gut wenn man sein Nichtwissen überspielen will ) Bewusstsein ist für mich zuerst ein mentaler Zustand der sich nach dem Grad der Wachheit richtet, also von Neurotransmittern gesteuert wird. Erst dann beginnen die Sinnesorgane zu arbeiten bzw. findet eine Bewertung der Außenwelt statt, was dann das Denken aktiviert. Und das, was wir DENKEN, wird von Gedanken bestimmt die wir konditioniert haben. Unser Bewusstsein ist also ein Spiegelbild unserer Konditionierungen durch Gesellschaft, Religionen, subjektiven Falschmustern etc….So gesehen bestimmt das SEIN das Bewusstsein bzw. das, was man umgangsprachlich dafür hält.(Man könnte auch Vokusssiertheit dafür sagen)

  27. @Tobias Jeckenburger // 15.01.2021, 17:53 Uhr

    » Naja, erstmal brauchen wir ein Haus, damit das Licht des Hauses eine Abbildung in der Optik der Kamera erzeugen kann. «

    Schon klar, aber wie funktioniert es umgekehrt? („Wechselwirkung“).

  28. @Elektroniker

    Würde dies berücksichtigt, sollte das „Bieri-Trilemma“ aufgelöst werden können, alle 3 Aussagen scheinen gleichzeitig korrekt.

    Das Bieri-Trilemma lässt sich immer ganz leicht auflösen, einfach indem man eines der 3 Lemmata abstreitet, oder durch etwas anderes, nicht gleichwertiges ersetzt.

    Wechselwirkungen zwischen Körper und Geist sind natürlich möglich

    Dann folgt, mit Lemma 3 (der Bereich physikalischer Phänomene ist kausal geschlossen), erstens, was nebensächlich ist, dass sie von physikalischen Wechselwirkungen sprechen, und zweitens, was wichtig ist, denn da schlägt das Trilemma zu, dass mentale Phänomene physikalische Phänomene sind.

    Dass sich auch physikalische Phänomene noch in Unterkategorien aufgliedern lassen (wie vielleicht in Bild und Abgebildetes, oder Großes und Kleines), sollte nicht ablenken.

    Wenn sie beide Gedanken in sich hegen, sowohl ‘Physikalisches und Mentales gehören zu unterschiedlichen Kategorien’ als auch ‘Physikalisches und Mentales gehören zur gleichen Kategorie’, handelt es sich dabei um eine kognitive Dissonanz. Eine solche lässt sich durch Verwendung schwammiger Begriffe, verbunden mit Bedeutungswechseln leicht auflösen. Insofern haben wir doch einen Fall gefunden, wo Unverständliches sehr hilfreich werden kann.

    Es geht nicht darum, das Trilemma zu beseitigen. Stecken sie da nicht zuviel Ehrgeiz rein. Bieri hat damit nur Klarheit geschaffen, wo eines der Probleme liegt, zu einer Lösung des Leib-Seele-Problems zu kommen. Diese muss eines der drei Lemmata verneinen, das ist logisch zwingend.

  29. off topic @Grams
    Ich schicke Ihnen morgen einen Brief, in dem beschrieben wird, wie das Gehirn einen einzelnen Reiz systematisch und strukturiert verarbeitet – nach Fulda an die Hochschule. Dass wir bewusst erleben können, wie das Gehirn einen einzelnen Reiz verarbeitet – ermöglicht es, seine Funktionsweise und wichtige Details (wie ´Bewusstsein´) besser zu verstehen.

    Wenn Sie die Texte lesen, können Sie viel über die Arbeitsweise des Gehirns lernen.

  30. @ Stephan Schleim 15.01.2021, 15:06 Uhr

    Freue mich, Herrn Grams Sichtweise zu lesen. Er muss natürlich gemäß dem Stand der Wissenschaft vorsichtig argumentieren.

    Er ist kein weitgehend anonymer Desperado wie ich. Beim Argumentieren falle ich fast schon aus dem Fenster. Bin sehr „brainstormig“ mit meinen „Denkmustern“ unterwegs, die ich ganz locker durch die „Luft wirbeln“ lasse.

    Zu den folgenden vier Prämissen:

    Zitat:
    (1) Der Geist ist ein nichtphysikalisches Ding.
    (2) Der Körper ist ein physikalisches Ding.
    (3) Geist und Körper interagieren miteinander.
    (4) Physikalische und nichtphysikalische Dinge können nicht miteinander interagieren.

    Die vier Annahmen führen zu einem Widerspruch und können daher nicht gleichzeitig wahr sein.

    Zu 1. Diese Aussage sollte „verboten“ sein, wie z.B. die Division durch 0, was dem Vernehmen nach die Mathematiker nur „zähneknirschend“ akzeptieren können. Geistiges kann, muss aber nicht, physikalisch „abgebildet“ werden.

    Zu 4. Dürfte überholt sein. Interaktionen zwischen z.B. „geistiger“ Software und physikalischer Hardware sind heutzutage alltäglich.

    Interpreter Mechanismen machen dies erklärbar.

  31. @ Balanus 15.01.2021, 17:25 Uhr

    Zitat: »Wechselwirkungen zwischen Körper und Geist sind natürlich möglich….«
    Um bei Ihrer Analogie zu bleiben: Wie wechselwirkt die Abbildung des Hauses mit dem abgebildeten Haus?“

    Z.B. mittels Videoübertragung. Die Mechanismen in der Kamera „interpretieren“ die Bildpixel auf dem Bildsensor und wandeln sie in ein digitales Abbild der Realität um. Die Information wird von der alten physikalischen Realität auf eine andere physikalische Realität, z.B. einen Bildschirm übertragen.

    Der Baupolier (oder irgendwann ein riesiger 3 d Drucker) überträgt umgekehrt die Abbildung auf dem Plan in die Realität, er baut das reale Haus.

    Wechselwirkungen zwischen Körper und Geist sind möglich, aber nicht zwingend.

  32. ein etwas anderer Denkansatz

    Ist der Kranke bei Bewusstsein ? Das fragt der Rettungssanitäter. Er meint damit, dass der Kranke lebt, aber vielleicht nicht ansprechbar ist.
    Bewusstsein hat also auch mit Sprache zu tun. Auf Dänisch heißt bewußt „“klar over“
    Es hat also auch mit Verständnis zu tun.

    Hat ein Tier Bewusstsein ? Klar , hat das Tier Bewusstsein. Es ist ansprechbar und es weiß, dass es da ist. Sonst würde die Katze bei einem Hund nicht fauchen, bei einer Maus faucht die Katze nicht, sie fängt sie. Die Katze kann ihre Situation verstehen und handelt danach. Und die Katze hat bewusst gehandelt. Wenn sie nämlich satt ist, dann reagiert sie bei der Maus anders.
    Wir hatten einmal einen Kater, der war sehr wasserscheu. Als es draußen stark regnete, da war er im Zwiespalt, ob er auf die Wiese laufen sollte, er musste nämlich pieseln. An seiner Körpersprache konnte man deutlich ablesen in welchem Zwiespalt er war. Das kann man bewusst nennen.

  33. @hwied (Zitat): „ Klar , hat das Tier Bewusstsein. Es ist ansprechbar und es weiß, dass es da ist. ….Die Katze kann ihre Situation verstehen und handelt danach. „
    Und zweites Zitat: „ Ist der Kranke bei Bewusstsein ? Das fragt der Rettungssanitäter.“
    Absolut richtig. Humanmediziner unterscheiden unterschiedliche Bewusstseinstufen oder um eine entsprechende Website zu zitieren:

    Der Bewusstseinszustand (Level if consciousness = LOC) ist ein medizinischer Begriff, der angibt, wie wach, aufmerksam und sich seiner Umgebung bewusst jemand ist.Er beschreibt auch den Grad, in dem eine Person auf Standardversuche reagieren kann, um ihre Aufmerksamkeit zu erlangen.

    Die Art wie Mediziner den Bewusstseinszustand beurteilen ist meiner Ansicht nach ebenfalls relevant für eine philosophische Diskussion über dieses Thema. Da möchte ich hwied recht geben.
    Nun zum bewussten Verhalten einer Katze. Es kann keinen Zweifel geben dass eine Katze sich bei einfachen Dingen wie der Futtersuche und dem Verhalten gegenüber Feinden und Freunden sich durchaus ähnlich verhält wie ein Mensch. Und es gibt keinen Grund anzunehmen eine Katze sei insoweit nur ein Automat, ein Mensch aber hochbewusst und reflektiert. Beides, das menschliche und tierische Verhalten ist in vielen Situationen ähnlich und ich behaupte auch mental läuft ähnliches ab. Beim Menschen kommen einfach noch höhere Bewusstseinsstufen/ zustände hinzu, er kann beispielsweise über sich selber nachdenken. Doch es gibt einen gemeinsamen Kern. Mit anderen Worten: es gibt einen Gradualismus. Es gibt Tiere mit komplexeren Bewusstseinsphänomenen und solche mit nur einfachen. Eine klare Grenze zwischen animalischem umd menschlichem Bewusstsein existiert aber nicht.

    Was da hwied in seinem kurzen Kommentar geschrieben hat ist wirklich sehr gut.

  34. @Stephan Schleim

    Da würde ich dich ja gerne einmal in meine Vorlesung zum Leib-Seele-Problem einladen.

    Gerne. Du weißt, ich bin bekennender Determinist und mein Glauben lässt sich sicher einer Spielart des Naturalismus zuordnen. Damit lässt sich dialektisch vermutlich gut zusammenarbeiten.

    Darf ich auch dich noch einmal an ‘Das kleine Einmaleins des Leib-Seele-Problems’ erinnern?

    An deinen Artikel kann ich mich erinnern, hatte ihn gelesen, konnte mich aber nicht aufraffen mitzudiskutieren. In solchen Diskussionen muss ich mich ja meistens mehr über andere ‘Naturalisten’ ärgern, die den gleichnamigen Ismus doch oft viel zu naiv vertreten, mehr zumindest als über ‘Idealisten’ wie Herrn Wicht – obwohl der auch hin und wieder lateinische Brocken fallen lässt, die ich nicht verstehe.

    Die Aufteilung in die 4 Punkte, wie Westphal das macht, kann zum Verständnis hilfreich sein, auf jeden Fall, aber das Trilemma bringt es dann doch besser auf den Punkt, wie ich meine. Damit bietet sich uns ein einfaches Schema an, wie man Bereiche, in denen Lösungen gesucht werden, wie soll ich sagen – kategorisieren kann.

    Da muss ich gleich noch ein Hühnchen mit dem Autor rupfen.

  35. Eben habe ich einen fire-side chat mit Geoffrey Hinton auf you-tube geschaut. Geoffrey Hinton ist einer der fürhrenden Köpfe hinter Deep Learning und hat dafür den Turing-Preis erhalten (zusammen mit Joshua Bengio und Yann Le Cun).

    In diesem Fire-Side Chat sagt er, so wie gewisse Leute noch über Bewusstsein gesprochen haben, so haben Leite früher über den élan vital, also über die magische Kraft, die Dinge mit Leben erfüllt, gesprochen. Und genau so wie heute niemand mehr über die „Lebenskraft“ spricht, so wird bald schon niemand mehr über die heutige philosophische Bedeutung von Bewusstsein sprechen.

  36. @ Joker 15.01.2021, 18:07 Uhr

    Das 1. Lemma ist definitiv unzulässig. Jedenfalls in der Informatik. Man kann nicht sinnvoll und allgemein gültig, Objekte unterschiedlicher Kategorien gleichsetzen.

    Ich sehe es vom Standpunkt der Informatik. Würde ich als Programmierer ein Programm hinsichtlich der Kategorien derart fehlerhaft „deklarieren“, wie es im 1. der 3 Lemmata erfolgt, das Programm würde mir um die Ohren fliegen, bzw. der Compiler z.B. 1000 Seiten Fehlermeldungen ausdrucken, die mir dann der Gruppenleiter auf dem Schreibtisch knallt….

    Die Philosophen können sich die Haare raufen, es hilft nichts.

    Wechselwirkungen zwischen Objekten verschiedenen Kategorien z.B. Prozessor – Prozess – Information, sind unter Einhaltung bestimmter Regeln der Informatik realisierbar. Allerdings müssen links und rechts des „Gleichheitszeichen“ die verknüpften Objekte gleichen Kategorien angehören, sonst kann man sie nicht gleichsetzen.

    Meiner Meinung nach könnten Sie vermutlich das „Blau einer Parkbank“ Pixelweise mittels Licht zur Netzhaut übertragen, dort die für blau zuständigen „Zapfen“ aktivieren und die Farbinformation, abgebildet als elektrische Signale, kann z.B. auf der „informellen Ebene“ letztlich physikalisch neuronal verarbeitet werden…., wie alle so ein langenden Informationen.

    Damit wären mentale Phänomene im Bereich physikalischer Abbildung kausal wirksam, weil sie auf die „gleiche Abbildungsebene transformiert“ wurden.

    Der Bereich physikalischer Phänomene wäre, auf der korrekten Betrachtungsebene, auch kausal geschlossen.

    Physikalisches und Mentales müssen auf die gleiche „Kategorienebene transformiert“ werden z.B. die Abbildungsebene.

    Ich wage es nicht auszuschließen, das „Fehlerhaftes mit Fehlerhaftem“ verknüpft „Richtiges“ ergibt.

    Das Hardware Software, besser das Problem „Prozessor – Prozess – Information“, dürfte die Informatik jedenfalls gelöst haben.

    Ich würde vermuten, dass man es beim Leib – Seele Problem als „Vorbild“ nutzen kann.

  37. @Timm Grams

    Der Naturalist[] lehnt 2 ab; also bleiben 1 und 3. Da er nur eine Welt anerkennt, gibt es keine nichtphysikalischen Phänomene. Folglich kann es keine mentalen Phänomene geben. Die Aussage 1 wird damit bedeutungsleer.

    Ich fühle mich als Naturalist, lehne aber 1 ab. Was mache ich falsch?

    Ein Naturalist ist doch nicht darauf festgelegt, die Existenz mentaler Phänomene schlechthin zu leugnen. Das macht doch nur ein eliminativer Materialist.

    Jemand der 2 ablehnt wäre nach meinem Verständnis z.B. ein Epiphänomenalist.

    Was mir bei dieser Gelegenheit auffällt, Philosophen, die entweder mentale oder physikalische Phänomene als nichtexistent betrachten (oder gleich beide Arten) umgehen das Trilemma tatsächlich.

  38. Wenn der Naturalismus so einfach widerlegbar ist, dann frage ich mich, warum die Mehrzahl der Philosophen Naturalisten sind! Sind die alle so dumm?

    Kein Naturalist behauptet, das Welträtsel Bewusstsein gelöst zu haben. Aber es gibt auch keinen Beweis, dass es unlösbar ist. Dagegen ist beweisbar, dass unser Wissen über diese Welt immer unvollständig bleiben wird (Gödel).

  39. @Elektroniker

    Das 1. Lemma ist definitiv unzulässig.

    Es ist definitiv zulässig.

    Es steht nur für eine andere Metaphysik als ihre.

    Ich würde vermuten, dass man es beim Leib – Seele Problem als „Vorbild“ nutzen kann.

    Kaum.

    Das reicht vielleicht für einen ersten Schritt auf einer kilometerlangen Strecke zur Lösung, aber wie jetzt die auf Hardware ablaufende Software Bewusstsein erzeugen soll, geschweige denn Selbstbewusstsein, dazu kann die Informatik doch noch gar nichts sagen. Ignoramus.

  40. @Elektroniker // 15.01.2021, 18:38 Uhr

    » Der Baupolier (oder irgendwann ein riesiger 3 d Drucker) überträgt umgekehrt die Abbildung auf dem Plan in die Realität, er baut das reale Haus. «

    Ich wollte nicht wissen, was ein Polier oder 3-D-Drucker macht, sondern auf welche Weise, Ihrer Meinung nach, das Abbild selbst auf die Abbildungsvorlage einwirkt (von wegen der behaupteten Wechselwirkung zwischen realem Haus und Abbild des Hauses).

    (Dass das Abbild selbst wiederum als Vorlage für ein Abbild dienen kann, steht nicht in Frage).

  41. @hwied
    Ihre ersten zwei Absätze gehe ich mit.

    @Reutlinger
    Gödels Unvollständigkeit könnte man als ‘Positivbeweis’ benennen, das Paradoxon als ‘Negativbeweis’.

    Widerspruchsfreiheit dürfte nur ein Teil der Bewusstheit sein,des Seins als Voraussetzung des sein.

    @Holzherr
    Was schlägt er denn als Alternative vor?

    Aber:wenn Naturalismus u.a. als Physikalismus interpretiert bzw. assoziiert wird,was ist dann die physikalische Entität, die diese Grundlage vereinen könnte?

  42. Jegliche Metaphysik sorgt für eine Einschränkung des Denkens.

    Da gab es doch schon mal Versuche, die Metaphysik aus dem Denken zu vertreiben. – Meines Wissens alle gescheitert.

    Ich würde das eher so formulieren: Eines jeden Metaphysik gibt den Bereich vor, in dem er eine Lösung der Probleme sucht und in deren Rahmen er Erklärungen akzeptieren könnte, auch für das Leib-Seele-Problem.

    Überzeugende Erklärungen bringen einen dazu, sich die zugehörige Metaphysik anzueignen. Fehlende Erklärungen lassen einen den Blick schweifen.

    Er [der Naturalist] scheitert schon auf der Ebene der Logik

    Das hört man oft.

    Natürlich können es meine begrenzten Fähigkeiten sein, logisch zu denken, die mich im Eindruck verharren lassen, noch nicht gescheitert zu sein. Wo man bisher meinte, mir Selbstwidersprüchlichkeit nachweisen zu können, waren es jedoch eher Annahmen die zum Widerspruch führten, die ich gar nicht vertrete. Da waren dann nicht explizit genannte Voraussetzungen in Argumente eingebunden, die der Andere als gegeben betrachtet hat, die oft einfach aus seiner Metaphysik stammten, zumindest aber nicht aus meiner. In Ihrem Text sehe ich da (noch) keine Ausnahme.

    Letztlich wurde also immer nur nachgewiesen, dass Unterschiede oder Widersprüche zwischen verschiedenen Glaubenssystemen bestehen. Dem wiederum würde ich vorbehaltlos zustimmen, auch ohne Beweis.

  43. Das selbe nochmal, mit Zitatfunktion

    Jegliche Metaphysik sorgt für eine Einschränkung des Denkens.

    Da gab es doch schon mal Versuche, die Metaphysik aus dem Denken zu vertreiben. – Meines Wissens alle gescheitert.

    Ich würde das eher so formulieren: Eines jeden Metaphysik gibt den Bereich vor, in dem er eine Lösung der Probleme sucht und in deren Rahmen er Erklärungen akzeptieren könnte, auch für das Leib-Seele-Problem.

    Überzeugende Erklärungen bringen einen dazu, sich die zugehörige Metaphysik anzueignen. Fehlende Erklärungen lassen einen den Blick schweifen.

    Er [der Naturalist] scheitert schon auf der Ebene der Logik

    Das hört man oft.

    Natürlich können es meine begrenzten Fähigkeiten sein, logisch zu denken, die mich im Eindruck verharren lassen, noch nicht gescheitert zu sein. Wo man bisher meinte, mir Selbstwidersprüchlichkeit nachweisen zu können, waren es jedoch eher Annahmen die zum Widerspruch führten, die ich gar nicht vertrete. Da waren dann nicht explizit genannte Voraussetzungen in Argumente eingebunden, die der Andere als gegeben betrachtet hat, die oft einfach aus seiner Metaphysik stammten, zumindest aber nicht aus meiner. Letztlich wurde also immer nur nachgewiesen, dass Unterschiede oder Widersprüche zwischen verschiedenen Glaubenssystemen bestehen. Dem wiederum würde ich vorbehaltlos zustimmen, auch ohne Beweis.

  44. @ Balanus 15.01.2021, 20:35 Uhr

    Zitat: „… auf welche Weise, Ihrer Meinung nach, das Abbild selbst auf die Abbildungsvorlage einwirkt (von wegen der behaupteten Wechselwirkung zwischen realem Haus und Abbild des Hauses).“

    Wechselwirkungen zwischen Körper und Geist sind möglich, aber nicht zwingend. (Steht am Ende meines Textes).

    Das „Bild des Hauses“ (Photo) will eben nicht mit dem „Haus“ eine Wechselbeziehung aufnehmen.
    Ein Roboter würde es vielleicht „wollen“.

    Es ist wie beim Menschen. Wenn Sie z.B. visuelle, oder verbale Information (Geistiges) aufnehmen wollen, so machen Sie es eben. In Ihrem Gehirn entstehen dieses Wissen abbildende synaptische Verknüpfungen, also ein „Abbild“ im mathematischen Sinne. (E. Kandel).

    Gemäß dem Konzept von W. McCulloch („Perzeptron“) können Sie (aus der „Abbildung des Geistigen“) einen Output, auch als Wechselbeziehung) generieren.

  45. Den Satz Mentale Phänomene sind nichtphysikalische Phänomene muss man hinterfragen.

    Es gilt zu unterscheiden zwischen den Objekten, mit dem sich ein mentaler Prozess beschäftigt und dem mentalen Prozess selbst.
    Beispiel: Ein Mathematiker, der sich beispielsweise mit Primzahlen beschäftigt, der beschäftigt sich mit etwas nicht physikalischem, denn Primzahlen sind keine physikalischen Objekte. Doch der mentale Prozess, der dabei abläuft, der setzt einen Körper und ein Hirn voraus und beides sind physikalische Objekte.
    Das mentale Phänomen „Primzahl“ (woran gedacht wird) hat etwas nichtphysikalisches , etwas Fiktives als Objekt des Gedankens, aber der Gedanke selbst, der mentale Prozess selber läuft auf/in einem physikalischen Objekt (Körper+Hirn) und beinhaltet physikalische Prozesse.

  46. @Joker
    Für Naturalisten, die Vollmer oder Mahner folgen, gibt es nur eine Welt, nämlich die materielle. Sie werden den Satz “Mentale Phänomene sind nichtphysikalische Phänomene” nicht nur ablehnen, was ja immer möglich ist, sondern sogar für sinnlos halten, weil es in ihrer einen Welt keine nichtphysikalischen Phänomene gibt.

    Der von mir angesprochene Widerspruch bezieht sich nur auf diese Spielart des Naturalismus. Andere Spielarten mögen ungeschoren davonkommen.

    @Reutlinger
    “Die Mehrzahl der Philosophen [sind] Naturalisten.” Ohne statistische Belege werde ich das so nicht weitergeben. Wenn ich Kollegen frage, ob sie Naturalisten sind, höre ich meistens tatsächlich “ja”. Erzähle ich ihnen dann, was Vollmer und Mahner darunter verstehen, ist es mit der Zustimmung vorbei.

  47. @ Joker 15.01.2021, 20:29 Uhr

    Zitat: „Es ist definitiv zulässig.“

    O.k. Aber dann kommen Sie eben auf Widersprüche.

    Ich habe einmal gelernt, man müsse, z.B. in der Mathematik, solange „herum formulieren“ bis man auf Widersprüche stößt, die nicht aufzulösen sind.
    Dann muss man eine These „beerdigen“.

    Die „andere Metaphysik“ Ruhe in Frieden!

    Im Vertrauen, ich würde mich freuen, könnte jemand meine „Empfindungs Thesen“ im Zusammenhang mit „Bewusstsein“ („Hirnkino“), wie oben angeführt, sachlich „zerlegen“. (So dass mir sozusagen vor Staunen das Gebiss aus dem Mund fällt….)

    Sonst wäre die restliche Wegstrecke recht kurz geworden….

  48. @Elektroniker // 15.01.2021, 21:48 Uhr

    »Gemäß dem Konzept von W. McCulloch („Perzeptron“) können Sie (aus der „Abbildung des Geistigen“) einen Output, auch als Wechselbeziehung) generieren.«

    Nach meinem Verständnis handelt es sich bei dieser „Abbildung“ um einen Sinneseindruck, also das Bild, das im Kopf „entsteht“, wenn wir etwas betrachten. Also das „Abbild“, das vor unserem „geistigen Auge“ (eine schöne Metapher, übrigens) erscheint oder aufscheint. Ich kann mir nicht vorstellen, wie eine solche Erscheinung auf irgendetwas in der physikalischen Welt einwirken könnte.

    Vielleicht haben Sie weiter oben mit „Abbildung“ ja etwas anderes gemeint, womöglich die neuronale Aktivität, die dem bewussten Erleben zugrunde liegt (nach naturalistischer Auffassung). Aber diese wäre ja Teil der physikalischen Welt und stellt somit kein philosophisches Problem dar.

  49. @Elektroniker

    O.k. Aber dann kommen Sie eben auf Widersprüche.

    1) führt nur dann zu Widersprüchen, wenn ich auch noch 2) und 3) gelten lasse.

    Dann kann man aber nicht sagen, ob 1), 2) oder 3) Schuld daran haben, kein Lemma ist ausgezeichnet. Zusammen sind sie halt nicht widerspruchsfrei. Das steht ja eigentlich alles schon richtig beschrieben im Text.

    Versuchen Sie doch mal einen Widerspruch aufzuzeigen, wenn nur 1) und 2) gilt, oder wenn nur 1) und 3) gilt.

    Wie gesagt, ein Epiphänomenalist wird vielleicht die letzte Auswahl übernhemen wollen und 2) bestreiten. Auch wenn wir uns beide dieser Philosophie nicht anschließen wollen, logisch widersprüchlich ist sie nicht.

  50. @anton reutlinger // 15.01.2021, 20:18 Uhr

    »Wenn der Naturalismus so einfach widerlegbar ist, dann frage ich mich, warum die Mehrzahl der Philosophen Naturalisten sind! «

    We are all naturalists now!” hat Sellars 1922 proklamiert.

    Leider (oder glücklicherweise) gibt es keine allgemeingültige Festlegung, was genau unter ‚Naturalismus‘ zu verstehen wäre.

    Ich glaube, der von Sellars (oder z. B. John Dewey) gemeinte Naturalismus lässt sich nicht so leicht widerlegen.

  51. Zitat:

    “ für mich spitzt sich alles auf die Frage zu, ob mein Nachbar die Farbe Rot genauso empfindet wie ich, oder ob sich sein Erleben derselben Situation von dem meinen unterscheidet.“

    Das kann man in der Tat nicht wissen. Noch weniger kann man wissen wie ein Tier, das die Farbe rot erkennen kann, dies erlebt.
    Wir wissen aber auch nicht wie ein künstliches neuronales Netz, das gelernt hat, rote von blauen Bällen zu unterscheiden, diese Farben/Farbunterschiede intern repräsentiert. Unterschiedliche neuronale Netze können das unterschiedlich handhaben. Alles was wir zuerst einmal erfahren ist, ob sie die gestellte Aufgabe lösen können, ob sie also etwa die Frage beantworten können: „was für eine Farbe hat die Kugel ganz links im Bild“. Neuronale Netze sind nämlich weitgehend „Black Boxes“, so wie das Menschen auch sind.
    Eines aber ist sicher: Wenn ein Mensch, ein Tier oder ein neuronales Netz eine rote Kugel erkennt, dann gibt es dafür ein transientes „neuronales Korrelat“, also eine Aktivierung von Neuronen/Zellen. Unsere Sinnesempfindungen haben also eine physikalische Grundlage, denn ein neuronales Korrelat ist etwas, was für kurze Zeit, nämlich für die Zeitdauer der Wahrnehmung, als physikalisches Objekt existiert.

  52. @Balanus (Zitat):

    “Leider (oder glücklicherweise) gibt es keine allgemeingültige Festlegung, was genau unter ‚Naturalismus‘ zu verstehen wäre.“

    Es hat keinen Sinn über etwas zu sprechen von dem jeder eine andere Vorstellung hat. Heute ist es naheliegend die Wikipedia als Referenz zu wählen, wobei dann noch die Frage bleibt ob es die deutsche oder englische Version sein soll. Im konkreten Fall, der Frage also was Naturalismus in der Philosophie bedeute bevorzuge ich die englische Version wo man gleich schon als ersten Satz folgendes liest (übersetzt aus dem Englischen)::

    In der Philosophie ist Naturalismus die Idee oder der Glaube, dass nur natürliche Gesetze und Kräfte (im Gegensatz zu übernatürlichen oder spirituellen) im Universum wirken.[1] Anhänger des Naturalismus behaupten, dass Naturgesetze die einzigen Regeln sind, die die Struktur und das Verhalten der natürlichen Welt bestimmen, und dass das sich verändernde Universum in jeder Phase ein Produkt dieser Gesetze ist.

  53. @Mussi (Zitat):

    “ Was schlägt er denn als Alternative vor?“

    Antwort: Geoffrey Hinton schlägt als Alternative für den Bewusstseinsbegriff wie er in der Philosophie verwendet wird, vor, das Hirn als Denkorgan zu verstehen und es als Netz von neuronalen Netzen aufzufassen. Hinton glaubt, dass wir alles was im Hirn passiert und alle Phänomene inklusive Gefühle und Empfindungen nachbilden können, dass also künstliche neuronale Netze zu allem fähig sind, wozu auch wir fähig sind.

  54. @Joker, Grams: Trilemma & Naturalismus

    Ich würde das auch so sehen, dass Materialisten am ehesten Prämisse 1 ablehnen: Dann ist 2 kein Problem mehr, denn dann bleibt die Kausalität sozusagen dem Physikalischen vorbehalten und bleibt somit auch 3 gewahrt…

    …dann hat man eben nur das Problem, zu erklären, wie genau aus Körperlichem Geistiges hervorgeht (und worauf sich unser psychologisches Vokabular bezieht).

    Naturalismus ist für mich übrigens eine epistemische Position, sagt also etwas über die Aussagekraft der Naturwissenschaften aus, nicht so sehr darüber, “was die Welt im Innersten zusammenhält”, also der Form: Die Welt lässt sich vollständig naturwissenschaftlich erklären.

    Dann wird auch sofort klar, dass es sich um einen philosophischen und keinen naturwissenschaftlichen Satz handelt. Er könnte wahr oder falsch sein; wahrscheinlich ist er falsch.

    Kürzlich schrieb ich selbst, mir vorstellen zu können, dass Bewusstsein naturwissenschaftlich erklärt werden kann – es wird bloß eine andere Naturwissenschaft sein müssen.

  55. @Timm Grams

    Für Naturalisten, die Vollmer oder Mahner folgen, gibt es nur eine Welt, nämlich die materielle.

    Das gilt in gewisser Weise auch für mich (*), ohne dass ich da einem von beiden in allen Punkten folgen würde.

    Sie werden den Satz “Mentale Phänomene sind nichtphysikalische Phänomene” nicht nur ablehnen, was ja immer möglich ist, sondern sogar für sinnlos halten, weil es in ihrer einen Welt keine nichtphysikalischen Phänomene gibt.

    Das bezweifle ich. Auch wenn es keine nichtphysikalischen Phänomene gibt, kann es Unterklassen geben, die anders bezeichnet werden.

    Ich selbst erkenne durchaus an, dass in der Welt verschiedenartige Phänomene auftreten, z.B. meteorologische, akustische, politische oder eben auch mentale. Die ersten lassen sich vermutlich recht leicht als eine Unterklasse physikalischer Phänomene beschreiben und verstehen, bei den letzten beiden ist das eben eher knifflig (politische Phänomene als physikalische zu verstehen, scheint mir davon übrigens die größere Herausforderung zu sein).

    Vollmer vertritt, was das Mentale betrifft, eine Identitätstheorie. Er schreibt in dem auch von Ihnen angegebenen ‘Gretchenfragen an den Naturalisten’, S.59:

    Nach der Identitätstheorie ist jeder mentale Vorgang mit einem neuronalen Vorgang verbunden, letztlich ja sogar mit ihm identisch. (Die Umkehrung gilt nicht: Die meisten neuronalen Vorgänge sind uns nicht bewusst.)

    Ich vermute, es würde ihm nicht schwerfallen, statt von mentalen Vorgängen, an dieser Stelle auch einfach von mentalen Phänomenen zu sprechen, die durch neuronale Vorgänge erzeugt werden. Zwei Seiten weiter spricht er ja sogar ganz unbefangen über Qualia.

    (*)
    Gleichzeitig lasse ich auch Schopenhauer gelten, “bei gleicher Umgebung lebt doch jeder in einer anderen Welt.” Was sich ja hier scheinbar bestätigt. Also zugegeben, auch ich leide teils unter kognitiven Dissonanzen.

  56. @Balanus 15.01. 17:53 / 18:03 / 22:39

    „Um bei Ihrer Analogie zu bleiben: Wie wechselwirkt die Abbildung des Hauses mit dem abgebildeten Haus?“

    „Schon klar, aber wie funktioniert es umgekehrt? („Wechselwirkung“).“

    Um beim Passbild zu bleiben: das Motiv wird sich alle Mühe geben, auf dem Foto gut auszusehen. So wirkt der Mensch auf sein Abbild. Und wenn die Person sich das Bild ansieht, und für gut hält, wird er es demnächst als Passbild in seinen Ausweisen mit sich tragen, und gibt dem Foto damit Gelegenheit, auf alle Betrachter einzuwirken, z.B. auf die Zöllner, um diese nach Möglichkeit dazu motivieren, einen durchzuwinken. Wir haben also in diesem speziellen Fall eine wirkliche Wechselwirkung bezüglich Motiv und Foto.

    Es kommt einfach auf das Foto an, ob es was bewegt, oder nicht. Hier sollen durch Fotos von Kriegsopfern schon Kriege beendet worden sein.

    „Also das „Abbild“, das vor unserem „geistigen Auge“ (eine schöne Metapher, übrigens) erscheint oder aufscheint. Ich kann mir nicht vorstellen, wie eine solche Erscheinung auf irgendetwas in der physikalischen Welt einwirken könnte.“

    Natürlich nicht direkt, sondern durch meine Handlungen, die ein internes Bild motivieren kann. Wenn die intern abgebildete schöne Frau in mein Beuteschema passt, werde ich vielleicht gucken, wie ich ihr näherkommen kann. Und dies mit allen Nebenbedingungen abgleichen, etwa ob dies überhaupt praktikabel ist. Wenn die schöne Frau im Zug gegenüber sitzt, der gerade in die andere Richtung weiterfährt, werde ich nichts weiter veranlassen. Sitzt sie aber im selben Abteil mir gegenüber, dann gibt es eventuell was zu unternehmen. Hier werden meine Hormone gleich eine ganze Kaskade von spezifisch männlichem Verhalten in Gang setzen. Dennoch ist das eigene Bewusstsein die entscheidende Schaltstelle auch für recht instinktives Verhalten.

    Unsere innere Erlebniswelt ist eben nicht nur wie ein ablaufender Film, dass ist doch mindestens so interaktiv, wie ein Computerspiel. Die Idee, unser Bewusstsein wäre ein reines Epiphänomen, und hätte rein gar nichts mit unserer Handlungsebene zu tun, ist wohl ziemlich weltfremd. Und auch irgendwie ziemlich respektlos.

    So oder so, was ich nicht brauche ist:

    3. Der Bereich physikalischer Phänomene ist kausal geschlossen.

    Dieses kann ich eigentlich problemlos aufgeben. Das erscheint mir doch am einfachsten zu sein, wenn ich mir mein Leben betrachte. So fügt sich einfach alles besser zusammen.

    1. Mentale Phänomene sind nichtphysikalische Phänomene.
    2. Mentale Phänomene sind im Bereich physikalischer Phänomene kausal wirksam.

    Ist für mich in jedem Fall widerspruchsfrei und eben auch ausreichend.

  57. Der Satz ´Mentale Phänomene sind nichtphysikalische Phänomene´ ist eine unsinnige Behauptung/Aussage.

    Denn mentale Phänomene können bei Menschen nur auftreten, wenn die drei Voraussetzungen für die Entstehung von Bewusstsein erfüllt sind:
    a) ein Mensch muss lebendig sein
    b) verschiedene Gehirnareale müssen vernetzt zusammenarbeiten
    c) es muss eine bestimmte Aktivitätsschwelle überschritten werden (z.B. EEG Alpha-Wellen > 8 Hz)

    Wenn diese drei Bedingungen nicht erfüllt sind, sind mentale Phänomene – also bewusste Wahrnehmung / ein Bewusstsein – nicht möglich. Jeder Neurologe kann mit Aktivitätsmessungen des Gehirns belegen, dass unser Gehirn auch in einem unbewussten Zustand aktiv ist.

    Unsere Neuronen sind eindeutig materiell-physikalischer Natur. Deshalb kann es keine nichtphysikalischen Bewusstseinsphänomne geben.
    Sondern nur eine Fehlinterpretation von Effekten – wie die Behauptung in dem obigen Satz.

  58. Jeder zumindest Ausdauersportler macht Erfahrung mit Endorphinen bis hin zu SuchtErscheinungen und depressiven Zügen bei Entzug.

  59. Epigenetische Erkenntnisse weisen auch eher darauf hin,dass Körper=Geist bzw. Geist=Körper wahrscheinlicher ist.

  60. Psychische Phänomen wie Phantasie,Imagination,Glaube und Autosuggestion sind wo?
    Mir fällt es schwer,sie außerhalb des Lebens zu finden.

  61. Zitat: “ Für Naturalisten, die Vollmer oder Mahner folgen, gibt es nur eine Welt, nämlich die materielle.
    Doch: Geoffrey Hinton ist ein Materialist, glaubt er doch das Hirn, die Hirnleistungen liessen sich nachbauen, wenn man nur die richtigen Legosteine zusammensetze, doch gleichzeitig basiert das Gebiet auf dem er arbeitet, die künstlichen neuronalen Netze nämlich, auf dem mathematischen Gebiet der Linearen Algebra und gehört damit nicht zur materiellen Welt.

    Folgerung: Wer an die Naturwissenschaften glaubt und alles damit erklärt, also alles „materiell“ erklärt, verwendet fast immer Mathematik und damit geistige Gebilde, die selbst nicht materiell sind.

  62. Was ist ‘materiell’? Nach Einsteins Äquivalenzprinzip und Higgs-Boston ist Materie Energie.
    Folglich kann Geist Energie sein und somit physikalisch.

  63. zwei Beispiele zu unserem Weltbild bzw unserer Wahrnehmung:

    A) Wenn man eine Batterie in die Uhr einlegt oder ein Federwerk aufzieht, dann läuft die Uhr indem diese zugefügte potentielle Energie in kinetische Energie umgewandelt wird – wodurch ein Zählwerk angetrieben und als Sekundäreffekt eine Kalenderanzeige bewegt wird, wo wir dann Kalenderdaten ablesen die wir als ´Uhrzeit´ bezeichnen.
    ´Zeit´ vergeht indem wir einen einseitig gerichteten Fluss von Energie haben: kurz – das Wesen von Zeit ist Energie
    Wir haben uns aber den Fehler angewöhnt unter ´Zeit´ die angezeigten Kalenderdaten (= Uhrzeit) zu betrachten. In anderen Worten: für unser Weltbild gehen wir von einer 100%ig falschen Annahme aus.
    Das sind wir so gewohnt und deshalb stört sich auch niemand daran

    Die Buddhisten (vor 2500 Jahren) und Bischof Augustinus (vor 1600 Jahren, in ´Bekenntnisse´) gehen davon aus, dass ´Zeit´ keine Dauer hat und legen Argumente für diese Behauptung vor. Auch diese Hinweise werden ignoriert.

    B) Wir wissen genau, dass im Kinofilm nur unbewegte Fotos gezeigt werden. D.h. unsere Wahrnehmung von bewegten Bildern ist zu 100% falsch. Dass unsere Wahnehmung zu 100 % falsch ist (mehr all 100% Fehler geht nicht), stört uns nicht. Wir haben uns an diese Fehlleistung unseres Gehirns gewohnt.

    Diese beiden Beispiele zeigen, dass hierbei unsere Wahrnehmung bzw. unser Weltbild zu 100 % auf Fehlannahmen beruht – ohne dass wir uns daran stören.

    Diese Tatsachen sollte man auch in Überlegungen zum Thema ´Bewusstsein´ einbeziehen. Denn es könnte sein, dass die Idee vom ´Bewusstsein´ nur ein Ergebnis von Fehlannahmen ist.
    Denken Sie daran wenn Sie die bewegten Bilder eines Kinofilms genießen.

    Beim Thema ´Bewusstsein´ muss man sich auch mit dem Phänomen ´Zeit´ befassen – denn man muss z.B. klären, wie lange ein Gedanke überhaupt dauern kann.

  64. @Mussi (Zitat): „ Nach Einsteins Äquivalenzprinzip und Higgs-Boston ist Materie Energie.“
    Für unsere Betrachtungen ist Energie auch materiell, denn eigentlich geht es darum ob etwas physikalisch oder nicht physikalisch ist. Im englischen Sprachraum spricht man deshalb häufig vom Physikalist und ein ontologischer Materialist aus Wien ist in London ein Physikalist.

  65. @KRichard (Zitat): “ Denn es könnte sein, dass die Idee vom ´Bewusstsein´ nur ein Ergebnis von Fehlannahmen ist.
    Materialisten wie Geoffrey Hinton halten das, was die Philosophen unter dem Begriff „Bewusstsein“ diskutieren für eine Illusion. Die Illusion liegt für Materialisten vor allem darin, dass man dem Bewusstsein eine eigene „Welt“ zuschreibt so als würde man Autos eine eigene Welt zuschreiben. Dabei basiert sowohl die Welt des Bewusstseins als auch die Welt des Autos im Materiellen.

  66. @Holzherr
    Wir können es nicht ausschließen. Agnostizismus.Fertig.
    Der ‘Rest ist Spielerei’.
    Nach heutiger Kenntnis.

  67. @Schleim
    Kürzlich schrieb ich selbst, mir vorstellen zu können, dass Bewusstsein naturwissenschaftlich erklärt werden kann – es wird bloß eine andere Naturwissenschaft sein müssen.

    Diese Aussage ist sicher nicht falsch – aber bringt sie uns weiter? Die Kugeltheorie der Erde ist eine andere Naturwissenschaft als die Scheibentheorie, die Relativitätstheorie ist eine andere Physik als die klassische.

    Dass der Naturalismus noch keine Lösung hat für das Bewusstsein ist kein Argument gegen ihn. Was wären denn die Alternativen zum Naturalismus, wären sie besser, hätten sie eine Lösung? Man kann empirische Wissenschaften von nichtempirischen unterscheiden, aber auch das hilft uns nicht weiter.

    Die allermeisten Argumente des Antinaturalismus oder Antimaterialismus sind nichts weiter als billige Wortspiele, die sich bei näherer Betrachtung in Aggregationen, in Synonymen und Homonymen verflüchtigen. Ein Auto ist ein Aggregat seiner Bauelemente, das andere Eigenschaften und Funktionen hat, als wenn man die Bauelemente nebeneinander legt.

    Wir müssen uns nun mal damit abfinden, dass wir die Welt nicht vollständig erkennen und erklären können – ignorabimus. Eine weiße Figur vor einem weißen Hintergrund ist nicht erkennbar, es sei denn, man weiß vorab, dass es eine weiße Figur vor einem weißen Hintergrund ist. Uns fehlt das Vorwissen, um die Welt vollständig erkennen zu können, siehe die “reine Vernunft” bei Kant.

  68. @Mussi (Zitat): „ Folglich kann Geist Energie sein und somit physikalisch.“
    Ja. Wobei Philosophen die von Bewusstsein sprechen damit nicht einverstanden wären. Denn für sie ist Geist auch nicht physikalische Energie.

    Zurück in die Vergangenheit: Früher erzählten Leute von Menschen, die sich nur von Licht ernährten und Licht galt als etwas nicht-materielles, als etwas Geistiges. Wer sich von Licht ernährte war nicht mehr von dieser Welt.
    Zurück in der Jetzt-Zeit: Heute wissen wir, dass derjenige, welcher sich nur von Licht ernährt nicht verhungern muss, denn auch Photovoltaikanlagen „ernähren sich“ nur von Licht und das reicht für allerhand Kraftanstrengungen (ja auch E-Autos können Muscle-Cars sein).

  69. @Joker
    Für Gerhard Vollmer gehören zum Naturalismus sowohl der ontologische Naturalismus (Welt unabhängig vom Bewusstsein) als auch die monistisch-materialistische Identitätstheorie. Darin sehe ich einen Widerspruch.

  70. @Holzherr
    Ja,ist doch witzig!
    Schön,dass wir das auch salutogenetisch sehen können und nicht pathogenetisch müssen!

  71. @Martin Holzherr // 16.01.2021, 10:18 Uhr

    »… geistige Gebilde…«

    Was genau sind „geistige Gebilde“?

    Für Naturalisten sind das doch nichts weiter als biophysikalische Gehirnzustände.

    Für Naturalisten gibt es keinen Gedanken, keine Idee und keine Erklärung außerhalb (jenseits) materieller Entitäten. Oder, mit Blick auf Timm Grams Tabelle, keine Innenwelt („Diesseits“) ohne die Außenwelt („Jenseits“). Grams Innenwelt ist einfach nur Teil der „einen“ Welt des Naturalisten.

  72. Am Anfang eines Montagebandes liegen die Bauelemente des Autos, am Ende steht das fertige Auto. Was geschieht dazwischen? Wird ihm Geist eingesprüht, damit das Auto Geräusche abgibt, damit sich die Räder drehen, dass es eigenwillig losfährt am Hang oder manchmal stehenbleibt? Die Antinaturalisten sollten Erklärungen haben.

  73. Holzherr,
    wir sind auf einer Ebene

    Denkansatz 2, was ist Bewußtsein ?

    Beobachten wir einmal Säuglinge. Die spucken den Brei aus, der ihnen nicht schmeckt.
    Kleinkinder zeigen ganz deutlich, wenn ihnen etwas nicht passt, sie verziehen das Gesicht, spitzen den Mund und los geht das Geschrei. Und wenn sie erst einmal das Wort „Nein“ entdeckt haben, dann haben sie ihren Willen entdeckt.

    Sie haben entdeckt, dass sie auch jemand sind, sie haben das Ich entdeckt. Und sie haben ihren Willen entdeckt. Und wehe der Mutter, die ständig nachgibt, die erzieht sich Tyrannen.

    Die Kinder tun das ganz bewusst und sie erkennen ihre Stärke. Wollen wir hier das Wort Bewußtsein verwenden, noch nicht. Es geht um das Ich und es geht um den Willen.

    Und wenn sie den Willen ganz bewusst durchsetzen, dann zeigen sie Bewusstsein, man kann es auch Selbstbewusstsein nennen. Und manche Kinder sind so findig, dass sie ihre Eltern manipulieren können.
    Wir befinden uns gerade in der Entwicklungspsychologie, Da ist kein Platz für Spekulationen, ob das Bewusstsein eine Illusion ist. Wer so denkt, der hat noch nie mehrere Kleinkinder hüten müssen.

    Verdict: Bewußtsein = Ich + Wille

  74. @Balanus (Zitat):

    „ Was genau sind „geistige Gebilde“?

    Für Naturalisten sind das doch nichts weiter als biophysikalische Gehirnzustände.

    Ein Mathematiker wäre wohl nicht einverstanden wenn man sein Lieblingsgebiet in der Mathematik als (Zitat) „ nichts weiter als biophysikalische Gehirnzustände“ bezeichnen würde (dafür würde er wohl nicht einmal einen Forschungskredit erhalten).
    Ein Mathematiker kann aber trotzdem ein Materialist sein.

  75. Solche Beiträge amüsieren mich. Ich stelle mir dann vor, ein Raumschiff mit Aliens, die uns mental und technisch haushoch überlegen sind, beobachten uns. Ich höre sie sagen, ‘was ist das für eine seltsame Spezies da unten. Sie schlagen sich gegenseitig die Köpfe ein und machen ihren Planeten kaputt, glauben aber, sie seien die Krone der Schöpfung. Was für eine Schöpfung? Sie glauben, jemand hätte sie erschaffen und wenn sie tot sind, leben sie trotzdem weiter, weil ihr, wie sie es nennen, Bewusstsein oder Geist, nicht stirbt. Aber warum denken sie denn so etwas? Ihre Neuronen sind nur niederdimensional verknüpft, sie denken überwiegend assoziativ. Schade, dass so ein schöner Planet solche einfachen Bewohner hervorbringt.’

  76. @hwied (Zitat 1): “Bewusstsein = Ich + Wille;“
    Zitat 2: “ Sie haben entdeckt, dass sie auch jemand sind, sie haben das Ich entdeckt. Und sie haben ihren Willen entdeckt. “
    Antwort: Bewusstsein ist keine Illusion, eine Illusion aber ist, Bewusstsein spiele sich auf einer ganz anderen Welt ab, es sei ausserhalb der materiellen Welt.
    Wie gesagt, es gibt unterschiedliche Grade von Bewusstsein und gewisse höhere Bewusstseinszustände, in der man sich etwa seiner selbst bewusst wird, müssen nicht bei allen Lebewesen vorkommen. Doch es gibt keine starke Trennlinie zwischen dem Bewusstsein einer Katze und der eines Menschen. Das Bewusstsein ist von dieser Welt und wer mehr davon hat, besitzt bildlich gesprochen vielleicht über ein fortgeschritteneres Vehikel (einen luxury car?)

  77. @hwied
    Das Wort “Illusion” kann ganz unterschiedlich gedeutet werden: ein unerfüllbarer Wunsch, eine irreale Idee oder Vorstellung, eine Wahrnehmungstäuschung, und mehr. Das ist eben ein Problem im nichtempirischen oder abstrakten Bereich von Diskussionen. Was genau ist mit einem Begriff gemeint, besonders bei historisch, interkulturell oder fremdsprachlich entstandenen Begriffen? Wenn das nicht klar ist, dann entstehen Folgefehler und noch größere Missverständnisse bis zum Zusammenbruch eines Diskurses. Darin liegt die Begründung der Analytischen Philosophie.

  78. Stegemann,

    Menschen denken überwiegend assoziativ. Das ist richtig und so können sie das Angenehme mit dem Nützlichen verbinden.
    Die Naturwissenschaftler, die denken auch analytisch, manche Ehefrauen übrigens auch, wenn sie in der Hosentasche ihres Mannes einen Zettel mit einer Telefonnummer finden.

    Die Aliens , die denken wie die Ameisen, ohne Humor, ohne Liebe, nur nützlich.
    Die sollten wir nicht als Vergleich heranziehen.

  79. Martin Holzherr,
    Auf ihren Gedanken mit der Welt außerhalb, komme ich bei “Bewußtsein als Reflexion ” zu sprechen. Geduld.

    Reutlinger,
    Tatsächlich ist der Begriff Illusion mehrdeutig.
    Und wir müssen prüfen, ob wir eine materialistische, naturwissenschaftliche Sichtweise anwenden, oder ob wir eine idealistische Sichtweise zugrunde legen.
    Später, Hunger ist keine Illusion !

  80. @hwied
    Hunger ist doch ein gutes Beispiel,wie ein Energiedefizit die Handlung und Phantasie anregen kann. Auf was für Ideen man da kommen kann.
    In welcher ‘Welt’ findet das statt?

  81. @Stegemann
    Ja,ist schon immer wieder erstaunlich,wie es bei der ‘Flucht aus der Realität’ einen durch die eigene Bedingheit ‘auf die Erde’ zurückwirft. 🙂

  82. @ anton reutlinger 16.01.2021, 11:15 Uhr

    Zitat: „Am Anfang eines Montagebandes liegen die Bauelemente des Autos, am Ende steht das fertige Auto. Was geschieht dazwischen? Wird ihm Geist eingesprüht, damit das Auto Geräusche abgibt, damit sich die Räder drehen, dass es eigenwillig losfährt am Hang oder manchmal stehenbleibt? Die Antinaturalisten sollten Erklärungen haben.“

    Heutzutage ist es nicht mehr ganz so wie Sie vermuten.

    Einem modernen Auto wird tatsächlich „Geist“ (neueste Software) eingespielt, z.B. auch noch beim 1. Service. Ein Auto muss schließlich „wissen“, wie es sich in jeder Situation zu „benehmen“ hat. Z.B. beim Bergabfahren spritzt es keinen Treibstoff ein und ist besonders umweltfreundlich, es passt sich total an die jeweiligen, sehr zahlreichen Bedingungen (Hitze, Kälte, Regen, Eis, Nacht, Tag, bergauf, bergab, schnell, langsam, … ) an. Natürlich auch auf dem Prüfstand, da muss zum Beispiel auch ausgeschlossen werden, dass das Auto von der Hebebühne herunterfällt, oder einen Techniker überfährt.

    Kann mir nicht vorstellen, wie es sinnvoll sein soll, wenn die Grünen und die Juristen die Techniker dazu zwingen wollen, ausgerechnet am Prüfstand, wo das Auto kaum Treibstoff benötigt, besonders zu stinken …

  83. @ anton reutlinger 16.01.2021, 11:15 Uhr

    Übrigens: Fast geräuschlosen – E Autos wird tatsächlich Geist (Software) eingespielt um Geräusche zu machen, weil sich Fußgänger erschrecken könnten, wenn sich ein fast geräuschloses E – Auto von hinten „anschleicht“ ….

  84. @Stephan Schleim 16.01. 00:43

    „Kürzlich schrieb ich selbst, mir vorstellen zu können, dass Bewusstsein naturwissenschaftlich erklärt werden kann – es wird bloß eine andere Naturwissenschaft sein müssen.“

    Meine Idee von den gezielten Zufällen beim Zusammenbruch der Wellenfunktionen wäre dann auch eine andere, bzw. eine nur ein wenig erweiterte Naturwissenschaft. Sollte dieses mal Experimentell bestätigt werden können, wären dann vielleicht auch die Naturalisten aller Art zufrieden?

    Immerhin gehörte es dann zu den Konsequenzen, dass unsere innere Welt eine andere Prominenz bekommt. Ich denke, das die zukünftige Hirnforschung genau diese Bestätigung tatsächlich liefern kann. Derweil ist der Agnostizismus aber wohl die realistischste Haltung.

  85. Mussi,
    so langsam laufen Sie zur Höchstform auf.
    Der Hunger erinnert uns daran, dass wir als Philosophen verhungern würden.
    Und er erinnert uns daran, dass wir Gefangene des Fleisches sind. Große Geister in alterndem Fleisch. Der Rauswurf aus dem Paradies ist eine treffende Beschreibung unserer Situation. “Im Schweiße eures Angesichtes sollt ihr euer Brot verdienen” heißt es in der Bibel .
    Das Ganze findet in unserer Welt statt, die aber nicht nur zeitlich-räumlich zu denken ist, sondern auch ideal-moralisch, also gedacht.

  86. @ Wolfgang Stegemann 16.01.2021, 11:59 Uhr

    Ich würde meinen, den „Aliens“ fällt bei den „Erdlingen“ auf, dass sie besonders in der Nacht, eifrig damit beschäftigt sind ihre “Software” (die Gene) mit besonderem Vergnügen „zu sichern“. So dass die „Software“ in einer „fast unendlichen Kette“ immer wieder neues Leben generiert, auch wenn das „alte Leben“ endet. Natürlich sichern sie sogar ihre „Meme“ einerseits durch „Überlieferung“ oder durch Medien (Bücher, …).

    Die „Aliens“ würden sich wundern, dass viele „Erdlinge“ trotzdem nicht an die Existenz einer fast „unkaputtbaren Software“ glauben…

    Sie würden sich auch wundern, dass einige „Erdlinge“ glauben, grundsätzlich ohne jeglichen „Schöpfungsprozess“, ohne jegliche „Naturgesetze“ angefangen haben zu existieren, obwohl sie doch selber den Begriff „Schöpfung“ ausdrücklich für den Prozess, dass etwas aus dem Nichts entsteht, „deklariert“ haben, z.B. auch bei der Geldschöpfung….

  87. @Martin Holzherr // 16.01.2021, 10:18 Uhr

    » Ein Mathematiker wäre wohl nicht einverstanden…«

    Darauf kommt es nicht an. Sondern darauf, ob es Argumente gibt, die für eine nicht naturalistische Erklärung mathematischer Konstrukte sprechen. Gibt es welche?

  88. @Reutlinger: Naturalismus und Unredlichkeit

    Ob alles naturwissenschaftlich erklärt werden kann, wissen wir genau dann, wenn wir alles naturwissenschaftlich erklären können. Ich halte es schlicht für unredlich, den Naturalismus vorher schon als Wahrheit zu verkaufen, während er in Wirklichkeit spekulative Philosophie ist. Die Vermischung von Philosophie (oder Ideologie) und Naturerkenntnis ist brandgefährlich!

    Ich bezog mich hier speziell aufs Bewusstsein. Der Naturalismus ist in seiner Allgemeinheit wahrscheinlich falsch, weil es in der Welt eben auch soziale und kulturelle Phänomene gibt, für die den Naturwissenschaften schlicht die analytischen Kategorien fehlen.

    Warum müssen wir hier die banalsten Dinge immer wieder wiederholen? Mir wäre diese hirnlose Naturalismus-Diskussion mehr oder weniger egal, würden seine Vertreter nicht mit so einer peinlichen Arroganz auftreten.

  89. @Mussi: Körperliches und Psychisches

    Dass körperliche Vorgänge (seien es Rituale, Substanzkonsum oder Verletzungen) mit bestimmten psychischen Vorgängen einhergehen, wussten schon die Naturvölker lange vor der Moderne; gerade deshalb haben sie ja z.B. Rituale durchgeführt oder Substanzen konsumiert (und versucht, Verletzungen zu vermeiden oder sie ihren Gegnern beizubringen).

    Warum muss man immer wieder so tun, als wäre das eine neue Erkenntnis, die wir der Hirnforschung zu verdanken hätten? (Übrigens war die Säfte-Lehre, die sogenannte Humoralpathologie, die Hippokrates [5.-4. Jh. v. Chr] und Galen [2. Jh. n. Chr.] entwickelten, wahrscheinlich aber bis ins Alte Ägypten zurückreicht, von ihren Annahmen her ähnlich der heutigen Biologischen Psychiatrie, die eben von anderen Körpersäften spricht, nämlich Neurotransmittern.)

    Interessant ist aber doch, dass selbst heute ein Neurochirurg, der am offenen Gehirn elektrisch stimuliert, nicht genau vorhersagen kann, was die elektrische Stimulation bewirken wird. Erst hinterher, wenn man z.B. einen “Lachen-” oder “Weinen-Knopf” im Gehirn gefunden hat, gibt man damit an. Wer nicht weiß, wie solche Aufnahmen entstehen, der missversteht leicht die Aussage.

  90. @Stegemann: Außerirdische

    Was wäre denn Ihre Antwort?

    Ich denke, aus philosophischer Sicht sollte man anerkennen, dass sich über manche Themen schlicht naturwissenschaftlich keine Aussagen treffen lassen. Und damit gut.

  91. @Balanus: Von den Füßen auf den Kopf!

    Mal wieder ein echter Balanus: Mit mathematischen Konstrukten werden eigentlich Naturvorgänge beschrieben; das nennt man dann zum Beispiel “Naturgesetz”.

    Nun sollen – nach balanesischer Logik – mathematische Beschreibungen von Naturvorgängen herhalten, um mathematische Konstruke zu erklären? Ein Ding der Unmöglichkeit!

    Man sollte so ehrlich sein, Mathematik, zumindest die reine Mathematik, als das anzuerkennen, was sie ist: nämlich Geisteswissenschaft, so wie die Logik und Philosophie auch.

  92. @Balanus

    ob es Argumente gibt, die für eine nicht naturalistische Erklärung mathematischer Konstrukte sprechen. Gibt es welche?

    Sowas vielleicht: Die mathematsche Konstruktion des Unendlichen kann nicht einer naturalistisch beschränkten Welt entstammen.

  93. @Balanus (Zitat): “ Sondern darauf, ob es Argumente gibt, die für eine nicht naturalistische Erklärung mathematischer Konstrukte sprechen. Gibt es welche?“
    Es gibt Mathematiker, die denken, mathematische Aussagen würden nicht erfunden, sondern entdeckt. Sie glauben also, mathematische Zusammenhänge könnten schon immer existiert haben und ein Mathematiker entdeckt sie nun wie ein Botaniker eine neue Pflanzenart entdeckt.

    Hier muss man allerdings berücksichtigen, dass ein wichtiger Teil der Mathematik auch dazu in der Lage ist die Physik des Universums zu beschreiben. Daneben gibt es aber auch Mathematikgebiete, die wirklich frei erfunden erscheinen und wo es in der Natur kaum Anwendungen oder Entsprechungen gibt.

  94. Hausaufgaben @alle Möchtegernnaturalisten

    Satz S: Die Summe der Winkel von Dreiecken ist immer 180°.

    Aufgabe: Rein(!) naturwissenschaftliche Erklärung von S.*

    Preis: Eingang in die Annalen von MENSCHEN-BILDER.

    * Also sich in der Erklärung bitte auf Elektronen, Protonen und Neutronen sowie Kernkräfte, Gravitation & Elektromagnetismus beschränken, wie es der Theoretische Physiker Sean Caroll erklärt.

  95. Aliens,

    Es tut mir leid, dass ich Euch Humorlosigkeit unterstellt habe. Intelligenz bringt meistens auch Humor mit sich.
    Der Herr Stegemann hat euch nur benützt um seine Bestürzung über die Dummheit seines eigenen Geschlechtes zum Ausdruck zu bringen.
    Und er hat euch unterstellt, nicht an die Unsterblichkeit des Geistes zu glauben.
    Dabei ist es doch sonnenklar, dass die „Naturgesetze“ schon vorhanden waren, bevor der große Big Bang eintrat

  96. @Holzherr
    Der goldene Schnitt ist so ein Phänomen.

    @Schleim
    bleiben Sie mal locker…
    Es deutet vieles eher darauf hin,dass es ein ‘Zusammenspiel’ von ‘Geist und Materie’ gibt.
    Woher,weshalb und warum?
    Wissen wir nicht und vermutlich werden wir es nie erfahren…
    Aber danach suchen ist ein gute Beschäftigung.

  97. @Stephan Schleim 16.01. 14:40

    „Nun sollen – nach balanesischer Logik – mathematische Beschreibungen von Naturvorgängen herhalten, um mathematische Konstrukte zu erklären? Ein Ding der Unmöglichkeit!“

    Ich denke schon, dass die physikalischen Eigenschaften der Natur vieles in der Mathematik inspiriert haben.

    Andererseits halte ich es für ein Wunder des Kosmos, das hier vieles so schön geordnet daher kommt, dass man es mit menschlich-mathematischen Werkzeugen beschreiben kann.

    Diese so prominente Beschreibbarkeit ist für mich sogar ein weiterer Grund, der Wirklichkeit eine geistige Ordnung zuzugestehen. In diesem Sinne ist Mathematik als Geisteswissenschaft noch mal aufgewertet.

    Kaum jemand wäre vor Newton auf die Idee gekommen, dass so vieles auf Erden so mathematisch ordentlich beschrieben werden kann. Die Welt sieht auf den ersten Blick so unordentlich aus, dass man da kaum eine so konsequente Ordnung erwartet. Der Sternenhimmel ist da anders: Die Planeten sind sich schön regelmäßig sich stets wiederholend gegenseitig am umkreisen, während die Sterne unbewegt als ewiger Hintergrund erscheinen, zumindest, wenn man kein Fernrohr benutzt.

  98. @hwied (Zitat): “ Dabei ist es doch sonnenklar, dass die „Naturgesetze“ schon vorhanden waren, bevor der große Big Bang eintrat“
    Nicht unbedingt, denn ein „vor dem Big Bang“ muss es gar nichts geben. Wenn alles beginnt mit dem Big Bang, dann beginnt mit dem Big Bang auch die Zeit und ein vorher gab es nicht.
    Der Big Bang kann also eine echte Singularität sein. Eine Singularität ist etwas was es nur in der Mathematik nicht aber in der Physik geben kann, denn in der Physik gibt es keine Unendlichkeiten. Ein schwarzes Loch ist in der Beschreibung der allgemeinen Relativitätstheorie eine Singularität, denn im Zentrum des schwarzen Loches gibt es keine Raumzeit mehr. Wenn man mit der allgemeinen Relativitätstheorie rechnet. Die Physiker folgen daraus und aus der Tatsache, dass es schwarze Löcher gibt, dass die Allgemeine Relativitätstheorie noch nicht die letzte Lösung für die Beschreibung unserer Raumzeit ist, dass sie nur eine Annäherung an die Wahrheit sei.
    Doch beim Big Bang wird es schwieriger. Denn welche Theorie soll vor dem Big Bang gelten, wenn es doch vor dem Big Bang gar keine Zeit gab.

    Der Big Bang kann also eine echte Singularität sein, denn vor dem Big Bang gab es keine Physik und ohne Physik, rein in der Mathematik, kann es Singularitäten geben.

  99. @Schleim
    Wie steht es mit Ihrer eigenen Arroganz?

    Satz S: Die Summe der Winkel von Dreiecken ist immer 180°.

    Aufgabe: Rein(!) naturwissenschaftliche Erklärung von S.*

    Legen Sie in Ihrem Garten ein geradliniges Dreieck aus und messen Sie die Winkel. Die Zahl 180 stammt von einem rein historischen Winkelmaß, eingeführt von griechischen Astronomen der Antike, naturwissenschaftlich ohne Bedeutung.

  100. Martin Holzherr,
    die Logik und die Gesetze der Mathematik brauchen keinen Big Bang um gültig zu sein. Sie sind gültig, weil sie logisch sind.

    Der Big Bang war nur eine Materialisation um das Wesen des Geistes zu erklären.
    Die Logik gehört einer anderen Kategorie an, als die Energie und die Materie.

    Hier wird ja versucht, Bewusstsein naturwissenschaftlich zu erklären. Das funktioniert ja auch bei der Hirnforschung. Aber die Erklärung findet ihre Grenze , wenn es um Phantasie , schöpferische Phantasie geht, das was wir als geistig bezeichnen.

  101. @Martin Holzherr // 16.01.2021, 14:49 Uhr

    »Daneben gibt es aber auch Mathematikgebiete, die wirklich frei erfunden erscheinen und wo es in der Natur kaum Anwendungen oder Entsprechungen gibt. «

    Ein Naturalist würde wohl nicht bestreiten wollen, dass der Mensch über die begreifbare Natur mit ihren Gesetzmäßigkeiten hinausdenken kann. So wie etwa beim Begriff des Unendlichen (@Joker). Er hat sich ja auch Gottheiten ausgedacht.

    Ich schätze, das eben sind die sogenannten „geistigen Gebilde“, von denen Sie oben sprachen, Ergebnisse menschlicher Denkprozesse, für die wir keine außer- oder übernatürlichen Erklärungen bemühen müssen—oder auch die Annahme einer gesonderten Welt jenseits (oder diesseits?) der natürlichen.

  102. Freier Wille bedeutet für mich, frei von allem, also losgelöst von allen Einflüssen. Was bedeuten würde, dass wir nicht existieren, da wir und jedes andere Lebewesen abhängig sind von Einflüssen jeglicher Art, wie Luft, Nährstoffe, klimatische Bedingungen und vieles mehr . Wie bitte soll da ein freier Wille existieren? Wo unser Wille von diesen Dingen abhängig ist. Wir entscheiden nach vorgegeben Mustern, was neurologisch gesehen kein Problem mehr darstellt. Sollten dennoch einmal zwei oder mehrere Möglichkeiten bestehen sich zu entscheiden, übernimmt ein Zufallsgenerator die Entscheidung. Wir entscheiden nach kausal determinierten Vorgängen im Gehirn.

  103. @hwied
    die Logik und die Gesetze der Mathematik brauchen keinen Big Bang um gültig zu sein. Sie sind gültig, weil sie logisch sind.

    Nein, das stimmt so nicht. Die Mathematik hat ihre Begründungen in der Natur unseres Universums. Die Logik kann die Natur transzendieren, aber sie sollte ihr nicht widersprechen. Es wäre ein anderes Universum denkbar, in dem andere Naturgesetze und damit eine andere Mathematik gelten könnten.

    Müssen sich auch die Götter an unsere Logik halten – oder ist sie schon von ihnen vorgegeben? Wenn Letzteres gelten würde, dann müssten die Menschen die Logik nicht mühsam erlernen!

  104. @Balanus, hwied (Zitat): „ Aber die Erklärung [für das Bewusstsein] findet ihre Grenze , wenn es um Phantasie , schöpferische Phantasie geht, das was wir als geistig bezeichnen.“
    und “ Ein Naturalist würde wohl nicht bestreiten wollen, dass der Mensch über die begreifbare Natur mit ihren Gesetzmäßigkeiten hinausdenken kann. So wie etwa beim Begriff des Unendlichen (@Joker). Er hat sich ja auch Gottheiten ausgedacht.“

    Ich denke, wenn man das Unmögliche und Unendliche nicht denken kann, kann man überhaupt nicht oder nur sehr eingeschränkt denken.
    Die Frage ist nun: 1) Kann erst der Mensch Fiktionen konstruieren und 2) Kann man innerhalb einer rein materiell/physikalisch basierten Welt keine Fiktionen konstruieren?

    Mir scheint tatsächlich, dass die materielle Welt auch Fiktionen zulässt. Diese Fiktionen passieren in Informationen verarbeitenden Systemen fast zwingend und lassen sich kaum vermeiden. Auch Hunde leiden beispielsweise unter optischen Illusionen und haben wohl verrückte Träume von Dingen die es nicht gibt.
    Höhere Tiere können ja Informationen verarbeiten und damit (oft unfreiwillig) Fiktionen erzeugen . Fiktionen kann man verschieden sehen. Die einen sehen darin Irrtümer („Er hat sich auch Gottheiten ausgedacht“), andere verdienen ihr Geld damit und wieder andere glauben gar, ohne Fiktionen, ohne Phantasie wäre alles nichts und Fiktionen/Phantasie könne es in einer rein materiell basierten Welt gar nicht geben.

    Nun, vielleicht sind Fiktionen häufig Irrtümer. Und gewisse Menschen sind dann auf die Idee gekommen, diese Irrtümer seien überhaupt erst das, was das Leben lebenswert macht. Es gibt ja sogar Sprüche wie: Der Irrtum eines Genies kann der Grund für eine geniale Entdeckung sein. Vielleicht leben wir ja alle um uns zu irren.

  105. @Jeckenburger: Ordnung & Chaos

    Schöner Gedanke. Doch man muss aufpassen, hier keine Tautologie zu begehen:

    Wir nehmen und behalten eben die mathematischen Verfahren, die uns ein bestimmtes Muster bestätigen. Dann wundern wir uns: “Oh, ein Muster! Die Welt ist erklärbar!” Die vielen anderen Vorgänge, die in kein von uns bekanntes Muster passen, blenden wir dann aus.

    Mal ein anschauliches Beispiel: Ein Biologieprofessor, Molekularbiologe, mit dem ich vor vielen Jahren zusammenarbeitete, erzählte einmal: Da mache man vielleicht dreißig, vierzig Aufnahmen mit dem Mikroskop – und die beiden “Schönen” (d.h. die die Hypothese stützen) wählt man aus und publiziert man. Ich will hiermit keine unlauteren Methoden unterstellen; aber interessant ist doch, dass die 38 anderen Abbildungen ebenso die Natur wiedergeben, vom Menschen aber als “uninteressant” abgetan werden, weil sie nicht zur Methode/Erwartung passen.

    Oder: In der bildgebenden Hirnforschung haben wir statistische Modelle zur Datenanalyse aufgestellt. Prinzipiell kann man wohl (annähernd) unendlich viele Modelle auf seine Daten loslassen. Die Teilmenge der Modelle mit “interessanten” Ergebnissen dürfte i.d.R. >1 sein. Wie wählt man dann aus? Und das, was nicht in (dieses) Modell passt, das landet im “Fehlerterm”. Auch hier ist es so: Den Teil der Naturvorgänge, der zu den Erwartungen des Menschen passt, wird hervorgehoben, und der Rest wird als uninteressant abgetan.

    Zudem haben die Menschen schon vor der modernen Wissenschaft Regelmäßigkeiten wahrgenommen und beschrieben: Denken Sie an Tag und Nacht, die Jahreszeiten, die Bewegung der Sterne, der Zyklus allen Lebens und noch viel mehr. In der Wissenschaftsgeschichte haben manche Naturwissenschaftler – würde mich nicht wundern, wenn Newton oder Darwin dazu zählen – diese Ordnung gerade als Beweis für eine höhere Macht, einen Gott angesehen.

  106. Martin Holzherr,
    Zustimmung

    Denkansatz 3
    was ist Bewusstsein oder über das Denken nachdenken.

    Die einfachste Methode zu denken ist die direkte Beobachtung. Man lässt sich vom Geschehen direkt inspirieren. Diese Form der Naturbeobachtung ist unentbehrlich aber sie hat ihre Grenzen.
    Worauf soll man achten, Welche Zusammenhänge sind wichtig.

    Unser Denkapparat hat da eine bewährte Methode, wir denken in Begriffen, d.h. Wir greifen auf schon Gedachtes zurück und müssen nicht immer wieder von vorn anfangen.
    Wir kombinieren also Beobachtetes mit Begriffen, Materielles mit schon Gedachtem.

    Schwierig wird es, wenn wir über Gedanken nachdenken, also reflektieren.
    Das kann schnell in geistige Inzucht ausarten, wir drehen uns im Kreis. Wir denken das, was wir denken wollen oder sogar sollen.
    Da ist es erfrischend, mal andere Gedanken zu lesen. Bücher sind der beste Freund des Menschen, so sagt es der Volksmund.

    Jetzt kommt so ein Begriff wie Bewusstsein und wir sollen darüber nachdenken, was er bedeuten könnte, suchen sprachliche Parallelen, oder wir definieren einfach den Begriff nach unserem Willen.

    Hier in diesem blog ist die Mehrheit der Meinung Bewusstsein ist naturwissenschaftlich erklärbar, oder es ist ein Scheinbegriff, dem keine Realität entspricht.

    Meine Meinung ist, der naturwissenschaftliche Anteil an der Erklärung ist groß, es bleibt ein unerklärbarer Rest, der aber eine ganz neue Dimension erschließt, die Religion.

    Reutlinger,
    “Die Mathematik hat ihre Begründungen in der Natur unseres Universums”
    Die Logik ist unabhängig vom Universum. Sie ist deshalb unabhängig, weil die Logik selbst eine Abstraktion ist, eine Schöpfung des Geistes.
    Abstraktionen sind immer richtig, sonst sind sie keine Abstraktionen.
    U = d mal Pi, das gilt immer, in jedem Universum, weil die Gravitation die Abstraktion nicht verbiegen kann. Geist ist stärker als jede Naturgewalt.

  107. @Mussi, Reutlinger: Arroganz

    Manche Menschen finden es schon Arrogant, wenn man es wagt, konsistent einen eigenen Standpunkt zu vertreten. Tja, da kann ich dann auch nichts machen. Und als Blogbetreiber und Hochschullehrer nehme ich mir manchmal heraus, offenkundigen Unsinn als solchen zu bezeichnen.

  108. @Tobias Jeckenburger // 16.01.2021, 01:19 Uhr

    » So wirkt der Mensch auf sein Abbild. Und wenn die Person sich das Bild ansieht, und für gut hält, wird er es demnächst als Passbild in seinen Ausweisen mit sich tragen, und gibt dem Foto damit Gelegenheit, auf alle Betrachter einzuwirken, z.B. auf die Zöllner, um diese nach Möglichkeit dazu motivieren, einen durchzuwinken. «

    Ich hatte bereits zugestanden, dass ein materielles Abbild selbst wieder zur Vorlage für ein Abbild werden kann. Das hat aber nichts mit der behaupteten *Wechsel*wirkung zwischen Motiv und Bild (als Analogie zu Körper und bewusst erlebtes Abbild) zu tun. Das Bild wirkt nicht auf das Motiv zurück. Das Motiv bedingt die Abbildung, aber die Abbildung bedingt nichts an/bei dem Motiv.

    » So oder so, was ich nicht brauche ist:
    3. Der Bereich physikalischer Phänomene ist kausal geschlossen.
    «

    Na, dann ist ja besonders mutig, wenn Sie aufs Rad steigen oder mit dem Zug fahren. Zu den Unwägbarkeiten, die es selbstredend auch in einer kausal (ab)geschlossenen Welt gibt, kämen dann ja noch akausale Ereignisse hinzu, die schon aus rein prinzipiellen Gründen überhaupt nicht einkalkuliert werden könnten.

  109. @Timm Grams

    Die Aussage des Naturalisten [dass identische Ursachen notwendigerweise identische Folgen haben, mithin es gäbe keine Willensfreiheit (*)] ist metaphysisch, unwissenschaftlich, Glaubenssache.

    Da haben Sie meine Zustimmung. Nur sollten Sie zugestehen, dass die Aussage von wem auch immer, dass identische Ursachen nicht notwendigerweise identische Folgen haben, mithin es gäbe Willensfreiheit (*), ebenfalls metaphysisch, unwissenschaftlich und Glaubenssache ist.

    Wenn Sie meinen, ‘es mag wohl so sein, dass identische Ursachen notwendigerweise identische Folgen haben’, dann müssten Sie für sich klären, warum das im Fall des menschlichen Willens, und vielleicht sogar nur speziell dort, nicht gültig sein soll. Im Denken des Naturalisten ist an dieser Stelle kein Bruch.

    Wie @Helmut Wicht als Antwort auf @Tobias Jeckenburger korrekt angemerkt hat, ist es auch nicht hinreichend 3) abzulehnen und/oder Zufall in Form von Quantenfluktuationen zu berücksichtigen, um der Willensfreiheit auf die Sprünge zu helfen.

    Jedoch lässt sich keine Versuchsanordnung denken, mit der die Entscheidung einer Person durch Herstellen identischen Bedingungen überprüft werden könnte. Es müsste schon dieselbe Person sein. Aber nach dem ersten Durchlauf ist deren mentale Ausstattung bereits verändert, so dass sie für den folgenden Prüflauf nicht infrage kommen kann.

    Streng genommen lässt sich überhaupt keine Versuchsanordnung denken, in der dasselbe unveränderte Objekt unter identischen Bedingungen überprüft werden kann. “Man kann nicht zweimal in den selben Fluss steigen”. Das liegt genauso am veränderten ‘man’, wie auch am veränderten ‘Fluss’. So ein Blitzdings, ein Neuralisator, bekannt aus Man in Black, wäre für Versuche mit Menschen sehr hilfreich.

    (*)
    Es gibt Unternehmen, den Begriff Willensfreiheit so auszulegen, dass man deren Existenz auch als Naturalist bejahen kann. Diese Auslegung verschleiert meiner Meinung nach aber nur das, was im Allgemeinen und Ursprünglichen mit der Behauptung von Willensfreiheit gemeint ist. So ist der Begriff hier also nicht gemeint.

  110. @Schleim
    Da kommen wir ja zum Kern: die eigene Position durchzubringen.
    Ihre letzte Stellungnahme ist das schlechteste aller Argumente.

  111. Timm Grams schreibt:

    Wenn es nur eine Welt gibt, die es zu erkennen gilt, wo finden die Gedanken über diese Welt dann ihren Platz? In der Welt kann ihr Platz nicht sein, denn die Welt soll ja unabhängig vom Bewusstsein und den darin aufgehobenen Gedanken existieren.

    Dieser Gedankengang ist mir zu hoch.

    Wieso kann das Erkenntnisorgan, in dem die Denkprozesse ablaufen, nicht Teil der erkennbaren Welt sein? Welche Logik steckt dahinter?

    Mir will auch nicht einleuchten, dass die (naturalistische) Annahme einer vom Denken unabhängigen Welt, oder besser: Umwelt, eine besondere metaphysische Annahme und somit eine „Einschränkung des Denkens“ wäre.

    Bevor in der Menschheitsgeschichte das große Nachdenken begann, waren alle Individuen im Grunde Naturalisten, etwas anderes als die natürliche Umwelt gab es für sie nicht. Erst mit dem Erwachen des Bewusstseins begannen die metaphysischen Spekulationen, Regen und Dürre waren nun nicht mehr vom Denken unabhängige Naturphänomene, auf die man keinen Einfluss hatte, sondern man hoffte, sie durch allerlei Aktionen günstig beeinflussen zu können.

    Kurzum, wenn einer metaphysisch auf Sparflamme kocht, dann ist es, so scheint mir, der Naturalist (auch wenn es *den* Naturalisten so nicht gibt).

    Und bevor ich’s vergesse: Danke für den anregenden Beitrag, Herr Grams!

  112. Ein interessanter Artikel(austausch), hier im Forum. Wenn ich nur die Bereitschaft oder die Kapazität aufbringen könnte!

  113. Balanus,
    Die Formulierung von Grams lässt sich leicht auflösen, wenn man die Subjekt-Objekt- Spaltung anwendet.

    Alles was es zu sehen gibt, das ist das Objekt. Der , der das Objekt beobachtet , ist das Subjekt. Wichtig: Das Subjekt betrachtet das Objekt, nicht sich selbst, das ist die Vereinbarung.

    Bei der Reflexion macht das Subjekt sich selbst zum Objekt.
    Damit wird Geistiges zum Objekt. So begründet sich auch der Idealismus.
    Das Gedachte ist damit als Objekt zugänglich.

  114. @hwied
    Abstraktionen sind immer richtig, sonst sind sie keine Abstraktionen.

    Da lehnen Sie sich aber weit aus dem Fenster. Was genau verstehen Sie unter Abstraktionen? Wenn ich weit genug abstrahiere, bis die Abstraktion immer richtig ist, dann habe nicht mehr die Taube in der Hand, sondern den Spatz auf dem Dach.

    Wenn ich von den spezifischen Eigenschaften von Autos abstrahiere und sage “Autos sind Blechkisten”, dann ist es eine Abstraktion, aber eine falsche, denn Autos sind auch aus Kunststoff. Abstrahiere ich weiter zu “Autos sind Kisten”, dann ist es immer richtig, aber sinnlos.

  115. @hwied // 16.01.2021, 18:00 Uhr

    » Das Gedachte ist damit als Objekt zugänglich. «

    Und deshalb haben die Gedanken über die Welt keinen Platz in der (einen) Welt des Naturalisten, oder wie?

  116. Nur ein paar einfache Fakten und Überlegungen zur Diskussion.
    Der Kosmos existiert seit mehr als 13 Milliarden Jahren, das Sonnensystem seit etwa 4 ½ Milliarden Jahren, der Mensch als Homo Sapiens seit (großzügig) 100000 Jahren.
    Damit ist doch klar, dass die Welt außerhalb unseres Denkens existiert und zwar schon die allermeiste Zeit. Ohne Menschen existiert auch alles außer uns weiter.

    Das bedeutet jedoch nicht, dass die Denkprozesse nicht von unserer Welt sind.
    Die Bausteine des Nervensystems, die Nervenzellen, sind bereits in ganz primitiven Lebewesen zu finden und da funktionieren sie so wie beim Menschen, und zwar durch chemische Prozesse.
    Die Entwicklung des Nervensystems und des Gehirns ist recht gut bekannt, und es kommen keinerlei neue Mechanismen hinzu. Alle Prozesse im Gehirn sind also materiell bedingt. Die höheren geistigen Fähigkeiten des Menschen im Vergleich zu Quallen, Würmern etc. sind also nur durch den enormen Zuwachs an Neuronen und durch die Struktur, also die Kopplung der Neuronen im Gehirn zu erklären. Was also „geistig“ genannt wird, hat eine biochemische Grundlage. Unser Denken ist also gebunden an die Materie.

    Bewusstsein im Sinne von dem Gefühl, dass man ein Individuum ist, muss nicht erlernt werden wie Sprache, Schreiben, Lesen und vieles andere. Es ist also ein Teil der grundlegenden Struktur unseres Gehirns und daher offenbar ein deutlicher evolutionärer Vorteil. Vermutlich haben sehr viele Tiere auch diese Art von Bewusstsein, besonders Säugetiere.

    Wenn man dies alles betrachtet, erscheint die Vermutung sehr wahrscheinlich, dass die Trennung in unbeseelte Welt einerseits und beseelte und denkende Welt andererseits den gleichen Weg nehmen wird wie die Trennung in unbelebte und lebende Materie: es gibt keine Trennung.

  117. @Balanus / 16.01.2021, 17:43 Uhr

    »Dieser Gedankengang ist mir zu hoch.«

    Eine als Gesamtheit aller Gegenstände möglichen Erfahrungswissens vorgestellte “Welt” ist selbst kein Gegenstand möglichen Erfahrungswissens. Der Begriff “Welt” bezeichnet nur eine Idee, eine gedankliche Vorstellung, aber ganz gewiss keinen vom Denken unabhängig bestehendes Ding oder Phänomen.

    Und wenn ein Naturalist an diesem nur von ihm selbst gedanklich geschaffenen Ding namens “Welt” dann eine Eigenschaft wie “kausal geschlossen” erkennen will, dann möchte man ihn doch fragen, wie und woran woran er das eigentlich feststellen will. Mit naturwissenschaftl. Argumenten geht das jedenfalls nicht. Also was meinst Du, wie soll das gehen?

  118. @Balanus
    “Dieser Gedankengang ist mir zu hoch.” So kommentieren Sie meine Auffassung, dass die Gedanken über die Welt im Weltbild des Naturalisten keinen Platz finden. Etwas präziser bin ich in einer Antwort an Joker geworden: Ich sehe den Widerspruch darin, dass der Naturalist von der einen Welt spricht, die unabhängig vom Bewusstsein ist, und dass er andererseits auch die monistisch-materialistische Identitätstheorie befürwortet. Die Gedanken werden also zum Bestandteil der Welt, so dass diese nicht unabhängig von den Gedanken sein kann.

    @Joker
    “Nur sollten Sie zugestehen, dass die Aussage von wem auch immer, dass identische Ursachen nicht notwendigerweise identische Folgen haben, mithin es gäbe Willensfreiheit (*), ebenfalls metaphysisch, unwissenschaftlich und Glaubenssache ist.”
    Warum sollte ich? Ich tretet hier als Skeptiker auf und das Erbauen kühner Gedankengebäude fällt nicht in seine Kompetenz. Ihm reicht es zu sagen, dass es keine wissenschaftlichen Gründe gibt, die Willensfreiheit zu negieren. Ein Plädoyer für die Willensfreiheit ist nicht beabsichtigt, weder wissenschaftlich noch metaphysisch. Als Skeptiker ziehe ich mich auf einen streng agnostischen Standpunkt zurück. (In einer anderen Rolle trete ich dann doch schon einmal leidenschaftlich für die Freiheit in den Ring.)

  119. @Stephan Schleim

    Sie schreiben:

    Als Bindemittel sehen sie das oszillatorische Feuern der Nervenzellen.

    Dieser Satz ist unsinnig – unter Oszillation versteht man ein Schwingen der Zellen in gleicher Phase. Jede Zelle hat eine Eigenschwingung, aber ab dem Moment, in dem Neurone sich darauf geeinigt haben, an der Repräsentation des selben Gegenstands beteiligt zu sein, zeigen sie dies an, indem sie gemeinsam schwingen, bzw. oszillieren. Das Feuern der Zellen hat damit nichts zu tun. Die Oszillation kann also kein Bindemittel sein, sie zeigt nur die vorübergehende Zusammengehörigkeit an.

    Das “Bindemittel” – wenn man es denn so überhaupt nennen kann – ist eine Regel (ich habe es gestern in Ihrem vorherigen Beitrag erklärt, worin Sie mir nicht folgen wollten).

    Also:

    Synchronisiertes Feuern hebt die Zusammengehörigen von den übrigen ab – so lautet die Hypothese. Die Teilrepräsentationen können sich so zu einem Gesamtbild fügen. Das aber hebt die Repräsentation nicht über die physikalische und physiologische Ebene hinaus.

    Letzteres ist richtig. Deshalb sind aber nicht nur Neurone in diese Vorgänge involviert, sondern auch Gliazellen. Eine Reihe von Wissenschaftlern, darunter Alfredo Pereira* von der Universidade do Minho, beschäftigt sich mit ihnen, um das Bewusstsein zu erklären ( er nannte meinen Vorschlag mit der Regel “a great idea” ;-), aber er ist ja auch Psychologe). Gliazellen sind Teil der tripatiten Synapsen, die sie quasi umschließen. Sie haben ihr eigenes Netzwerk auf Calcium-Basis.

    Spannend finde ich dazu die Forschungen von Lissek et al**, die in Tierversuchen festgestellt haben, dass unter der Gabe von Anästhetika das Gehirn keineswegs weniger aktiv ist, sondern es auf der unteren Ebene (Wahrnehmung eines Objekts)sogar vermehrt zu Oszillationen kommt – die allerdings unsinnig sind – mit dem Ergebnis: der Patient verliert das Bewusstsein.

    *Pereira, A. (2007) Astrocyte-Trapped Calcium ions: the Hypothesis of a Quantum-Like Conscious Protectorate . Quantum Biosystems 2 p.80-92
    http://www.quantumbiosystems.org/admin/files/QBS2%2080-92.pdf

    **Lissek, Thomas, Obenhaus Horst A., Ditzel, Dèsirèe A. W., Nagai, Takeharu, Miyawaki, Atsushi, Sprengel, Rolf, Masahir T. Hasan: General Anesthetic Conditions Induce Network Synchrony and Disrupt Sensory Processing in the Cortex. Front. Cell. Neurosci., 14 April 2016 | https://doi.org/10.3389/fncel.2016.00064 in:
    https://www.frontiersin.org/articles/10.3389/fncel.2016.00064/full

  120. @Physiker: Nervensysteme

    Sie scheinen mir hier vorauszusetzen, was man erst zeigen müsste: dass sich Bewusstsein auf neuronale Vorgänge reduzieren lässt.

    Wenn man dies alles betrachtet, erscheint die Vermutung sehr wahrscheinlich, dass die Trennung in unbeseelte Welt einerseits und beseelte und denkende Welt andererseits den gleichen Weg nehmen wird wie…

    Hier scheinen Sie sich zu widersprechen: Denn erst schrieben Sie ja von der langen Zeit, in der es keine Menschen gab. Und davor eine Zeit, in der es gar keine Lebewesen gab; noch nicht einmal unser Sonnensystem. Oder sind Sie Panpsychist?

    …die Trennung in unbelebte und lebende Materie: es gibt keine Trennung.

    Das leuchtet mir auch nicht ein: Man unterscheidet doch sehr deutlich zwischen beispielsweise anorganischer und organischer Chemie. Oder einer lebenden und einer toten Zelle: In einer toten Zelle gibt es etwa weniger (insb. zielgerichtete) Bewegung.

  121. Anton Reutlinger,
    scharf erkannt, die x-te Abstraktion ist tatsächlich der Spatz auf dem Dach.
    “Autos sind Blechkisten”, das ist keine Abstraktion, das ist eine Meinung.
    Das Wesentliche des Auto ist nicht das Blech, sondern , wie der Name “Automobil” schon sagt, das man mobil wird = beweglich, und das noch ohne Pferd, weil sich das Auto, auto = selbst (griechisch) bewegt. Für Kinder ist das Wesentliche die 4 Räder. Vier Räder und ein Sitz, und fertig ist das Auto.
    Slogan von 1926 : Ein bischen Blech , ein bischen Lack und fertig ist der Hanomag. Dabei war der Hanomag Kommisbrot aus Holz gebaut.
    Aber um diese Spitzfindigkeiten geht es mir nicht.
    Es geht darum, dass eine Abstraktion eines realen Gegenstandes kein Gegenstand mehr ist. Und diese Idee ist nirgends zu finden. Aber sie ist trotzdem vorhanden.
    So eine Idee ist auch die Kugel, die gibt es nicht wirklich, die ist eine geometrische Idee (nach Plato), die man aber überall finden kann, bei den Atomen, bei den Planeten, selbst das Universum kann man sich als Kugel vorstellen.

  122. Balanus,
    Gedanken haben in der Welt keinen Platz. Was sich Herr Grams dabei gedacht hat, das weiß ich auch nicht. Aber da liegt er falsch, Zustimmung.

  123. Physiker
    ” Unser Denken ist also gebunden an die Materie”, soweit es die Darstellung der Gedanken betrifft, aber nicht zwangsläufig ihr Inhalt.
    Die Komposition einer Sonate , die ist von ihrer Wirkung her betrachtet nicht von dieser Welt. Ein Liebeslied, das Menschen zu Tränen rührt, dieses Lied ist kein Ergebnis der Evolution, dieses Lied ist eine Schöpfung. Und daher kommt der Ausspruch: Und Gott schuf den Menschen nach seinem Bilde. Das heißt wir sind in der Tat den Göttern gleich, wir erschaffen etwas, was es vorher im ganzen Universum noch nicht gegeben hat. Das trifft übrigens auch auf andere Kunstgattungen zu.

  124. @hwied
    So etwas kommt heraus, wenn der Kontext weg ist: “Gedanken haben in der Welt keinen Platz. Was sich Herr Grams dabei gedacht hat,…” Es geht um die eine Welt des Naturalisten, die einmal unabhängig von den Gedanken sein und sie andererseits beherbergen soll.

  125. @Stephan Schleim
    Danke für den Kommentar. Das Einfachste zuerst.
    Sie schreiben:
    „Man unterscheidet doch sehr deutlich zwischen beispielsweise anorganischer und organischer Chemie. „
    Die Benennung ist rein historisch und bezeichnet zur Zeit nur die Chemie der Kohlenstoffverbindung als organische Chemie und die Chemie der übrigen Elemente als anorganische Chemie. Dabei handelt die organische Chemie überwiegend von Substanzen, die NICHT in Organismen vorkommen, die also nicht „organisch“ sind. Die Trennung ist nur sinnvoll, weil Kohlenstoffatome sich sehr variabel zu komplexen Molekülen zusammensetzen lassen, eine Flexibilität, die den übrigen Elementen (auch übrigens Silizium) fehlt. So gehören Polymere zum Beispiel zur organischen Chemie, kommen aber in Organismen nicht vor (zumindest nicht die industriell in großem Maßstab hergestellten Polymere).
    Nun zu den schwierigeren Themen.
    „Sie scheinen mir hier vorauszusetzen, was man erst zeigen müsste: dass sich Bewusstsein auf neuronale Vorgänge reduzieren lässt.“
    Ich denke, hier liegt die Beweislast nicht bei mir. Bisher sind ausschließlich neuronale Prozesse in Nervensystemen beobachtet worden, also Erregungszustände, die durch biochemische Prozesse hervorgerufen werden. Wie ich geschrieben habe, gilt das für die einfachsten Nervensysteme wie für die kompliziertesten: die Bauteile sind für alle Lebewesen mit Nervensystemen gleich. Da Bewusstsein, wie ich es beschrieben habe, sicher nicht auf Menschen beschränkt ist, wäre es eigenartig, wenn in den jeweiligen Nervensystemen etwas fundamental Anderes übersehen worden wäre. Also liegt die Beweislast bei denjenigen, die Mechanismen jenseits der bekannten neuronalen Prozesse postulieren. Penrose hat zum Beispiel dafür Vorschläge gemacht, die – soweit ich weiß – aber widerlegt worden sind. Solange es also keine (überprüfbaren) Vorschläge zu solchen Mechanismen gibt, ist es nicht sinnvoll sie in Betracht zu ziehen. Es sei denn, dass man aus anderen Überlegungen solche unbekannten Mechanismen als zwingend notwendig erachten müsste.
    Den letzten Punkt ihrer Argumentation kann ich nicht nachvollziehen. Das einfachste Leben auf der Erde ist erst einige Zeit nach der Entstehung des Sonnensystems nachweisbar, und höhere Organismen sind erst seit ca. 600 Millionen Jahren vorhanden. Nervensysteme haben Lebewesen also vermutlich auch erst seit dieser Zeit. Viele andere Himmelskörper haben nie Leben gehabt und werden es auch nie haben: die Sonnen zum Beispiel. Dennoch basieren alle auf den gleichen atomaren Bausteinen wie wir, nur in anderen Aggregatzuständen und in anderen chemischen Formen. In planetaren Nebeln finden allerdings viele chemische Reaktionen statt, in denen die Bausteine von Organismen erzeugt werden, aber eben keine kompletten Organismen. Die Trennung in unbelebte Natur und belebte Natur ist daher meiner Meinung nach künstlich, der Übergang ist fließend.

  126. @hwied:
    Da die Wirkung einer Sonate, eines Gemäldes, eines Gedichtes sich im Gehirn entfaltet, ist die Wirkung sehr wohl an die Materie gebunden. Unser Hirn ist eben ein Teil des Körpers, und wenn wir bestimmte Erregungszustände im Gehirn haben, werden Hormone ausgeschüttet: Streßhormone, Glückshormone… Alles eben Materie, chemische Substanzen, die an anderen Stellen im Körper Reaktionen hervorrufen: schneller Herzschlag, Erröten, Schweißausbrüche…
    Die Erzeugung von Kunstwerken ist auch nicht zwingend an Menschen gebunden, es gibt inzwischen eine Reihe von Kunstwerken, die von neuronalen Netzen produziert wurden. Diese sind auch durch Materie realisiert, in diesem Falle durch Schaltkreise in Computern.
    Das Ergebnis einer solchen Schöpfung ist ebenfalls an Materie gebunden, entweder in Form von Luftschwingungen oder Licht verschiedener Wellenlängen.
    Oder die Kunstwerke werden materiell fixiert, als Notenblätter, als CDs, … Die Möglichkeiten sind unendlich, aber immer durch Materie realisiert.

  127. @Physiker: Beweislast

    Nun ja, es gibt erstens sehr wenige gelungene Reduktionen in der Wissenschaftsgeschichte; und zweitens ist das, was man sieht, eine statistische Korrelation zwischen physiologischen und psychologischen Vorgängen.

    Eine Aussage der Form “psychologische Prozesse sind nichts als physiologische Prozesse” halte ich schon aus den Gründen für sinnlos, da einerseits gar nicht klar ist, wie man psychologische Prozesse (begrifflich) am besten fassen soll, und andererseits auch nicht feststeht, was die grundlegende Ebene für die Reduktion sein soll.

    Sie schrieben, A und B ließen sich nicht trennen. Darauf erwiderte ich, dass sie sich sehr wohl trennen lassen – und das die ganze Zeit gemacht wird. Wie nützlich diese Trennung ist, worauf Sie jetzt vielleicht abzielen, ist eine andere Frage. Lebende und tote Zellen lassen sich meines Wissens durchaus klar unterscheiden, auch wenn es ein paar Grenzfälle gibt (was ist z.B. mit Makroviren – leben die oder nicht?). Summa summarum ist auch dies eine Frage, die sich nicht und vielleicht auch nie auf grundlegende physikalische Vorgänge wird reduzieren lassen.

    Vielleicht ist der Hinweis hilfreich, dass es in meiner Argumentation primär um die epistemische Ebene geht, das heißt, was können wir von der Welt wissen? Wie die Welt “wirklich” ist (ontologische Ebene), das können wir meiner Meinung nach nie wissen.

    P.S. Darf ich zur Verdeutlichung meines Standpunkts auf Das kleine Einmaleins des Leib-Seele-Problems hinweisen?

  128. @Timm Grams

    Ich tretet hier als Skeptiker auf und das Erbauen kühner Gedankengebäude fällt nicht in seine Kompetenz. Ihm reicht es zu sagen, dass es keine wissenschaftlichen Gründe gibt, die Willensfreiheit zu negieren.

    Verstehe. Dafür hätte es allerdings doch schon gereicht, Michael Schmidt-Salomon vernünftig zu interpretieren. Er behauptet weder, es sei eine wissenschaftliche Aussage, dass identische Ursachen notwendigerweise identische Folgen haben, noch, dass Willensfreiheit wissenschaftlich widerlegt wurde. Er macht ja keine Experimente, sondern das, was Philosophen halt so machen, Begriffsklärung und Argumentieren. Eines seiner Argumente beinhaltet den Hinweis auf eine metaphysisch notwendige Annahme (die von Ihnen im Text zitiert wurde), der die Verfechter der Willensfreiheit verpflichtet sind.

    Von daher berichten Sie hier nichts Neues, nichts Strittiges und führen erst recht keinen Widerspruch im Denken dieses Naturalisten vor.

    Bleiben Sie skeptisch.

  129. @Stephan Schleim
    Danke, ich werde mir ihre Veröffentlichung sorgfältig durchlesen, aber nicht mehr heute Abend.
    Nur ein paar Anmerkungen: das Problem Geist – Körper darf man nicht nur auf Menschen beziehen, sondern man muss Tiere ebenfalls mit betrachten. Die Struktur des Nervensystems ist ähnlich, die Bauteile identisch.
    Wenn man eine Trennung einführt, muss das sinnvoll sein. Wenn es nicht nützlich ist, sollte man es bleiben lassen. Siehe Occams razor.
    Ich habe vermutlich eine Abneigung gegen solche Zweiteilungen. In der Physik hat es sich als unsinnig erwiesen, die Trennung von Teilchen und Welle aufrechtzuerhalten. Im Mikrokosmos gibt es diese Trennung nicht.
    Meiner Meinung nach ist auch die Trennung in Naturwissenschaft und Geisteswissenschaft nicht immer sinnvoll. Ist Mathematik das eine oder das andere oder etwas von beidem? Da die Natur sich nur durch Mathematik beschreiben lässt, kann Mathematik ja nicht nur Geisteswissenschaft sein…

  130. Der Unterschied zwischen belebter und unbelebter Materie besteht in den verschiedenen Arten von Selbstorganisation. Leben ist also sozusagen Selbstorganisation 2.0, wobei dort graduelle Unterschiede existieren. Einzeller organisieren sich anders, man könnte sagen proteinbildend, als Mehrzeller oder einfache Organismen und solche mit einem zentralen Nervensystem, die bioelektrisch arbeiten, also Selbstorganisation 2.1 bis 2.n.
    Noch etwas: Ich kann den Satz nicht mehr hören, die Naturwissenschaft könne den Menschen nicht erklären. Das will sie auch gar nicht. Jeder Wissenschaftszweig betrachtet den Menschen aus einer anderen Perspektive. Natürlich gehört es nicht zur Aufgabe der Naturwissenschaft, kulturelle Aspekte des menschlichen Zusammenlebens zu erklären.

  131. @Timm Grams // 16.01.2021, 18:56 Uhr

    » Ich sehe den Widerspruch darin, dass der Naturalist von der einen Welt spricht, die unabhängig vom Bewusstsein ist, und dass er andererseits auch die monistisch-materialistische Identitätstheorie befürwortet. Die Gedanken werden also zum Bestandteil der Welt, so dass diese nicht unabhängig von den Gedanken sein kann. «

    Ja, diese an @Joker gerichtete Erläuterung hatte ich gesehen. Das hat mir aber nicht weitergeholfen , weil ich bezweifle, dass Naturalisten generell der monistisch-materialistischen Identitätstheorie anhängen. Das ist das Eine.

    Das Andere ist, dass nach meinem Verständnis Gedanken (bewusste Denkprozesse) selbstredend Bestandteil der einen Welt sind, und dass die Beschaffenheit der Welt oder die Eigenschaften der Dinge unabhängig von diesen Gedanken sind (von den Gedanken, wohlgemerkt, nicht von Handlungen). Und da Naturalisten gemeinhin vernünftige Menschen sind, werden sie das ganz ähnlich sehen wie ich.

    Kurzum, ich versteh‘s leider immer noch nicht.

    Passend dazu die Einlassung von

    @Chrys // 16.01.2021, 18:45 Uhr

    » Der Begriff “Welt” bezeichnet nur eine Idee, eine gedankliche Vorstellung, aber ganz gewiss keinen vom Denken unabhängig bestehendes Ding oder Phänomen. «

    Dass die „Welt“ als Idee nicht unabhängig vom denkenden Subjekt ist, erscheint mir einleuchtend. Aber war von „Welt“ als Idee überhaupt die Rede?

    Mein Eindruck war vielmehr, und ist es eigentlich noch, dass Grams die wirkliche, reale, physikalische Welt meint (Umwelt, Umgebung, Lebensraum, Natur, Erde, Kosmos, solche Dinge halt), kurz, die den Sinnen zugängliche und darum erforschbare „Welt“. Schließlich sind naturwissenschaftliche Befunde für die meisten Naturalisten von zentraler Bedeutung.

  132. @Balanus
    “Ich bezweifle, dass Naturalisten generell der monistisch-materialistischen Identitätstheorie anhängen”, schreiben Sie.

    Manche, die sich für Naturalisten halten, werden nicht einmal wissen, was das ist. Ohne es immer wieder zu betonen, spreche ich von der in der Skeptikerbewegung wirksamen Philosophie (s. Quellenangaben).

    @Joker
    Wer sich auf die Wissenschaft beruft wie Michael Schmidt-Salomon, der muss sich nicht wundern, wenn auch seine Behauptungen an diesem Anspruch gemessen werden.

  133. Stephan 16.01. 16:23

    „Wir nehmen und behalten eben die mathematischen Verfahren, die uns ein bestimmtes Muster bestätigen. Dann wundern wir uns: “Oh, ein Muster! Die Welt ist erklärbar!” Die vielen anderen Vorgänge, die in kein von uns bekanntes Muster passen, blenden wir dann aus.“

    In diesem Sinne sind doch die Mathematischen Verfahren auch durch die Welt inspiriert. Nebenbei sind Physikalische Formeln ähnlich schön wie die Welt selbst, finde ich.

    Man muss hier gucken, mit welchem Fachgebiet wir es zu tun haben. Etwa in der Biochemie ist man glaube ich viel näher an der Wirklichkeit dran, weil eben das Biochemische gut zu der Reduktionistischen Strategie passt. In der Psychologie sieht das ganz anders aus. Die Medizin liegt so in der Mitte, hier gibt es die Harten Fakten, aber genauso die Persönlichkeit des Arztes, und die Qualität von Zuspruch, der ihm möglich ist. Man erkennt den Fehler im System vielleicht an dem fachspezifischem Kosmos, der all die Fragen umfasst, die noch nicht vernünftig erforscht sind.

    „…diese Ordnung gerade als Beweis für eine höhere Macht, einen Gott angesehen.“

    Ist nicht das einzige Argument, aber es ist Eins. Finde ich auch.

    @Balanus 16.01. 16:44

    „Zu den Unwägbarkeiten, die es selbstredend auch in einer kausal (ab)geschlossenen Welt gibt, kämen dann ja noch akausale Ereignisse hinzu, die schon aus rein prinzipiellen Gründen überhaupt nicht einkalkuliert werden könnten.“

    Nun ja, wenn ich die Motive kosmischer Geister für positiv einschätze, dann hoffe ich eher gerade auf eine Unterstützung bezüglich meiner Sicherheit im Straßenverkehr.

    @Physiker 16.01. 20:59

    „Es sei denn, dass man aus anderen Überlegungen solche unbekannten Mechanismen als zwingend notwendig erachten müsste.“

    Zum Beispiel spirituelle Erfahrungen aller Art?

  134. @Timm Grams

    Wer sich auf die Wissenschaft beruft wie Michael Schmidt-Salomon, der muss sich nicht wundern, wenn auch seine Behauptungen an diesem Anspruch gemessen werden.

    Stimmt, wundern muss sich Schmidt-Salomon nicht.

    Da ich schon einmal bei meiner Lieblingsbeschäftigung bin, beim Zerpflücken des Naturalismus, komme ich auf ein weiteres Welträtsel zu sprechen, dessen Existenz der Naturalist schichtweg abstreitet, anstatt es zu lösen [die Frage nach der Willensfreiheit].

    Ihr Anspruch, den Naturalismus zu zerpflücken, ist Ihnen, wie in meinem letzten Kommentar geschildert, zumindest an dieser Stelle nicht gelungen, der steht nach wie vor in voller Blüte.

    Es scheint mir auch nicht richtig zu sein, dass der Naturalist die Frage nach der Willensfreiheit nicht als interessantes Problem, bzw. Rätsel, ansieht, warum sollte er sich damit sonst so ausführlich beschäftigen? Lösen wird er es allerding nicht können, schon gar nicht mit wissenschaftlichen Methoden, das sollten auch Sie eigentlich wissen.

    Der Naturalist hat auch hier eine einfache Lösung

    Wenn dem so wäre, hätten Sie sich widersprochen (vrgl. “anstatt es zu lösen”).

    Die Antwort, die der Naturalist auf die Frage nach der Willensfreiheit gibt, fügt sich nur in eine kohärente, widerspruchsfreie Weltsicht ein, mit sparsamer Metaphysik. Sie gilt zunächst dort. Muss man nicht mögen.

  135. @Joker
    Der einführende Blog-Beitrag hat zwei konkrete suggestive Ziele, die auch benannt werden:
    1) das Bewusstsein muss unerklärbar bleiben
    2) das Zerpflücken des Naturalismus

    Solch eine Vorgehensweise ist nichts anderes als Esoterik-/ Querdenker-Argumentation

    Esoteriker/Querdenker lehnen sachliche/wissenschaftliche Argumentation grundsätzlich ab und bevorzugen Darstellung von Problemen als mystisch/unerklärbar.

    Der Herr Prof. Grams hat diese Absicht offen kommuniziert

  136. Timm Grams,
    entweder oder…..sehr gut. Damit wird der Unterschied zwischen Geistigem und Materiellen verdeutlicht. Ich bin da ganz auf ihrer Seite. Den Materialisten muss man klar machen, dass es ohne Geist nicht geht.

    Physiker,
    Gefühle zu Erregungszuständen erklären, da machen Sie es sich aber sehr einfach. Dann ist das Lesen und Verstehen eines Physikbuches auch nur ein Erregungszustand ?

    Und wie ist es , wenn Sie eine Grabinschrift in einer fremden Sprache zwar sehen können, aber Sie verstehen Sie nicht. ?

    In der Mathematik unterscheidet man zwischen der Darstellung einer Zahl, und dem Wert einer Zahl.
    Das erste ist materiell, das zweite ist immateriell.

    Bei fremden Sprachen ist das genauso. Sie hören die Sprache, physisch, aber sie verstehen sie nicht , aphysisch, geistig.

    Beim Bewusstsein haben wir es auch mit beiden Zuständen zu tun. Wir haben die physische Seite, die Gehirnwellen, und wir haben die geistige Seite, das Verstehen, die Einsicht. Die ist immateriell.

  137. @hwied
    Durch den gleichen Geist mit dem sie Erkenntnis erzielen durch Logik den Geist sowohl ein- als auch auszuschließen.
    man o man

  138. Mussi,
    Sie treffen den Nagel auf den Kopf.
    Beim Bewusstsein befinden wir uns genau auf der Grenze zwischen der realen physischen Welt und der transzendenten geistigen Welt.
    Beispiel :
    Heute Morgen war ich in der Kirche. Draußen liegt Schnee und in der Kirche , noch ganz leer, brennt das ewige Licht. Das ist die physische materielle Seite de Situation.
    Im Augenblick des Betretens der Kirche ergreift mich ein Gefühl des Glückes und der Zufriedenheit. Ich kniee nieder, spreche mein Gebet. Dabei richte ich eine Frage an das Kreuz mit dem gekreuzigten Jesus: “Jesus, bist du der Sohn Gottes?”. In diesem Augenblick antwortet er :”Ja”. Nicht akustisch, sondern nur in meinem Denken.

  139. @Physiker: Unterscheidungen & Sprache

    Ich glaube, wir hätten da noch viel auszudiskutieren, wie mir scheint; jetzt auch Ihre Verwendung von Occam’s Razor.

    Da die Natur sich nur durch Mathematik beschreiben lässt, kann Mathematik ja nicht nur Geisteswissenschaft sein…

    Dieser Logik kann ich nicht folgen: Die (jedenfalls: reine) Mathematik handelt nicht von der Natur; bestimmte mathematische Abstraktionen lassen sich schlicht auf natürliche Sachverhalte anwenden. Ebenso werden naturwissenschaftliche Sachverhalte (wie wissenschaftliche überhaupt) in menschlicher Sprache ausgedrückt.

    Es hilft, denke ich, wenn man die Natur, die man beschreiben will, also den Forschungsgegenstand, von den Instrumenten, womit man sie beschreiben will, unterscheidet: Letztere sind notwendigerweise menschengemacht und in diesem Sinne prinzipiell anders als Erstere. (Wieder eine Unterscheidung.)

  140. @Stegemann: Hirnforschung und der Mensch

    Noch etwas: Ich kann den Satz nicht mehr hören, die Naturwissenschaft könne den Menschen nicht erklären. Das will sie auch gar nicht.

    Nun ja, da äußern sich manche Naturwissenschaftler aber anders, wie ja nicht zuletzt in der Hirnforschung in den letzten 20-30 Jahren zu hören war (allerdings vor allem im Sinne von Versprechen). Das berühmte “Manifest” von 2004 ist Ihnen bekannt?

    Oder schauen Sie sich Sean Carroll an, Theoretischer Physiker am MIT: Der will schlicht alles erklären, was es gibt. Aber ob das jetzt eher Naturwissenschaft oder (schlechte) Philosophie ist, das lasse ich einmal offen.

  141. @Grams
    Bieri konstruiert ‘vorsätzlich’ ein Paradoxon.
    Das ist eben widerspruchsfrei nicht aufzulösen. Ist aber die Kehrseite der Logik und Erkenntnis im ‘selben’ Geist.

  142. @hwied
    Den Materialisten muss man klar machen, dass es ohne Geist nicht geht.

    So geistlos sind die Materialisten nicht. Ich bin immer wieder erstaunt über die Naivität (und gelegentlich die Überheblichkeit) der Antinaturalisten. Sie werfen mit Wörtern um sich, ohne sich über deren Intension und Extension zu kümmern.

    Extension und Intension (lateinisch extensio ‚Ausdehnung, Spannweite, Verbreitung‘ und lateinisch intensio ‚Mühe, Spannung, Anspannung‘) sind Begriffe aus der Semantik, mit denen verschiedene Dimensionen der Bedeutung sprachlicher Ausdrücke (Prädikate, Sätze) oder logischer Entitäten (Mengen, Begriffe, Propositionen) bestimmt werden. [..] In der Sprachphilosophie, den Sprachwissenschaften, der Logik und der Mathematik werden Extension und Intension oftmals unterschiedlich konzipiert. Für Prädikate und Begriffe sind die Ausdrücke Begriffsumfang und Begriffsinhalt unproblematische Übersetzungen.

    aus Wiki.

    Erzählen Sie uns etwas über den Geist, das nachweislich über die Hirnphysiologie hinaus ginge. Keiner der Antinaturalisten kennt das Gehirn vollständig. Aber sie behaupten penetrant und ohne Nachweise, dass der Geist hirnphysiologisch nicht zu erklären wäre.

    Zum Naturalismus gibt es keine Alternative. Die Welt ist so wie sie ist, etwas anderes ist grundsätzlich nicht erkennbar. Der Antinaturalismus beruht auf selbstüberhöhenden Phantasiegebilden. Und selbst die Phantasie und die Vorstellungskraft richtet sich an unserer realen Welt aus. Sogar einen Gott stellen wir uns in Menschengestalt vor!

    Ihre kirchlichen Gefühle beruhen auf Ihrer Erziehung, die sich in Ihrem Gedächtnis verfestigt hat. Wir alle haben Erinnerungen an schöne Erlebnisse, mit angenehmen Vorstellungen und Gefühlen, die wir gerne wiederholen würden. Daraus entstehen Wünsche, Absichten, Ziele, letztlich der Wille für das Verhalten. Und wir haben schlechte Erinnerungen, die wir als Wahrnehmungen und als Handlungen gerne vermeiden würden.

  143. @ Krichard 17.01.2021, 05:03 Uhr

    Zitat:
    1) das Bewusstsein muss unerklärbar bleiben
    2) das Zerpflücken des Naturalismus
    Solch eine Vorgehensweise ist nichts anderes als Esoterik-/ Querdenker-Argumentation“

    Das halte ich für eine Unterstellung.

    Ich meine eher, dass Prof. Grams vermutet, dass Bewusstsein nicht vollständig erklärbar ist. Es könnte zwar herausgefunden werden, unter welchen Umständen z.B. Empfindungen (Qualia) auftreten, aber man kann sich nicht vorstellen warum aus irgendwelchen Prozessen Empfindungen entstehen?

    Beschränkung auf den „Naturalismus“ bedeutet einfach, dass man freiwillig verzichtet immer mehr, möglichst (fast) alles, wissen zu wollen.

    Könnte z.B. aus der unterbewussten Angst kommen, die Existenzfähigkeit zu verlieren wenn man die „wahren“ Zusammenhänge erkennt, oder irrtümlich falsche Zusammenhänge für „wahr“ hält.

    Ich persönlich bin neugierig auf alle Denkmuster, manche interessieren mich, viele halte ich für originell kreativ, manche aus psychologischen Gründen für unzweckmäßig.

    Aber ich versuche sozusagen, mein eigenes „Betriebssystem“ möglichst abzuschirmen, wie man es auch, ganz sachlich, in der Computertechnik macht.

    Mein “Virenschutzprogramm” ist die Psychologie….

  144. @Grams
    Die Frage ist,welchen Schluss ich aus der Logik ziehe oder wo Erkenntnis stattfindet.

  145. Mussi, wie mir scheint, sehen sie das Paradoxon als Kehrseite der Logik an.
    Wenn man mit Logik die Alltagslogik meint, also einen Widerspruch im gesprochenen Satz oder auch geschrieben auf einem Blatt Papier., dann mag das stimmen.
    In der Aussagenlogik ist das Paradoxon kein überraschender Widerspruch, es gehört zu den Zirkelschlüssen, die die Eingangsprämissen verneinen.

    Das zu erkennen verlangt Intelligenz, die nur eine Teilmenge des Geistes ist. Paradoxi (stimmt das ?) kann man nicht auflösen, wenn doch, dann waren sie keine Paradoxone.
    Die Auflösung geschieht auf einer höheren Ebene, in der das Paradoxon als solches erkannt wird.
    Ein schönes Paradoxon ist die Aussage von Pinochio: „Meine Nase wächst gerade“.
    Paradoxon – Wikipedia
    „Im selben Geist,“ Sie meinem wohl im selben Sinn ?

  146. @Elektroniker
    Herr Prof Grams macht mit seiner Esoterik-/Querdenker-Zielsetzung nicht nur das wissenschaftliche Denken lächerlich. Man muss ihm zusätzlich auch noch kulturellen ´Rassismus´ vorwerfen (mir fällt kein besserer Begriff ein).

    Die buddhistische Philosophie lehrt seit 2500 Jahren dass Zeit keine Dauer (nicht die Kürzeste) sondern nur Intensität hat. Weil es keine Zeit-DAUER gibt können auch Gedanken und unser Bewusstsein nicht von Dauer sein.
    Die buddhistische Lehre geht deshalb davon aus, dass unser Bewusstsein daher nur eine Illusion/Täuschung ist, die sich aus der Arbeitsweise unseres Gehirns ergibt: durch eine rasche Abfolge von einzelnen Wahrnehmungen und reaktivierten Erinnerungen entsteht die Illusion(=Täuschung) von zusammenhängender Kontinuität.
    (Das ist Vergleichbar mit dem Sehen eines Kinofilms – wo wir die Illusion von wahrgenommener Bewegung empfinden – obwohl nur unbewegte Fotos gezeigt werden.)

    Weil Herr Prof. Grams mit keinem einzigen Wort auf diese gut bekannte Sichtweise des Buddhismus eingeht – wird das kulturelle Wissen der Buddhisten vorsätzlich ignoriert. Denn es sollte sich mittlerweile sogar bis Fulda herumgesprochen haben, dass es die buddhistische Philosophie gibt – und was da gelehrt wird.

  147. @hwied
    Ist doch mal eine schöne Aufgabe,einen Begriff und eine Definition dafür zu finden,wo Widerspruchsfreiheit scheitert.

  148. Anton Reutlinger,
    Man verzeihe mir meine Polemik gegen die Naturalisten. Sie hatte nicht Ziel , die Naturalisten als beschränkt hinzustellen, es geht um die Sache.
    Was ist Bewusstsein.?

    Und Bewusstsein hat eine transzendente Seite, die Sie ganz ausblenden wollen. Der Satz, „einen Gott stellen wir uns in Menschengestalt vor“ Er stimmt und er stimmt nicht.
    Im Dekalog steht explizit „Du sollst dir kein Abbild machen von Gott“. Damit wäre alles gesagt.
    Die Menschen machen aber dennoch ein Bild von Gott sie schaffen sich sogar ihren Gott (Dawkins)

    „Zum Naturalismus gibt es keine Aternative“ Zustimmung. Zu Gott auch nicht.

  149. @KRichard
    Dass ich nicht den Esoterikern zuzurechnen bin, habe ich “offen kommuniziert”. Wenn die Message nicht ankommt, muss das nicht am Sender liegen.

    Dafür, dass mir die Aussage “Das Bewusstsein muss unerklärbar bleiben” zugeschrieben wird, habe ich keine Erklärung und ich kann auch nicht finden, mich unklar ausgedrückt zu haben. Unter der Überschrift “Neue Ideen gefragt” steht das Gegenteil.

    @Joker
    Wortbedeutungen sind kontextabhängig. Der aufmerksame Leser sollte die richtige Auslegung finden. Dennoch: An einer Stelle hätte ich besser “Antwort” anstelle von “Lösung” geschrieben. Dann hätte es eine Gelegenheit für Missverständnisse weniger gegeben.

  150. Zur Definition der Widerspruchsfreiheit.

    In der Logik gilt eine Aussage als widerspruchsfrei, wenn sie keine falschen Schlüsse zulässt. (wikipdedia)
    Man könnte auch das Wort „eindeutig“ verwenden.
    Das ist eine vernünftige Forderung. Ist sie zu 100 % erfüllbar?
    Nein.
    Sogar in der Mathematik findet man einen Widerspruch, der sich nicht auflösen lässt, die Division durch 0.
    Man sagt dann einfach, die Division durch 0 ist nicht definiert.

    Widersprüche in der Aussagenlogik nennt man Paradoxon.
    Bei der Implikation sind sogar 50 % der Aussagen sowohl falsch als auch richtig. Nämlich immer dann, wenn die Eingangsbehauptung falsch ist.

    Kurt Gödel überträgt diese Einsicht auf die Sprache und sagt, jedes in sich geschlossene System ist offen und damit nicht widerspruchsfrei.
    Und da unsere Sprache eine große Redundanz hat, kann man ableiten, dass Sprache grundsätzlich nicht widerspruchsfrei ist.

  151. @Balanus / 16.01.2021, 22:38 Uhr

    Inwiefern könnte Deine »wirkliche, reale, physikalische Welt« denn etwas anderes meinen als das, was ich die Gesamtheit aller Gegenstände möglichen Erfahrungswissens ganannt hatte?

    Und die Position des (metaphysischen) Naturalismus wäre dann doch die, dass alles, was nicht zu eben dieser Welt zählt, bestenfalls ignorabler Hokuspokus ist. Oder genauer, dass alles, was nicht in dieses Weltbild passt, als inexistent erklärt wird. Ein Beispiel hatten wir ja bei früherer Gelegenheit schon mal mit Daniel Dennett und “Quining Qualia”. Für Dennett sind begriffliche Probleme mit Qualia ja kein Anlass zur Reflexion seines vorgefassten Weltbildes. Dass dieses vielleicht defizitär sein könnte, wird von ihm nicht erwogen, allfällige Probleme “löst” er einfach eliminativ — was nicht zum Dogma passt, wird passend gemacht. Davon muss aber nicht jeder begeistert sein, denn so wahnsinnig überzeugend ist diese Vorgehensweise letztlich nicht.

  152. Sind „Viele Welten“ mit dem Naturalismus vereinbar
    Dualisten sehen zwei Welten am Werk, eine geistige und eine materielle. Der Philosoph Markus Gabriel sieht in seinem Neuen Realismus sogar unbegrenzt viele Welten gleichzeitig am Werk – jede dieser Welten in ihren eigenen Bezügen, ihren eigenen Zusammenhängen. Eine Welt, die das alles unter einen gemeinsamen Hut setzt, sieht Markus Gabriel nicht. Deshalb hat er einem seiner Bücher den Titel gegeben: „Warum es die Welt nicht gibt“. Denn für Gabriel gibt es viele gleichberechtigte Welten, aber keine Welt die über alle regiert. Damit lehnt Gabriel wohl auch den Naturalismus/ontologischen Materialismus ab. Denn für einen Physikalisten, einen ontologischen Materialisten ist alles was real ist Teil des physikalischen Universums. Die materielle, die physikalische Welt ist für einen Naturalisten die dominierende Welt.

    In meinen Augen aber schliesst eine materialistische/naturalistische Position nicht aus, dass es „Viele Welten“ gibt. Es ist nur gerade der eliminative Materialismus der nichts anderes als die materielle Welt gelten lassen will. Der nicht eliminative Materialist aber denkt lediglich, dass alle Welten ihre Existenz letztlich der physischen Welt zu verdanken haben. Mit andern Worten Mathematik ist zwar ihre eigene Welt und lässt sich nicht auf Physik reduzieren, aber es gibt keine Mathematik ohne Mathematiker die sich die mathematischen Objekte ausgedacht haben. Es gibt in meiner Auffassung eine Art Nahrungskette, die im Falle der Mathematik so aussieht:
    physisches Universum –> Mathematiker —> —&;gt mathematische Objekte

    Im Kern:: Es gibt viele Welten, darunter beispielsweise die Welt der Mathematik und diese Welten lassen sich nicht auf die physische Welt reduzieren aber sie können nicht ohne diese physische Welt existieren.

    Schlussfolgerung: Descartes Idee, weil er sich seinen Geist ohne Körper vorstellen könne, müsse es eine Entität Geist losgelöst vom Körper geben, ist falsch. Aber die mental/geistige Welt folgt trotzdem ihren eigenen Regeln. Mit dem Geist ist etwas neue in die Welt gekommen – und zwar in ähnlicher Weise wie ein Kind, das geboren wird, neu in diese Welt kommt.

  153. @hwied
    Ist Gott nicht selbst, bzw. der Glaube an ihn, eine Alternative zum Naturalismus?

    Bewusstsein ist ein ständiger Vergleich zwischen Wahrnehmung und Erinnerung. Ich sehe ein Haus und weiß, dass es ein Haus ist, weil ich zuvor tausend Häuser gesehen habe. Die Wahrnehmung ist kein Einzelereignis, sondern ein dauerhafter Vorgang mit wechselnder Aufmerksamkeit und ständigen Vergleichen. Wahrnehmung und Erinnerung bilden einen ständigen Kreislauf der Signale. Die Wahrnehmung ist nicht ein einfaches Abbild der Außenwelt bzw. des eigenen Körpers, sondern eine Transformation oder Konstruktion in Vorstellung und in Sprache “ich sehe ein Haus”. Dass die Wahrnehmung mit der Realität korreliert, ist eine Folge der Evolution.

    Solche Vergleiche mit Entscheidungen finden auf molekularer Ebene statt, zwischen Neurotransmitter und Rezeptor, zwischen Enzym und Substrat. Davon gibt es in jeder Sekunde zig millionen im Gehirn. Ebenso gibt es millionen Feedbacks unterschiedlichen Umfangs im Gehirn. Allein die Anzahl und die Komplexität der Vorgänge im Gehirn übersteigt unsere Vorstellungskraft.

    Die ungelöste Frage ist, wie kommt man von den Vorgängen im Gehirn zu den Phänomenen, die wir selber als Bewusstsein deuten. Es ist eben die Gesamtheit oder die Integration der Vorgänge, die uns als Bewusstheit – von Gedanken, Wahrnehmungen, Erinnerungen – und Bewusstsein erscheint. Schon diese beiden Begriffe sollte man differenzieren. Darin liegt wohl die Quelle für die von Ihnen bezeichnete Transzendenz.

  154. Eben beim Lesen/surfen im Internet stieß m.o.w. zufällig ich auf das:

    Auf der anderen Seite des Mondes
    gehen
    in goldene Kleider gehüllt
    deine wirklichen Tage.

    Ist von Hilde Domin. Klickte dann Wikipedia. Ihr Grabspruch lautet:

    Wir setzten den Fuß in die Luft / und sie trug.

    Das kann man wohl als mentale Interpretation von naturalistischen Wahrnehmungen bezeichnen.

    Musste dann (“irgendwie”) an den Blogpost hier und die Kommentare denken.

  155. Anton Reutlinger,
    Zur Eingangsprämisse. Jetzt mal philosophisch.
    Gott ist nach dem AT weder eine Sache noch eine Person, er ist noch nicht einmal ein Objekt unserer Vorstellung. Er ist der Schöpfer des Universums, er ist der Ursprung des Geistes und der Sprache.
    Wie kann man dann sprachlich Gott erfassen ? Man kann es nicht. Deshalb das Verbot im Dekalog: Du sollst dir kein Abbild von Gott machen.

    Aber Gott hat sich geoffenbart, in seiner Schöpfung in jeder Sekunde, wo wir das Universum anschauen. Sogar der schwierige C.F. Gauss sagte einmal. “Zwei Dinge überraschen mich immer wieder, der gestirnte Himmel über mir und das moralische Gesetz in mir ”

    Naturalismus als Gegensatzpaar zu Gott zu sehen, das ist ausgeschlossen.
    Bei allem anderen stimme ich zu.
    Wie man unterschiedliche Sichtweisen zu Bewusstsein haben kann, daran arbeite ich gerade und will es auch formal logisch beweisen. Geduld.

  156. @hwied / 17.01.2021, 12:38 Uhr

    »Sogar der schwierige C.F. Gauss sagte einmal. “Zwei Dinge überraschen mich immer wieder, der gestirnte Himmel über mir und das moralische Gesetz in mir ”«

    Mon Dieu! Das ist nun aber garantiert nicht von Gauss.

    An falschen Zitaten sind schon etliche vielversprechende Karrieren gescheitert, das sollten Sie gegebenenfalls bedenken.

  157. @ hwied 17.01.2021, 08:15 Uhr
    @ Physiker

    Zitat: „Physiker, Gefühle zu Erregungszuständen erklären, da machen Sie es sich aber sehr einfach. Dann ist das Lesen und Verstehen eines Physikbuches auch nur ein Erregungszustand ?“

    Die Behauptung von „Physiker“ kann praktisch nicht „falsch“ sein. Es geht um die verknüpften „Kombinationen“ von, bei Menschen extrem vielen „Erregungszuständen“, die nicht alle so etwas wie „starke Empfindungen“ hervorrufen.

    Um Kombinationen von jeweiligen Schaltzuständen (0…1, die z.B. Erregungszustände entsprechen könnten) geht es den Informatikern. Sie haben ein abstraktes Konzept entwickelt, um in geeigneten technischen Maschinen gemäß einem Code, „abgebildete“ Information zu verarbeiten.

    Letztlich wird Input in „Maschinen geeignete Form“ umgewandelt, verarbeitet, gespeichert und in geeigneten Output umgewandelt.

    Beim Lesen und verstehen eines Physikbuches geht es, kurz gesagt darum, sequentiell über die „Buchstaben“ den Inhalt des Buches in die „baumartigen und vermaschten“ Strukturen des Gehirns „einzubringen“, wobei die vorhandenen „synaptischen Wissensstrukturen erweitert“ werden (E. Kandel).

    Letztlich ist das neue Wissen mehr oder weniger exakt, im neuronalen System „gespeichert“. Die Erregungszustände „bilden“ das Wissen strukturiert ab, einerseits die Begriffsempfindungen z.B. für das Wort „Atom“, aber auch die Kombinationen der strukturell verknüpften Assoziationen.

    Allerdings können keinesfalls alle jemals „gespeicherten“ Empfindungen bewirkende Erregungszustände gleichzeitig ins Bewusstsein gelangen. Die wirkliches „Bewusstsein“ ermöglichenden Strukturen dürften so etwas wie “Schnittstellen“, flächige Strukturen sein, über die die Ergebnisse von „Rechenoperationen“ geführt werden. Z.B. Netzhaut, oder andere Zwischenschichten. Genau dort, oder von dort aus, dürften besonders empfindungsfähige Strukturen angesteuert werden um die Empfindungen zu „realisieren“ (z.B. Zapfen, Stäbchen…).

    Ich würde davon ausgehen, dass es in technischen Computersystemen zwar elektrische „Erregungen“ gibt, die aber keine größeren Empfindungen mit besonderen Auswirkungen haben dürften.

    Selbst dann nicht, wenn das Bedienpersonal gelegentlich, wenn gehäuft bestimmte Fehlermeldungen am Kontrollbildschirm ausgegeben werden vermutet, dass ihre elektronischen „Lieblinge“ unter Depressionen leiden könnten ….

  158. @Grams
    Die buddhistische Philosophie hat schon vor 2500 Jahren erklärt, wie die Illusion eines Bewusstseins entsteht.
    Für die seit damals beschriebene Arbeitsweise finden Sie heute in der Fachliteratur (Psychologie, Kognitionswissenschaft) die ´predictive coding theory´.

    off topic:
    Mein Brief sollte morgen oder übermorgen an Ihrer Hochschule ankommen.

  159. @Reutlinger
    Es gibt schon viele Experimente (fMRT. Elektroden im Gehirn) welche zeigen, dass bei der Verarbeitung von neuen Reizen (= Lernen) bestimmte Neuronen aktiv sind.
    Und beim Wiedererinnern sind dann wieder die gleichen Neuronen aktiv.

    D.h. der neuronale Zustand beim Lernen und beim Erinnern ist weitgehend gleich. Diese Tatsache ist eine wichtige Grundlage der Gedächtnisforschung auf neuronaler Ebene.

    Dies bedeutet, dass wir kein extra Bewusstsein brauchen – weil es ausreicht, dass diese neuronalen Aktivitäten vorhanden sind.
    Der Unterschied zwischen unbewusster und bewusster Wahrnehmung liegt einfach nur in der Intensität der neuronalen Aktivität. Wenn eine bestimmte Aktivitätsschwelle überschritten wird – bekommen wir eine bewusste Wahrnehmung. Ist die Intensität geringer, sprechen wir vom Unterbewusstsein.

  160. @hwied

    Das angebliche Zitat von Gauss stammt in Wirklichkeit von I.Kant.

    Ein anderes ZItat von Kant ist für diese Diskussion von Bedeutung, als “Kopernikanische Wende” Kants bezeichnet:

    unsere Vorstellung der Dinge, wie sie uns gegeben werden, richte sich nicht nach diesen als Dingen an sich selbst, sondern diese Gegenstände vielmehr als Erscheinungen richten sich nach unserer Vorstellungsart.

    Das heißt eben, unsere Wahrnehmung der Welt ist nicht eine Abbildung derselben, sondern eine eigenständige Konstruktion des Gehirns, die selbstverständlich mit den Signalen der Sinnesorgane verknüpft ist. Diese Konstruktion ist nicht beliebig, sondern eine Folge von Phylogenese, Ontogenese, Ratiogenese zum Überleben in einer dynamisch veränderlichen Welt.

  161. anton reutlinger 17.01. 11:43

    „Die ungelöste Frage ist, wie kommt man von den Vorgängen im Gehirn zu den Phänomenen, die wir selber als Bewusstsein deuten. Es ist eben die Gesamtheit oder die Integration der Vorgänge, die uns als Bewusstheit – von Gedanken, Wahrnehmungen, Erinnerungen – und Bewusstsein erscheint. Schon diese beiden Begriffe sollte man differenzieren. Darin liegt wohl die Quelle für die von Ihnen bezeichnete Transzendenz.“

    Solange die Erklärungslücke so riesig ist, ist jede Menge Platz für verschiedene Weltanschauungen. Der Naturalismus ist nicht nachgewiesen, und in der Praxis schon gar nicht alternativlos.

    Was religiösere Ansichten betrifft, so gibt es zwischen denen nach wie vor erhebliche Unterschiede. Einen ganzen Kosmos von Konzepten kann hier der Religionswissenschaftler untersuchen.

    Ich kann mir sehr gut vorstellen, dass man in vielleicht 50 Jahren sehr viel mehr zum Thema sagen kann, wenn eben diese Erklärungslücke entscheidend kleiner geworden ist. Bis dahin bleibt eigentlich nichts anderes Vernünftiges übrig, als hier die Vielfalt zu respektieren.

    Wieso dass immer wieder schwer fällt, dass mag auch an der Wirklichkeit selbst liegen: Der Eine hat überhaupt keine persönlichen spirituellen Erfahrungen, und muss eine geistlose Welt vermuten, der andere hat diese oder jene spirituelle Erfahrung, und tendiert entsprechend auch noch zu verschiedenen religiösen Konzepten.

    Sollten wir zukünftig das Zustandekommen der spirituellen Erfahrungen von der Ebene der Nervenzellen aus nachvollziehen können, werden wir wohl auch Andere mit anderen jeweiligen Erfahrungen entsprechend einschätzen können. Und ggf. eher respektieren können. Solange der Eine meint, dass der Andere in einer Welt lebt, die unmöglich ist, dann geht das mit dem Respekt jedenfalls schlecht.

  162. Erfrischend, auch einige Wortmeldungen hier, danke, hierzu eine kleine Nachfrage :

    Du weißt, ich bin bekennender Determinist und mein Glauben lässt sich sicher einer Spielart des Naturalismus zuordnen. [Kommentatorenfreund “Joker” a.k.a. “Jolly”]

    Wie wird man denn ‘bekennender Determinist’?

  163. Hierzu noch kurz :

    „Für das Rätsel des Bewusstseins gilt etwas, was für sonstige Rätsel nicht gilt: Wir haben keine Vorstellung davon, was als Lösung, als Verstehen zählen würde.“

    Das Bewusstsein ist genau das, was begrifflich von erkennenden Subjekten konstruiert und so gemeint ist, dies als “Sehnsuchtsort”.

    Wer will kann auch anmerken, dass der Mensch nicht die Welt direkt, sondern seine Vorstellung von der Welt bearbeitet, anders scheint es nicht zu gehen, und insofern nicht bspw. den Hund, sondern die Idee von einem Hund bearbeitet.
    (Es sei denn der beißt gerade in die Waden und wird reflexhaft weggetreten, ganz ohne Ratio sozusagen.)

    Und das letztliche Unwissen über die Welt und ihren Betrieb ergibt sich dadurch, dass das Erkenntnissubjekt Weltteilnehmer und nicht Weltbetreiber ist.

    Interessant ist für mich die Frage, ob das Erkenntnissubjekt mit seinen Vorstellungen eine Welt bildet, die letztlich nicht unterscheidbar ist von der Naturwelt, außer dass sie se-ehr klein ist (aber nicht so klein sein muss, wenn an andere Erkenntnissubjekte gedacht wird, die allerdings zumindest zurzeit nicht vorliegen, also : gedankenexperimentell) und keinen Raum für besondere Entwicklung offen lässt, dortige selbstständige Erkenntnissubjekte meinend.

    Die Theorie über Welten ist interessant, ist es möglich in einer Welt eine Welt zu bilden, die komplexer ist als die umgebende Welt (in unserem Fall die Naturwelt)?
    (Antwort : Ja, das ist unter bestimmten Voraussetzungen möglich.)

    MFG
    Dr. Webbaer

  164. Du Bois-Reymond geht von seinem naturwissenschaftlichen Befund der Reizleitung im Nervensystem aus. Vermutlich sieht er mit den Methoden, die er zur Verfügung hat, keine empirische Perspektive, den Zusammenhang von neuronalem Geschehen und subjektivem Empfinden zu entdecken; m.E. kapituliert er deshalb als Naturwissenschaftler mit seinem “ignorabimus” vor der Philosophie.
    Das „Rätsel“ des Bewusstseins stellte sich in voller Schärfe erst mit dem Aufkommen der Naturwissenschaften am Beginn der Neuzeit, als Descartes seine zwei kategorial getrennten Substanzen lehrte, res cogitans und res extensa. Und dieser Dualismus beeinflusst in irgendwelchen Formen noch heute das Denken, z.B. auch das von Bieri.
    Ich weiß nicht, woher Grams seine Auffassung von Naturalismus bezogen hat. Ich finde es arrogant und ignorantisch, neben der Natur eine weitere Welt der mentalen Phänomene zu postulieren, eine geistige Welt, die kategorial von der Natur getrennt existieren soll, obwohl sie doch nur in dieser und mit ihr erfahrbar ist. Grams „Problem“ des Fremdpsychischen halte ich aus dem gleichen Grund für marginal, wenn nicht gar für ein Scheinproblem.
    Wir haben doch heute, auch mit Relativitätstheorie und Quantenmechanik, schon längst die Mittel, auf der Basis der Entdeckung von Bois-Reymond weiter zu forschen. Um es global zu formulieren: Die Kosmologie hat aufgezeigt, auf welche Weise Naturgesetze und –konstanten unseres Universums die Existenz von Materie ermöglichen, dieser besonderen Erscheinungsform von Energie. Nur Materie und Energie generieren Komplexität, und allein mit Materie und freier Energie wird höchste Komplexität realisiert: Leben und Geist.

  165. Nur sollten Sie zugestehen, dass die Aussage von wem auch immer, dass identische Ursachen nicht notwendigerweise identische Folgen haben, mithin es gäbe Willensfreiheit (*), ebenfalls metaphysisch, unwissenschaftlich und Glaubenssache ist. [Kommentatorenfreund “Joker” a.k.a. “Jolly”]
    Warum sollte ich? Ich trete[] hier als Skeptiker auf und das Erbauen kühner Gedankengebäude fällt nicht in seine Kompetenz. Ihm reicht es zu sagen, dass es keine wissenschaftlichen Gründe gibt, die Willensfreiheit zu negieren. [Gastautor, in diesem Fall, Moment, gleich wieder da, …, …, …, achja, Herr Prof. Dr. Timm Grams]

    Ganz genau, es wird zwar nicht so-o viel gewusst, aber es wird gewusst, was nicht gewusst wird, agnostizistischerweise, immerhin!

    Metaphysik ist allerdings nicht unwissenschaftlich, dies nur ganz am Rande angemerkt. Hat wohl auch niemand behauptet, Aufzählungen meinend.

  166. @Dr. Webbaer
    Hab sie schon vermisst!

    Wir wissen,was das klassische logische Schließen ist.
    Wenn aber die Axiomatik den Schluss ergibt,diese Axiomatik ist ein Paradox, können dann weitere Schlüsse aus einem Paradox wahr sein oder sind sie von vornherein unwahr?

    Was meinen Sie?

  167. @ Klaus Günther :

    Ich finde es arrogant und ignorantisch, neben der Natur eine weitere Welt der mentalen Phänomene zu postulieren, eine geistige Welt, die kategorial von der Natur getrennt existieren soll, obwohl sie doch nur in dieser und mit ihr erfahrbar ist.

    Ist halt der Philosophie, der Liebe zum Denken, der Denkmöglichkeit geschuldet, dieses “Ignorabimus!”, eigentlich so bereits im bekannten Höhlengleichnis angelegt.

    Direkt schön ist es so nicht, aber es ist angemessen, vs. ‘arrogant und ignorantisch’, oder?

    MFG
    Dr. Webbaer

  168. @ Kommentatorenfreund ‘Mussi’ :

    Also mit bereits derart problematischen Axiomatiken würde ich es nicht versuchen wollen … darauf etwas aufzubauen.
    Logik kann widerspruchsfrei sein, es wird ja auch nur behauptet, dass Widersprüchlichkeit in logischen Systemen auftritt, wenn eine gewisse Mächtigkeit erreicht wird, wobei diese Mächtigkeit dann genau der “Wurm” sein muss.

    Danke für die nette Anmerkung, ich bin abär schon bei den kommentierenden wie inhaltegebenden Kollegen hier, will hier keinesfalls irgendwie esoterisch aufscheinen.

  169. Hallo Dr. Webbaer,
    jetzt kommt auch noch der Determinismus ins Spiel
    Das ist das Stichwort für einen geschichtlichen Längsschnitt von der schönsten Frau der griechischen Antike, der schönen Helena, dann einem Sprung in das vorletzte Jahrhundert, der intelligenten Alice im Wunderland, dann zu dem eigenwilligen Cantor und seinen Epigonen hin zu Betrand Russell und weiter zu unserem Blog. Und am Ende werden wir erkennen, was Bewusstsein nicht ist (Popper).
    Die Geschichte beginnt in Griechenland, wo Naphtene als die Anmutigste unter den Schönen galt.
    Sogar Helena verblasste in ihrer Gegenwart. Schönheit ist nicht nur ins Gesicht geschnitten, sie zeigt sich in jeder Bewegung. Und wenn Naphtene eine Handbewegung machte, hörten die Vögel für einen Moment auf zu singen. Die griechische Sagenwelt wäre anders verlaufen wenn nicht Egoi
    eine andere Mitbewerberinnen einen schlauen Plan gehabt hätte.
    Sie schenkte Naphtene einen goldenen Spiegel. Als Naphtene sich zum ersten Mal im Spiegel betrachtete erstrahle ihr Lächeln, so dass sogar die Sonne neidisch wurde.
    Aber ihr Lächeln wurde von mal zu mal immer gestellter, ihre anmutigen Bewegungen wurden gezierter und ihre Anmut vertrocknete zur Eitelkeit.
    Was wir uns merken sollen: Wenn man sich seiner Schönheit bewusst wir, dann verliert man die Anmut für immer. Und so wurde Naphtene nur noch eine unter den Schönen. Fortsetzung folgt.

  170. Ehrlich geredet hasst Dr. Webbaer das Wort Determinismus ein wenig, mittlerweile, denn die Bestimmtheit von gegebener Entwicklung muss einem Bewusstsein von erkennenden Subjekten nicht entgegenstehen.
    Ich mag es, wenn die Frage nach der Determiniertheit dieser Welt als per se unbestimmbar vertagt oder als nebensächlich erledigt wird.

    Hierzu – ‘Solange die von uns erkannten Gesetzmäßigkeiten der Welt ein bewusstes Denken nicht zulassen, kann auch das Universum nicht gedacht werden. Es existiert demnach für uns nicht. Die heutige Wissenschaft ist offenbar nicht reichhaltig genug, um das Geistige zu verstehen.’ – fällt mir nicht viel ein, außer :
    1.) ‘bewusstes’ Denken / Leben kann determiniert sein, muss also nicht so sein
    2.) kann die Welt, das was “schaltet und waltet”, oder die Wirklichkeit (Meister Eckhart, das, was wirkt ist gemeint) oder das Universum, die physikalische Sicht ist gemeint, oder die Realität, die Konstruktion von Sachlichkeit ist gemeint, vorgenommen eben wieder von erkennenden Subjekten, “gedacht” werden
    3.) ‘existierend’, aus sich heraus sein, aus sich heraus stellend, ist nichts, wenn sich niemand findet, der so beschreibt
    4.) und zum letztzitierten Satz : Die szientifische Methode ist eine Methode des Geistes, die nicht rekursiv auf ihn, den Geist selbst, angewendet werden kann

    MFG

  171. @Jeckenburger
    Das Zustandekommen von spirituellen Erfahrungen kann man mit dem Begriff ´state dependent retrieval /zustandsabhängiges Erinnern´ erklären.
    Dieser Begriff besagt, dass das WAS und WIE wir erinnern, immer von zwei Zuständen abhängig ist
    A) von unseren körperlichen, intellektuellen und emotionalen Fähigkeiten zu dem Zeitpunkt wo wir eine Erfahrung machen und als Wissen im Gedächtnis abspeichern.
    B) von unseren körperlichen, intellektuellen und emotionalen Fähigkeiten zu dem Zeitpunkt wo diese Erfahrung beim Erinnern wieder reaktiviert wird.
    Dies bedeutet – dass unsere im Gedächtnis gespeicherten Erlebnisse beim Erinnern umgedeutet/verändert werden können.

    Hinzu kommt, dass unsere Erfahrungen in der zeitlichen Gegenwartsform erlebt, so im Gedächtnis gespeichert und genau so wieder reaktiviert werden. D.h. wir erleben das Ganze noch einmal.

    Weil die Wahrnehmung bei Menschen aller Kulturen gleichartig funktioniert, werden ähnliche Erlebnisse von allen Kulturen berichtet. D.h. hiermit hat man eine kulturübergreifend gemeinsame biologische Grundlage für ´spirituelle´ Erfahrungen – wenn bestimmte Erlebnisse kulturell als spirituell gedeutet werden.
    Werden spirituelle Erfahrungen dann in einer Sekundärinterpretation religiös gedeutet – dann versteht man, wieso es Religiosität in allen Kulturen gibt.

    Ein Tipp: Überlegen sie, was ein Mensch ab dem 5. Schwangerschaftsmonat erleben kann – und wie diese Erlebnisse dann wohl von einem Erwachsenen beim Erinnern reaktiviert und interpretiert werden.
    Machen Sie zwei Listen – Sie werden überrascht vom Ergebnis sein.

    Der Begriff ´state dependent retrieval´ erklärt auch, wie wir ALTE Erfahrungen in NEUES Wissen umwandeln – bzw. wieso wir uns lebenslang als die gleiche Person empfinden; obwohl wir uns dauernd verändern.

  172. Determiniert ist hier als Reihenfolge zu verstehen, eine Folge die auch anders hätte verlaufen können.
    2. Teil
    Von soviel Schönheit geblendet wenden wir uns einer anderen Frau zu, eigentlich kann man sie nicht als Frau titulieren, sie war eher ein gealtertes Kind noch vor der Pubertät, Alice.
    Alice war also der Erzählung nach im Wunderland, und in diesem Land gab es eine Königin, die Schwarze Königin.
    Sie verlangte von Alice, oder war es jemand anderes , dass man eine Landkarte ihres Königreiches anfertige, im Maßstab 1 : 1 . Und auf dieser Landkarte sollte alles aber auch alles, verzeichnet sein.
    Das sollte doch möglich sein, aber wenn alles darauf verzeichnet sein soll, dann muss doch auch diese Landkarte mit eingezeichnet werden, und auf dieser wieder die nächste gezeichnete Landkarte. Wie kann man das lösen. Es zeigte sich, dass das genaue Abbild gar nicht möglich ist.
    Warum ist das wichtig, weil die Mathematiker unter den Naturalisten von Abbildung sprechen, wenn sie eine Zuordnung machen oder etwas füllen wollen.
    Dabei sei auch angemerkt, dass wir das Wort Bewusstsein für den Container , das Gehirn , verwenden, als auch für den Inhalt, der Abbildung. Das ist sehr unklug würde Alice sagen, soviel passt in das Gehirn nicht hinein. Genauso wie wir nicht alles auf der Landkarte unterbringen könnten, selbst wenn wir im Maßstab 2:1 zeichnen.
    Wir merken uns Abbildung.

  173. Ich hab mal neben dem (eher gewohnten) “Schleim-Scan” in diesem Kommentariat den “Helmut Wicht-Scan” versucht und bin fündig geworden :

    Ich neige ja dazu – ich weiss, ich wiederhole mich ad nauseam – du Bois-Reymonds (Leib-Seele) Problem dadurch zu lösen, dass ich alles für beseelt halte […]
    […] und muss mich fragen lassen, wie es sich wohl anfühle, ein Stein zu sein.

    Genau diese Sicht kenne ich, von anderen vorgetragen, vor Jahrzehnten bereits, es gibt hier auch das Gaia-Konzept, Quelle :
    -> https://de.wikipedia.org/wiki/Gaia-Hypothese
    …es ist falsch, aber, lol, ich mag es.

    Es ist wohl so, dass alles potentiell beseelt ist.
    Ich rate allerdings nicht an so besonders auszubauen,
    es war abär eben schön,
    MFG
    Dr. Webbaer

  174. @Dr. Webbaer

    Wie wird man denn ‘bekennender Determinist’?

    Zum Deterministen bin ich geworden, durch Abbruch der Extrapolation des Ursache-Wirkungs-Gedankens an einer Stelle und der Dogmatisierung der dortigen Begründung.

    (Es musste so kommen, auch das anschließende Bekennen dazu.)

    Ich mag es, wenn die Frage nach der Determiniertheit dieser Welt als per se unbestimmbar vertagt oder als nebensächlich erledigt wird.

    Da haben Sie meine volle Unterstützung, und hoffe, das Nebensächliche damit erledigt zu haben.

  175. Dr. Webbaer,
    determiniert in unserer Welt sind nur die Denkgewohnheiten, determinierend wirkt auch die Evolution, die geistige Freiheit wird nicht angetastet.

  176. Mancher Kommentar wäre möglicherweise weniger giftig ausgefallen, wenn dieser Abschnitt meines Artikels mehr Aufmerksamkeit erhalten hätte:

    Der weiße Pfeil spielt auf diesen Gedanken Kants an: Da wir nur die Erscheinungen erfahren, und nicht „die Dinge an sich“, gruppieren wir die vermeintlichen Dinge der Außenwelt nach Maßgabe der Erscheinungen. Wir „denken“ also in Pfeilrichtung.

    Die Ursache-Wirkungsrichtung ist aber genau entgegengesetzt.

  177. Ja, ich will das mal so durchgehen lassen, Kommentatorenfreund “Joker”, a.k.a. “Jolly” :

    Wie wird man denn ‘bekennender Determinist’? [Ego]
    Zum Deterministen bin ich geworden, durch Abbruch der Extrapolation des Ursache-Wirkungs-Gedankens an einer Stelle und der Dogmatisierung der dortigen Begründung.
    (Es musste so kommen, auch das anschließende Bekennen dazu.) [Kommentatorenfreund “Jolly”]

    Zur Feststellung von Kausalität bleibt Ihr Kommentatorenfreund gerne bei Humes, bei der sog. Humeschen Metaphysik, hier so webverwiesen :
    -> https://de.wikipedia.org/wiki/INUS-Bedingung#Hintergrund
    ..wobei streng genommen, sofern Sie Ihr Langzeit-Kommentatorenfreund korrekt verstanden hat, so kein Determinismus angeleitet wird, sondern eher Relativismus und Nihilismus, dem Sie, wie auch ich, zu widerstehen vermochten, dem naturwissenschaftlichen Methodengedanken folgend.
    Utilitarismus im besten Sinne könnte dieses Vorgehen so genannt werden.

    MFG

  178. zz HWied:
    Ihre Vergleiche mit der griechischen Mythologie schrecken mich immer. “Wenn jemand sich seiner Schönheit bewusst wird…” Dann könnte dieser jemand entweder eitel sein oder ein Narzisst. Oder er glaubt dem von der Gesellschaft vorgegebenen Schönheitsideal ,was sich bei den alten Griechen ja vornehmlich auf die äußere Erscheinungen bezog, zu entsprechen. Der Schönheitsbegriff, der sich stetig wandelt, wird also von der Gesellschaft/Zeitgeist vorgegeben -und ist bei entwickelten ,nicht nur von niederen Trieben gesteuerten Gesellschaften – auch gepaart mit inneren Werten. Wenn sie, wie sie oben angeben ,mit Gott gesprochen haben und eine innere Stimme gehört haben, so kenne ich Literatur von Soldaten die an der Ostfront in den Schützengräben damals auch um Gottes Hilfe gebetet haben. Ihnen war leider nicht “bewusst” , dass ihr Bewusstsein von den Mächtigen gezielt gesteuert wurde und das Gott -wie üblich- im Sinne der Macht und des Geldes wegen instrumentalisiert wurde und sich nicht wehren konnte da selbst die oberen Kirchenvertreter dieses System unterstützten. Das Bewusstsein wird also von WERTEN bestimmt, gesteuert .
    So war Zeuss hinter jeder “Schönen” hinterher, die nicht rechtzeitig auf den Baum fliehen konnte…Mir ist also nicht das Bewusstsein ein Rätsel sondern der Umstand wie die Menschen sich immer wieder bewusst manipulieren lassen….

  179. Jegliche Metaphysik sorgt für eine Einschränkung des Denkens.

    Unser Denken hat Voraussetzungen, das Erkennen betreffende (Erkenntnistheorie) und das Sein bzw. die Wirklichkeit betreffende (Metaphysik). Ich verstehe diesen Satz von Ihnen nicht, ich vermute, Sie meinen Ontologie? Und ich vermute, Sie vermuten neben der Existenz der physischen (natürlichen) Gegebenheiten die Existenz von Übernatürlichem (Nichtphysischem), z.B. irgendeine Art von Bewusstseinssubstanz (res cogitans). Helmut Wicht erklärt sich sogar zum idealistischen Monisten.

    Mir ist nicht klar, wie Sie natürlich auftretende Phänomene wie bewusstes Erleben mit etwas gänzlich und prinzipiell Unerklärlichem erklären wollen.

    Würde sich mein Bewusstsein einem absoluten, substantiellen Bewusstsein verdanken, könnte ich niemals bewusstlos sein – was offensichtlich widersinnig ist.

    Das Bewusstsein gibt es so wenig wie das Rotsein. Sie reifizieren eine zeitweise auftretende Eigenschaft eines Teils der in biologischen Systemen ablaufenden Informationsverarbeitung, nämlich manchmal und in Teilen bewusst zu sein, also von dem betreffendem Subjekt als Empfindung, Vorstellung oder Gedanke erlebt zu werden.

    Wenn Sie an echte, d.h. primäre psychische Ursachen glauben, dann treten Sie nicht in Widerspruch zum Naturalismus, sondern zur Physik. Für alle beobachtbaren Bewegungen von Materie sind uns die hinreichenden physischen Ursachen weitgehend bekannt; menschliches Verhalten ist da m.W. nicht ausgenommen. Was sich bewegt oder ändert, bewegt oder ändert sich aufgrund (bekannter) physischer Gesetzmäßigkeiten. Sie können das aus pragmatischen Gründen mit komplexeren Terminologien beschreiben, aber Sie werden keinen Physiker finden, der für Bewegungen eines Organismus nichtphysische Ursachen annimmt.

    Solange die von uns erkannten Gesetzmäßigkeiten der Welt ein bewusstes Denken nicht zulassen, kann auch das Universum nicht gedacht werden. Es existiert demnach für uns nicht. Die heutige Wissenschaft ist offenbar nicht reichhaltig genug, um das Geistige zu verstehen.

    Wir haben heute in der Kosmologie und Evolutionsbiologie Erkenntnisse aus Zeiträumen, in denen (menschliches) Bewusstsein noch gar nicht vorhanden war. Selbstverständlich muss man davon ausgehen, dass die Natur auch bewusstes Erleben und Denken ermöglicht; alles andere wäre ein performativer Selbstwiderspruch und kein Naturalist behauptet dergleichen.

    Der Naturalismus ist der Versuch ausgehend von einer unumwundenen Anerkennung der Gültigkeit der Physik und auch der Biologie zu philosophieren, auch wenn sich daraus Zumutungen für den menschlichen Alltagsverstand ergeben sollten.

  180. @Chrys // 17.01.2021, 11:35 Uhr

    » Inwiefern könnte Deine »wirkliche, reale, physikalische Welt« denn etwas anderes meinen als das, was ich die Gesamtheit aller Gegenstände möglichen Erfahrungswissens ganannt hatte? «

    Die von mir gemeinte Welt (Lebenswelt) ist keine vom menschlichen Denken abhängige bloße „Vorstellung“, sondern Vorbedingung für jegliche Vorstellungen.

    In dieser meiner Welt kann alles Messbare Gegenstand empirischer Forschung sein. Für die (messtechnische) Erfassung der „Gesamtheit aller Gegenstände“ fehlen uns hingegen die Mittel.

    Siehe auch meine nachfolgende Antwort an Timm Grams, dort streife ich das Thema ein wenig.

  181. @Timm Grams // 16.01.2021, 23:45 Uhr

    » Manche, die sich für Naturalisten halten, werden nicht einmal wissen, was das [die monistisch-materialistischen Identitätstheorie] ist. «

    Wenn ich dem Wikipedia-Eintrag trauen darf, dann war die ursprüngliche Identitätstheorie aus den 1950er Jahren in sich schon nicht sehr stimmig. Da sollte nämlich, als Analogie, bereits ein Molekül wie H2O mit Wasser „identisch“ sein. Das ist natürlich eine problematische Gleichsetzung, aber welche Analogie aus der beobachtbaren Welt trifft mit Blick auf mentale Phänomene schon exakt die Verhältnisse im hoch entwickelten Gehirn? Meines Wissens keine.

    Davon abgesehen meine ich jetzt verstanden zu haben, wieso Ihnen naturalistische Vorstellungen über den Zusammenhang von neuronaler Aktivität und mentalen Empfindungen in sich widersprüchlich erscheinen.

    Ihre Ausführung zum Bieri-Trilemma machen m. E. deutlich, wo der Denkfehler liegt: Der Naturalismus-Kritiker setzt (völlig unbegründet) voraus, dass es eine von der physikalischen Welt geschiedene nichtphysikalische Welt gibt, in der die mentalen Phänomene, die per Definition ja nichtphysikalische Phänomene sein sollen, zwangsläufig ihren Platz haben. Die Existenz einer solchen Welt wird aber von den Naturalisten bestritten, woraus dann gefolgert wird, dass es demzufolge auch keine mentalen Phänomene geben könne, denn ohne nichtphysikalische Welt keine nichtphysikalischen, mentalen Phänomene. Damit schließt sich der Kreis, und der agnostische Skeptiker meint, damit habe sich der Naturalismus selbst widerlegt.

    Ich glaube, der Fehler liegt darin, wie im Bieri-Trilemma Prämisse 1 formuliert wird:

    1 Mentale Phänomene sind nichtphysikalische Phänomene.

    Mir scheint, dieser Satz ist falsch und wahr zugleich, je nachdem, was man unter ‚mental‘ und ‚nichtphysikalisch‘ versteht. Der Skeptiker meint, ein Naturalist müsse diese Prämisse für wahr halten. Das kann er m.E. aber nur, wenn ‚nichtphysikalisch‘ nichts weiter bedeutet als „nicht mit physikalischen Mitteln messbar“.

    Wenn sich Helmut Wicht als Idealist also fragt, wie es sich wohl anfühlt, ein Stein zu sein, dann haben wir offenkundig nicht die Mittel, das mittels physikalischer Instrumente herauszufinden. Die Gefühle und Empfindungen des Steins bleiben obskur.

    Nicht anders verhält es sich mit den Gefühlen und Wahrnehmungen eines lebenden Objekts: sie sind schlicht nicht (direkt) messbar. Aber wer würde, zumindest im Falle des Menschen, aufgrund dieser Tatsache bestreiten wollen, dass sie im Organismus tatsächlich existieren, also Teil unserer realen physikalischen, mithin einen Welt sind?

    Allem Anschein nach hat sich die Naturalismus-Kritik hier argumentativ selbst ausgetrickst, sich im Gestrüpp mentaler und physikalischer Begriffe heillos verheddert. Vielleicht sollte sie dem „gesunden Menschverstand“ doch etwas weniger trauen.

  182. Golzower,
    die Schönen in der griechischen Mythologie, die gab es nicht wirklich.
    Sie dienen nur dazu Ideen zu transportieren.
    Und was Bewusstsein ist oder nicht ist,, das soll erst über diesen neuen Denkansatz herausgefunden werden.
    Sie haben mir Recht angemerkt, dass unser Bewusstsein mit verschiedenen Inhalten gefüllt sein kann, und dass diese Inhalte oft manipuliert worden sind.
    Der Soldat im Schutzengraben, der hat auch gebetet, vielleicht für seine Familie, wenn er realistisch war, dann hat er lieber den Kopf eingezogen.
    Bewusstsein soll trotzdem nicht verwechselt werden mit Erfahrungen.

  183. @ Kommentatorenfreund ‘Bal’ :

    Die von mir gemeinte Welt (Lebenswelt) ist keine vom menschlichen Denken abhängige bloße „Vorstellung“, sondern Vorbedingung für jegliche Vorstellungen. [Quelle : hiesige Kommentatorik]

    Derartige ‘Vorbedingung’ ist ja von Ihnen oder von anderen zur Erkenntnis bereiten, befugten Subjekten geschaffen.
    Sie müssen, wie einige finden, dies beizeiten anerkennen.

    Nach ca. zehn Jahren, in denen sich Dr. Webbaer mit Ihnen beschäftigt hat, wird allerdings hier nicht mehr daran geglaubt, dass Sie es schaffen.
    Helmut Wicht ist aus diesseitiger Sicht zwar nett, aber auch vulgär, Dr. W bleibt hier schon gerne bei Timm Grams, der sozusagen noch netter ist.
    >:->

    Ischt ein Problem, diese Erkenntnis, die szientifische, die qua angehäufter Evidenz zu untermauern ist, Dr. W weiß dies.
    Amüsiert sich insofern eher zusammen mit Herrn Dr. Stephan Schleim (einstmals Numero 3 auf einem nur von Dr. W gepflegten Ranking), mit “Chrys” und natürlich auch ein wenig zusammen mit dem Genie “Joker”, a.k.a. “Jolly”, sometimes.

  184. @Balanus
    Sie schreiben:

    Der Naturalismus-Kritiker setzt (völlig unbegründet) voraus, dass es eine von der physikalischen Welt geschiedene nichtphysikalische Welt gibt, in der die mentalen Phänomene, die per Definition ja nichtphysikalische Phänomene sein sollen, zwangsläufig ihren Platz haben.

    Das trifft nicht meine Standpunkt. An keiner Stelle spreche ich von einer eigenständigen Welt des Mentalen. Ich habe nur die Begriffe des erlebten Bewusstseins der Diesseits-Klasse zugeordnet und diejenigen der physischen Bewusstseinsprozesse der Jenseits-Klasse. Das sind reine Sprachregelungen; sie implizieren keinerlei Ontologie. Eine Vergegenständlichung des Mentalen liegt mir genauso fern wie Ihnen.

  185. @KRichard 17.01. 16:05

    „Weil die Wahrnehmung bei Menschen aller Kulturen gleichartig funktioniert, werden ähnliche Erlebnisse von allen Kulturen berichtet. D.h. hiermit hat man eine kulturübergreifend gemeinsame biologische Grundlage für ´spirituelle´ Erfahrungen – wenn bestimmte Erlebnisse kulturell als spirituell gedeutet werden.“

    Wenn wir annehmen können, das Spiritualität sowieso zu den Eigenschaften der Wirklichkeit gehört, dann brauchen wir jedenfalls keine biologische Grundlage eventueller Fehlwahrnehmungen. Was nicht heißt, dass es diese nicht auch noch gibt.

    Dass die Naturwissenschaften keine Spiritualität kennen, ergibt sich daraus, dass sie danach einfach nicht gucken. Das Vorgehen, für irgendwelche Sachverhalte mathematische Formeln zu finden, ist so erfolgreich, dass hier versäumt wird, nach Effekten überhaupt zu suchen, die eben auch mal individuell sind, weil sie geistig bedingt sind.

    @Balanus 17.01 17:09

    „Nicht anders verhält es sich mit den Gefühlen und Wahrnehmungen eines lebenden Objekts: sie sind schlicht nicht (direkt) messbar.“

    Das eigene Bewusstsein und seine Rekonstruktion der Außenwelt ist doch zweifelsfrei und unmittelbar selbst-erfahrbar. Und weil es sich so konsequent neben der wirklichen Außenwelt zeigt, kommt man schnell auf die Idee: „1 Mentale Phänomene sind nichtphysikalische Phänomene.“ Das ist ziemlich gut denkbar, meine ich.

  186. » Nach ca. zehn Jahren, in denen sich Dr. Webbaer mit Ihnen beschäftigt hat, …«

    Ich fühle mich geehrt … 🙂

  187. Eine Vergegenständlichung des Mentalen liegt mir genauso fern wie Ihnen.

    Vgl. :
    -> https://scaleofuniverse.com (nette NASA-Animation, sie ist in einigen sog, Browsern heutzutage ausgeschlossen, wegen “Flash-Player”)
    -> https://www.youtube.com/watch?v=uaGEjrADGPA

    Der hier gemeinte Primat, der Mensch, ist sozusagen zur Sachlichkeit verdammt, weil er nicht anders fühlen kann.
    Die Vergegenständlichkeit, ‘Vergegenständlichung’ ist ihm auferlegt, sozusagen.

    Es gibt also schon Welten, die der Naturwelt nur näherungsweise, ausschnittsartig und an Interessen (!) gebunden, entsprechen, um dann idf näherungsweise, ausschnittsartig und an Interessen (!) gebunden theoretisiert zu werden.

    MFG

  188. @ “Bal” :

    Opi W mag Sie ebenso, Sie müssten aus seiner Sicht nur ausbauen, damit es mit den Nachbarn sozusagen klappt.

  189. @Balanus / 17.01.2021, 17:04 Uhr

    »Die von mir gemeinte Welt (Lebenswelt) ist keine vom menschlichen Denken abhängige bloße „Vorstellung”, sondern Vorbedingung für jegliche Vorstellungen.«

    Nein, da hast Du noch nicht hinreichend reflektiert, was Du sagst.

    Der Welt-Begriff meint in jedem Zusammenhang eine “Idee der Vernunft” (Kant), der eine rein regulative Funktion hat. “Welt” ist quasi ein Container-Begriff, der uns dazu dient, verschiedene Erkenntnis- oder Diskursbereiche zu definieren. Und zwar nicht, weil uns diese Bereiche so vorfindlich wären, sondern weil unser Verstand eine gewisse Ordnung in eine zunächst unstrukturierte Vielfalt von Sinneseindrücken bringen muss, um sich irgendwie zurechtzufinden.

    Ob Umwelt, Tierwelt, Arbeitswelt, Lebenswelt, Unterwelt, Begriffswelt, Weltraum, … nichts davon bezeichnet einen Gegenstand, den Du mir im Bedarfsfall zeigen könntest, wie Du mir nach einigem Nachdenken gewiss wirst bestätigen können. In der Terminologie der (naiven) Mengentheorie wären das Mengen, also vom Verstande konstituierte Zusammenfassungen von Gegenständen unser Wahrnehmung oder unseres Denkens zu einer begrifflichen Gesamtheit.

    Und Vorbedingung dafür für all das sind die diversen Sinneseindrücke — sowie hinreichend Verstand, um sich dann einen Reim auf diese Eindrücke zu machen.

  190. @Timm Grams // 17.01.2021, 17:42 Uhr

    »Das trifft nicht meine Standpunkt. An keiner Stelle spreche ich von einer eigenständigen Welt des Mentalen.«

    Ich eigentlich auch nicht (ich schrieb „geschiedene Welt“).

    Tatsächlich heißt es in Ihrem Beitrag:

    Da er [der Naturalist] nur eine Welt anerkennt, gibt es keine nichtphysikalischen Phänomene. Folglich kann es keine mentalen Phänomene geben.

    Das heißt doch im Umkehrschluss: Würde er nicht nur eine Welt anerkennen, sondern zwei oder viele, dann wäre auch Platz für (scheinbar) nichtphysikalische Phänomene wie Gedanken und Empfindungen, und er würde sich nicht „rettungslos verheddern, wenn es um das Geistige geht“.

    —-
    @Timm Grams // 17.01.2021, 16:54 Uhr

    Diesen Ihren Kommentar hatte ich leider übersehen, weil er zwischen unwichtige Wortmeldungen geraten war.

    Wenn Sie also anerkennen, dass die Ursache-Wirkungsrichtung von der Außenwelt (neuronale Aktivität) zur Innenwelt (Erscheinung, Vorstellung) geht, wo genau ist dann das Problem mit dem Naturalismus? Wie passt das zu Ihrer Naturalismus-Kritik?

    Offenkundig bin ich nicht der Einzige, der Ihren Beitrag nicht zur Gänze so versteht, wie er von Ihnen möglicherweise intendiert war.

  191. @Jeckenburger
    Die meisten Menschen haben manchmal Träume während des Schlafens. Wie ist das zu erklären? Es sind von der Realität zeitlich und empirisch losgelöste Erfahrungen. Sind es also nicht auch spirituelle Erfahrungen?

    DIe meisten Spekulationen oder Theorien von Antinaturalisten, Esoterikern, Spiritualisten usw. haben ganz banale Ursachen, Erklärungen oder Hintergründe, bis hin zu psychischen oder cerebralen Störungen. Man schaue sich Experimente der Wahrnehmungspsychologie, der Gestaltpsychologie und der Psychologie des Unbewussten an, sie sind erhellend und erstaunlich.

    Die derzeitigen Ereignisse in USA um die Erstürmung des Kapitols, die Leugnung der Wahlergebnisse und die QAnon-Verschwörungen zeigen deutlich die Narrheiten des menschlichen Bewusstseins. Die neurobiologische Funktionsweise des Bewusstseins ist bei allen Menschen im gesunden Zustand gleich, aber seine Inhalte …

    Ja, die mentalen Inhalte können losgelöst sein von jeder physischen oder außerweltlichen Realität! Es gibt keine allgemeinen Vorschriften zur Deutung des Weltgeschehens, das ist individuell und frei, manchmal von der sogenannten Erziehung oder der zwischenmenschlichen Kommunikation beeinflusst.

  192. Kommentatorenfreund ‘Chrys’ und hierzu kurz :

    Der Welt-Begriff meint in jedem Zusammenhang eine “Idee der Vernunft” (Kant), der eine rein regulative Funktion hat. “Welt” ist quasi ein Container-Begriff, der uns dazu dient, verschiedene Erkenntnis- oder Diskursbereiche zu definieren. Und zwar nicht, weil uns diese Bereiche so vorfindlich wären, sondern weil unser Verstand eine gewisse Ordnung in eine zunächst unstrukturierte Vielfalt von Sinneseindrücken bringen muss, um sich irgendwie zurechtzufinden.

    Die Welt meint das “Schalten und Walten” eines Betriebs, der für Weltteilnehmer erst einmal i.p. Verständnis außen vor steht.

    “Vernunft” ist so nicht gemeint, Vernunft dient der Beschreibung des Vorgefundenen

    Dies hier – ‘“Welt” ist quasi ein Container-Begriff, der uns dazu dient, verschiedene Erkenntnis- oder Diskursbereiche zu definieren. Und zwar nicht, weil uns diese Bereiche so vorfindlich wären, sondern weil unser Verstand eine gewisse Ordnung in eine zunächst unstrukturierte Vielfalt von Sinneseindrücken bringen muss, um sich irgendwie zurechtzufinden.’ – war natürlich (“geboren”) nett oder korrekt angemerkt und genau so auch von mir vorgestellt.

    Die Welt selbst muss nicht vernünftig sein, es ist der Verstand der ggf. entstehenden Subjekte, der Welten Sinn zuspricht, derart entwickelt.
    Ich rege in diesem Zusammenhang auch an Welten, erdachte Welten aus dem Hause Philip K. Dick und Terry Pratchett, auch magische Welten im Erfahrungsbereich, im theoretischen, zK zu nehmen, auch diese Welt kann denkmöglicherweise partiell magisch sein, also dem bloßen Willen von erkennenden Subjekten unterworfen, Stichworte : Doppelspaltversuch und Verschränkung, jeweils die Rolle des Beobachters meinend.
    Womit von mir im esoterischen Sinne nicht geworben werden soll, wenn so angemerkt worden ist.

    Die Welt ist, Kommentatorenfreund ‘Chrys’, auf vielfältige Art und Weise greifbar, wir bleiben hoffentlich bei der szientifischen Methode, andere Vorgehensweisern werden von uns beobachtet und bei Gelegenheit verdammt und bei besonderer Gelegenheit besonders begutachtet, auch eigenen möglichen Irrtum meinend, im Sinne des Skeptizismus.
    Kommentatorenfreund “Joker”, a.k.a. “Jolly”, ist womöglich der Einzige hier, der dies genau versteht.

  193. Das Zitat

    Da er [der Naturalist] nur eine Welt anerkennt, gibt es keine nichtphysikalischen Phänomene. Folglich kann es keine mentalen Phänomene geben.

    ist aus dem Zusammenhang gerissen. Es steht unter der Prämisse der ersten Aussage des Bieri-Trilemmas.

    Meine Naturalismus-Kritik setzt am Unabhängigkkeitspostulat an: Aufgrund der physiologischen Bewusstseinsprozesse kann die eine Welt nicht vom Bewusstsein unabhängig sein. Wohlgemerkt: Ich spreche hier nicht von den mit den Prozessen verbundenen Bildern (Erscheinungen).

  194. @ Balanus 17.01.2021, 17:09 Uhr

    Zitat: „1 Mentale Phänomene sind nichtphysikalische Phänomene.

    Mir scheint, dieser Satz ist falsch und wahr zugleich, je nachdem, was man unter ‚mental‘ und ‚nichtphysikalisch‘ versteht. “

    Das kann man genau so sehen wie Sie es formuliert haben.

    Eben weil die Begriffe „mental“ und „physikalisch“ unklar „deklariert“ sind. Anfänger in „Programmierung“ machen immer derartige Fehler, weil das im normalen Leben nicht so wichtig ist. Aber in Grenzfragen zwischen Materie und Geist (und auch Prozessen) kommt es auf absolut klare eindeutige „Deklarationen“ an.

    Die Philosophen wundern sich, dass sie in absurde Widersprüche geraten, wenn sie Fehler bei der „Deklaration“ machen, bzw. auf exakte „Deklarationen“ verzichten und einfach, letztlich unklare Begriffe einfach übernehmen.

    Mental ist zwar eher etwas Geistiges, kann aber so wie ich den Begriff deuten würde, nur in Verbindung mit etwas „wirklich materiellem“ (Molekülen) und in Verbindung mit Prozessen“ auftreten. Eine eindeutig Trennung in Prozessor – Prozess – Information ist, um Kategorienfehler zu vermeiden unbedingt nötig.

    Physikalisches braucht immer Materie, allenfalls Prozesse, allenfalls eine Beschreibung und eine Prozesssteuerung. Die Prozesse, Beschreibung und erst die Prozesssteuerung sind so etwas wie „selbstverständliches Beiwerk“, im normalen Leben keiner besonderen Erwähnung wert.

    Die Elektroniker arbeiten sozusagen an der „Front“, praktisch immer wollen sie die „Materie“ vom „Beiwerk“ trennen und dieses „Beiwerk“ getrennt, aber praktisch immer mit „physikalischen Werkzeugen und Methoden“ behandeln. Sie nutzen z.B. die Stimme und übertragen sie über Telefon- oder Radiotechnik, kein Stäubchen Materie wird mit übertragen, nur dass was Techniker eben „Information“ nennen.

    Mentales hat den Charakter von „Empfindungen im Gehirn“ ist vermutlich nicht nur rein geistige Information, benötigt unbedingt vermutlich ein biologisches Gehirn. Geistiges ist eher so deklariert dass es eben völlig von Materie frei ist.

    Deklarieren kann man übrigens was man will, nur logische Fehler (besonders auch Kategorienfehler) sollten nicht auftreten …..

  195. Hey, Kollego (flapsige Umgangsform bitte ich an dieser Stelle gleich zu entschuldigen, vielen Dank für Ihre Arbeit!), und hierzu kurz :

    ‘Meine Naturalismus-Kritik setzt am Unabhängigkkeitspostulat an: Aufgrund der physiologischen Bewusstseinsprozesse kann die eine Welt nicht vom Bewusstsein unabhängig sein.’


    Ist sich bereits einmal mit dem Konstruktivismus beschäftigt worden?
    Ich frage da mal, stets ein wenig dull, manchmal auch ein wenig (nur künstlich – vgl. bspw. mit meiner Handhabung von Kommentatorenfreund “Joker”, a.k.a. “Jolly”) künstlich aufgeregt bis apodiktisch an.

    MFG

  196. @Chrys // 17.01.2021, 18:25 Uhr

    » Der Welt-Begriff meint in jedem Zusammenhang eine “Idee der Vernunft” (Kant), «

    Ich war aber so frei, ein bisschen was anderes zu meinen. Nämlich das, wofür der Begriff steht, was er bezeichnet, was uns Anlass gab, diesen Container-Begriff zu prägen. Gibt es da was, oder gibt es da nichts, wofür der Begriff „Welt“ steht?

    »Ob Umwelt, Tierwelt, Arbeitswelt, Lebenswelt, Unterwelt, Begriffswelt, Weltraum, … nichts davon bezeichnet einen Gegenstand, …«

    Habe ich auch nicht behauptet. Aber es sind Gegenstände, auf die ich zeigen kann, die zu solchen Kategorien oder Klassen oder Konstrukte, wie auch immer, zusammengefasst werden.

    Schlussendlich geht es mir um die Vorbedingungen, ohne die diverse Sinneseindrücke gar nicht möglich wären, welche dann Vorbedingung für menschengemachte Kategorien sind.

    Und komme mir bitte nicht mit dem Einwand, dass ich ohne Sinneseindrücke überhaupt nicht über die Vorbedingungen der Sinneseindrücke nachdenken könnte. Das wäre allzu trivial.

    Und dann bleibt immer noch die Frage, was genau Timm Grams mit „Welt“ im Zusammenhang mit der einen Welt der Naturalisten gemeint hat.

  197. @ Balanus 17.01.2021, 17:09 Uhr
    @ Elektroniker 17.01.2021, 19:04 Uhr

    Ich möchte meinen Text ergänzen.

    Selbstverständlich mache ich den Philosophen keine Vorwürfe wegen der problematischen Deklaration, man weiß einfach zu wenig über Empfindungen und die zugehörigen Prozesse.

    Ich gehe kurzerhand von zwar naheliegenden, aber absolut unbewiesenen Voraussetzungen, was das Entstehen von Empfindungen betrifft, aus.

  198. Es gilt beizeiten definitorisch zu werden, Kommentatorenfreund “Bal”, ich habe bereits weiter oben versucht auf den Veranstaltungscharakter von Sprache, insbesondere auch auf die Altvorderen Bezug nehmend, aufmerksam zu machen, vgl. :

    -> https://www.etymonline.com/search?q=world

    Zum hier gemeinten “Container-Begriff” habe ich ebenfalls bereits weiter oben erklärt, Kommentatorenfreund “Bal”.

    Tatsächlich liegt die Antwort von Frage oft in der Bedeutung der Frage, nice1!

    Lernen Sie bitte in Schichten zu denken, Kommentatorenfreund “Bal”, bleiben Sie gerne skeptisch und versuchen insbes. auch Altvorderen im Sprachlichen zu folgen, behalten Sie oder entwickeln Sie erst : Phantasie.

    Danke, ich muss mich nun langsam ausklinken, Fressie naht, auch das Schlafbett, danke.

    Nie döfer sein als unbedingt nötig, Kommentatorenfreund “Bal”!

    MFG

  199. Anton Reutlinger,
    mit dieser ihrer Aussage kommen wir dem Phänomen Bewusstsein auf die Spur.
    “Die ungelöste Frage ist, wie kommt man von den Vorgängen im Gehirn zu den Phänomenen, die wir selber als Bewusstsein deuten. Es ist eben die Gesamtheit oder die Integration der Vorgänge, die uns als Bewusstheit – von Gedanken, Wahrnehmungen, Erinnerungen – und Bewusstsein erscheint. Schon diese beiden Begriffe sollte man differenzieren.
    Bewusstsein wäre demnach das Integral oder die Summe
    von Gedanken, Erlebnissen, Wahrnehmungen.
    Ist diese Vorstellung in sich logisch oder führt sie zu einem Widerspruch.
    Wir müssen dazu Georg Cantor bemühen, der sich mit solchen Gedanken beschäftigt hat. Er nennt die Summe von wohl definierten Dingen eine Menge.
    Und er stellt sich die Frage, kann es eine Menge geben, die alle Erfahrungen, Gedanken, Erlebnisse beinhaltet.
    Sein Urteil ist nein, nicht weil es an der Größe dem Umfang der Erfahrungen scheitert, sondern weil es eine Menge, die alles enthält nicht geben kann, weil sie offen bleibt. Die Elemente einer Menge kann man nämlich sortieren, zeitlich, räumlich oder nach Themen. Das Umgruppieren führt zu Teilmengen. Und wenn man die neuen Teilmengen abzählt, dann ist deren Summe höher als die Summe der Ausgangsmenge. Wenn ich also über meine Erlebnisse nachdenke erhöht sich der Speicherplatz innerhalb meines Gedächtnisses. Cantor sagte das natürlich anders. Er sagte, die Menge aller Mengen ist logisch unmöglich.
    Unser Bewusst sein als Menge aller Erlebnisse ist also logisch gesehen nicht möglich.
    Bertrand Russel hat diese Auffassung übernommen und damit auch die Widersprüche bei Paradoxien erklärt.
    Übertragen auf unser Bewusstseinsproblem heißt das, ein Bewusstsein anzunehmen als Summe aller Gedanken und Geschehnisse führt zu einem logischen Widerspruch, deshalb gibt es dieses Bewusstsein nicht.
    Ein überraschendes Ergebnis oder nicht.

  200. @Klaus Günther: Moralisierend

    Ich finde es arrogant und ignorantisch, neben der Natur eine weitere Welt der mentalen Phänomene zu postulieren, eine geistige Welt, die kategorial von der Natur getrennt existieren soll, obwohl sie doch nur in dieser und mit ihr erfahrbar ist.

    Geht es vielleicht auch eine Nummer kleiner? Etwas weniger moralisierend? Wir wollen hier schließlich Argumente austauschen.

    Wie halten Sie es eigentlich mit Abstrakta? Denken wir an den Kreis. In der natürlichen Welt gibt es beispielsweise keine perfekten Kreise. In unseren Gedanken und in der Mathematik aber schon. Oder noch abstrakter: Wahrheit. Gerechtigkeit. Schönheit.

    Wenn Sie von Materie und Energie sprechen, können Sie dann beispielsweise Wahrheit, Kreis, Gerechtigkeit, Schönheit und viel mehr in Materie und Energie definieren?

    P.S. Warum nur kommt Ihr Name mir so bekannt vor?

  201. @Hoppe: Kausalität und Physik

    Welches Naturgesetz wäre denn verletzt, wenn dort, wo ein Prozess nicht physikalisch determiniert ist, der Ausgang psychisch determiniert würde?

    P.S. Schön, dass du hier vorbeischaust.

  202. @Timm Grams

    Wortbedeutungen sind kontextabhängig. Der aufmerksame Leser sollte die richtige Auslegung finden.

    Von Skeptiker zu Skeptiker gefragt, was macht Sie so sicher, den Kontext der Wortbedeutungen immer hinreichend zu würdigen?

    Vor allem soll diese Welt – die Realität – „außerhalb unseres Denkens“ angesiedelt sein (Mahner, 2018, S. 46); sie ist folglich „in ihrer Existenz und ihren Eigenschaften unabhängig von unserem Bewusstsein“ (Vollmer, 2013, S. 22).

    Mein Verdacht ist, dass Sie hier speziell bei ‘Welt’, ‘Realität’, ‘außerhalb’ und ‘unabhängig’ nicht zur richtigen Auslegung gefunden haben, wenn Sie dann fortfahren,

    Mir widerstrebt zu erklären, dass das nicht zusammenpasst. Es ist zu offensichtlich.

    Mir ist es gelungen, eine Auslegung zu finden, in der sich alles völlig harmonisch zusammenfügt. Ich möchte ein Gleichnis benutzen, um diese zu beschreiben.

    Im Gebirge hat es einen Erdrutsch gegeben, der nun ein Tal versperrt. Der eben noch frei fließende Bach staut sich in der Folge zu einem See. Auf dessen Oberfläche spiegeln sich nun die Wolken und umliegenden Berge.

    Können Sie hier zustimmen, dass die Existenz und Eigenschaften der Wolken und Berge unabhängig von den Spiegelungen auf dem See sind, unabhängig davon, wie durch diese die Oberfläche des Sees affiziert wird?

    Einige Zeit später hat sich das Wasser einen Weg durch die Erdmassen freigespült, der See verschwindet. Nichts kann sich mehr an seiner Oberfläche spiegeln.

    Welchen Einfluss hat das auf die Existenz und Eigenschaften der Wolken und Berge? Können Sie zustimmen, dass es eine Welt ohne Spiegelungen im See gibt, ohne affizieren von dessen Oberfläche, dass es eine Realität ‘außerhalb’ von Spiegelungen gibt?

    Falls Sie jeweils zustimmen können, dann verschwindet auch gleich der Widerspruch, den Sie bisher gesehen haben. Denn auch der See und die Spiegelungen gehören zur Welt, bzw. zur Realität.

    Mit etwas gutem Willen lässt sich für alles ein Ort finden, auch für das Mentale.

  203. @hwied
    Ob Unterbewusstssein oder Bewussstsein – unser Gehirn arbeitet immer gleich.

    Eine bewusste Wahrnehmung (Bewusstsein) entsteht dann, wenn bestimmte Aktivtätsschwellen überschritten werden. z.B.
    – wenn sich das Gehirn auf die Verarbeitung eines Reizes konzentriert
    – weil sich ständig neuronale Aktivierungswellen über den Cortex bewegen (wie eine LaOla-Welle). (Dadurch kann es zu spontanen Ideen kommen (Tagtraum) – die uns wegen dieser Aktivierung bewusst werden)

  204. @Schleim
    Die Frage ist,wo finden wir es IN Materie und Energie,sondern DURCH.
    Was ist mit Geschmack und Geruch und dadurch verursachte emotionale mentale Zustände?

  205. Was ist Bewusstsein 5. Denkansatz

    Wir gehen einmal vom öffentlichen Bewusstsein aus, was man auch als öffentliche Meinung sehen kann. Die hängt zusammen mit der Kultur einerseits und den aktuellen politischen Zuständen andererseits.
    Damit muss man rechnen wenn man ein erfolgreicher Politiker sein will.
    Bewusstsein als Arbeitsbegriff !

    Und wir betrachten die 100 wichtigsten Persönlichkeiten der Weltgeschichte, und wir versuchen zu extrahieren, warum diese Persönlichkeiten so erfolgreich waren. Der Extrakt müsste dann erahnen lassen, was Bewusstsein und was öffentliches Bewusstsein ist.
    Hier die 10 wichtigsten:
    1. Mohammed 2. Newton, 3. Jesus 4. Buddha 5. Konfuzius 6. Paulus 7. Cai Lun 8. Gutenberg 9. Kolumbus 10-Einstein

    Adolf Hitler kommt auf Platz 39 und Beethoven auf Platz 45.

    Was ist den ersten 10 Persönlichkeiten zu eigen ?
    Sie haben der Menschheit etwas hinterlassen. Und das waren in der Mehrzahl geistige Güter.
    Geistige Güter überdauern die Zeiten.
    Oder anders formuliert, das Geistige wird höher geschätzt als materielle Dinge.

    Wenn also eine Korrelation besteht zwischen einem einzelnen Menschen und der Menschheit, dann ist sie geistiger Natur. Der Geist ist die Klammer, die die Kultur begründet.
    Man kann auch sagen , das Bewusstsein begründet die Kultur.
    Die 100 – Wikipedia

  206. @Joker
    Auch ich meine, dass die Welt (die Realität) fast immer ohne mein Bewusstsein existiert und folglich unabhängig davon ist. Aber nur “fast immer”. Sowie ich bin und darüber nachdenke, gehören die physiologischen Korrelate meiner. Gedanken zur Welt. Dann ist es mit der Unabhängigkeit vorbei.

  207. Timm Grams,
    zur realen Welt gehören auch die Menschen. Und die Menschen verändern die Welt z.B. durch Umweltzerstörung.
    Man kann diesen Prozess nur aufhalten, wenn Jeder Einzelne sein Bewusstsein überprüft und danach handelt. Dann muss die Gesellschaft als Ganzes ihr Bewusstsein überprüfen und Folgerungen daraus ableiten.
    Man sollte die Geschichte der Osterinseln in den Lehrplan der Schulen aufnehmen. Dort hat sich eine Kultur selbst ausgerottet.

  208. @hwied
    Die Gesamtheit unserer Wahrnehmungen, Erfahrungen, Erinnerungen im Verlauf des Lebens formt unser Bewusstsein. Die Veränderungen des Bewusstseins können verändertes Verhalten und im Feedback der Erfahrungen wieder Veränderungen des Bewusstseins hervorbringen. Das sind einfache, empirische Wahrheiten.

    Neben dem Bewusstsein gibt es ein Unterbewusstsein und ein Unbewusstsein. Nur ein kleiner Teil unserer Lebenserfahrungen wird uns voll bewusst. Trotzdem ermöglicht das Bewusstsein die notwendige Anpassung des Lebens an die dynamischen Veränderungen der Lebenswelt, einschließlich unseres eigenen Körpers.

    Für das Verständnis des Bewusstseins ist es unerlässlich, es zu zerlegen in die Funktionsweise und die phänomenalen Inhalte. Die Funktionsweise wird eines Tages naturwissenschaftlich und neuropsychologisch enträtselt werden. Für die phänomenalen Inhalte gilt sehr wahrscheinlich das “ignorabimus”, denn dafür ist keinerlei Kausalität erkennbar.

    Ebenso wenig können wir erklären, warum die Welt in Bewegung ist. Außer Bewegung von Körpern gibt es physikalisch Druck und Spannung zwischen Körpern mit Verformungen derselben. Wir wissen, dass die Kräfte der Physik Bewegungen oder Verformungen hervorbringen und wir können sie messen und berechnen, aber das Warum bleibt unerklärbar. Die Kausalität als physikalische Erklärung ist eine Krücke für die Unerklärbarkeit.

  209. Anton Reutlinger,
    allgemeine Zustimmung.
    Gut ist die Trennung von Bewusstsein als Container und Bewusstsein als Inhalt des Containers.
    “Für die phänomenalen Inhalte gilt sehr wahrscheinlich das “ignorabimus”, denn dafür ist keinerlei Kausalität erkennbar.”
    Da machen Sie es sich zu einfach.
    Beim “Lebenswille, Überlebenswille” ist auch keine Kausalität erkennbar, der ist einfach vorhanden, wie der Mensch einfach vorhanden ist.
    Bei den Naturalisten ist das ganze Universum einfach vorhanden.
    Sie haben es netter erklärt, dieser Abschnitt war für die anderen Mitleser gedacht.
    Gut,, dass wir eine klare Frontlinie gefunden haben.

    Wenn wir nämlich auf die Wirkung des Bewusstseins zu sprechen kommen, dann hat der irrationale Anteil in der Vergangenheit die größere Wirkung gezeigt.
    Jetzt, wo sich die naturwissenschaftliche Denkweise durchsetzt, da verkümmert der moralische Anteil am Bewusstsein. Und der Mangel an Moral, sprich Mangel an Verantwortungsbewusstsein der Umwelt gegenüber. der wird sich fataler auswirken als der Nutzen des naturwissenschaftlichen Denkens.

  210. @Timm Grams

    Aber nur “fast immer”

    Der schlichte Gebrauch des Wortes ‘unabhängig’ impliziert eben nicht, was Sie unterstellen, dass die Unabhängigkeit ‘immer’ und überall gegeben sein muss, dass sie absolut sein muss, es keinerlei Wechselwirkung gibt (@Balanus: oder Wirkung in eine Richtung).

    Elternunabhängiges BAFÖG bedeutet nicht, dass die Steuern der Eltern nicht zu dessen Finanzierung herangezogen werden. Wenn ich unabhängig von meiner Freundin in Urlaub fahre, schließt das nicht aus, dass ich manchmal an sie denke oder sogar mit ihr telefoniere chatte.

    Mehr ist auch nicht gesagt, wenn Naturalisten von einer vom Denken und vom Bewusstsein unabhängigen Existenz der Dinge sprechen.

  211. @hwied
    Der Überlebenswille ist eine Fiktion, ein willkürlicher Begriff für eine subjektive Verhaltensweise in bestimmten Situationen. Das kann Flucht oder Kampf oder etwas anderes sein. Mit den phänomenalen Inhalten des Bewusstseins, die bei allen Menschen vorhanden und erfahrbar sind, ist das nicht zu vergleichen.

    Die Diskussionen um Bewusstsein kranken vor allem an missverständlichen Begriffen und individuell verschiedenen Deutungen, sowie an subjektiven Voreinstellungen, insbesondere Dualismus oder Monismus. Der Dualismus ist so gut wie widerlegt.

  212. @KRichard

    wenn sich das Gehirn auf die Verarbeitung eines Reizes konzentriert

    Die Konzentration des Gehirns, muss ich mir die so vorstellen, wie die Konzentration einer starken Base?

    die uns wegen dieser Aktivierung bewusst werden

    Müssten Sie nicht schreiben, um die Konsistenz der Base ihrer Aussagen zu bewahren, die unserem Gehirn bewusst werden?

  213. @anton reutlinger

    Die Diskussionen um Bewusstsein kranken vor allem an missverständlichen Begriffen und individuell verschiedenen Deutungen

    Zustimmung. Das wird durch ihre Deutungen ja immer wieder eindrucksvoll belegt.

    Der Dualismus ist so gut wie widerlegt.

    Der Dualismus kann nicht widerlegt werden. Es handelt sich dabei um eine metaphysische Annahme.

    Der Glaube daran kann allerdings aussterben. Davon sind wir wiederum noch weit entfernt.

  214. @Stephan Schleim:

    wo ein Prozess nicht physikalisch determiniert ist

    Du meinst mit “Prozess” einen materiellen/physischen Vorgang? Dann erklärt er sich aus (bekannten) physikalischen Gesetzmäßigkeiten.

    Der Begriff “determiniert” kommt so in der Physik gar nicht vor. Die Physik behauptet einfach die Gültigkeit ihrer Erkenntnisse für alle materiellen/physischen Prozesse.

    In bestimmten (unfassbar komplexen) Fällen beschreiben wir derartige Vorgänge mit psychologischen Termini, rekurrieren z.B. auf Tatmotive usw. Das ändert aber nichts daran, dass die dabei stattfindenden physischen Vorgänge in der Physik bereits ihre vollständige prinzipielle Erklärung gefunden haben.

    Hinsichtlich materieller/physischer Vorgänge bleibt kein unerklärlicher Rest, der dann z.B. psychologisch erklärt werden könnte.

  215. @anton reutlinger 17.01. 18:45

    „Die meisten Spekulationen oder Theorien von Antinaturalisten, Esoterikern, Spiritualisten usw. haben ganz banale Ursachen, Erklärungen oder Hintergründe, bis hin zu psychischen oder cerebralen Störungen.“

    Was nicht davon abhält, die Welt anders zu deuten, wenn man aufgrund wirklich spiritueller Erfahrung manches eben anders einordnet.

    „Ja, die mentalen Inhalte können losgelöst sein von jeder physischen oder außerweltlichen Realität! Es gibt keine allgemeinen Vorschriften zur Deutung des Weltgeschehens,…“

    Eben, man muss sich in vielen Sachfragen gar nicht viel streiten, man kann es entweder so und so sehen, oder man kommt immer zur selben Antwort, wenn man ehrlich ist.

    @hwied 18.01. 11:31

    „Wenn wir nämlich auf die Wirkung des Bewusstseins zu sprechen kommen, dann hat der irrationale Anteil in der Vergangenheit die größere Wirkung gezeigt.“

    Und das ist wohl immer noch so. Ohne dass es ein Problem sein muss. Egal inwieweit man sich Selbst und Teile des Geschehens dieser Welt als geistig oder nicht erlebt, so bleiben wir dabei immer noch in den Gefilden des Denkbaren, sonst wären wir dazu ja nicht fähig.

    Vielleicht ist hier die Einstellung gegenüber dem Anderen viel wichtiger, ob hier Redlichkeit und Rücksichtnahme dominiert, oder ob man den Anderen sozusagen abschlachtet, ob nun nur geistig oder auch physisch, weil man meint, mit ihm nichts zu tun zu haben. Und was letztlich zwischenmenschlich angeraten ist, das lässt sich auch auf den Umgang mit anderen Spezies wie mit der ganzen Natur übertragen.

    Das ist doch eher unabhängig davon, wie spirituell oder auch gar nicht spirituell man hier versucht, seine Welterfahrung zu deuten. Aber man muss sich auch selber da abholen wo man selber gerade ist, eine gewisse Sturheit hilft in der Lebenspraxis, und ist öfter notwendig.

  216. @Joker
    Mit ´Konzentration´ ist gemeint: worauf das Gehirn den Fokus seiner Aufmerksamkeit richtet

    suchen Sie per Google [Gorilla-Experiment]
    Versuchspersonen bekamen einen Film mit hell bzw. dunkel gekleideten Basketballspielern gezeigt – und sollten z.B. die Ballkontakte des dunklen Teams beim Spiel zählen. Filmdauer 1 Min.
    Zusätzlich zu den Basketballspielern ging eine als Gorilla verkleidete Person in die Mitte des Spielfelds, stellte sich direkt vor die Kamera, klopfte sich auf die Brust und verließ das Bild.

    Ein Großteil der Versuchspersonen sahen diesen Gorilla nicht, obwohl er quer über das Spielfeld ging und sich direkt vor die Kamera stellte.

    Gehirn/Aufmerksamkeit der Versuchspersonen hat sich so darauf konzentriert die Ballkontakte zu zählen – dass andere Sinneseindrücke nicht bemerkt wurden.
    Dieses Beispiel zeigt, dass wir nur wahrnehmen worauf sich unsere Aufmerksamkeit konzentriert und aber auch dass unsere Wahrnehmung extrem fehlerhaft ist

    Auch dies ist ein schönes Beispiel dafür, wie fehlerhaft unsere Wahrnehmung ist. Uns stören solche Fehler aber nicht – wie ich in früheren Beiträgen (Bewegungwahrnehmung beim Kinofilm = 100 % Fehler) schon erwähnt habe.

  217. Jeckenburger,
    “üb immer Treu und Redlichkeit”, so hat man früher gesungen.

    Reutlinger,
    vielen Dank für ihre vielen ausführlichen Antworten. Im Prinzip liegen wir nicht weit auseinander. Damit verabschiede ich mich.

  218. @Joker
    Der Dualismus kann nicht widerlegt werden. Es handelt sich dabei um eine metaphysische Annahme.

    Auch eine metaphysische Annahme muss begründet werden, sonst ist sie wissenschaftlich bedeutungslos und sinnlos. Die Begründung kann u.U. logisch widerlegt werden. Ist sie nicht widerlegbar oder falsifizierbar, dann ist es bloß religiöser Glaube.

  219. @ Christian Hoppe 18.01.2021, 13:21 Uhr

    Zitat: „Das ändert aber nichts daran, dass die dabei stattfindenden physischen Vorgänge in der Physik bereits ihre vollständige prinzipielle Erklärung gefunden haben.
    Hinsichtlich materieller/physischer Vorgänge bleibt kein unerklärlicher Rest, der dann z.B. psychologisch erklärt werden könnte.“

    Die Aussage „vollständige prinzipielle Erklärung“ finde ich nicht vollständig, damit nicht völlig korrekt.

    Beispiel: Ein absolut gesunder Junge fällt in die Hand einer „Kriegerbande“ und wird psychisch völlig ruiniert, zu einem „Killer dressiert“ und das ist er danach auch.

    Die psychischen Vorgänge und Mechanismen die dabei wirksam werden gehören zum Fachgebiet der Psychologie. Für Physiker wäre dies naheliegend, aber ihr Fachgebiet wäre es zweifellos nicht.

    Es stimmt offensichtlich, dass der erzieherische Einfluss die neuronalen („physikalischen“) Strukturen verändert hat und er deswegen falsch handelt. Aber der entscheidende Grund ist praktisch die Erziehung.

  220. @KRichard

    worauf das Gehirn den Fokus seiner Aufmerksamkeit richtet

    Erstaunlich, dass es Ihrer Aufmerksamkeit entgangen ist, worauf ich hinweisen wollte: Es ist Ihre Aufmerksamkeit, nicht die ihres Gehirns, Sie richten den Fokus aus, nicht ihr Gehirn.

    Auch dies ist ein schönes Beispiel dafür, wie fehlerhaft unsere Wahrnehmung ist. Uns stören solche Fehler aber nicht.

    Mich stören solche Fehler.

    (Ich hoffe, Sie können die Bezüge für ‘dies’ uns ‘solche’ fehlerfrei zuordnen. Falls Sie unsicher sind, bitten Sie einfach ihr Gehirn um Hilfe.)

  221. anton reutlinger
    hwied

    Ich stimme auch, außer im Punkt der getrennten Hardaware -Software, oft mit Herrn Reutliger überein.

    Auf irgend eine Art habe ich praktisch mein gesamtes Berufsleben davon gelebt, „Materie“ vom „informellen Beiwerk“ zu trennen und nur dieses „Beiwerk, die Information“ getrennt, aber praktisch immer mit „physikalischen Werkzeugen und Methoden“ zu behandeln.

    Nachrichtentechniker/Elektroniker nutzen z.B. die Stimme und übertragen sie über Telefon- oder Radiotechnik, kein Stäubchen Materie wird mit übertragen, nur dass was Techniker eben „Information“ nennen.

    Es wird praktisch bestritten, was ich in meinem Berufsleben gemacht habe, davon zu leben dass Information von der Materie getrennt werden kann.

    Allerdings mussten Berufskollegen vor rund 60 Jahren auch noch damit rechnen, wegen Betruges eingesperrt zu werden, weil derartiges unmöglich schien.

  222. @Elektroniker
    Es wird praktisch bestritten, was ich in meinem Berufsleben gemacht habe, davon zu leben dass Information von der Materie getrennt werden kann.

    Eine konkrete Information “Biden wird Präsident” kann von einer Materie auf eine andere kopiert werden, sogar mehrfach oder über mehrere Stufen und auf verschiedene Arten von Materie. Das geschieht alltäglich über die Medien oder durch Kopieren von Daten oder Software. Aber die Information wird eben nur von einer Materie auf eine andere übertragen, z.B. über Licht und die Augen in das Gehirn, aber sie wird nicht von Materie getrennt. Im Original bleibt sie dabei sogar erhalten. Ohne Materie/Energie kann Information nicht dargestellt, nicht übertragen und nicht gespeichert werden.

    Das ist eine Widerlegung des metaphysischen Dualismus.

  223. @Balanus / 17.01.2021, 19:11 Uhr

    »Und komme mir bitte nicht mit dem Einwand, dass ich ohne Sinneseindrücke überhaupt nicht über die Vorbedingungen der Sinneseindrücke nachdenken könnte. Das wäre allzu trivial.«

    Empirisch forschen kann nur, wer auch beobachten kann, d.h., wer beispielsweise die Zeigerstellung eines Messinstruments als Sinneseindruck wahrnehmen kann. Insofern ist die Befähigung zu Sinneseindrücken doch eine methodische Vorbedingung für jegliches Betreiben von empirischer Wissenschaft.

    Mein Urteil «Je vois» ist synthetisch a priori, vergleichbar mit Descartes’ Urteil «Je pense». Die wissenschaftl. Feststellung, dass ich Augen habe, die mir das Sehen ermöglichen, ist hingegen synthetisch a posteriori und könnte sich sogar als falsch erweisen. Nämlich falls sich irgendwann herausstellen sollte, dass ich nur so etwas wie ein “Brain In a Vat” ohne Sinnesorgane bin, dem der Eindruck, einen Körper mit Augen zu haben, nur als eine Simulation suggeriert wird.

    Wenn ich unversehens einen elektr. Viehzaun berühre, dann vermittelt mir das einen Sinneseindruck und ich zeige eine Reaktion, von der man dann vielleicht sagen kann, dass sie mit der gleichen Zwangsläufigkeit von Reiz-Reaktions-Mechanismen erfolgt, durch die auch das Zucken der Beine eines toten Frosches beim Anlegen einer elektr. Spannung erklärt werden kann. Mein subjektiver Sinneseindruck wird für diese Erklärung nicht gebraucht, er ist für jemand anderen als mich auch gar nicht feststellbar. Umgekehrt liefert mir diese Erklärung aber auch keinerlei Einsichten über das Zustandekommen meines Sinneseindrucks. Doch es besteht keinerlei Widerspruch zwischen subjektiver und intersubjektiver Sicht, das ist miteinander kompatibel. Wo also sollte da nun eigentlich ein Problem sein? Für einen Naturalisten wie Dennett ist da offenbar eines, weshalb er die subjektive Sicht dann unbedingt “quinen” will — erscheint Dir das vernünftig?

  224. @Joker
    Wenn Sie die fehlerhafte Arbeit des Gehirns stört – dann sollten Sie z.B. keinen Kinofilm ansehen: dass wir dort Bewegung ´sehen´ ist ein 100%iger Fehler

    Sie können auch bei Wikipedia [Erinnerungsverfälschung] dazu nachlesen, wie leicht sich unser Gedächtnis manipulieren lässt.

    Es gibt zahlreiche Experiemente, welche zeigen, wie fehlerhaft unsere Wahrnehmung, unsere Erinnerung und sogar unsere Ich-Identität sind.

    Je mehr man sich über die Arbeitsweise unseres Gehirns informiert um so mehr Fehler entdeckt man. Das Gehirn ist dazu da, dass wir irgendwie weiterleben können – Fehler sind dabei egal; ganz gleich wie groß/schwerwiegend sie sind.

  225. @Timm Grams // 18.01.2021, 09:54 Uhr

    » Sowie ich bin und darüber nachdenke, gehören die physiologischen Korrelate meiner. Gedanken zur Welt. …«

    Das würde wohl kein Naturalist bestreiten wollen.

    »…Dann ist es mit der Unabhängigkeit vorbei. «

    Wie das?

    Ich sehe nicht, inwiefern die Perzeption der empirisch zugänglichen Realität, die zwangsläufig mit neuronaler Aktivität verbunden ist, in irgendeiner Weise eine Abhängigkeit der Realität von eben diesen neurophysiologischen Prozessen begründen könnte.

    (@Joker: Noch nicht mal hin und wieder.)

    Aber vielleicht übersehe ich ja auch etwas, nur was?

  226. @Chrys

    Ok, ich kucke mir Dennetts Aufsatz gerne mal an, dann bilde ich mir ein Urteil… 😉

  227. Der Körper Geist Dualismus beruht in Wirklichkeit auf der Verwechslung der Perspektiven. Spreche ich vom Körper, dann bewege ich mich im Bereich der Physik, Chemie oder Biologie. Spreche ich vom Geist, bewege ich mich im Bereich der Psychologie oder Philosophie. Ich thematisiere also zwei unterschiedliche Blickwinkel auf ein und denselben Gegenstand und meine, es wären zwei unterschiedliche Welten. Der Dualismus findet also im Kopf des Betrachters statt, nicht im Gegenstand.

  228. @anton reutlinger

    Auch eine metaphysische Annahme muss begründet werden, sonst ist sie wissenschaftlich bedeutungslos und sinnlos.

    Da stimme ich zu. Das wird sicher auch in den Geisteswissenschaften so gesehen. Ich möchte nur ergänzen, dass selbst begründete metaphysische Annahmen naturwissenschaftlich nicht unbedingt eine Bedeutung erlangen.

    Ist sie nicht widerlegbar oder falsifizierbar, dann ist es bloß religiöser Glaube.

    In diesem Satz verquirlen sie wieder mal so viel, dass ich ihn weniger zu den falschen, sondern eher zu den bedeutungslosen und sinnlosen zählen möchte.

    Widerlegbar ist eine einzelne Annahme meistens auch dann nicht, wenn sie sinnvoll erscheint. Man kann in der Regel nur einen logischen Widerspruch zwischen mehreren Annahmen aufzeigen, wie es am Bieri-Trilemma verdeutlicht wird, oder wie das der Autor dieses Textes für den Naturalismus und dessen metaphysischen Annahmen versucht.

    Nicht falsifizierbar zu sein, ist etwas, was andere Annahmen von einer naturwissenschaftlichen unterscheidet. Mit dem ‘sie’ beziehen Sie sich aber gerade auf eine solche, eine metaphysische Annahme. Da lese ich also: ‘Wenn die Annahme nicht das ist, was sie ist, ist sie bloß religiös.’ Das wirkt abstrus.

    Glaube ist beteiligt, wenn man sich eine metaphysische Annahme zu eigen macht, wie z.B. den Monismus oder den Dualismus, oder an der Entscheidung, welche Aussage des Bieri-Trilemmas man verneint. Aber nicht jeder Glaube ist religiös, auch nicht in den Geisteswissenschaften. So gibt es für religiösen Glauben z.B. die Forderung nach Widerspruchsfreiheit nicht (siehe allmächtig oder allgütig).

    HTH

  229. @Christian Hoppe 18.01. 13:21

    „Hinsichtlich materieller/physischer Vorgänge bleibt kein unerklärlicher Rest, der dann z.B. psychologisch erklärt werden könnte.“

    Wenn man über entsprechende spirituelle Erfahrungen verfügt, dann findet man doch unerklärliche Reste. Auch die Parapsychologie hat solche Reste längst gefunden, wird aber einfach ignoriert.

    „In bestimmten (unfassbar komplexen) Fällen beschreiben wir derartige Vorgänge mit psychologischen Termini, rekurrieren z.B. auf Tatmotive usw““

    Das sind hier keine Ausnahmen, sondern das ist fast die Regel, wenn es um psychische Fragen geht. Insbesondere kann von der Ebene der Nervenzellen her nur sehr wenig konkret erklärt werden, weil das Konnektom noch nicht entschlüsselt ist. Darauf einfach vorzugreifen, und so zu tun, als wüssten wir hier schon alles und es ist 100% sicher, dass sich die naturalistische Sichtweise fraglos bestätigen wird, das ist jetzt wirklich voreilig, meine ich.

    Ich kann mir sogar vorstellen, dass dieses Konnektom Beweise für einen geistigen Anteil unseres Bewusstseins liefern kann, oder eben den Beweis, dass hier wirklich alles rein materiell ist, was da im Kopf passiert. Wie kann man da so einfach vorgreifen? Warten wir mal ab, was zukünftige Forschung bringt.

    Nochmal das Trilemma von Bieri:

    1. Mentale Phänomene sind nichtphysikalische Phänomene.
    2. Mentale Phänomene sind im Bereich physikalischer Phänomene kausal wirksam.
    3. Der Bereich physikalischer Phänomene ist kausal geschlossen.

    Der Naturalismus meint, 1. muss falsch sein, währen der spirituell Orientierte meint, 3. muss falsch sein.

    Wo ist hier das Problem, wir haben noch keine Beweise für beides nicht, aber die Perspektive, dass wir sie noch bekommen werden. Ein vollständiges, im Computer simuliertes menschliches Konnektom wird unsere Frage beantworten können, meine ich. Vielleicht reicht sogar ein Mäusekonnektom, um diese Frage zu klären. Das könnten wir sogar selbst noch miterleben. Die Geduld habe ich noch, darauf zu warten. Und ich muss mich da auch jetzt nicht drauf festlegen.

    Tippen tue ich eher auf die geistige Variante, dass das ganz ohne geistige Aspekte nicht funktionieren kann, einfach weil das so gut zu meiner Lebenserfahrung passt. Ist noch Glaubenssache, kann aber bestätigt oder auch nicht bestätigt werden.

  230. @Stephan Schleim

    Zugegeben, ich habe mich durch die Formulierungen von Grams, besonders der ersten drei Absätze nach dem Zwischentitel „Sackgassen“, provozieren lassen und etwas unangemessen geantwortet. Das tut mir Leid.

    Es ändert aber nichts daran, dass ich die betreffenden Ausführungen falsch finde. Von der Annahme nur einer Welt auszugehen ist m.E. gerade keine Metaphysik, vielmehr ist im Gegenteil die Postulierung einer Welt hinter der physischen meta ta physika einzuordnen. Damit kommt man nicht nur zu falschen Schlussfolgerungen, sondern enthebt sich auch der Mühe nachzuvollziehen, wie im Lauf der Evolution zu den physischen Phänomenen der Lebewesen unserer einen Welt notwendig die mentale Dimension hinzugekommen ist.

    Ob Sie sich einen idealen Kreis vorstellen oder Ihren realen Schreibtisch, ob Sie einen Sachverhalt bewerten oder von einem Ereignis emotional berührt werden, in allen Fällen handelt es sich um neuronale, mithin physische, energetische Vorgänge in ihrem Gehirn, immer von körperlichen Reaktionen begleitet, die von unmerklich bis erschütternd reichen können.

    Es gibt keine Information ohne einen realen Träger. Denn eine solche müsste man als „reinen Geist“ auffassen; dieser wäre jedoch nirgendwo anders zu verifizieren als in Ideen, also philosophischen Denkfiguren – die ihrerseits aber immer höchst materieller Träger bedürfen.

    Ich wüsste nicht, dass wir schon einmal konferiert haben; ich hatte einmal einen Leserbrief in „Spektrum“ und äußere mich ansonsten nur in einem einzigen anderen Forum.

  231. Eine geistige Welt gibt es nicht. Die Materie/Energie bringt Information hervor, ohne sie gibt es keine Information und keinen Geist. Information ist die nichtzufällige Anordnung von Elementen der Materie/Energie, die ggf. auf die Sinnesorgane einwirkt und schließlich auf die Nervenzellen. Nicht die Information bestimmt die Anordnung, sondern die Anordnung bestimmt die Information.

    Die Oberfläche von Körpern der Außenwelt emittiert Licht unterschiedlicher Wellenlängen, das auf die Netzhaut einwirkt, Nervensignale erzeugt und darüber die Nervenzellen affiziert. Es gibt keine Interaktion zwischen mentalen und physikalischen Prozessen oder Zuständen, weil es keine mentalen Prozesse gibt. Im Lauf des Lebens lernen wir bestimmte Anordnungen zu erkennen und zu verstehen, z.B. als Schrift oder Sprache, als mentale Prozesse.
    Mentale Prozesse sind nichts anderes als physikalische Prozesse der Transformation solcher Anordnungen!

  232. @anton reutlinger

    Ich möchte mich korrigieren, obwohl @KRichard meint, Fehler seien egal; ganz gleich wie groß/schwerwiegend sie sind.

    Ich lese also bei Ihnen:
    ‘Wenn die Annahme das ist, was sie ist, ist sie bloß religiös.’ Das wirkt abstrus.

    Sie werden es vielleicht schon bemerkt haben, da war ursprünglich ein ‘nicht’ zu viel. Mein Gehirn bittet um Entschuldigung.

    Lebbe geht weider. (Stepanovic)

  233. @Hoppe: Determinismus in der Physik

    Es ging um die Frage, nun in meinen Worten formuliert, ob die Physik Raum für mentale Verursachung lässt. Meine Frage war eher rhetorischer Art: denn natürlich können wir nicht ausschließen, dass dort, wo physikalische Prozesse indeterministisch sind (bsp. radioaktiver Zerfall, Doppelspaltversuch, Schrödingers Katze), eine von uns noch nicht verstandene Größe die Vorgänge determiniert. Irgendwann zerfällt das Atom ja, das einzelne Photon fliegt nach links oder rechts, die Katze ist (wenn wir nachschauen) lebendig oder tot.

    In diesem Zusammenhang darf man durchaus noch einmal an Einsteins beflügeltes Wort denken, dass Gott nicht würfelt.

    Jetzt meinst du, Determinismus (bzw. “der Begriff determiniert”) sei in der Physik kein Thema. Das stimmt wirklich nicht: Sowohl in der Klassischen Mechanik, als auch in der Allgemeinen Relativitätstheorie und in der Quantenmechanik geht es (auch) darum. Für eine Übersicht verweise ich auf: The Status of Determinism in Physical Theories (Stanford Encyclopedia of Philosophy)

    In der Wissenschaftstheorie hat sich beispielsweise John Earman ausführlich mit dem Thema beschäftigt, meines Wissens zuletzt in seinem Aufsatz: Aspects of Determinism in Modern Physics (2007, erschienen in: Handbook of the Philosophy of Science, Volume 2: Philosophy of Physics, pp. 1369-1434).

  234. @Jeckenburger

    Ich sprach ausschließlich von den materiellen/physischen Vorgängen. Diese mögen einem psychischen Phänomen (z.B. einer Handlung) zugrunde liegen oder nicht. Das ändert nichts daran, dass alles was dort physisch geschieht in der Physik bereits seine vollständige Erklärung gefunden hat. – Physische Vorgänge können Sie durch Hinweis auf mentale Phänomene genau dann überhaupt nicht mehr erklären, wenn Sie sich diese mentalen Phänomene nicht von vornherein als physisch realisiert denken.

    Jede psychisch bedeutsame Situation, jedes psychische Phänomen umfasst materielle/physische Vorgänge – diese sind bereits hinreichend erklärt. Der subjektive Rest ist faszinierend, auch sicherlich noch nicht erklärt, aber er hat *als solcher* keine kausale Wirkung. Das ist auch der Grund, warum wir dem anderen seine Qualia nicht ansehen können: *als solche* wirken sie nicht kausal auf uns ein. Wir sehen immer nur das materielle/physische Verhalten unseres Gegenübers.

    Daher muss uns Naturwissenschaftler das philosophische Qualia-Problem in keiner Weise beunruhigen. (Die einseitige Abhängigkeit des Erlebens von hirnmateriellen Vorgängen halte ich für erwiesen – auch wenn hier einige im Forum offensichtlich den Parawissenschaften anhängen.)

  235. @Klaus Günther: Erkenntnis

    …in allen Fällen handelt es sich um neuronale, mithin physische, energetische Vorgänge in ihrem Gehirn…

    Vielleicht stimmt das. Tatsache aber ist, dass wir diese Vorgänge (wie Gedanken, Gefühle usw.) nicht als “neuronale, mithin physische, energetische Vorgänge” beschreiben können.

    …die ihrerseits aber immer höchst materieller Träger bedürfen.

    Hier setzen Sie schlicht voraus, was zu zeigen wäre: dass es nämlich nichts ohne “materiellen Träger” geben kann (in der Philosophie nennt man diesen Argumentationsfehler “petitio principi” oder “question begging”). Im Übrigen scheint Materie doch gar nicht das grundlegende Prinzip in der Physik zu sein.

    Insofern kann ich Ihnen nur freundlich empfehlen, Ihre Grundannahmen gründlich und kritisch zu überprüfen; für Sie denken kann ich nämlich nicht.

  236. @Joker
    Meine Behauptung “Die Begründung kann u.U. logisch widerlegt werden. Ist sie nicht widerlegbar oder falsifizierbar, dann ist es bloß religiöser Glaube.” bezog sich auf die Begründung einer metaphysischen Annahme, welche nicht notwendig metaphysisch sein muss.

  237. @Balanus: 19.000 Kommentar

    Ich sah gerade, dass du hier den 19.000 Kommentar bei MENSCHEN-BILDER hattest. Hast du vielleicht einen Themenwunsch?

    Für den 20.000 würde ich ein Freiexemplar meines nächstens Buchs in den Ring werfen; und wenn das in dem Tempo weitergeht, dann haben wir den wahrscheinlich erreicht, bevor das Buch erschienen ist.

  238. @ anton reutlinger 18.01.2021, 14:44 Uhr

    Zitat: „Das geschieht alltäglich über die Medien oder durch Kopieren von Daten oder Software. Aber die Information wird eben nur von einer Materie auf eine andere übertragen, ….“

    So ist es. Theologen haben nach Entdeckung der „Seele“, die Softwarecharakter hat und lang bevor die Genetik etabliert wurde, ungefähr in dem Sinne argumentiert: „Dass die „Software“ in einer „(fast) unendlichen Kette“ immer wieder neues Leben generiert, und das „alte Leben“ endet. Natürlich existieren auch „Meme“ weiter, einerseits durch „Überlieferung“ über die Generationen, oder durch Medien (Bücher, …).

    Die weitere Existenz wesentlicher Teile der „Software“ (auf „junger Hardware“), obwohl die „alte Hardware“ endet, ist deren bemerkenswerte und für manche Menschen tröstliche Aussage. Und das alles lange bevor diese Aussagen durch die Genetik, Memetik …. wissenschaftlich bestätigt wurden.

    Selbst wenn Theologen gelegentlich einen „Bock geschossen“ haben, sie hatten höchstes geistiges Niveau.

  239. @Klaus Günther
    Ihnen ist, wie manch anderem hier, vermutlich entgangen, dass ich in meinem Artikel keinen Standpunkt einnehme, weder einen monistischen noch einen dualistischen oder was sonst noch. Den Naturalismus betreffend setze ich versuchsweise die Prostulate einer “marktgängigen” Metaphysik (Ontologie) voraus und leite daraus nach Möglichkeit einen Widerspruch her. Skeptikerdenken ist das, mehr nicht!

    Die Diskussion hat gezeigt, wie man damit umgehen kann: Mit dem Widerspruch leben, ihm durch Veränderung der Postulate entgehen, oder ihn ignorieren. Das Verhalten zu bewerten, fällt wiederum nicht in die Kompetenz des Skeptikers.

  240. @anton reutlinger

    Meine Behauptung […] bezog sich auf die Begründung.

    Oh, jetzt sehe ich den Gorilla.

  241. @Joker
    Nun, ich gebe zu, meine Formulierung war nicht astrein. Wenn eine Aussage nicht widerlegbar ist, dann kann sie tautologisch oder wahr sein. Gemeint war die logische oder empirische Überprüfbarkeit oder Falsifizierbarkeit im Popperschen Sinn. Der religiöse Seitenhieb war etwas überspitzt.

    Nach wie vor bin ich aber überzeugt, dass die Annahme des metaphysischen Dualismus widerlegt ist, weil die Information als zentrales Element des Geisteslebens zwingend an Materie/Energie gebunden ist. Die physikalische Ansicht von Materie spielt dabei keine Rolle. Es zählt die sinnlich wahrnehmbare Materie, sei es Papier oder Bildschirm, oder die neurobiologisch wahrnehmbaren elektrisch/chemischen Nervensignale.

  242. @Grams / Schleim / Hoppe
    Das Komplementaritätsprinzip wird nach Bohr mit Welle/Teilchen verbunden. Welle/Teilchen ist Elektromagnetismus.
    Reduktionistisch müsste somit das Komplementaritätsprinzip im Elektro-/Magnetismus(Wechselwirkung/Grundkraft) verortet werden.
    Was ist IN der Wechselwirkung?
    Insofern hat Stephan Schleims Frage schon Berechtigung.
    Ist der Wechselwirkungsmechanismus (Polarität) deckungsgleich mit erlebter gesellschaftlicher Polarisation?
    Auf der abstrakt konkreten Ebene spreche ich als Pole/Wechselwirkung Konkurrenz und Kooperation an.
    Was meinen sie? Ein ‘scheinbares’ Phänomen?

  243. @Schleim
    ” denn natürlich können wir nicht ausschließen, dass dort, wo physikalische Prozesse indeterministisch sind (bsp. radioaktiver Zerfall, Doppelspaltversuch, Schrödingers Katze), eine von uns noch nicht verstandene Größe die Vorgänge determiniert.”
    Doch, das genau das können wir inzwischen ausschließen. Es gibt inzwischen Versuche, die diese verborgenen Parameter ausschließen. Einstein hatte eben doch nicht recht mit seiner Vermutung, dass es diese Parameter geben müssen. Siehe Bellsche Ungleichung.

  244. @Mussi
    “Das Komplementaritätsprinzip wird nach Bohr mit Welle/Teilchen verbunden. Welle/Teilchen ist Elektromagnetismus.”
    Nein, das gilt allgemein in der Quantenmechanik, auch für andere Wechselwirkung, nämlich der starken und der schwachen Wechselwirkung und nicht nur für die elektromagnetische WW.

  245. @anton reutlinger

    Nachdem Sie mich auf meinen Fehler aufmerksam gemacht haben – wer lesen kann ist klar im Vorteil – lassen Sie mich nun noch etwas anderes versuchen.

    Ist sie [die Begründung einer metaphysischen Annahme] nicht widerlegbar oder falsifizierbar, dann ist es bloß religiöser Glaube.

    Ich halte ihn zwar nicht mehr für bedeutungslos und sinnlos, aber:

    Dieser Satz ist falsch. (*)

    Im Fall des Bieri-Trilemmas gibt es gute Begründungen für alle drei Lemmata. Nehmen wir uns nur einmal eines davon vor,

    Der Bereich physikalischer Phänomene ist kausal geschlossen.

    Schon die enthaltene, etwas versteckte Annahme, es gäbe auch nur manchmal so etwas wie eine Kausalität zwischen einigen physikalischen Objekten (wie z.B. Billardkugeln), ist eine metaphysische Annahme. Dass sie sich in vielen praktischen Bezügen bewährt hat (in einigen soll sie das ja auch nicht getan haben), kann man als Begründung für deren Annahme sehen. Wie schon Hume gezeigt hat, ist diese Begründung aber weder logisch abgesichert, noch abzusichern, wie es sich vielleicht später Kant erhofft hat, der Kausalität als Notwendigkeit betrachtet. Logisch widerlegbar scheint sie mir aber auch nicht zu sein.

    Es lässt sich auch kein Experimentum crucis denken, was als Letztbegründung dienen könnte, mit dessen Hilfe man das Prinzip der Kausalität falsifizieren könnte.

    Die Annahme einer Kausalität in der Natur ist demnach durch keine Begründung abgesichert, die logisch widerlegbar oder empirisch falsifizierbar wäre.

    Wenn ich mit dem Gesagten recht habe, wie soll es dann eine Begründung mit diesen, von Ihnen geforderten Kriterien geben, für die Annahme, die Welt wäre im Bereich physikalischer Phänomene sogar kausal geschlossen?

    Ich halte Lemma 3. dennoch nicht für religiösen Glauben, Sie?

    (*)
    Mit Selbstbezüglichkeit von ‘dieser’ würde sich das Lügner-Paradox ergeben, was hier nicht gemeint ist.

  246. @Mussi

    p = m x v, wobei p und v Vektoren sind.

    Der Impuls ist eine Erhaltungsgröße wie Energie.

  247. @Physiker
    formel: einfach so hinschreiben geht didaktisch methodisch in diesem Blog nicht. Wir sind nicht alleine..
    Erläuterung? !

  248. @Schleim
    “Hier setzen Sie schlicht voraus, was zu zeigen wäre: dass es nämlich nichts ohne “materiellen Träger” geben kann (in der Philosophie nennt man diesen Argumentationsfehler “petitio principi” oder “question begging”). Im Übrigen scheint Materie doch gar nicht das grundlegende Prinzip in der Physik zu sein.”

    Um es ganz klar zu sagen: unsere Welt besteht NUR aus Materie und ihren Interaktionen in Form von vier Wechselwirkungen (zu dem, was eventuell oder vermutlich noch fehlt, morgen mehr, heute ist es zu spät). Jeder der etwas anderes behauptet, muss es beweisen. Wenn die Philosophen etwas anderes behaupten ohne diesen Beweis antreten zu können, ist die Behauptung gegenstandslos. Das grundlegende Prinzip der Physik ist eine ganz andere Sache, davon – wie gesagt – später mehr.

  249. @Mussi
    Dann muss die Frage genauer formuliert werden. Impuls ist das Produkt aus Masse und Geschwindigkeit, das ist die Definition. Was “bedingt” bedeutet, ist mir unklar.
    Die Erhaltung des Impulses ergibt sich aus einem Symmetrieprinzip, der Homogenität des Raumes, sie ist also sehr fundamental.

  250. @Physiker: bisschen Dogmatisch?

    Ihre Diskussionsstrategie scheint darin zu bestehen, Dinge schlicht zu behaupten, nicht für Ihren Standpunkt zu argumentieren. Das Gegenüber soll dann mal das Gegenteil beweisen. Ja, so einfach geht das.

    Das klinkt für mich doch etwas dogmatisch. Überzeugend ist es jedenfalls nicht.

  251. @Mussi
    Die Beschleunigung von Materie relativ zum Bezugsystem.
    Aber ehrlich, was soll das? Soll ich eine allgemeinverständliche Einführung in die Physik empfehlen?

  252. @Schleim
    Ich wollte das erst morgen schreiben, aber da Sie heute schon antworten:
    es scheint mir in dieser Diskussion bei einigen Personen eine grundlegende Unkenntnis von physikalischen Tatsachen vorzuliegen. Ich kann nur empfehlen, sich einmal in entsprechende Literatur zu vertiefen, und zwar kann man in Roger Penrose: “The Emperor’s New Mind” (fast) alles Relevante zu der Diskussion finden, und noch eine ganze Menge an wichtigen Gedanken mehr.
    Wenn man die fundamentalen Erkenntnisse der Physik in Frage stellt, die experimentell abgesichert sind, ist es nicht möglich zu diskutieren.

  253. @Schleim
    In der Naturwissenschaft entscheidet das Experiment. Daher behaupte ich nur, was experimentell nachgewiesen ist. Frei aus dem Nichts heraus zu diskutieren ist das Privileg der Philosophen. Das führt ausschließlich dann zu etwas Sinnvollem, wenn man irgendwann einmal harte Evidenz nachfüttern kann.

  254. @Mussi

    Wie Newton uns schon sagte:
    F = m x a

    Die Geschwindigkeit kommt durch die Beschleunigung . Die Beschleunigung erfolgt durch die Kraft, die wiederum verschiedene Ursachen haben kann. Wenn man das bis zum Ursprung verfolgt, ergibt es sich durch Austausch von Photonen oder anderen Elementarteilchen, zum Beispiel Gluonen, W oder Z Bosonen oder – wie vor kurzem experimentell bestätigt- Gravitonen.

  255. @Joker // 18.01.2021, 22:01 Uhr

    »Schon die [im dritten Bieri-Lemma] enthaltene, etwas versteckte Annahme, es gäbe auch nur manchmal so etwas wie eine Kausalität zwischen einigen physikalischen Objekten (wie z.B. Billardkugeln), ist eine metaphysische Annahme. «

    Liegt die metaphysische Annahme nicht vielmehr darin, dass der gesamte Bereich der Physik als kausal geschlossen angenommen wird? Es also nie unverursachte Erstereignisse geben kann? (Als das wirklich erste, vermutlich unverursachte Ereignis kann der Big Bang gelten).

    Denn tatsächlich handelt es bei der Kausalität (dem Ursache-Wirkungs-Prinzip) um eine Urerfahrung des Menschen: Wenn ich (als Kleinkind) mir den Kopf an der Tischkante stoße, tut es weh. Wenn Mama mich tröstet, lässt der Schmerz nach. Und so geht es weiter, Kausalitätserfahrung folgt auf Kausalitätserfahrung, meist wahre, mal vermeintliche, und als Erwachsene bauen die die einen dann auf die evidenzbasierte Medizin, die anderen auf homöopathische Mittel.

  256. @Timm Grams

    Eine Ihrer Thesen ist, der Naturalist würde im Bieri-Trilemma Option 2 verneinen.

    Mentale Phänomene sind im Bereich physikalischer Phänomene kausal wirksam.

    Das hat ja nicht nur mich überrascht.

    Weiter oben hatte ich bereits Vollmer zitiert, u.a. mit:

    Nach der Identitätstheorie ist jeder mentale Vorgang mit einem neuronalen Vorgang verbunden.

    Bei Schmidt-Salomon findet sich im ersten Kapitel des Manifests, unter der Überschrift “Fundamentale Kränkungen”

    – die neurobiologische Kränkung ([…] “Geistiges” beruht auf “Körperlichem” […])

    Beide Naturalisten deute ich daher so, dass sie mentale Phänomene nicht leugnen, auch wenn der eine nur von mentalen Vorgängen spricht, und der andere es in Gänsefüßchen gesetzt Geistiges nennt. Das scheint mir nur eine terminologische Frage zu sein (offensichtlich sind beide keine Phänomenologen).

    Es ist wohl auch anzunehmen, dass Körperliches, bzw. etwas spezifischer, neuronale Vorgänge als physikalische Vorgänge verstanden werden, die kausal wirksam werden.

    Lässt sich da ihre These überhaupt halten? Ist nicht jeder Vertreter einer Identitätstheorie sogar verpflichtet Option 2 zuzustimmen? Können Sie mir gegebenenfalls Textstellen benennen, aus denen für mich ersichtlich wird, dass Naturalisten diese Option bestreiten?

    Bin gespannt, ob sie mit dem Widerspruch leben, ihm durch Veränderung der Postulate entgehen, oder ihn ignorieren. Aber wer weiß, vielleicht gelingt es Ihnen ja auch, den Widerspruch als nur scheinbaren zu entlarven und aufzulösen.

  257. Die ganze Diskussion hier läuft auf zwei Interpretationen von Dualismus hinaus:
    Die eine lautet: es gibt einen immateriellen Geist und ist der Religion und Esoterik zuzuordnen, die andere behauptet, es gibt eine materielle Welt, welche die Physik beschreibt sowie eine Welt der Ideen, Bedeutungen etc., welche nicht von den Naturwissenschaften beschrieben werden kann. Letzteres ist eine Binsenweisheit. Kein ernstzunehmender Naturwissenschaftlicher, auch Sean Carroll nicht, würde behaupten, man könne Begriffe, wie ‘Marketing’ mit Begriffen der Physik beschreiben. Das Bewußtseinstheater, das im Gehirn aufgeführt wird, läßt sich physikalisch beschreiben hinsichtlich der Beschaffenheit der Bühne, der Dynamik der Akteure und der Größe der Kulisse, der Inhalt der Geschichte, ihre Symbolhaftigkeit etc. ist nicht ihr Thema.

  258. @Physiker / 18.01.2021, 20:12 Uhr

    »@Schleim … Doch, das genau das können wir inzwischen ausschließen. Es gibt inzwischen Versuche, die diese verborgenen Parameter ausschließen. Einstein hatte eben doch nicht recht mit seiner Vermutung, dass es diese Parameter geben müssen. Siehe Bellsche Ungleichung.«

    Aus Ihren spärlichen Worten darf man schliessen, dass Sie hier deterministisch mit nicht-lokal verwechseln.

    Die Bohmsche Mechanik liefert eine deterministische Interpretation der Quantenmechanik, das wollen wir doch mal unmissverständlich festhalten. Und Stephan Schleim geht’s nur um `deterministisch’, über `nicht-lokal’ hat er sich meines Wissens noch nie irgendwo beschwert.

  259. @Chrys
    Die Bohmsche Mechanik ist nicht die allgemein akzeptierte Interpretation der QM. Ich bin allerdings nicht im klaren darüber, wie die Bohmsche Interpretation und die Bellschen Ungleichungen zusammenhängen, da wäre ich für Informationen dankbar. Insbesondere bezüglich der Interpretation der dazu gemachten Experimente.

  260. @Wolfgang Stegemann
    Das ist auch meine Überzeugung, aber es gibt da noch tiefere Probleme, die mit den prinzipiellen Fähigkeiten des Gehirns zu tun haben. Siehe das Buch von Penrose.

  261. @Joker
    ein Bewusstsein bzw. bewusste Wahrnehmung tritt nur im Zusammenhang mit einem biologisch aktiven Gehirn auf, wenn verschiedene Gehirnareale vernetzt zusammen arbeiten und dabei eine bestimmte Aktivitätsschwelle überschritten wird (z.B. EEG Alpha Wellen > 8Hz).
    D.h. es müssen reale physikalische Grundlagen vorhanden und Bedingungen sein. Sind diese Grundlagen nicht vorhanden, gibt es kein Bewusstsein – daher ist die Idee eines immateriellen/nichtpgysikalischen Bewusstseins auszuschließen

    (Bei schlafenden oder narkotisierten Menschen ist kein Bewusstsein erkennbar – obwohl sie lebendig sind. Hier sind biologische (physikalische) Grundvoraussetzungen nicht vorhanden, damit ein Bewusstsein entstehen kann.)

    Ich habe schon mehrmals darauf Aufmerksam gemacht, dass unser Gehirn extrem fehlerhaft arbeitet (z.B. die Bewegungswahrnehmung beim Kinofilm = 100 % falsch). Wer meint, dass es ein immaterielles Bewusstsein oder einen immateriellen Geist geben könne – muss sich über dessen Grundlagen Gedanken machen: Etwas immaterielles von einer nachweisbaren und nachprüfbaren 100 % falschen Wahrnehmung zu haben – was soll das?

  262. @Physiker: Experimente und Interpretationen

    Sie hören sich für mich wie jemand an, der nicht übers Diplom hinausgekommen ist, ergo nie wirklich selbst eigenständig geforscht hat: Da hat man vielleicht Lehrbuchwissen verinnerlicht und hält es für DIE WAHRHEIT – aber nie mitbekommen, wie an den Grenzen der Wissenschaft mehrere Sichtweisen konkurrieren.

    Sie beziehen sich hier auf “fundamentalen Erkenntnisse der Physik” ohne zu nennen, welche Erkenntnisse das sein sollen und inwiefern sie hier für die Diskussion eine Rolle spielen; Sie sprechen in Rätseln, wie die Sphinx. Und dann schlussfolgern Sie: Wer nicht so denkt, wie Sie, mit dem “ist es nicht möglich zu diskutieren.” Inwiefern ist das überhaupt eine Diskussion, wenn es von Anfang an neben Ihrer Sichtweise sowieso keine andere geben kann?

    Und dann ergänzen Sie noch: “In der Naturwissenschaft entscheidet das Experiment. Daher behaupte ich nur, was experimentell nachgewiesen ist.”

    Erstens stelle ich hier einmal fest, dass Sie hier vor allem zu behaupten scheinen, DIE WAHRHEIT zu verkünden. Zweitens scheinen an Ihnen sämtliche Erkenntnisse der Wissenschaftstheorie des 20. Jahrhunderts vorbeigegangen zu sein – und da waren genug Physiker dabei –, dass beispielsweise experimentelle Daten der Interpretation bedürfen und das immer im Lichte bestimmter Theorien geschieht.

    Die Diskussionen bei MENSCHEN-BILDER sind offen für alle. Aber Sie diskutieren nicht, Sie dekretieren vielmehr. Daran habe ich jedenfalls keinen Bedarf.

  263. @Physiker: Die Ideen von Penrose sind bisher noch reine Spekulation. Und zur Erklärung von Bewußtsein, um das es hier geht, brauchen wir sie nicht, denn dort geht es um eine ganz andere Skala.

  264. Mein Schonbezug und das Bewusstsein (6)

    Was hat mein Autoschonbezug mit dem Bewusstsein zu tun ?
    Versuchen wir es mal sprachlich. Die Vorsilbe be- bedeutet im Deutschen „mit“, sowie con im span. mit bedeutet. Conscience , dämmert es ?
    Bewusstsein setzt sich als zusammen aus „ mit“ und „wissen“. Aber welches Wissen ? Ich weiß , ?

    Der Schonbezug setzt sich zusammen auch be und ziehen. Und man kann es sich bildhaft vorstellen, wie wir den Stoff über den Autositz ziehen. Die Vorsilbe be bekommt dann die Bedeutung von „mit Stoff“ überziehen.

    Bei Bewußtsein überziehen wir auch etwas mit Wissen, das Ich. Wir holen uns das Wissen aus dem Gedächtnis und überziehen uns damit. Damit sind wir bewusst, aber wessen ?
    Das hängt von unserer Absicht ab, was wir vorhaben.
    Der Richter holt sich am Morgen sein Wissen über Recht aus dem Gedächtnis, er macht sich dessen bewusst und er zeigt es auch nach außen , indem er seine Robe anzieht.

    Das ist der Sinn der Berufskleidung, jeder sieht, wessen sich der Richter bewusst ist, des Rechtes, der Gesetze.

    Bewusstsein ist also nicht mehr unbestimmt, es nimmt die Gestalt unserer Absicht an. Deshalb haben wir viele Bewusstseine in unserem Gedächtnis. Die gleiche Person hat am Morgen das Bewusstsein des Familienvaters, und diese Rolle spielt er auch, am Nachmittag hat er das Bewusstsein des Verkehrsrichters und er spielt diese Rolle auch.

    Bewusstsein ist also ein Schonbezug für das Ich, dass uns schützt vor den Gefahren .
    Und dieser Schonbezug bekommt im Laufe des Lebens Flecken, Löcher und Risse.
    Man kann die Beschädigungen als Scham, Schuld und Fehler bezeichnen.
    Manche Beschädigungen kann man beheben, durch eine Reinigung, die die Katholiken die Beichte nennen.
    Manche Beschädigungen kann man nicht beheben, die führen dann zu Psychosen.
    Das Ich nennen die Christen übrigens dann die Seele. Die sollte nicht beschädigt werden, beschädigt wird meistens nur das Bewusstsein.

  265. @Joker

    Die Annahme einer Kausalität in der Natur ist demnach durch keine Begründung abgesichert, die logisch widerlegbar oder empirisch falsifizierbar wäre.

    Das stimmt so nicht. Die Kausalität ist selber eine metaphysische Annahme, das kann man gelten lassen. Sie ist ein Denkkonzept zur Ordnung des Weltgeschehens und sie hat sich bewährt. Ihre Anfänge reichen über Leibniz bis zu Aristoteles. Alle Naturphilosophen müssen sich damit beschäftigen.

    Die Begründung der Kausalität ist aber rein empirisch, in jedem einzelnen Experiment könnte sie widerlegt werden. Oder kennen Sie irgendeine Beobachtung oder ein Experiment, in dem ein nichtkausales Ereignis nachweisbar wäre? Es müsste spontan und zufällig aus dem Nichts kommen. Die Grenze der Kausalität liegt in ihrer Erstbegründung, dem BigBang.

    Die Regelhaftigkeit der Naturgesetze ist eine logische Begründung für die Kausalität. Dass es auch zufällige Ereignisse gibt, ist kein Gegenbeweis. Mit der universellen Gültigkeit der physikalischen Kausalität ist die Geschlossenheit der Physik logisch bewiesen.

  266. @anton reutlinger betreffend Kausalität: strenge Kausalität im Sinne von „jedes Ereignis B hat die Ursache in Ereignissen A1, A2, … und ist determiniert durch A1, A2,…“ gibt es nicht: allein schon wegen den nicht streng determinierten Beobachtungen/Messungen bei Quantensystemen .

    Interessant scheint mir noch die herausragende Rolle, die die Zeit bei der Kausalität spielt: Kausalität impliziert zeitliche Aufeinanderfolge und in einer von Kausalitäten bestimmten Welt, in der Ereignisse andere Ereignisse verursachen, ist es äusserst schwierig bis unmöglich Zeitreisen zu unternehmen ohne die Welt durcheinander zu bringen. Wenn das aber so ist, bedeutet das für mich auch, dass man Zeit nicht den anderen Dimensionen gleichsetzen kann. Einsteins Relativitätstheorie legt es ja nahe die Zeit einfach als vierte Dimension anzusehen. Bis zu einem gewissen Grad ist diese Gleichsetzung erlaubt. Allerdings kann man die räumlichen Dimensionen beliebig bereisen, die zeitliche aber nicht.

  267. @ anton reutlinger: das gilt allerdings nicht unbedingt für die Quantenwelt und selbst für die klassische Physik sagt Maxwell: „Es ist ein metaphysischer Grundsatz (doctrine), dass dieselben Vorgänger (same antecedents) stets dieselben Nachfolger (same consequences) haben. Niemand kann dies leugnen. Nur ist das nicht von viel Nutzen in dieser Welt, in der dieselben Vorgänger niemals wieder auftreten und nichts zweimal geschieht. Ein physikalischer Grundsatz verwandter Art lautet, dass ähnliche (like) Vorgänger stets ähnliche Nachfolger haben. Hier aber sind wir von Gleichheit (sameness) zu Ähnlichkeit (likeness) übergegangen, von absoluter Genauigkeit zu mehr oder weniger guter Näherung. Es gibt mancherlei Erscheinungen, bei denen ein kleiner Fehler in den Daten ein nur geringes Verschieben im Ergebnis bewirkt. Der Geschehensablauf heißt in diesen Fällen stabil. Es gibt aber auch andere Klassen von Phänomenen […]. In solchen Fällen können Einflüsse, deren physische Größe zu klein ist, um von irgendeinem endlichen Wesen bemerkt zu werden, Ergebnisse von größter Wichtigkeit hervorrufen.“ Das weist darauf hin, daß die Naturgesetze, die wir machen, Konstrukte sind, die zwar der Realität offenbar sehr nahe kommen, aber nicht mit ihr identisch sind. Wir leben also nicht in einem mathematischen Unoversum, sondern wir mathematisieren es bloß.

  268. @KRichard
    Im TV und in Kinofilmen gibt es stroboskopische Effekte bei Fahrzeugen mit Speichenrädern, z.B. Traktoren oder Autos mit entsprechenden Radkappen. Wenn die Umdrehungszahl der Räder mit der Frequenz der Aufnahmetechnik synchron ist, gibt es interessante Effekte: scheinbarer Stillstand oder Rückwärtsdrehen der Räder bei Vorwärtsfahrt des Fahrzeugs. Die Erklärung ist, dass die einzelnen Speichen der Räder nicht unterscheidbar sind. Da auch Atome und Elementarteilchen nicht unterscheidbar sind, lässt sich dieser Effekt leicht auf die Physik übertragen, mit geradezu philosophischen Folgerungen!

  269. @Holzherr
    Die Kausalität hat mit dem Determinismus nichts zu tun. Die Zeit kommt nur über die Lichtgeschwindigkeit ins Spiel. Ich sehe da keine direkten Zusammenhänge, außer dass die Kausalität quantitativ der Konstanz der Lichtgeschwindigkeit unterliegt. Zeitreisen sind rein spekulativ. Die Natur kennt kene Zeit, auch sie ist eine metaphysische Annahme, wohlbegründet durch die Eigenheiten des Weltgeschehens.

  270. @Physiker: Inhalt

    Ich habe etwas zur Bellschen Ungleichung gelesen und bekomme, wie Chrys hier schon andeutete, den Eindruck, dass Sie am Thema vorbei schreiben: Was haben die Ausführung hier mit Lokalität und Realismus (im physikalischen Sinne) zu tun?

    Nach dem Ausschlussverfahren gehe ich nun davon aus, dass Sie das Thema dieses Blogbeitrags nicht verstanden haben, bis mir jemand das Gegenteil beweist. Dieses Argumentationsmuster dürfte Ihnen ja bekannt sein (mit dem Unterschied, dass ich mir Ihr Argument wenigstens einmal angeschaut habe).

    Übrigens hatte der MIT-Physiker Sean Carroll (Theoretische Physik), auf den ich mich hier hin und wieder beziehe, vor rund drei Jahren ein Gespräch mit dem Buddhisten und Religionswissenschaftler (PhD Stanford) Alan Wallace, vom Studium her auch Physiker, über die Natur der Realität. Ihr Argumentationsmuster erinnert mich an das von Carroll. Ich habe versucht, mir das Gespräch so neutral wie möglich anzuschauen, kann aber zu keinem anderen Schluss kommen, als dass der Buddhist – insbesondere in der zweiten Gesprächshälfte – mehr vom Thema versteht als der Physiker.

  271. @all: Zwischenbilanz

    Wie passend, dass mir am Sonntag eine Vorlesung über Nietzsche auf den Weg kam:

    Ich gewinne hier den Eindruck, dass jetzt, wo wir im Westen nicht mehr Gott haben, viele ein anderes Fundament brauchen, wie es gerade wieder hieß: Materie.

    Was aber, wenn es gar kein Fundament gibt?

  272. @Stegemann
    Kein Widerspruch dazu. Wir müssen uns damit begnügen, dass wir die Natur oder die Welt nicht vollständig erkennen können. Mesoskopisch zumindest ist die Annahme der Kausalität bewährt und unverzichtbar.

    Wir wissen, dass regelhafte, begrenzte Veränderungen ein stabiles Gleichgewicht als scheinbar unveränderlichen Zustand erzeugen können. Der Thermostat ist ein banales Beispiel dafür. Das Gleichgewicht anziehender und abstoßender Kräfte bringt stabile Körper wie Atome hervor. Es ist geradezu das Wesen der Physik, den Anschein von der Realität zu unterscheiden.

    Die Naturwissenschaft erzeugt ein Netzwerk von Naturgesetzen, die in ihrer Gesamtheit widerspruchsfrei sein müssen. Je dichter dieses Netzwerk wird, desto zuverlässiger wird es. Das ist aber keine Garantie für die “totale Wahrheit”.

    Auch folgendes Zitat ist eine Beschreibung von Kausalität:

    Wir machen uns innere Scheinbilder oder Symbole der äußeren Gegenstände, und zwar machen wir sie von solcher Art, dass die denknotwendigen Folgen der Bilder stets wieder die Bilder seien von den naturnotwendigen Folgen der abgebildeten Gegenstände.

    Heinrich Hertz (1857-1894)

  273. @Joker

    Meine Feststellung, dass der Naturalist die zweite Bieri-Aussage („Mentale Phänomene sind im Bereich physikalischer Phänomene kausal wirksam“) verneine, steht tatsächlich im Gegensatz zu dem Vollmer-Zitat: „Nach der Identitätstheorie ist jeder mentale Vorgang mit einem neuronalen Vorgang verbunden“ (Vollmer, 2013, S. 59).

    Auch stimme ich zu, wenn Sie schreiben „Es ist wohl auch anzunehmen, dass Körperliches, bzw. etwas spezifischer, neuronale Vorgänge als physikalische Vorgänge verstanden werden, die kausal wirksam werden.“

    Um Ihre Fragen zu beantworten, muss ich etwas ausholen.

    Der von mir aufgezeigte Widerspruch steckt in der Kernaussage der Naturalisten: Es gibt die eine Welt, die Realität, die außerhalb unseres Denkens existiert (Mahner) und die von unserem Bewusstsein unabhängig ist (Vollmer). Diese Kernaussage stellen diese Naturalisten im Allgemeinen an den Anfang ihrer Ausführungen; sie wird offenbar für besonders wichtig gehalten. Aus diesen Kernaussagen entwickle ich den Widerspruch.

    In weiteren Ausführungen präzisieren Mahner und Vollmer ihre Standpunkte.

    Bei Martin Mahner ist es der Fiktionalismus (Mahner, 2018, S. 35). Demnach betrachtet der Naturalist die reinen Denkinhalte als fiktional und damit als nicht real. Ich weiß nicht, wie er auf diese Weise den von mir aufgezeigten Widerspruch loswerden will.

    Gerhard Vollmer übernimmt die Identitätstheorie und geht zu einer Art Aspektdualismus über (Vollmer, 2013, S. 58): „Innen- und Außenaspekt [sind] sind verschiedene Projektionen von Gehirnvorgängen.“ Damit entgeht er dem Verdikt der Widersprüchlichkeit. Die Identitätstheorie jedoch passt nicht zur eingangs postulierten Unabhängigkeitsannahme.

    Damit hoffe ich, alle Ihre Fragen beantwortet zu haben. Ich bleibe vorerst bei meinen Thesen.

  274. @Wolfgang Stegemann // 19.01.2021, 10:03 Uhr

    Aus Ihrem Maxwell-Zitat:

    » Es gibt aber auch andere Klassen von Phänomenen […]. In solchen Fällen können Einflüsse, deren physische Größe zu klein ist, um von irgendeinem endlichen Wesen bemerkt zu werden, Ergebnisse von größter Wichtigkeit hervorrufen. «

    Wie wahr, wie wahr.

    Ein eindrückliches Beispiel dafür sind willentliche Entscheidungen, die von vielen für „frei“ gehalten werden, eben weil die physische Größe bestimmter Einflussfaktoren extrem klein ist und sie darum nicht bemerkt werden—obschon sie natürlich vorhanden sind.

    Da mag mancher „gesunde Menschenverstand“ (T.G.) nicht folgen.

  275. @Timm Grams
    Das Auge kann sich selber nicht sehen, das Gehirn kann sich selber nicht denken. Wir und unsere Instrumente sind aus derselben Materie wie das Universum. Die hypothetisch kleinsten Teilchen des Universums interagieren mit den kleinsten Teilchen unseres Gehirns und unserer Instrumente. Wie können wir das Verhalten dieser kleinsten Teilchen erkennen, wenn wir nicht wissen, wie sie interagieren und folglich kein Instrument bauen können, das deren Verhalten messen kann? Ein Widerspruch! Trotzdem können wir als Mensch mit der Welt erfolgreich interagieren.

  276. @Grams, Joker: Naturalismus und Bieri-Trilemma

    Davon abgesehen, dass ich den Naturalismus als epistemische These begreife und auf der ontologischen Ebene eher vom Materialismus/Physikalismus (einem physikalischen Monismus) reden würde, finde ich diese Analyse des Bieri-Trilemmas nicht geglückt:

    Ontologisch gesehen bestreitet der Materialismus doch vor allem die erste Prämisse, den mentalen Realismus (MR). Das, was manche “mentale Vorgänge” nennen, sind laut Materialist nichts als physikalische Vorgänge (bzw. Gehirnprozesse).

    Wenn dem so ist, dann ist die zweite Prämisse, also mentale Verursachung (MV), kein spezifisches Problem mehr, denn das, was in der zweiten prämisse als MV angesehen wird, ist schlicht physikalische Verursachung. Wieso sollte der Materialist also diese Prämisse ablehnen? Sie fällt vielmehr auf das allgemeinere Problem zurück, physikalische Kausalität zu erklären (die angesichts der Komplexität von Gehirnprozessen wohl allenfalls statisitsch beschrieben werden kann; dahingehend bewegen sich ja aber ohnehin die Kausalbegriffe in den Lebenswissenschaften).

    Knackpunkt für den Materialisten ist meiner Meinung nach MR, nicht MV: Er muss nicht erklären, wie “Mentales” auf “Körperliches” wirkt, woran schon Descartes scheiterte, dafür aber, wie aus “Körperlichem” “Mentales” (bzw. dann präziser: psychische Vorgänge) und vor allem Bewusstsein entsteht.

  277. @Timm Grams

    Der von mir aufgezeigte Widerspruch steckt in der Kernaussage der Naturalisten: Es gibt die eine Welt, die Realität, die außerhalb unseres Denkens existiert (Mahner) und die von unserem Bewusstsein unabhängig ist (Vollmer).

    Es mag sein, dass viele, vielleicht sogar die meisten Naturalisten auch Realisten sind, aber das gilt nicht für alle. Es gibt z.B. durchaus Physiker, die sich zu der Frage nach der Realität einer “äußeren Welt” gar nicht äußern, also auch die Physik erst einmal nicht ontologisch, sondern vorsichtiger epistemisch interpretieren. Dennoch würden sie die Welt vielleicht nicht dualistisch in eine physische/natürliche und eine nichtphysische/übernatürliche Sphäre aufteilen – und blieben in diesem Sinne Naturalisten bzw. Physikalisten.

    Die Physik bietet auch heute keinerlei Anlass für die Vermutung, dass an irgendeiner Stelle *materielle* Vorgänge durch so etwas wie “psychische/mentale Ursachen” beeinflusst werden. Vielmehr gilt, dass alle Vorgänge, wenn man sie einmal nur unter ihrem materiellen Aspekt betrachtet, in dieser Hinsicht ihre wohl vollständige Erklärung in der Physik finden. Für unsere meso-/makroskopische Alltagswirklichkeit scheint mir das auf jeden Fall zuzutreffen. Ich wüsste auch nicht, inwiefern hier “Bewegung/Veränderung” im Sinne von “Verhalten” eine Ausnahme bilden sollte.

    Materiell realisierte, am ehesten als informationsverarbeitende Systeme zu charakterisierende Organismen zeigen materielle Interaktionen mit ihrer physischen, teils sozialen Umwelt. Aus einem uns bisher nicht bekannten Grund, wird ein Teil dieser informationsverarbeitenden Prozesse manchmal (z.B. tagsüber) von den betreffenden Organismen subjektiv als Empfindung, Gefühl, Gedanke, Vorstellung usw., letztlich also als “Welt” erlebt – ohne dass dieser Umstand als solcher die physischen Vorgänge, die stur und ausnahmslos den bekannten Gesetzmäßigkeiten folgen, beeinflussen könnte.

    Die Vorstellung eines von diesen Vorgängen unabhängigen Bewusstseins erklärt nichts: 1. weil dieses übernatürliche Bewusstsein ja seinerseits völlig unerklärlich wäre, 2. weil ich dann nicht mehr bewusstlos sein könnte (mein absolutes Bewusstsein wäre ja seiner Natur nach notwendigerweise stets bei Bewusstsein, ein bewusstloses Bewusstsein wäre ein Widerspruch in sich selbst), und 3. weil eine Interaktion zwischen immateriellen und materiellen Prozessen physikalisch heute nicht vorstellbar ist. (Es würde sich im Übrigen die Frage stellen, warum Ihr Bewusstsein eigentlich nicht in der Lage ist, meinen Körper oder gleich direkt Gegenstände zu beeinflussen.)

    Denkbar mag vieles sein, sogar ein Solipsismus!, aber wenn wir nüchtern den heute bekannten Tatsachen ins Auge blicken, dann können wir einige denkbare Optionen gut begründet ausschließen bzw. sie den Philosophen und Theologen überlassen; ich meine sogar, dass wir das redlicherweise tun müssen. Diese Optionen sind allerdings, soweit ich sehe, duchwegs die religiös favorisierten Optionen …

    Ich selbst glaube nicht mehr an die Kompatibilität von religiösem Glauben und Vernunft, sondern ich sehe hier einen unausweichlichen Konflikt. Stephans quietistischer Agnostizismus mag zwar sympathischer rüberkommen, aber wenn die Zusammenhänge wissenschaftlich klarer werden, dann sollte man auch bereit sein, die Konsequenzen zu ziehen.

    Wir sind alle in einem dunklen Raum, wir wissen nicht, was kommt. Gläubige meinen, dass in diesem dunklen Raum ein Gott sei, der uns sieht, während wir ihn nicht sehen. Sie bemühen sich herauszufinden und zu erraten, was er von ihnen will usw. Ich stelle mir vor, dass irgendwann das Licht angeht und dann zeigt sich, dass immer nur wir in dem Raum waren, sonst niemand. Ich ahne, dass Gläubige das im Tiefsten ihres Herzens sogar wissen – aber sie können oder wollen einfach nicht von ihrer irgendwie kindlichen Vorstellung lassen.

    Es ist noch nichts Gutes aus religiösen Gründen getan worden, das nicht ebenso gut oder sogar besser aus vernünftigen Gründen hätte getan werden können.

  278. @Timm Grams

    Wenn Gedanken auf neuronaler Aktivität beruhen, dann sind Gedanken ganz offensichtlich nicht von physischen Vorgängen unabhängig.

    Gedanken sind somit (virtueller?) Bestandteil der postulierten einen Welt, der Realität.

    Ergo: Keine Unabhängigkeit.

    Bleibt die Frage, ob die Welt/Realität, die _außerhalb_ unseres Denkens existiert, von unserem Denken _unabhängig_ existiert.

    Diese Frage wird gemeinhin bejaht.

    Ergo: Unabhängigkeit.

    Zwei Fragen, zwei Antworten, kein Widerspruch.

  279. @Physiker / 19.01.2021, 01:37 Uhr

    Das zuvor schon verlinkte Review Paper* von R. Tumulka gefällt mir recht gut als Übersicht zur Bohmschen Theorie. In Sec. 6, Non-Locality, wird der Zusammenhang mit Bell’s Theorem erläutert

    * The Routledge Companion to the Philosophy of Physics, für den der Artikel geschrieben ist, erscheint laut Verlag 2021, ist aber aktuell noch nicht zu haben.

  280. Christian Hoppe,
    Entschuldigung , dass ich mich einmische.
    Sie stellen Bewußtsein als etwas Dingliches hin, dass schon erkärt ist und noch weiter erforscht werden kann. Das ist aber nur eine Arbeitshypothese.

    Das Beispiel mit dem leeren Raum und Gott, das ist ja wohl eine sehr kindliche Vorstellungswelt.
    Gott ist doch kein Objekt, den ich im Raum plaziere, und dann, wenn ich ihn nicht sehe, den Beweis habe, dass es ihn nicht gibt.

    So geht es nicht.
    Das ist der Kardinalfehler der Naturalisten. Sie versuchen Gott zu beweisen.

  281. @Stephan Schleim
    Es kommt darauf an, ob es ein Naturalist ist, der die Identitätstheorie akzeptiert oder ob es einer ist, der auf der Unabhängigkeit der Realität vom Denken (Mentalen) besteht. Letzterer versucht wohl eine naturalistische Antwort auf den Dualismus zu geben und gerät dadurch meines Erachtens in die beschriebenen Schwierigkeiten. Der Verdacht wird genährt durch die Behandlung der Übernatur durch Martin Mahner.

  282. @Schleim
    Bewegung ist ein grundlegendes Phänomen der Welt. Wir wissen, dass physikalische Kräfte Bewegung verursachen. Aber warum tun sie das, warum verursachen sie nicht irgend etwas anderes, z.B. Farbspiele wie Nordlichter oder Regenbogen? Das weiß niemand. Trotzdem käme niemand auf die Idee, Götter oder andere außernatürliche Wesenheiten für Bewegung verantwortlich zu machen.

    Nebel ist ein gelegentlich auftretendes und allgemein zu beobachtendes Phänomen. Jeder weiß, dass kleinste Wassertröpfchen die Ursache sind. Nebel kann auch Ursache oder Begründung für mentale Phänomene wie depressive Zustände oder für Angst sein. Auch hier käme niemand auf die Idee, übernatürliche Ursachen für den Nebel zu suchen oder den Nebel als eigenständige Wesenheit zu betrachten.

  283. @Stephan Schleim // 19.01.2021, 11:26 Uhr

    » Er [der Materialist] muss nicht erklären, wie “Mentales” auf “Körperliches” wirkt, woran schon Descartes scheiterte, dafür aber, wie aus “Körperlichem” “Mentales” (bzw. dann präziser: psychische Vorgänge) und vor allem Bewusstsein entsteht. «

    Ich meine, er muss erklären, wie das, was wir als psychische Vorgänge beschreiben, mit den zugrundeliegenden physischen Vorgängen zusammenhängt.

    „Bewusstsein“ kann als eine Systemeigenschaft aufgefasst werden. Systemeigenschaften „entstehen“ bzw. ergeben sich bekanntlich aus dem wechselseitigen Zusammenspiel der Systemkomponenten. Wie weit man diesen Zusammenhängen auf die Spur kommen wird, wird die Zukunft zeigen.


    @Kommentar 19.000

    Lieben Dank für das Angebot. Aber ich finde, ich habe Dich schon genug gequält mit vielen meiner Kommentare, da muss ich nicht noch das Thema eines Blog-Beitrags vorgeben.

    Oder vielleicht: Wie kommt es, dass uns runde Zahlen, insbesondere solche die mit vielen Nullen am Ende, als etwas Besonderes erscheinen?

  284. @Balanus

    Liegt die metaphysische Annahme nicht vielmehr darin […]

    Wer sagt denn, dass rund um Kausalität nur eine metaphysische Annahme vonnöten ist?

    Fundamental ist halt die Annahme, dass überhaupt ein Ursache-Wirkungs-Verhältnis vorliegt und nicht nur reine Korrelation. Wichtig im Zusammenhang mit der Willensfreiheit, das wird schon im Blogbeitrag erwähnt, ist auch die Frage, haben identische Ursachen notwendigerweise identische Wirkungen? Das muss man annehmen, beweisen oder belegen kann man es nicht.

    Kausalitätserfahrung folgt auf Kausalitätserfahrung

    Das hat ja auch deutliche Spuren in meinem Denken hinterlassen (*) Nichtsdestotrotz ist Induktion eben kein wahrheiterhaltendes Verfahren, kein logisch gültiges Schließen.

    Die metaphysischen Wurzeln des Naturalismus zu erkennen, Mind Blowing Big Bang.

    @anton reutlinger

    […] die Geschlossenheit der Physik logisch bewiesen.

    Gute Nachricht, danke für die Info. Rettet mir den Tag.

    (*)
    Nicht nur Spuren im Denken. Die als Kleinkind erworbene Narbe auf der Stirn stammt allerdings von der Begegnung mit einem Heizkörper, nicht mit einer Tischkante.

  285. @Schleim: wie erklären Sie, “wie aus “Körperlichem” “Mentales” (bzw. dann präziser: psychische Vorgänge) und vor allem Bewusstsein entsteht.”

  286. @Hoppe: Erklärungen & Agnostizismus

    Die Vorstellung eines von diesen Vorgängen unabhängigen Bewusstseins erklärt nichts: 1. weil dieses übernatürliche Bewusstsein ja seinerseits völlig unerklärlich wäre…

    Hier müsste man aufpassen, finde ich, dass man sich die gewünschte Lösung nicht implizit zurechtdefiniert: Wenn du eben “Erklärung” mit “physikalische Erklärung” gleichsetzt, dann folgt trivialerweise, dass Nichtphysikalisches “völlig unerklärlich” wäre. Deiner Definition muss man aber nicht zwingend folgen und wir verwenden im Alltag ständig nicht-physikalische Erklärungen, etwa soziologischer, ökonomischer oder auch moralischer Natur. (Jetzt zu behaupten, diese seien im Grunde auch physikalisch, wäre wieder Spekulation.)

    Stephans quietistischer Agnostizismus mag zwar sympathischer rüberkommen, aber wenn die Zusammenhänge wissenschaftlich klarer werden, dann sollte man auch bereit sein, die Konsequenzen zu ziehen.

    Hmm, an welche Konsequenzen denkst du?

    Ich bin davon überzeugt, dass religiöse und (natur-)wissenschaftliche Aussagen nur eine kleine Schnittmenge haben: Und selbst wenn Naturalisten die Bewegung jedes einzelnen Atoms bis zum Urknall zurückverfolgen und erklären könnten, was sie nicht können, dann bliebe noch die Frage nach dem, was vor dem Urknall war.

    Mich wundert, warum sich so viele Menschen – übrigens sowohl auf religiöser als auch auf (natur-)wissenschaftlicher Seite – über den Beweis von Aussagen streiten, die meiner Meinung nach nicht beweisbar sind. Erfreut euch doch lieber am Leben! Und lasst dem Anderen seinen Glauben.

    Es ist noch nichts Gutes aus religiösen Gründen getan worden, das nicht ebenso gut oder sogar besser aus vernünftigen Gründen hätte getan werden können.

    Das mag auf dem Papier so stimmen. In der Lebenswelt ist es aber für viele Menschen doch ein Unterschied, ob es einen Jesus, einen Herrgott, eine Maria, einen Allah, einen Ganesha, einen Shiva, Naturgeister usw. im transzendentellen Sinn gibt. Für manche ist es schlicht ein Prinzip Hoffnung; das Einzige. Was hätte deine Weltsicht denen denn als Alternative anzubieten?

  287. @Balanus: Materialismus

    Dass psychische und physische Prozesse irgendwie zusammenhängen, dürfte den Materialisten nicht befriedigen; das behaupteten ja schon Vertreter des psychophysischen Parallelismus (z.B. Leibniz) oder Okkasionalisten im 17. bis 18. Jahrhundert.

    P.S. Und das mit den Zahlen ist eben Konvention: Warum feierten wir an der Uni Groningen den 400. Geburtstag viel intensiver als den 399.? Warum halten wir Menschen einen 40. Geburtstag für so viel besonderer als den 41.? Warum haben wir überhaupt nur einen Geburtstag im Jahr, wo wir doch jeden Tag gewissermaßen neu geboren werden, wenn wir aufwachen? Konvention.

  288. @Stegemann
    Genau diese Frage kann der Naturalist nicht beantworten, weil für das “Entstehen” keinerlei Kausalität erkennbar ist. Es ist das “explanatory gap” (Joe Levine, *1951) der Bewusstseinsphilosophie bzw. der Hirnforschung.

    Wie von mir oben beschrieben, ist Bewegung die Folge physikalischer Kräfte. Wo Bewegung auf Widerstand stößt, entstehen Druck und Spannung und ggf. Verformungen. Es ist wohl kein Geheimnis, dass diese Phänomene auch im Gehirn auftreten, als Bewegung von Ionen, als Bewegung von Signalmolekülen, als Druck und Spannung, als Dichte von Gasen in oder zwischen Nervenzellen. Warum sollten diese physikalischen Phänomene nicht als mentale Phänomene in Erscheinung treten?

  289. Stephan Schleim,
    Ich finde es gut, dass sie keine sture materialistische Haltung einnehmen, die behauptet, Naturalismus und Religion schließen sich grundsätzlich aus.
    Wenn man da offen bleibt, dann bleiben sie Ansprechpartner für beide Seiten.

    Meine Erfahrung: Wenn man Gott zu nahe kommt, wird man von ihm gefangen.

  290. @Timm Grams

    Die ontisch-monistische These von der Existenz einer allumfassenden Welt W ist in jedem Fall logisch inkonsistent. Denn Existenz wird dabei bestimmt als Zugehörigkeit zu W, d.h. x existiert per definitionem genau dann, wenn x ∈ W gilt. Demnach muss für die Welt selbst W ∈ W gelten, da sie ja laut Annahme existiert.

    Die Bedingung, die Welt sei allumfassend, impliziert indessen, dass Grösseres als die Welt nicht existiert, das heisst, W ∈ x ⇒ x ∉ W. Damit ergibt sich für die Welt aus W ∈ W ⇒ W ∉ W ein Widerspruch. Die Annahme der Existenz einer allumfassenden Welt impliziert also deren Nichtexistenz.

    Die mit allumfassenden Gesamtheiten verbundenen Antinomien hat allerdings Kant bereits herausgestellt, das ist also alles andere als neu. Vor allem ist das keine von Markus Gabriel erstmals erdachte Argumentation, wie manche nach meinem Eindruck zu glauben scheinen. Doch wie auch immer, ontischer Monismus in jeglicher Erscheinungsform ist als philosophische Position längst reif für die Tonne.

    N.B. Bei dieser Gelegenheit auch von mir noch besten Dank für Ihren Gastbeitrag, der mir sehr zusagt.

  291. @Joker // 19.01.2021, 12:24 Uhr

    » Fundamental ist halt die Annahme, dass überhaupt ein Ursache-Wirkungs-Verhältnis vorliegt und nicht nur reine Korrelation. «

    Wie gesagt, ich würde hier eher von einer fundamentalen Erkenntnis sprechen. Und diese tritt sehr früh auf, bereits im zartesten Kindesalter: Mein Tun hat Folgen. Dahinter können wir nicht zurück.

    Davon unbeschadet ist die später auftretende Erkenntnis, dass zwei aufeinanderfolgende Ereignisse nicht notwendigerweise kausal (naturgesetzlich) verknüpft sein müssen. Um zu dieser Erkenntnis zu gelangen, bedarf es schon einer gewissen Denkarbeit..

    Ich denke, man sollte die Klassifizierung „metaphysische Annahme“ auf solche Annahmen beschränken, die völlig aus dem Rahmen naturwissenschaftlicher oder empirischer Erkenntnismöglichkeiten herausfallen.

  292. @hwied

    Sie stellen Bewußtsein als etwas Dingliches hin, dass schon erkärt ist und noch weiter erforscht werden kann. Das ist aber nur eine Arbeitshypothese.

    Verstehe ich nicht? Ich stelle Bewusstsein gerade NICHT als etwas Dingliches hin. Bewusstsein gibt es genausowenig wie Rotsein. Einen Teil der in uns ablaufenden Informationsverarbeitung erleben wir (aus bisher unbekannten Gründen) bewusst, den weitaus größten Teil nicht. “Bewusst” ist also ein Adjektiv, eine mögliche Eigenschaft einiger biologischer, informationsverarbeitender Systeme. Eine Eigenschaft sollte man nicht zu einem Ding machen – wie die Dualisten (vgl. res cogitans).

    Das Beispiel mit dem leeren Raum und Gott, das ist ja wohl eine sehr kindliche Vorstellungswelt. Gott ist doch kein Objekt, den ich im Raum plaziere, und dann, wenn ich ihn nicht sehe, den Beweis habe, dass es ihn nicht gibt.

    Ähm, der dunkle Raum ist, quite obviously, eine Metapher?

    Ich finde diese Metapher gerade hinsichtlich des Versuchs religiöser Menschen, zu verstehen, was Gott, den niemand je gesehen hat, hier und jetzt von ihnen persönlich will, äußerst treffend. Manche kommen dabei auf coole Ideen und gründen eine caritativ tätige Organisation – andere auf entsetzliche, z.B. mit dem Lkw in einen Weihnachtsmarkt rasen oder Frauen bei lebendigem Leibe verbrennen. Lassen wir die Vernunft – Fakten, Argumente, Konsensfindungsprozesse usw. – unsere Richtschnur in wichtigen Fragen sein und halten wir den lieben Gott da raus – das ist doch so schon alles schwierig genug …

    (Die Gottesfrage ist hier übrigens relevant, weil das Leib-Seele- bzw. Gehirn-Geist-Problem das Gott-Welt-Problem auf der individuellen Ebene spiegelt und die jeweiligen Argumente miteinander korrespondieren.)

    Das ist der Kardinalfehler der Naturalisten. Sie versuchen Gott zu beweisen.

    Oops, jetzt geht hier aber einiges durcheinander … vielleicht besser noch einmal in Ruhe nachdenken?

  293. @ Schleim, Reutlinger: das sehe ich vollkommen anders. Wie ich bereits sagte, kann die Naturwissenschaft psychologische Dinge nicht erklären, und zwar, weil es sich um eine andere Beschreibungsweise handelt. Während die Naturwissenschaft das Verhalten von Neuronen misst, kümmert sich die Psychologie um bedeutungshafte Phänomene, Bedeutungen, die wir den Dingen zuschreiben. Ich kann mir aber sehr gut vorstellen, daß beide Beschreibungsweisen eines fernen Tages konvergieren. Das setzt seitens der Psychologie eine Operabilisierbarkeit der Phänomene voraus, auf seiten der Naturwissenschaft eine Theoretisierung in der Modellbildung (von den technischen Voraussetzungen einmal abgesehen). Das Manko der naturwissenschaftlichen Humanwissenschaft besteht darin, daß sie Unmengen an empirischen Puzzleteilchen sammelt, aber nicht das Gesamtbild kennt, das daraus entstehen soll. Einen Katalysator dafür sehe ich in einer Systemtheorie des Menschen.

  294. @Stephan

    Wir sind hier auf den Scilogs. Ich bestreite die weit verbreitete und gern geglaubte These, dass religiöser Glaube mit naturwissenschaftlicher Erkenntnis vereinbar sei. Falls jemand an dieser Vereinbarkeit gar kein Interesse hat – z.B. weil ihn Naturwissenschaft im Grunde überhaupt nicht interessiert oder weil ihn sein Glaube ganz doll überzeugt -, darf er von mir aus sehr gerne religiös bleiben. (Ich persönlich mag kompetente, kooperative, engagierte Menschen – ob religiös oder nicht, ist mir wurscht.)

    Was hatte Darwin den armen Leuten zu bieten, als er ihnen mit seiner Evolutionstheorie die angenehme Vorstellung einer direkten Schöpfung des Menschen durch das göttliche Wort nahm? Pragmatisch betrachtet ist es wahrscheinlich empfehlenswert, religiös zu bleiben, wenn man es ist; man kriegt nur Ärger und fühlt sich immer schrecklich allein so ohne Gott. Dieser Pragmatismus ist aber, wie Dir bekannt ist, epistemisch irrelevant (und sein Outcome ist auch nicht frei von Ambivalenzen …).

  295. @Chrys: Logik und Welt

    Warum nur musste ich “Gabriel” denken, bevor ich den Namen sah? 😉

    Aber im Ernst: Was sollen Sätze der Form “Die Welt ist Teil der Welt” bedeuten? Das Verrät uns vielleicht mehr über die Logik (sprich: unsere Denk-Welt) als über die natürliche Welt.

  296. @Stegemann: Psychologie und Hirnforschung

    Auf einer gewissen Ebene stimme ich Ihnen zu – und diesen Standpunkt vertrete ich auch in meinen Vorlesungen, wobei ich dann problematisiere, dass unsere psychologische Begrifflichkeit eher pragmatischer Natur und daher nicht in Stein gemeißelt ist…

    …aber irgendwie denke ich auch, dass wir den lebendigen Körper – oder Leib, wie man das früher nannte – besser verstehen müssen und die im vorherigen Absatz genannte Begrifflichkeit vor allem ein Spiel unserer Ideenwelt ist, unseres Denkens (wo es dann auch zu solchen “Einsichten” kommt wie “Die Welt ist Teil der Welt”).

    Man sollte der Phänomenologie noch einmal eine Chance geben.

  297. @Stegemann

    Anerkennen Sie denn, dass hinsichtlich aller an einem psychischen Vorgang (z.B. einer Handlung) beteiligten *materiellen* Prozesse (z.B. die Bewegung eines Arms) die Physik bereits die vollständige Erklärung liefert? Akzeptieren Sie die Gültigkeit der Physik für physische Prozesse einschl. derer bei Verhalten?

    Und falls ja: Was genau bleibt noch zu erklären übrig für die Psychologie?

    Selbstverständlich können Sie quasi “auf-zoomen” und betrachten einen extrem komplexen physischen Vorgang unter seinem psychologischen Aspekt; Sie können auf dieser Beschreibungsebene auch Erklärungen finden usw. Alles richtig. Aber Sie werden keine *primären* psychischen Ursachen finden – weil die Ursachen der beobachtbaren Bewegungen ja bereits physisch hinreichend aufgeklärt waren. Es gibt, was die beobachtbaren materiellen Vorgänge angeht, einfach keine Erklärungslücke, “wo die Psyche dann wirksam wird”.

  298. @Hoppe: Religion und Wissenschaft

    Einen Widerspruch kann es – schon aus logischen Gründen – nur dort geben, wo sich die Aussagenbereiche überschneiden.

    Wir sind auf den SciLogs, stimmt; wir tun aber nicht nur so, als seien wir hyperrationale, unfehlbare Wissenschaftler, sondern wir sind in letzter Instanz schlicht: Menschen – und als solche verletzlich, unvollendet und abhängig.

    Wenn die absolute Wahrheit für uns unerreichbar ist, könnte es dann nicht aus pragmatischen Gründen sinnvoll sein, sich ein Überzeugungssystem zu wählen, mit dem man besser durchs Leben kommt?

  299. @Timm Grams

    Bei Martin Mahner ist es der Fiktionalismus (Mahner, 2018, S. 35). Demnach betrachtet der Naturalist die reinen Denkinhalte als fiktional und damit als nicht real.

    Markus Gabriel sagt, “Es gibt […] sogar Einhörner auf dem Mond”. (Warum es die Welt nicht gibt, S. 9). Ich stelle mir in meinem Denken gerade solche vor, genau dort. Wieso um alles in der Welt sollte ein Naturalist sagen, die Einhörner seien real?

    Ich habe verstanden, dass es eigentlich nicht um Ihre Sicht der Dinge geht, aber ich bin schon etwas neugierig. Sind Sie geneigt, zu sagen, die Denkinhalte seien real? Falls ja, auf welchem Mond sind die Einhörner gerade; die Rede (bzw. die Denke) ist ja von ‘dem’ Mond, nicht von irgendeinem. Führt uns die Sprache an dieser Stelle in die Irre?

    Eine generelle Diskussion über Feinheiten der Ontologie des Naturalisten oder den Existenzbegriff im Allgemeinen scheint mir für ihren Versuch einer Widerlegung auch nicht nötig zu sein.

    Um die physikalische Wirksamkeit mentaler Phänomene zu behaupten, reicht es meines Erachtens, das durch diesen Gedanken ausgelöste Schmunzeln als eine von vielen messbaren Folgen anzuerkennen.

    Ich weiß nicht, wie er auf diese Weise den von mir aufgezeigten Widerspruch loswerden will.

    Zunächst bin ja ich es, der den loswerden will. Dazu hatte ich bereits einen Kommentar geschrieben, bezüglich der Interpretation von ‘unabhängig’. Eventuell ist dieser ihrer Aufmerksamkeit bisher entgangen.

    Was soll’s, alle anderen wiederholen sich ja auch ständig:

    Aber nur “fast immer”

    Der schlichte Gebrauch des Wortes ‘unabhängig’ impliziert eben nicht, was Sie unterstellen, dass die Unabhängigkeit ‘immer’ und überall gegeben sein muss, dass sie absolut sein muss, es keinerlei Wechselwirkung gibt (@Balanus: oder Wirkung in eine Richtung).

    Elternunabhängiges BAFÖG bedeutet nicht, dass die Steuern der Eltern nicht zu dessen Finanzierung herangezogen werden. Wenn ich unabhängig von meiner Freundin in Urlaub fahre, schließt das nicht aus, dass ich manchmal an sie denke oder sogar mit ihr telefoniere chatte.

    Mehr ist auch nicht gesagt, wenn Naturalisten von einer vom Denken und vom Bewusstsein unabhängigen Existenz der Dinge sprechen.

    Bei dieser Gelegenheit möchte ich explizit darauf aufmerksam machen, um einen inneren Widerspruch aufzuzeigen, sind Sie verpflichtet die Begriffsbedeutungen im Sinne der Naturalisten zugrunde zu legen. Es würde also nicht reichen, dass Sie ‘unabhängig’ als ‘total unabhängig’ verstehen.

    @Balanus

    Aber vielleicht übersehe ich ja auch etwas, nur was?

    Wärmestrahlung, elektromagnetische Felder und Gravitation. Zugegeben, alles schwer zu sehen.

    Alles hängt mit allem zusammen! (Ein Physiker würde vielleicht noch ergänzen, im Lichtkegel.)

  300. @Stephan // 19.01.2021, 12:44 Uhr

    » Dass psychische und physische Prozesse irgendwie zusammenhängen, dürfte den Materialisten nicht befriedigen;… «

    Nicht „irgendwie“, inzwischen haben wir ja eine gute Vorstellung davon, wie und warum das eine (die Physis) das andere (das Psychische) bedingt. Zu Leibniz‘ Zeiten war das noch unklar.

  301. @ Hoppe: Anerkennen Sie denn, dass hinsichtlich aller an einem psychischen Vorgang (z.B. einer Handlung) beteiligten *materiellen* Prozesse (z.B. die Bewegung eines Arms) die Physik bereits die vollständige Erklärung liefert?
    Natürlich nicht. Ansonsten könnten wir aufhören zu forschen. Aber Sie haben mich offenbar nicht verstanden. Wie soll die (derzeitige) Physik z.B. Eifersucht vollständig erklären. Dafür hat sie keinerlei Begriffe. Braucht sie auch nicht, für das, was sie derzeit tut. Fragen Se einen Physiker, was Leben ist. Daran hat sich schon Schrödinger die Zähne ausgebissen.

  302. @Stegemann
    Naturwissenschaft und Psychologie liefern unterschiedliche Beschreibungsweisen für Phänomene, völlig korrekt. Mein Beispiel vom Nebel sollte genau diesen Sachverhalt erläutern: der Nebel ist nichts anderes als eine Wolke von Wassertröpfchen, die wir nicht als einzelne physikalische Wassertröpfchen, sondern eben als undurchsichtigen, ausgebreiteten Nebel wahrnehmen. Da der Zusammenhang von Wassertröpfchen und Nebel aber allgemein bekannt ist, wird er nicht infrage gestellt! Oftmals taucht der Nebel scheinbar aus dem Nichts auf oder man fährt mit dem Auto plötzlich und unerwartet in eine Nebelbank. Wundert sich jemand darüber?

    Das Gehirn ist extrem komplex und kompliziert. Man kann es bei der Arbeit nicht beobachten, außer mit den relativ grobkörnigen Instrumenten der Hirnforschung. Es gibt eine große Zahl verschiedener Nervenzellen, verschiedener Neurotransmitter und Rezeptoren und es gibt zigtausende verschiedener Substanzen. Daher ist seine Funktionsweise noch lange nicht ganz bekannt. Es mit Elektronik oder Computern zu vergleichen, ist geradezu naiv und erschwert das Verständnis noch zusätzlich. Unverständnis erweckt wilde Spekulationen.

    Ein künstliches Gehirn hätte mit Sicherheit andere Formen von Bewusstsein, wenn überhaupt.

  303. @Hoppe: Lebensweisheit

    Ich musste an eine Textstelle des belgischen Sanskritologen Prof. Winand Callewaert (Uni Leuven; *1943) denken, über die ich mich beim ersten Lesen maßlos geärgert habe. Callewaert wurde übrigens als junger Mann von den Jesuiten nach Indien geschickt, um Inder zum katholischen Glauben zu missionieren. Die Mission war nicht sehr erfolgreich (was wahrscheinlich viel mit der Arroganz der Missionare zu tun hatte). Callewaert war anders: studierte die indischen Sprachen und kehrte gewissermaßen in den Hinduismus verliebt zurück nach Europa. Er schrieb, ich übersetze:

    Im Zusammenhang mit dem Karma des Hinduismus wird schnell suggeriert, dass es ein einfaches System ist, um die Armen weiterhin zu unterdrücken, dass es “Opium” ist, um Menschen, denen es jetzt nicht gut geht, für später ein besseres Leben vorzuspiegeln. Es ist fesselnd zu sehen, dass genau diese Argumentation von den Marxisten verwendet wurde, um die Unterdrückung durch die Kirche im vorherigen Jahrhundert [gemeint ist wohl das 19.] zu verurteilen. Darf man hierbei allerdings anmerken, dass eine Ideologie, die eine heilvolle Zukunft vorspiegelt, vielleicht nicht immer zu unrecht die Basis dafür war, ein schwieriges Leben leichter zu überstehen?

    aus: India: Betoverende verscheidenheid (2001)

    Gilt nicht nach wie vor die Lebensweisheit, dass man ändern soll, was man ändern kann, und akzeptieren soll, was man nicht ändern kann? Und bei dieser Akzeptanz kann einem ein Glaubenssystem vielleicht helfen, oder nicht?

  304. @Stephan / 19.01.2021, 14:15 Uhr

    Übersetzungsregel: “x ∈ W” bedeutet hier “x existiert“. Nicht mehr und nicht weniger.

    Das kommt eben dabei heraus, wenn Existenz begriffen wird als Zugehörigkeit zu einem bestimmten Gegenstandsbereich, vulgo “Welt”. Und dass sich der Existenzbegriff so nicht fassen lässt, ist ja auch eine zuerst von Kant formulierte Einsicht. Die sich leider nicht überall herumgesprochen hat. Speziell nicht bei Materialisten wie Mario Bunge — möge er in Frieden ruhen.

  305. @Chrys // 19.01.2021, 14:55 Uhr

    Übersetzungshilfe:

    Wofür steht das x ? Für „Existenz“? Oder ebenfalls für „Welt“?

  306. @Joker // 19.01.2021, 14:31 Uhr

    » Um die physikalische Wirksamkeit mentaler Phänomene zu behaupten, reicht es meines Erachtens, das durch diesen Gedanken ausgelöste Schmunzeln als eine von vielen messbaren Folgen anzuerkennen. «

    Wie darf ich das verstehen? Von wessen Gedanken ist die Rede?

    Deine Gedanken lösen bei mir kein Schmunzeln aus, aber was ich von Dir lese, schon.

    Oder geht es nur darum, dass Du schmunzeln musstest, als Du diese(n) Gedanken gehabt hast?

    Wenn ich lese, was Da nach einigem Nachdenken niedergeschrieben hast, und wenn ich dann Schmunzeln muss, sind es dann Deine oder meine Gedanken, die kausal wirksam werden?

  307. Christian Hoppe,
    wenn wir bei Metaphern sind, dann sind wir schon sehr weit gekommen.
    Wie kann man Farben erklären, wenn ein Mensch farbenblind ist.
    So und jetzt muss ich polemisch werden.
    Wie soll man einem Mensch Religion erklären, wenn er religionsblind ist.

    Für einen Menschen, der eine Gotteserfahrung gemacht hat, ist Gott existent.
    Einem Menschen, der verliebt ist, dem können sie nicht klarmachen, dass dies nur eine hormonelle Störung ist. Die Liebe ist eine Gottesgabe. Ohne sie wäre die Menschheit ärmer.

    Das Gott – Welt – Problem spiegelt nicht nur auf der individuellen Ebene, dann wäre es in der Tat im Bereich des Bewusstseins angesiedelt.

    Reicht das Wunder Mensch nicht als Beweis für einen Schöpfergott. Was wollen sie noch ? Verlangen Sie, dass er sich vor ihnen rechtfertigt ?

  308. @Hoppe
    Der Beitrag von Herrn Prof. Grams ist ein eindeutiger Esoteriker-/Querdenker-Text der das erklärte Ziel hat die wissenschaftliche Denkweise (Naturalismus) lächerlich zu machen und zu suggerieren, dass es unerklärbare/übernatürliche Phänomene gibt.

    Für das was suggeriert werden soll, gibt es keine vorgelegten Beweise/Belege – welche nachvollziehbar wären.

    Aus diesem Grund nützt es nichts, wenn Sie versuchen, naturwissenschaftlich und sachlich zu argumentieren: Sie werden es NIE schaffen, etwas zu widerlegen, was es gar nicht gibt.

  309. @ Stephan Schleim 19.01.2021, 14:25 Uhr

    Zitat: „Wenn die absolute Wahrheit für uns unerreichbar ist, könnte es dann nicht aus pragmatischen Gründen sinnvoll sein, sich ein Überzeugungssystem zu wählen, mit dem man besser durchs Leben kommt?“

    Das optimale „Überzeugungssystem“ zu wählen, mit dem man besser durchs Leben kommt, das auch ethisch halbwegs korrekt scheint, dürfte heutzutage bei uns am öftesten von sehr vielen Menschen realisiert werden.

    Kausal, determiniert, oder zufällig das ist heutzutage oft die Frage.

    Ist ein System, auch ein Mensch immer „kausal“ in seinem Handeln ist er berechenbar und kann „ausgenutzt“ werden.

    Handelt ein System immer „zufällig“ ist es unbrauchbar, außer man benutzt es absichtlich als „Zufallsgenerator“.

    So scheint es „optimal“ geworden zu sein, dass viele Menschen kausal handeln, aber wenn sie merken z.B. manipuliert, ausgenutzt, ausgebeutet zu werden, so „schalten“ sie auf „zufällig“ oder ganz krass, auf grundsätzlich „konträr“ als Protest. Wie meiner Meinung nach derzeit unter Trump in Amerika.

    Meiner Meinung nach ist eine Art von Mischsystem aus „kausal“ und „zufällig“ die Grundlage der Evolution. Die vielen Zyklen (Tag – Nacht, Jahreszeiten, Herzschlag, zyklische Signalkaskaden im Gehirn, ….) die es gibt, generieren im Zusammenwirken mit Schaltprozessen bei der Genexpression „determinierte“ (sozusagen „vorausberechenbare“) Zufallsvariable, allenfalls aber auch wirklich fast absolut zufällige „Mutationen“, die bedeutsam für die Evolution waren.

  310. KRichard das 6. Rätsel

    Alte Inschrift auf einem Hauseingang: Ich bin, ich weiss nicht wer. Ich komme, ich weiss nicht woher. Ich gehe, ich weiss nicht wohin. Mich wundert, dass ich so fröhlich bin. 

  311. @Balanus / 19.01.2021, 15:27 Uhr

    »Wofür steht das x ?«

    Das x ist eine Variable, für die Du alles als grammatisches Subjekt einsetzen darfst, was Dir in den Sinn kommt und aus der Aussageform “x existiert” einen grammatikalisch korrekt geformten Satz macht. Ob der so gebildete Satz dann wahr oder falsch ist, hinge dann nur davon ab, ob das jeweils eingesetzte “Ding” prinzipiell in der Welt zu finden ist oder nicht.

  312. @Joker
    Auf Ihren Kommentar zu “fast immer” habe ich zunächst nicht geantwortet, da ich im großen und ganzen zustimme.

    Über Feinheiten der Wortbedeutung (‘unabhängig’, ‘total unabhängig’) sollten wir nicht weiter streiten. Das läuft sonst auf Sophisterei hinaus; ich stimme Ihnen ja zu: Man kann’s so sehen – aber eben auch anders.

    Zu den (reinen) Denkinhalten zählt Martin Mahner nicht nur Einhörner und dergleichen, sondern unter anderem auch mathematische Gegenstände und Strukturen. Ohne diese gäb’s keine Flugzeuge, keine der eleganten Schrägseilbrücken, eigentlich fast nichts von dem, was unser modernes Leben ausmacht. Also haben sie ganz reale Auswirkungen. Sogar die Einhörner finden wir in einschlägigen Läden als Figuren wieder – ganz real.

  313. @Christian Hoppe 19.01. 14:19

    Es gibt, was die beobachtbaren materiellen Vorgänge angeht, einfach keine Erklärungslücke, “wo die Psyche dann wirksam wird”.

    Man kann hier aber eine Lücke sehen: Wenn die Quantenzufälle auch mal gezielt sein können, dann kann man z.B. wenn man Computerkunst macht, statt der üblichen mathematisch generierten Zufallszahlen besser „analoge“ Zufallszahlen verwenden. Die kann man sich ziemlich einfach z.B. aus dem Pixelrauschen einer abgedunkelten Webcam extrahieren.

    Jetzt hoffe ich als Künstler einfach mal, das ich damit etwas bessere Computerbilder bekomme. Und offline hoffe ich als begeisterter Skatspieler auf bessere Karten. So viel mehr verspreche ich mir auf der Ebene der physischen Vorgänge gar nicht von Seiten eines vermuteten kosmischen Geistes. Und der ist ja nicht mal psychisch, sondern gehört zur Ausstattung der Kosmos dazu, und wäre, wenn es ihn denn gibt, wohl an der Psyche nur beteiligt aber ansonsten auch anderweitig mit der „Schöpfung“ beschäftigt.

    Das Bewusstsein selbst als Geistig zu betrachten ist noch mal eine ganz andere Geschichte. Hier weis ich nicht, was da wirklich abläuft, und hoffe da mittelfristig, dass man mal ein Mäusekonnektom so genau untersuchen kann, dass man da entweder keine Erklärungslücke mehr hat, und damit der Naturalismus in diesem Fall gut begründet wäre. Oder man merkt, dass die Simulation des Mäusekonnektoms nur „aufwachen“ kann, wenn die Zufallseffekte, die hier sicherlich vielfältig beteiligt sind, in der Simulation mit analogen Zufallszahlen umgesetzt werden.

    Ich halte es für unnötig voreilig, hier entsprechende Ergebnisse vorweg zu nehmen. Wir wissen es nicht, aber wir werden es wissen können, meine ich.

    @Chris 19.01. 16:15

    „Ob der so gebildete Satz dann wahr oder falsch ist, hinge dann nur davon ab, ob das jeweils eingesetzte “Ding” prinzipiell in der Welt zu finden ist oder nicht.“

    Eben auf die gezielten Zufälle angewendet: Man kann sie mal suchen und finden, aber auch in vielen Bereichen nicht finden. Je sporadischer diese Zufälle auftauchen, desto eher sieht man sie nicht. Ich würde gerne „3. Der Bereich physikalischer Phänomene ist kausal geschlossen.“ nur prinzipiell ausschießen, im Sinne von Ausnahmen von der Regel. Viele physikalischen Phänomene sind kausal geschlossen, aber eben nicht alle bzw. nicht immer vollständig geschlossen.

    Genug Platz für innere Geisteswelten, darum geht es eigentlich, mir zumindest. Das ist einerseits auch eine Frage von Freiheit, und andererseits eine von Geborgenheit.

  314. @Jeckenburger
    Wissenschaftliche Erkenntnisse können weder Freiheit noch Geborgenheit bieten, das sind ganz andere Kategorien. Sie werfen hier Kraut und Rüben in einen Topf, wie “gezielte Zufälle” von Quanten, um Ihre “innere Geisteswelt” zu verteidigen. Diese kann Ihnen ohnehin niemand nehmen.

    Die Entzifferung eines Mäusekonnektoms würde nichts nützen, denn das Fremdpsychische kann damit nicht entlarvt werden, d.h. man würde nicht erkennen können, ob die Maus gerade an Eifersucht leidet. Mensch und Maus haben übrigens 99% gemeinsame Gene, bezogen auf das Mausgenom.

    Der Zusammenhang von mentalen Zuständen mit physischen Zuständen des Nervensystems ist durch die individuelle Ontogenese bestimmt und nicht naturgesetzlich gegeben. Die Reize der individuellen Umwelt prägen die physische Struktur des Nervensystems innerhalb der möglichen Variationsbreite. Es schließt nicht aus, dass bestimmte Typen physischer Zustände mit bestimmten Typen psychischer Zustände korrelieren (Typenidentität), z.B. Angst auslösende Reize und Angstgefühl.

  315. Es ist bekannt und belegt dass es bewusste Erlebnisse bzw. ein Bewusstsein nur im Zusammenhang mit konkreten physikalischen/neuronalen Aktivitäten gibt.
    Als konkreter Beleg gibt es dafür aktuell 7,8 Mrd. Menschen – wo man jeweils erkennen kann: im Wachzustand gibt es ein Bewusstsein – im Schlaf ist das Bewusstsein weg.

    Für die Idee eines nichtphysikalischen Bewusstseins gibt es keinen einzigen konkreten Beleg/Nachweis.

  316. @anton reutlinger 19.01. 18:12

    „Sie werfen hier Kraut und Rüben in einen Topf, wie “gezielte Zufälle” von Quanten, um Ihre “innere Geisteswelt” zu verteidigen.

    Ich brauche meine innere Geisteswelt nicht verteidigen, bin aber auf der Suche nach deren Einordnung, auch in die Möglichkeiten, die die Physik offenlässt.

    „Die Entzifferung eines Mäusekonnektoms würde nichts nützen, denn das Fremdpsychische kann damit nicht entlarvt werden, d.h. man würde nicht erkennen können, ob die Maus gerade an Eifersucht leidet.„

    Man könnte doch sehen, ob die Maussimulation wach ist, und sich entsprechend so verhält wie das natürliche Vorbild. Wenn man mit echten Mäusen psychologische Experimente macht, hat man auch das Problem des Fremdpsychischem, und kann trotzdem Forschungsergebnisse erzielen.

  317. @KRichard

    Danke. Sie haben vermutlich Recht.

    Ich selbst habe 30, 40? Jahre gebraucht, um schlicht und einfach die Gültigkeit der Physik für materielle Phänomene anzuerkennen – ohne phil.-theol. Ausflüchte (ich bin ja auch Theologe). Es scheint eine besonders große Schwierigkeit darin zu liegen, inbesondere Verhalten *auch* als materiellen Prozess zu sehen (was immer es noch sein mag). Wenn es dann aber schließlich doch gelingt, hat man für das allermeiste, was *real* und *beobachtbar* geschieht, eine *prinzipielle* Erklärung in der Physik gefunden. Dass das dann – mit einem anderen epistemischen Zoom auf die komplexe Realität – auch chemisch, molekularbiologisch, neurophysiologisch usw. und am Ende psychologisch fortgeführt werden muss, steht außer Frage – aber alle diese Wissenschaften setzen die Gültigkeit der Physik selbstverständlich voraus und kennen keine anderen Wechselwirkungen (“Kräfte”) als die in der Physik beschriebenen. Insbesondere gibt es keine psychologischen Kräfte (z.B. Vorstellungskraft oder Willenskraft).

    Wie gesagt, mir ist völlig wumpe, was jemand glaubt, wenn er damit die Wissenschaft nicht verdirbt und wenn er sich kompetent und kooperativ in die Gemeinschaft einbringt.

  318. @Chrys // 19.01.2021, 16:15 Uhr

    » Das x ist eine Variable,… «

    Danke, da hätte ich auch selber drauf kommen können (mit etwas mehr Nachdenken; da waren wohl einige Synapsen blockiert… 🙂

    In Deinem heutigen Kommentar (13:52 Uhr) hast also vorgeführt, wie leicht man eine allumfassende Welt „quinen“ kann—sofern man das logische Rüstzeug dafür besitzt.

    Wäre denn die Existenz einer ontisch-monistischen Welt zu retten, wenn sie nur ein winziger Teil einer größeren Welt wäre? Also Teil des dann wirklich allumfassenden Multiversums?


    @18.01.2021, 14:52 Uhr

    »Doch es besteht keinerlei Widerspruch zwischen subjektiver und intersubjektiver Sicht, das ist miteinander kompatibel. Wo also sollte da nun eigentlich ein Problem sein? Für einen Naturalisten wie Dennett ist da offenbar eines, weshalb er die subjektive Sicht dann unbedingt “quinen” will — erscheint Dir das vernünftig? «

    Mir erscheint es höchst vernünftig, dass und vor allem, wie er es versucht, mit welchen Argumenten (15 „intuition pumps“). Denn eins ist ja mal klar: Mit den subjektiven Eindrücken kann niemand etwas anfangen, außer man selber.

    Danke für den Hinweis auf dieses schöne Stück Philosophie!

  319. @Balanus: Mäuse

    Das kann man jetzt aber auch umdrehen und schlussfolgern, dass die Probleme nicht auf der genetischen Ebene liegen können, wenn etwa die Mäuseforschung in der Psychiatrie seit Jahrzehnten so ergebnislos ist und bleibt.

    Danke für das gute Beispiel!

  320. @Chrys: Logik, zweiter Versuch

    Dass “x ist ein Element der Welt” gleichbedeutend ist mit “x existiert”, war mir klar; es ging doch ausdrücklich um den anderen Satz, “die Welt ist ein Element der Welt”.

    Ich denke zudem, dass hier eine Äquivokation vorliegt und das erste “Welt” etwas Anderes bedeutet als das zweite “Welt”, nämlich erstens die real existierende Welt und zweitens eine abstrahierte Welt. Ob dann der logische Schluss noch funktioniert, lasse ich mal im Raum stehen.

    Wie dem auch sei: Aus dieser sprachlich unklaren und umständlichen Formulierung eine ontologische Schlussfolgerung über die Unmöglichkeit des Materialismus ziehen zu wollen, halte ich für sehr gewagt!

    P.S. Gibt Gabriel eigentlich die älteren Quellen des Schlusses an oder verkauft er das als seine eigene Idee?

  321. @Stephan // Gene und Gehirn

    Ich meine, irgendwo mal gelesen zu haben, dass die genetischen Unterschiede zwischen Mensch und, ich glaube, es ging um einen Primaten, bei regulatorischen Genen am größten ist, und zwar bei jenen, die an der Hirnentwicklung beteiligt sind.

    Passend dazu noch ein Fund aus Bonn:

    Intelligenzdefizit: Forscher schließen von der Maus auf den Menschen


    Noch kurz hierzu, bevor ich die Augen zu mache:

    » Ich denke zudem, dass hier eine Äquivokation vorliegt und das erste “Welt” etwas Anderes bedeutet als das zweite “Welt”, nämlich erstens die real existierende Welt und zweitens eine abstrahierte Welt. «

    Guter Einwand, gefällt mir, finde ich einleuchtend.

    (Sofern es eine real exisiterende Welt überhaupt gibt–was wir aber alle mal hoffen wollen…)

  322. @Jeckenburger @Reutlinger

    Mit dem US-Präsidenten Trump haben wir einen konkreten Beleg für die menschliche Dummheit: er konnte jahrelang beliebig viele Lügen und Unsinn verbreiten und fand dann immer Leute, die ihm kritiklos zustimmten und seine Aussagen für eine Wahrheit hielten.

    Der Grund dafür ist klar: Die menschliche Dummheit ist grenzenlos.

    Einen ähnlichen Mechanismus haben wir bei der Idee vom nichtphysikalischen Bewusstsein.
    Es wird immer Leute geben, welche diese Idee nicht hinterfragen und nachvollziehbare Belege verlangen – sondern es vorziehen daran zu glauben.
    Notfalls hat das Ganze dann quantenphysikalische Ursachen.

  323. noch eine Anmerkung:
    Wenn vermutet wird, dass ein ´nichtphysikalisches Bewusstsein´ eventuell quantenphysikalische Ursachen/Grundlagen haben könne – dann ist diese Vermutung/Sichtweise ein Unsinn.

    Quantenphysikalische Phänomene sind immer physikalischer Natur!

  324. @Hoppe
    Sie haben den Prozess Ihrer “Mühen” auf scilogs mitgeteilt. Ich war beeindruckt, dass sie das öffentlich machten.
    Für mich ist es ein Hinweis darauf, nochmals intensiv über Prägung/Autosuggestion/Lernen nachzudenken.
    In diesem blog geht es um das Bewusstsein und was sich in ihm (uns) abspielt und es geht rund um das Thema Widerspruch im/ind diesem Bewusstsein: Widerspruchsfreiheit, Logik, Paradox, Komplementarität, Wechselwirkung, scheinbare und wirkliche Widersprüche, Kausalität, Zufall, Grenzen und eben auch Glaube.
    Eine Hypothese, eine Annahme, ist der “naheliegend vermutete” Glaube an etwas Wirkliches und somit Vorraussetzung für Forschung/Wissen. Glaube spielt somit für Prägung/Lernen/”Wissen”, auch für Kognitive Verzerrungen, eine sehr wichtige Rolle.
    Auch Wissen und Glaube sind in meinen Augen komplementär, ohne jetzt beides ausdefinieren zu wollen.
    Aber was wäre das für ein Leben, wenn tatsächlich alles wirklich kausal wäre? Das Faszinosum, die Faszinität für das Leben, Konflikt und vermutlich Humor usw wäre weg. Es wäre die totale Linearität wohin auch immer. Völlig unspektakuläre und die SPANNUNG (!) wäre weg.
    Auch und gerade wenn wir feststellen, dass vermutlich “Geist” viel mit den Wechselwirkungen/ Komplemetarität zu tun hat (nichtlösbare aber entsprechende „Gegensätze“ : Kooperation-Konkurrenz/ Sieg-Niederlage / Gewinn-Verlust/ Abstraktion-Konkretheit usw.), scheint mir Kooperation im komplett gesellschaftlichen Wesen – noch – zu sehr unterrepräsentiert, kaum wirklich wahrgenommen, obwohl sie „allgegenwärtig und -mächtig“ ist.
    Ihnen alles Gute und danke für die Darstellung Ihres stellvertretende öffentlichen Prozesses des „Kampfes“ !

  325. @Mussi
    Man darf Gegensätze nicht mit Komplementarität verwechseln. Gegensätze sind gleichartige Sachverhalte, die sich gegenseitig ausschließen, schwarz oder weiß; Komplementarität sind unterschiedliche Sachverhalte, die sich gegenseitig ergänzen (oder vertreten), Ort oder Zeit.

  326. @Mussi

    Dass Glauben eine epistemische Einstellung ist, die das Denken und Erleben jedes Menschen prägt, stimmt – aber das doch bitte nicht Verwechseln mit religiösem Glauben. Jeder muss im o.g. Sinne glauben – aber niemand muss (und sollte) an übernatürliche Wesenheiten und die Wirkung paranormaler Praktiken glauben; das ist alles Blödsinn und Zeitverschwendung. Oder genauer: die faszinierende Psychologie der Dissoziation.

    Ebenso spricht die von mir nie bestrittene Existenz psychischer Phänomene in keiner Weise gegen die durchgängige Gültigkeit der Physik für alle physischen Vorgänge (die z.B. auch beim Lachen unübersehbar beteiligt sind).

    Wir sind heute weiter. Die Neurowissenschaften der letzten 200 Jahre waren noch einmal ein wichtiger Schritt. Es ist an der Zeit, Religion etc. jetzt hinter uns zu lassen. Im Grunde ist es auch nicht traurig, wenn man eine Illusion fallen lässt, sondern befreiend.

  327. @ Christian Hoppe 19.01.2021, 18:54 Uhr

    Zitat: „…alle diese Wissenschaften setzen die Gültigkeit der Physik selbstverständlich voraus und kennen keine anderen Wechselwirkungen (“Kräfte”) als die in der Physik beschriebenen. Insbesondere gibt es keine psychologischen Kräfte (z.B. Vorstellungskraft oder Willenskraft).“

    Wie möchten Sie das Faktum anders als metaphorisch beschreiben, dass große „Anziehungskräfte“, allenfalls auch „Abstoßungskräfte“ z.B. zwischen Menschen unterschiedlichen (allenfalls auch gleichen Geschlechts) bestehen, beschreiben?

    Für mich und ich bin da vermutlich absolut in der Minderheit, dürften Empfindungen auf physikalische, eigentlich quantenphysikalische Effekte zurückgeführt werden können.

    Wenn durch irgend einen „auslösenden“ Einfluss in („sensorischen“) Molekülen sich „innere“ physikalische Kraftwirkungen verändern, so dass z.B. Elektronen aus ihren „Bahnen fliegen“ und frei werden, so dass sie im neuronalen System ausgewertet und z.B. lokalisiert werden. Dabei die bekannten Signalmuster die den generativen Prozess einleiten und steuern, aktiviert werden….

    Jedenfalls hat es die Natur und auch der Mensch geschafft, Information abbildende Objekte zu schaffen, bei denen nur die „abstrakte Information von Bedeutung“ ist, die immer gemäß den Gesetzmäßigkeiten der Physik verarbeitet und gespeichert wird. Es ist aber nur die „beschreibende“ oder „Prozess steuernde“ Information von Bedeutung, erst in 2. Linie die verwendete Physik. Information kann in Stein gehauen, mit Bleistift auf einem Stück Papier geschrieben, oder eben auf der DNA vorhanden sein…

    Information und deren Verarbeitung hat in der Informatik und der Technik immer größere Bedeutung erlangt. Von „Hardcore Materialisten“ wurde lange bestritten, dass es derartiges überhaupt gibt und die „Pfaffen“ die ebenfalls derartiges (wie die Existenz von „Geistigem“) behaupteten, wurden gerade noch geduldet, der Verkauf von Software (wobei nur die Information, kaum aber der Datenträger von Wert war) galt lange Zeit als Betrug….

  328. K.Rrichard,

    „Für die Idee eines nichtphysikalischen Bewusstseins gibt es keinen einzigen konkreten Beleg/Nachweis. „

    Das ist eine Leerformel. Sie benutzen den Begriff des Bewusstseins sowohl für den Container, das Gehirn, als auch dessen Inhalt, die Ideen.

    Wenn man sich eine Idee, einen Gedanken als ein elektromagnetisches Phänomen denkt, dann stimmt das soweit. Was sie vergessen, den Inhalt der Idee, den Sinn der Idee.
    Wie unterscheidet sich dann ein logischer Gedanke physikalisch von einem unlogischen Gedanken.

    Und womit wir täglich zu kämpfen haben, das öffentliche Bewusstsein, das wir Kultur nennen.
    Wenn sie einen Toten sezieren, dann finden sie kein Bewusstsein.
    Das liegt dann auf der gleichen Vorstellungsebene, wo man nach dem Gottesgen sucht.

    Sie kommen mir vor wie ein Automechaniker, der mit Ratsche und Schraubenzieher den Sinn eines Steuergesetzes verstehen will. (kleiner Spaß)

  329. Da in diesem Blogpost viel von Bewusstsein die Rede ist (und auch Buddhismus erwähnt wird), erlaube ich mir unten einen Auszug aus einem (englischsprachigen) buddhistischen Handbuch zu dem Thema anzuführen (übersetzt mit Hilfe von Google Translate und DeepL sowie Änderungen von mir). Erwähnen möchte ich, dass ich kein “100%“-iger (oder permanenter) Buddhist bin (es gibt Anlässe zu Kritik). Ich halte den Buddhismus aber für, Mal so formuliert, wahrlich das noch “geringste Übel“ unter all diesen “Spiritualitäten“. Darüber hinaus interessiere ich mich auch für Naturwissenschaften. Sollte Bedarf zu Anmerkungen bestehen die über den Rahmen dieses Blogposts hinausgehen (off topic), hier meine E-Mail: akasataki@gmail.com.

    (2) Abhängig von karmischen Gestaltungen entsteht Bewusstsein: Das heißt, die karmischen Gestaltungen – die neunundzwanzig heilsamen und unheilsamen Willensäußerungen – bedingen das Entstehen der zweiunddreißig Arten des resultierenden Bewusstseins. Im Moment der Empfängnis erzeugt eine besonders starke karmische Gestaltung, die sich im mentalen Kontinuum des verstorbenen Wesens angesammelt hat, eine der neunzehn Arten des Wiedergeburtsbewusstseins in dem Bereich, der für die Reifung dieses Karmas geeignet ist. Danach erzeugen im Laufe der Existenz andere angesammelte Karmas je nach den Umständen andere resultierende Bewusstseinsarten [..].

    (3) Abhängig vom Bewusstsein entsteht Geist und Materie: Während sich in Schritt (2) Bewusstsein ausschließlich auf das resultierende Bewusstsein bezieht, bedeutet es hier sowohl das resultierende Bewusstsein als auch das karmische Bewusstsein früherer Leben. Der Begriff “Geist” bezeichnet die mentalen Faktoren, die mit dem resultierenden Bewusstsein verbunden sind, der Begriff “Materie” bezeichnet materielle Phänomene, die durch Karma erzeugt werden. In der Existenz mit fünf Bestandteilen, d.h. in jenen Bereichen, in denen alle fünf Aggregate (Materie, Gefühl, Wahrnehmung, mentale Gestaltungen, Bewusstsein) gefunden werden, bedingt das Bewusstsein sowohl den Geist als auch die Materie. Aber in der Existenz mit vier Bestandteilen, den immateriellen Bereichen, bedingt es nur den Geist. Und in einer einteiligen Existenz, dem Bereich der nicht wahrnehmenden Wesen, bedingt es nur die Materie. Im Falle einer Wiedergeburt mit fünf Bestandteilen, wenn das Wiedergeburtsbewusstsein im Moment der Wiedergeburtsverknüpfung entsteht, entstehen gleichzeitig die anderen drei geistigen Aggregate von Gefühl, Wahrnehmung und mentalen Gestaltungen zusammen mit einer bestimmten Verbindung materieller Phänomene – im Falle des Menschen die materiellen Zehnergruppen des Körpers, des Geschlechts und der Herzbasis. Da das Bewusstsein das Haupt dieser koexistierenden geistigen und materiellen Elemente ist, wird gesagt, dass das Bewusstsein Geist und Materie bedingt.

  330. @Axel Krüger

    (3) Abhängig vom Bewusstsein entsteht Geist und Materie

    Wenn es Bewusstsein ist – wer erlebt denn da? Sie können sich doch nicht ein bewusstloses Bewusstsein (wie so eine Art Sauce) vorstellen?!!! Wenn dieses Bewusstsein Ihr Bewusstsein erklären soll – wieso waren Sie denn dann nicht immer schon bei Bewusstsein, vor der Erschaffung der Materie oder auch nur Ihres Körpers? Wieso können Sie überhaupt zeitweise ohne Bewusstsein sein??

    Wo ist denn ein einziges Beispiel dafür, dass Bewusstsein nicht nur das Erkennen von Materie ermöglicht, sondern Materie hervorbringt? – Während es doch offensichtlich anders herum ist, wenn Sie nur an die Narkose denken! Selbstverständlich hängt bewusstes Erleben EINSEITIG von Hirnprozessen ab!

    Merken Sie denn nicht, dass das alles ein gigantisches Blabla ist von Leuten, die glauben die Welt vom Sitzkissen aus verstehen zu können, dabei aber nicht einmal die einfachsten wissenschaftlichen Erkenntnisse verstehen??

  331. @Chrys, Joker
    Hat man erst einmal akzeptiert, dass unser Denken in der einen Welt, stattfindet, dann fangen die Schwierigkeiten eigentlich erst an. (Dabei können wir die Frage, wie die Binnensicht der Denkprozesse zustande kommt, das fünfte Welträtsel, einstweilen beiseite lassen.)

    Es kommt zur Selbstreferenz, die ins Paradoxe führt. Chrys hat mit seiner Anspielung auf die Paradoxien der Mengenlehre – das Russellsche Paradoxon ist eins von ihnen – angedeutet, was da auf uns zukommt.

    Ich zitiere aus dem Buch “Philosophy and the Mirror of Nature” von Richard Rorty (1979/2009, S. 297). Rorty gibt Jay Rosenberg wieder, der schreibt: “To put it crudely, we must come to see the pyhsical universe as an integrated physical system which necessarily “grows knowers” and which thereby comes to mirror itself within itself.”

  332. @Hoppe: Neurowissenschaften und das Leben

    Wir sind heute weiter. Die Neurowissenschaften der letzten 200 Jahre waren noch einmal ein wichtiger Schritt. Es ist an der Zeit, Religion etc. jetzt hinter uns zu lassen. Im Grunde ist es auch nicht traurig, wenn man eine Illusion fallen lässt, sondern befreiend.

    Nun ja – wir hatten mit Feuerbach, Marx, Nietzsche und Freud auch schon treffende Religionskritik. (Gott ist Opium für das Volk; Gott ist eine Projektion des menschlichen Geistes usw.)

    Aber dann zurück zu meiner Frage: Was soll denn dann kommen, nachdem wir die “Illusion Religion” fallengelassen haben und wir Bewusstsein als Produkt von Neuronenfeuern im Gehirn auffassen? Welches Menschen- oder Weltbild soll uns das vermitteln?

    Ist das neurowissenschaftliche Wissen nicht vielmehr völlig irrelevant für die Ausgestaltung unseres Lebens? (Wenn man von der Neurologie und der Behandlung von Erkrankungen des Nervensystems absieht.)

  333. @ Krichard 20.01.2021, 05:01 Uhr

    Zitat: „Mit dem US-Präsidenten Trump haben wir einen konkreten Beleg für die menschliche Dummheit: er konnte jahrelang beliebig viele Lügen und Unsinn verbreiten und fand dann immer Leute, die ihm kritiklos zustimmten und seine Aussagen für eine Wahrheit hielten.
    Der Grund dafür ist klar: Die menschliche Dummheit ist grenzenlos.“

    Ich fürchte Sie sehen das nicht realistisch genug.

    Amerika wird einfach das neu „Schlachtfeld“ des Klassenkampfes. Die halbe Welt wird „angezogen“ vom Wohlstand Amerikas und will dorthin. Einwanderer wollen sich nicht mehr brav ausbeuten lassen wie die Einwanderer früher, sondern vom Rauschgifthandel leben oder sich einfach mit Demos auf der Straße holen, was ihnen vermeintlich „zusteht“.

    Trump war noch kaum weg und schon setzte der große „Run auf Amerika“ wieder ein. In Honduras haben sich Berichten zufolge mehrere Tausend Menschen zu einer Migrantenkarawane zusammengeschlossen und ziehen nach Amerika….

    Die „andere Seite“ hat aber nicht dafür gearbeitet um sich wieder alles abnehmen zu lassen, wie die fleißigen Bienchen. Auch sie gehen neuerdings auf die Straße, was Zuwanderer können, Druck machen, Straßenzüge verwüsten, das können sie schon lange…..

    Die „Wahrheit der Medien“ und die Träume vom „Gutmenschentum“ (das zu Lasten derjenigen geht die sich Wohlstand erworben haben) interessiert sie nicht mehr im geringsten, Trump macht das erst so richtig transparent mit seinen 100 000 Lügen.

    Besonders die Medien sollte die Realität sehen und nicht Träumen nachhängen. In Amerika ist der „Zug abgefahren“, Richtung Bürgerkrieg 2.0….

  334. Christian Hoppe,
    “Wir sind heute weiter. Die Neurowissenschaften der letzten 200 Jahre waren noch einmal ein wichtiger Schritt. Es ist an der Zeit, Religion etc. jetzt hinter uns zu lassen. Im Grunde ist es auch nicht traurig, wenn man eine Illusion fallen lässt, sondern befreiend.”
    Wir haben Feuerbach, Nietzsche, Marx, Popper, Dawkins verinnerlicht und was übrig bleibt, das ist der Gottesglaube und die Frohe Botschaft des Neuen Testamentes. Auch ein Papst kann daran nicht rütteln, der Klerus wird sich auch mal dessen bewusst werden.

    Religion hinter sich lassen. Was soll denn das heißen ? Sie können Gott nicht ablegen wie einen gealterten Schuh. Die Probleme der Zukunft, die werden den Hochmut der Menschheit wieder zurecht stutzen.

  335. @Krüger: Karma

    Was halten Sie denn von dem hier formulierten Gedanken, dass man sich Karma ausgedacht hat, um Menschen in unteren Schichten besser zu kontrollieren? Und die Herrschaft der oberen Schichten zu rechtfertigen? Ähnlich wie bei uns die Sache mit dem Paradies und dem Gottesgnadentum (Herrschaft eines Königs von Gottes Gnade).

  336. @Reutlinger

    Wenn Komplementarität (Komplementaritätsprinzip) Teil der Erkenntnistheorie ist, dann müsste es nach der Komplemenatrität heissen:
    1.) Köper ist entweder oder Geist
    2.) Körper ist sowohl als auch Geist.

    Das kommt der Wirklichkeit aus meiner Sicht ziemlich nahe (sonst gäbe es kein Konnektom).
    Es entspräche auch der Kommentarlage/Positionen ( wenn man sie “zusammenbringt”).

    Und damit hätten wir ein neues Dilemma/Paradoxon.
    Es ist nicht eindeutig zu klären.

  337. Was in der Diskussion bis jetzt zu kurz kam:

    Wie wurde früher über Bewusstsein diskutiert und wie wird heute darüber diskutiert.
    Christian Hoppe erwähnte den Substanzdualismus und damit eine früher verbreitete Art einer „Lösung“ des Leib-Seele-Problems. Zitat:
    Der Substanzdualismus hält Materie und Geist überdies für Substanzen, d.h. für grundsätzlich nicht aufeinander reduzierbar.

    Christian Hoppe verwies darauf, dass bewusst zu sein, besser als Prozess aufgefasst wird. Tatsächlich wird in der Humanmedizin das genau so gehandhabt und man spricht dort vom Level of Consciousness, was implizit annimmt, dass Bewusstsein nicht einfach da ist, sondern quasi an- und abgeschaltet werden kann (mit allen Zwischenformen natürlich).

    Auch Stephan Schleim hat jetzt in mehreren Kommentaren von einem Prozess gesprochen, wenn es ums Bewusstsein ging. Das aber ist nur ein Beispiel von vielen dafür, wie sich auch in der Philosophie die Art wie über Bewusstsein gesprochen wird, ändert.

    Geändert hat sich auch die Zuschreibung von Bewusstsein zu Lebewesen. Für Kant hatten Tiere kein Bewusstsein weil ihnen die Fähigkeit zum Denken fehlt. Einige heutige Philosophen sehen das allerdings inzwischen anders. Vor allem natürlich diejenigen, die für Tierrechte eintreten wie etwa Peter Singer über den man in diesem Zusammenhang folgendes liest:

    Das Argument für das tierische Bewusstsein und die Notwendigkeit, ihre Interessen anzuerkennen, sagte Singer, wird durch die Ähnlichkeiten zwischen Tieren und Menschen belegt – anatomisch, physiologisch und verhaltensmäßig. Er wies auch auf die gemeinsame Evolutionsgeschichte der beiden hin.
    “Wir sind Tiere”, sagte Singer und zitierte Darwins Evolutionstheorie.

    Bei Timm Grams bin ich mir allerdings nicht sicher wie er zur Idee tierischen Bewusstseins steht. Er schreibt etwa:

    @Chrys, Joker
    Hat man erst einmal akzeptiert, dass unser Denken in der einen Welt, stattfindet, dann fangen die Schwierigkeiten eigentlich erst an. (Dabei können wir die Frage, wie die Binnensicht der Denkprozesse zustande kommt, das fünfte Welträtsel, einstweilen beiseite lassen.)

    Es kommt zur Selbstreferenz, die ins Paradoxe führt. Chrys hat mit seiner Anspielung auf die Paradoxien der Mengenlehre – das Russellsche Paradoxon ist eins von ihnen – angedeutet, was da auf uns zukommt.

    Diese Sätze machen in meinen Augen nur Sinn, wenn Timm Grams annimmt, erst Menschen könnten in einer Art Denken, welche einen Dualismus geistige/materielle Welt nötig macht. Begründung: das Problem der Selbstreferenz taucht erst bei relativ komplexen Denkprozessen auf. Bei Denkprozessen, die es bei Tieren selbst dann noch nicht gibt, wenn man auch Tieren einfache Denkvorgänge zugesteht.

    Doch gerade das ist für mich ein deutlicher Hinweis darauf, dass eben auch das Geistige eine materielle Grundlage hat und nicht ausserhalb dieser Welt liegt. Denn für mich gibt es keinen Zweifel, dass der Übergang vom Tier zum Mensch ein fliessender, gradueller ist. Dass mit dem Mensch der Dualismus in die Welt kam, scheint mir nicht sehr plausibel. Denn der Mensch ist für mich nicht das Besondere dieser Welt. Er ist für mich nur ein Geschöpf unter anderen.

  338. In Herrn Prof. Grams´ Text fehlen wichtige sachbezogene Punkte:

    – Für die suggerierte Idee eines nichtphysikalischen Bewusstseins gibt es keine nachvollziehbaren Belege.
    – Es fehlen Hinweise auf Experimente in Schlaflabors, wo man erforschte, wann/wie das Bewusstsein schwindet und wann/wie es wieder entsteht
    – Im Wachzustand haben wir ein Bewusstsein – im Schlaf haben wir kein Bewusstsein: Wo ist ein nichtphysikalisches Bewusstsein während des Schlafes?
    – Durch stufenweise Gabe von Narkosemitteln, konnte hat man bei Versuchspersonen untersuchen (fMRT), was passiert wenn das Bewusstsein schwindet und wann es wieder kommt
    – der Buddhismus beschäftigt sich seit 2500 Jahren mit dem Bewusstsein und stuft es als eine Illusion ein; die noch nicht einmal eine Dauer hat. Die Frage nach der Dauer ist eine zentrale Fragestellung: weil das Bewusstsein ständig neu entsteht und sofort wieder vergeht.
    – Unsere Wahrnehmung und unsere Erinnerungen sind extrem fehlerhaft – dies bedeutet, dass unser Bewusstsein die gleiche fragwürdige Qualität hat. (Tipp: Wenn die Museen wieder öffnen dürfen – besuchen Sie ein ´Museum der Illusionen´ (Stuttgart?) )
    – Dass man die Idee eines nichtphysikalischen Bewusstseins als Grundlage für irgendein Menschen-/Weltbild brauchen würde – ist durch den Buddhismus längst widerlegt. Diese Menschen sind genau so freundlich, nett, emphatisch, … wie Menschen anderer Kulturen bzw. Religionszugehörigkeiten.

  339. @ Mussi 20.01.2021, 12:05 Uhr

    Zitat: „Wenn Komplementarität (Komplementaritätsprinzip) Teil der Erkenntnistheorie ist, dann müsste es nach der Komplemenatrität heissen:
    1.) Köper ist entweder oder Geist
    2.) Körper ist sowohl als auch Geist.“

    „Körper“ und „Geist“ haben „umgangssprachliche“ Bedeutung, was normalerweise völlig reicht.

    In der Informatik wären derartige „Objektdeklarationen“ eine „Katastrophe“ und völlig ungeeignet, wenn man nur einige wenige, aber etwas “tiefer gehende“ Verknüpfungen mit ihnen anstellt, käme man sozusagen in „Teufels Küche“ …. Den Philosophen scheint es ähnlich zu gehen, aber sie haben das Problem nicht grundlegend gelöst, wie die Informatiker.

    Man kann z.B. informelle Objekte nur dann gleich setzen, wenn sie gleicher, sehr gleicher, „Kategorie“ angehören.

    Beispiel: Es ist in der Informatik ein schwerer Fehler wenn sie z.B. behaupten a=b („3“=“3“), wenn sie z.B. die linke Variable (a) als „natürliche (ganze) Zahl „deklariert“ haben, und die rechte Seite (b) z.B. als „Fließkommazahl“ deklariert haben.

    Selbst wenn Sie die linke als 3 stellige und die rechte Seite als 6 stellige Fließkommazahl deklariert hätten, würde der „Compiler“ die Verknüpfung entrüstet zurückweisen (Seitenlange Fehlermeldungen ausdrucken).

    Das ist den Informatikern recht bald klar, weil sie in frühen Praktika einen Compiler (eine Art von Übersetzungsprogramm von Programmiersprache in Maschinensprache) „basteln“ müssen.

    Sie könnten selbstverständlich mit einem „kleinen Programm“ eine natürliche Zahl in eine Fließkommazahl umwandeln und danach die Fließkommazahlen vergleichen. Auch umgekehrt, nur fallen dann die Kommastellen „unter dem Tisch“.

    Das ist letztlich der Grund, warum Informatikern die korrekten Kategorien so wichtig sind. „Geist“ und „Gehirn“ gehören unterschiedlichen Kategorien an, dürfen nicht gleichgesetzt werden.

    Bin deswegen so penibel, weil ich mir fast ein „Trauma“ aufgerissen habe, als ehemals ein Informatiklehrer fast ausgeflippt ist, weil ich einen „Zähler“ als Fließkommazahl deklarieren wollte.

    Die Informatiker Trennen nicht z.B. in “körperlich oder geistig” um die “Materialisten” zu ärgern oder “reich” zu werden (das wohl auch…), sondern darum, um korrekt und ohne Widersprüche Information verarbeiten zu können.

  340. @Stephan / 19.01.2021, 23:29 Uhr

    Wenn Dich die Mengen-Schreibweise irritiert, das lässt sich alles auch äquivalent mit einem trditionellen Existenzprädikat E sagen. Damit auch klar ist, wovon die Rede sein soll, beginnen wir mal beim Lemma `Welt’ aus dem UTB Handwörterbuch Philosophie:

    Welt Griech. kosmos, lat. mundus, engl. world, fr. monde: In der Philosophie bezeichnet der Begriff seit alters in äußerster Verallgemeinerung die Gesamtheit dessen, was ›ist‹.

    Mit den Definitionen W := {x | x ›ist‹} für die Welt sowie Ex :⇔ "x ›ist‹" für das Existenzprädikat ergibt sich die zuvor schon genannte Übersetzungsregel nun als formale Äquivalenz

    x ∈ W ⇔ Ex

    zwischen der Welt W und der sie charakterisierenden Eigenschaft E. Das führt dann zu EW ⇒ ¬EW und bleibt kontradiktorisch. Oder eben in ontologischem Deutsch: Wenn die Welt ›ist‹, dann ›ist‹ sie nicht.

    Die Texte von Gabriel habe ich jetzt gerade nicht zur Hand. Sein für philosoph. Laien geschriebenes “Warum es die Welt nicht gibt” darf man sicherlich als Eigenwerbung für seinen “Neuen Realismus” sehen, wo er sich selbst ins rechte Licht zu setzen bemüht ist. Er hatte zuvor aber auch ein Lehrbuch zur Erkenntnistheorie verfasst, wo er u.a. sein Argument gegen ontischen Monismus in, sagen wir mal, akademisch solider Form bringt und wohl nicht plagiiert. Ideen von Kant zu plagiieren wäre wohl auch eine eher blöde Idee, damit fliegt man zu schnell auf.

    @Balanus / 19.01.2021, 18:57 Uhr

    »Mir erscheint es höchst vernünftig,…«

    Quod erat expectandum.

    »Mit den subjektiven Eindrücken kann niemand etwas anfangen, außer man selber.«

    Wer beispielsweise Zahnschmerzen hat, dem ist es nach meiner Erfahrung absolut egal, dass kein anderer damit etwas anfangen kann. Und ich halte es auch für unglaubhaft, dass Dennett, falls er mal nachts vor Zahnschmerzen wach im Bett sitzen sollte, sich dann sagt, “Okay, das ist alles nur Einbildung und existiert nicht. Ich schlaf’ mich jetzt trotztdem erst mal richtig aus, und morgen sehen wir dann weiter.

    Hast Du schon mal erwogen, zum Apostaten zu werden, falls das mit dem metaphysischen Naturalismus unhaltbar wird? Die methodolgische Variante zerlegt sich zumindest nicht so ganz offenkundig selbst. Wär’ das nicht was? Als Apéritif, hier etwas W.V.O. Quine für Einsteiger: Im Naturalismus gefangen 😉

  341. @KRichard

    Quantenphysikalische Phänomene sind immer physikalischer Natur!

    So wie ich das verstehe, ist deren Wirkung physikalisch, was die Phänomene als solche erkennbar macht. Über die Ursachen wird noch geforscht, nicht immer ergebnisoffen, und viel philosophisch spekuliert. Manchen gelten zumindest einige der Phänomene als unverursacht, was gegen deren physikalische Natur sprechen würde.

  342. @Elektroniker: Definition
    Da haben sie recht. Ich meine den Geist im Bewusstsein.
    Alle anderen “Geister” kann ich mir sehr gut damit vorstellen. 🙂

  343. @ Stephan Schleim 20.01.2021, 11:51 Uhr

    Zitat: „Was halten Sie denn von dem hier formulierten Gedanken, dass man sich Karma ausgedacht hat, um Menschen in unteren Schichten besser zu kontrollieren? Und die Herrschaft der oberen Schichten zu rechtfertigen? Ähnlich wie bei uns die Sache mit dem Paradies und dem Gottesgnadentum (Herrschaft eines Königs von Gottes Gnade).“

    Sehe es im Prinzip auch so wie Sie. Es kommt sozusagen auf das „optimale“ psychische Betriebssystem des Menschen an. Es ist allerdings tatsächlich so, dass unsere Generation weitaus besser lebt als frühere Generationen, auch länger lebt.

    Wegen der Umweltproblematik und der zunehmenden Weltbevölkerung sind allerdings Probleme absehbar ….

    Ist man gegen eine vernünftige Ordnung, so führt das nur zu Unzufriedenheit, Revolution und Bürgerkrieg…

  344. @Chrys: der pure Empirismus löst keine umfassenden Probleme. In der Krebsforschung sucht man seit vielen Jahrzehnten vergeblich nach den Ursachen und findet sie nicht, weil Krebs nicht lokal bzw. lokalisierbar entsteht, wie etwa ein Knochenbruch, sondern durch komplexe Prozesse. Wenn man nur, wie gesagt, Puzzleteile sucht, aber das Bild nicht kennt, das daraus entstehen soll, gelangt man kaum ans Ziel.

  345. @KRichard 20.01. 05:01

    „Der Grund dafür ist klar: Die menschliche Dummheit ist grenzenlos.
    Einen ähnlichen Mechanismus haben wir bei der Idee vom nichtphysikalischen Bewusstsein.
    Es wird immer Leute geben, welche diese Idee nicht hinterfragen und nachvollziehbare Belege verlangen – sondern es vorziehen daran zu glauben.
    Notfalls hat das Ganze dann quantenphysikalische Ursachen.“

    Nun gut, es gibt eben Menschen, die spirituelle Erfahrungen haben, und die entsprechend versuchen, diese in ihr Weltbild zu integrieren. Dazu gehört dann auch die Idee, dass unser Bewusstsein geistige Anteile hat. Das heißt ja noch lange nicht, dass Bewusstsein keine neuronalen Grundlagen hat. Der Geist kommt hier eben noch dazu. Wenn ich sage, wir sind eine Synthese eines kosmischen Geistes mit unserem lokalem Gehirn, dann widerspreche ich damit nicht der Erfahrung, dass Menschen ein Gehirn für ihre Existenz brauchen.

    Es wäre dann beim derzeitigen Erkenntnisstand schon Dummheit, zu glauben, dass unser Gehirn keine Rolle spielt. Das sehe ich auch so. Aber Sie scheinen hier einfach allen Menschen Dummheit zu unterstellen, die nicht 100% materialistisch daherkommen.

    @Christian Hoppe 19.01. 18:54

    „Wie gesagt, mir ist völlig wumpe, was jemand glaubt, wenn er damit die Wissenschaft nicht verdirbt und wenn er sich kompetent und kooperativ in die Gemeinschaft einbringt.“

    Sie meinen, es würde die Wissenschaft verderben, wenn man glaubt, dass hier vielleicht doch Geist in der Welt mitspielt?

    Und umgekehrt, scheint es ihnen kein Glaubensinhalt zu sein, wenn sie meinen, das die bisher entdeckten physikalischen Gesetzmäßigkeiten auch für Bereiche gelten, die noch gar nicht hinreichend erforscht sind? Was Sie nicht wirklich wissen, das können Sie doch auch nur glauben.

    Ich meine, dass erst die Zukunft zeigen wird, ob der Kosmos wirklich geistlos ist oder nicht. Noch gibt es genug Spielräume, beides zu glauben. Verschiedene Lebenserfahrungen spielen hier eine große Rolle, und durchaus auch religiöse Traditionen. Die Wissenschaft kennt eben noch nicht alles.

    Ich habe den Eindruck, das sich die Wissenschaft gerne darauf beschränkt, sich mit Sachverhalten zu beschäftigen, die zu ihren Methoden passen. Das ist auch vollkommen in Ordnung. Aber daraus zu schließen, dass der ganze Kosmos in allen Einzelheiten so funktioniert, das ist meine ich dann doch mehr Glaubenssache, als dass man das zum gesichertem Wissen zählen könnte.

  346. @Elektroniker: Lebenserwartung

    Das kann man nicht so allgemein sagen: In den USA kehrt sich das schon wieder um, Menschen sterben wieder früher, insbesondere in ärmeren Bevölkerungsgruppen und Männer sowieso.

    Wie sich das in Europa entwickelt, muss sich noch zeigen. Die Corona-Pandemie verschärft die Gegensätze aber wieder weiter.

  347. @Schleim
    Das ist wohl der “dual/multiple use” einer Religion/Ideologie.
    Die einen werden unterdrückt, die anderen brauchen “Vordenker und Vorsager”/Hilfe für die Legitimation von “Lebensentscheidungen” (wie die USA gerade wunderbar zeigen).

    Ihre Überlegungen zu “was soll dann kommen…” ist die Frage nach “Was gibt Orientierung?”…sehr gute Frage!

  348. @Stephan

    Ich weiß nicht, ob wir wirklich noch Weltbilder oder gar ein Weltbild (bzw. Menschenbild) brauchen. Es besteht in der gesellschaftlichen Praxis heute doch in vielen Fragen ein erstaunlicher, auch generationsübergreifender Grundkonsens. Interessenkonflikte müssen mit den Mitteln der Demokratie entschieden werden. Den meisten ist klar, dass es in den Bereichen, die wissenschaftlich erforschbar sind, keinen Sinn mehr macht, wild herumzuspekulieren. Erfahrungen von Sinn und Trost und auch Hoffnung können nur wir Menschen einander vermitteln.

    Psychologisch finde ich interessant, ob wir die Möglichkeit der innerspsychischen Dissoziation – des sich selbst Gegenüber tretens bzw. der inneren (!) Ausrichtung auf etwas “Höheres” – nicht anders als religiös kultivieren und auch nutzbar machen können, also ohne magische/mythische Reifizierung und im vollen Bewusstsein dieses innerpsychischen Vorgangs.

  349. @Christian Hoppe (Zitat):

    ob wir die Möglichkeit der innerspsychischen Dissoziation – des sich selbst Gegenüber tretens bzw. der inneren (!) Ausrichtung auf etwas “Höheres” – nicht anders als religiös kultivieren und auch nutzbar machen können

    Mit Ikigai meinen die Japaner genau diesen persönlichen Lebenssinn, diese Ausrichtung auf etwas das Sinn stiftet. Ikigai brauchen alle, sowohl die die religiös sind als auch alle anderen. Religiösität allein gibt dem eigenen Leben ja noch keinen Sinn. Das tut erst die eigene Aktivität. Hier nun was man in der Wikipedia dazu liest:

    Ikigai (jap. 生き甲斐 Lebenssinn) ist frei übersetzt „das, wofür es sich zu leben lohnt“,[1] „die Freude und das Lebensziel“[2] oder salopp ausgedrückt „das Gefühl, etwas zu haben, für das es sich lohnt, morgens aufzustehen“.

  350. @Holzherr

    Ein Teil meines wach werden ist zumindest fundamental: mich/andere zu versorgen (z.B. Existenzsicherung)…

  351. @Martin Holzherr

    Ich meinte nicht die Bezugnahme auf einen abstrakten Sinn, sondern die Fähigkeit in sich selbst eine letztlich personale Bezugsgröße zu kreieren durch Dissoziation, sodass man in sich selbst ein ermutigendes, tröstendes, klärendes “Gegenüber” findet.

    Gläubigen ist nicht klar, dass sie genau das z.B. während des Gebetes machen: sie erschaffen sich ihren Gott, indem sie nicht diskursiv über ihn, sondern dissoziativ an ihn denken (“Andacht”).

    Anfangen könnte man damit, sich im Spiegel zu betrachten, sich selbst tief und ernsthaft in die Augen zu sehen und dann ein Gespräch mit sich zu führen. Dann könnte man aber auch an die berühmte “innere Stimme” oder den Sokratischen Eudaimonion denken. Und auch die Vorstellung eines “höheren oder besseren Selbst” könnte hier reinspielen.

    Dissoziation ist eine häufige, dann pathologische und leidvolle Traumafolgestörung. Vielleicht könnte es kultivierte und anerkannte Formen der Dissoziation geben, die zumindest zu Beginn der Verarbeitung einiges abfangen, bis dann wieder eine Re-Integration möglich wird.

  352. @ Christian Hoppe – heute – 11:26

    Tut mir leid. Ich verstehe Ihre beabsichtige Aussage im ersten Absatz nicht so recht. Insb. wenn sie sich auf den Satz *Abhängig vom Bewusstsein entsteht Geist und Materie* (der, zugegebenermaßen, nicht einfach ist) bezieht. Z.B. das “es“ in “Wenn es Bewusstsein ist“ … Bewusstsein erleben tut/hat ein lebendiges Wesen. Bewusstloses Bewusstsein ist eine “Assoziation“ (Interpretation) von Ihnen. Steht so nicht im Text. Ohne Bewusstsein sein kann man als lebendiges Wesen nicht. Dass man keine Wahrnehmungen hat halte ich für möglich.

    Dass Bewusstsein Materie hervorbringt hängt (gemäß Buddhismus) damit zusammen, dass wir einen ungeheuren Lebenswillen haben und demgemäß auch einen neuen weltlichen Köper gestalten (so à la *Wo ein Wille ist, ist auch ein Weg*) … Inwiefern ist Narkose ein Beispiel dafür, dass Materie Bewusstsein hervorbringt? Meiner Ansicht nach hängt bewusstes Erleben nicht einseitig von Hirnprozessen ab, sondern eben auch von Bewusstsein. Sie ergänzen sich (Komplementarität). Ich gebe zu, dass der Gedanke eines Bewusstseins ohne Gehirn sperrig ist. Möchte aber auch das anorganische/geistlose Organ Gehirn nicht überbewerten (beim allem Respekt vor der Komplexität/Wichtigkeit).

    Die Formulierung gigantisches Blabla ist – für mein Empfinden – überheblich (auch unhöflich/-kultiviert). Man kann auch trotz hoher Intelligenz Blabla gestalten (höflicher gesagt: Auf – jedoch nicht unverständlicherweise – falsche Gedanken kommen). Ist mir auch schon passiert.

  353. @ Stephan Schleim – heute – 11:51

    Ich denke nicht, dass der Buddha sich Karma ausgedacht hat – insb. um Menschen in unteren Schichten besser zu kontrollieren. Der Buddha hat das brahmanische Kastensystem abgelehnt! Dass es Unterschiede zwischen Menschen gibt (und warum) ist ein anderes Thema … Paradies und Gottesgnadentum sind dem Buddhismus fremde Begriffe. Dass man Nirwana mit Paradies assoziiert ist nachvollziehbar. Einen Schöpfergott gibt es im Buddhismus nicht – ist kein wichtiges Thema.

  354. @Balanus

    Deine Gedanken lösen bei mir kein Schmunzeln aus, aber was ich von Dir lese, schon.

    Das könnte man so erklären:

    Lesen ist Denken mit fremdem Gehirn

    (J.L. Borges)

    Ich denke, man sollte die Klassifizierung „metaphysische Annahme“ auf solche Annahmen beschränken, die völlig aus dem Rahmen naturwissenschaftlicher oder empirischer Erkenntnismöglichkeiten herausfallen.

    Dachte ich doch, das wäre bei der Kausalität so. Aber gut, ganz sicher bin mir selbst nicht. Heisenberg trägt dazu bei, er sagte ja schon vor längerer Zeit:

    Ich glaub’, ich hab’ das Kausalgesetz widerlegt.

    Wie konnte er das Kausalgesetz falsifizieren? Reichte ein Versuch, bei dem die theoretisch vorausgesagte Wirkung ausgeblieben ist? Würde eine Wiederholung, mit gleichem Resultat, nicht schon wieder für Kausalität sprechen?

    Wie sollen wir die physikalischen Annahmen nennen, die wir uns im Kindesalter empirisch erarbeiten? Was hältst du von hemiphysische Annahmen?

    Da könnte dann, neben der Kausalität, sowas wie die Objektpermanenz ihren terminologischen Platz finden. Zu den hemipyhsischen könnte man ebenfalls die Annahmen zählen, dass Dinge bevorzugt zu Boden fallen, von alleine langsamer werden, dass Raum und Zeit für alle immer gleich sind.

    Wäre das okay?

  355. @Joker, @Balanus
    Metaphysische Annahmen sind für die Naturwissenschaft das, was Axiome und Definitionen für die Mathematik sind: unverzichtbar, weder wahr noch falsch, weder beweisbar noch falsifizierbar, konventional. Die Frage ist, ob eine metaphysische Annahme notwendig, begründet, berechtigt, nützlich ist. Für die Biologie ist Leben eine metaphysische Annahme, für die Psychologie sind Psyche und Geist metaphysische Annahmen. Theologie ist insgesamt Metaphysik.

    Selbstverständlich kann man hier Einwände haben, aber ich denke, es ist vernünftig so.

  356. @Joker
    Heisenberg hat formuliert,was er entdeckt hat.
    Es ist etwas schwer,die Kausalität und Nichtkausalität als ‘Gleichzeitigkeit’ zu akzeptieren.
    Ich sehe es wie Elektroniker,dass Determinismus und Zufall die beiden Seiten der einen Medaille sind:komplementär.

  357. @ Stephan Schleim 20.01.2021, 14:13 Uhr

    Zitat: „Das kann man nicht so allgemein sagen: In den USA kehrt sich das schon wieder um, Menschen sterben wieder früher, insbesondere in ärmeren Bevölkerungsgruppen und Männer sowieso.
    Wie sich das in Europa entwickelt, muss sich noch zeigen. Die Corona-Pandemie verschärft die Gegensätze aber wieder weiter.“

    Sicherlich ist es so wie Sie es sehen.
    Der Ressourcenverbrauch unseres „guten Leben“ scheint einfach zu hoch und eine Art von „Sättigung“ tritt auf.

    Auch Erkrankungen (Pandemien) die mit der immer größer werdenden Bevölkerungsdichte und den zunehmenden Kontakten zusammenhängen, dürften große Probleme bereiten wie z.B. ehemals die Pest. Ich setze auf neue technische Lösungen, wie ehemals der Bau von Trinkwasser und Abwasserkanälen notwendig wurde.

    Faire, möglichst für die ganze Gesellschaft nützliche „Arbeit“ sollte als Therapie gesehen werden und staatlich gefördert, nicht ausgelagert werden. Modernste Technik sollte das „Arbeitsleid“ minimieren. Der Drogenverbrauch sollte minimiert werden.

    Übrigens: Ich bewundere Ihre ungeheure persönliche Arbeitsleistung im Job als Hochschullehrer, in der Forschung, als Verfasser von Büchern und auch noch als Betreuer dieses sehr großen „Forums“.

    Danke dafür Herr Dr. Stephan Schleim.

  358. @Christian Hoppe 20.01. 14:40

    „Psychologisch finde ich interessant, ob wir die Möglichkeit der innerspsychischen Dissoziation – des sich selbst Gegenüber tretens bzw. der inneren (!) Ausrichtung auf etwas “Höheres” – nicht anders als religiös kultivieren und auch nutzbar machen können, also ohne magische/mythische Reifizierung und im vollen Bewusstsein dieses innerpsychischen Vorgangs.“

    Finde ich auch interessant. Was man ohne magische/mythische Reifizierung hinbekommt, wäre sicher vorzuziehen. Gerade die Konzentration auf bewusst Innerpsychisches hört sich gut an.

    Mit Geist geht aber mehr, in der Tat vielleicht zu viel wie gut ist, das kann auch ein Nachteil sein. Insbesondere wenn hier Geist nur von außen her proklamiert wird, ohne dass es persönlich so erlebt wird.

    Aber wenn man nun eindeutig spirituelle Erfahrungen hat, warum sollte man die dann nicht einbinden? Zumal Pluralismus in Glaubensfragen kein Nachteil sein muss. Oder ist die Religionsfreiheit nur dem Unvermeidlichem geschuldet? Dass man sie nur gewährt, weil die Leute sowieso religiös unterwegs sind, und man schon froh ist, wenn sie sich nicht gewalttätig um den richtigen Glauben streiten.

  359. @Hoppe: Orientierung

    Für dich mag das ja funktionieren; doch andere Menschen werden ein System zur Orientierung suchen und brauchen.

    (Schreibe ich während der Zeremonie zur Vereidigung des neuen Präsidenten der USA, wo oft von Gott, Gebet, Amen die Rede war und gerade ein Pfarrer ein mehrminütiges, bewegendes Gebet sprach.)

  360. @Timm Grams

    Es kommt zur Selbstreferenz, die ins Paradoxe führt.

    Eher nicht. Die mir bekannten (angeblichen) Paradoxien bei Versuchen, Bewusstsein naturalistisch zu erklären, entstehen durch von Skeptikern hereingebrachte Begriffsverwirrungen, die dann in Aussagen gipfeln, wie der, es sei paradox, dass der Geist sich selbst erkennen könne. Ich z.B. würde es eher für eine Paradoxie halten, wenn der Geist, der erkennen kann, ausgerechnet sich selbst nicht erkennen könnte.

    Womit ich keinesfalls ausdrücken möchte, es gäbe bereits auch nur eine halbwegs befriedigende naturalistische Erklärung des Bewusstseins (*), oder wir könnten jemals zu einer solchen gelangen. Da sprechen andere Gründe dagegen, nichts Paradoxes.

    Chrys hat mit seiner Anspielung auf die Paradoxien der Mengenlehre – das Russellsche Paradoxon ist eins von ihnen – angedeutet, was da auf uns zukommt.

    Markus Gabriel benutzt nicht das Russellsche Paradoxon (und auch kein anderes), um zu erklären, warum es die Welt nicht gibt. Er definiert Existenz als vorkommen in der Welt, und schreibt dann

    Die Welt kann vielmehr prinzipiell nicht existieren, weil sie nicht in der Welt vorkommt

    (Warum es die Welt nicht gibt, Seite 22)

    Ich kann auch nicht erkennen, dass es zwangsläufig zu einer Paradoxie käme, würde jemand die Menge alles Existenten um dieses eine Element erweitern.

    Erst wenn jemand Welt definieren würde, als Menge aller Objekte in der Welt und aller Mengen, die aus diesen Objekten gebildet werden können, mit Ausnahme der alles umfassenden, – oder so ähnlich – hätte er sich ein Problem eingefangen.

    (*)
    Für Naturalisten könnte selbstverständlich nur eine naturalistische Erklärung überhaupt befriedigend sein.

  361. @Chrys // 20.01.2021, 13:42 Uhr

    »Wer beispielsweise Zahnschmerzen hat, dem ist es nach meiner Erfahrung absolut egal, dass kein anderer damit etwas anfangen kann.«

    Fein, dann stimmst Du mit mir also darüber ein, dass subjektive Empfindungen etwas höchst Privates sind.

    Schmerz scheint mir übrigens ein sehr überzeugendes Beispiel dafür zu sein, dass Empfindungen auf physischen Ereignissen beruhen, also erst die Physik, nachfolgend die Empfindung. So gesehen scheint der schmerzende Zahn doch ein bisschen mehr zu sein als bloß das physische Korrelat des Schmerzes.

    Des Weiteren halte ich es für völlig glaubhaft, dass z. B. Dennetts imaginärer Zwillingsbruder, dem ein Bein amputiert wurde und der des Nachts Schmerzen in diesem abben Bein verspürt, zu sich sagt: „Okay, das ist alles nur Einbildung und existiert nicht. Ich schlaf’ mich jetzt trotztdem erst mal richtig aus, und morgen sehen wir dann weiter.“

    Nicht, dass er keinen Schmerz verspüren würde, aber irgendwie fehlt in diesem Fall dann doch das physische Korrelat (zum Teil, versteht sich), aber bekanntlich kann man manche Schmerzen durch Autosuggestion ganz gut ausblenden.

    » Hast Du schon mal erwogen, zum Apostaten zu werden, falls das mit dem metaphysischen Naturalismus unhaltbar wird? «

    Als ob es eine Rolle spielen würde, dass eine Lehre oder Weltanschauung—in den Augen mancher Skeptiker und/oder Kritiker—unhaltbar ist. Schau Dich mal um, was und an was so alles geglaubt wird…

    Auch im verlinkten Artikel (Im Naturalismus gefangen), da möchte man schon nach dem fünften Satz aufhören zu lesen:

    Gehirnforschung und Neurologie beanspruchen, menschliches Handeln vollständig zu erklären, so dass für Freiheit und verantwortliches Handeln kein Platz mehr bleibt, weil das Gehirn bereits alles steuert.

    (Alexander Riebel)

    Doch zurück zum metaphysischen Naturalismus. Ich habe mal bei Geert Keil nachgeschaut (aus: „Naturalismus und menschliche Natur“. Velbrück Wissenschaft, 2008):

    Als ontologische oder metaphysische Position ist der Naturalismus eine These darüber, was es gibt oder wie die Welt beschaffen ist. Der metaphysische Naturalismus läßt sich durch die Parolen wiedergeben >Alles ist NaturAlles ist natürlichAlles, was es gibt, ist Teil der einen, natürlichen WeltNaturnatürliche Welt< gemeint ist.

    Insgesamt hält Keil den metaphysischen Naturalismus, in meinen Worten, für ziemlich witzlos:

    Manche Naturalisten verstehen unter Natur tatsächlich >alles, was es gibt<:[…] Diese Totalisierung ist sicherlich keine gute Idee, denn wenn Natur schon soviel heißt wie Seiendes oder Wirkliches, dann wäre der Naturalismus ein Ismus des Wirklichen und hätte überhaupt keine kritische Pointe mehr.

    Es scheint, als habe Geert Keil (und mit ihm viele andere) übersehen, dass sich der metaphysische Naturalismus logisch selbst widerlegt.

  362. @Stegemann
    Es deutet eigentlich mehr darauf hin,dass wir den ‘Phasenwechsel’ zwischen Mikro und Makro,Reduktion und Emergenz nicht ‘in den Griff’ bekommen.

  363. Balanus,
    „Befriedigende Erklärung“, darauf kommt es an. Wir können noch so viele Beriffsgaggeleien durchführen, wenn sie nicht befriedigen, dann bleibt das Bewusstsein unbefriedigt.
    Die Antworten der Bibel , oder sollte man besser sagen, die Bilder der Bibel sind tiefschichtig. Jeder,ob alt oder jung, dumm oder intelligent,liest in der Bibel etwas anderes heraus.
    Vergessen wir bei dem Begriff „Bewusstsein“ die Bibel nicht. Ein Appetithappen :
    Wissed ihr nicht, daß ihr der Tempel (das Haus) Gottes seid, und der
    GEIST GOTTES in euch wohnet? Wenn der göttliche Geist aus der Einheit
    heraus fällt ( …

  364. @Joker // 20.01.2021, 17:07 Uhr

    » Das könnte man so erklären:
    Lesen ist Denken mit fremdem Gehirn
    (J.L. Borges)
    «

    Ein schöner, poetischer Gedanke, informationstheoretisch gesehen aber problematisch.

    »Wie konnte er [Heisenberg] das Kausalgesetz falsifizieren? «

    Das sogenannte „Kausalgesetz“ stammt aus der klassischen Physik. Dort war es lange von Nutzen bei der Untersuchung makroskopischer Gegenstände.

    ,,Wenn von einem abgeschlossenen System alle Bestimmungsstücke zu einer gegebenen Zeit genau bekannt sind, so lässt sich für die Zukunft das physikalische Verhalten des Systems daraus eindeutig berechnen.“

    (aus Heisenbergs Aufsatz von 1931)

    Das hat mit der individuellen Kausalitätserfahrung, ohne die wir nicht unbeschadet durchs Leben kämen, nur ganz entfernt zu tun.

    Witzig finde ich, dass es eine Arbeitsgruppe gibt, die nun wieder den umgekehrten Weg geht wie Heisenberg: Sie startet bei der Unschärfe des Mikrokosmos und landet im Makrokosmos bei einem Ableger des „Kausalgesetzes“, genannt „Kausale Emergenz“.

    (An dieser Stelle Dank an Wolfgang Stegemann für den Hinweis)

    » Wie sollen wir die physikalischen Annahmen nennen, die wir uns im Kindesalter empirisch erarbeiten? «

    Wie wär’s mit Hypothese? („Immer wenn …, dann …“)

  365. @Stegemann
    Niemand bestreitet die Existenz der Kausalität.
    Nehmen Sie auch die ‘Kritik’ wahr?
    Absolute Kausalität führt in die absolute Linearität und absoluten Determinismus, in die absolute Vorhersage.
    Was finden Sie vor?

  366. @Stegemann
    Danach reguliert das Makro das Mikro, während das Mikro das Makro konstituiert.

    Das ist metaphysische Spekulation, die ich für falsch halte, weil es widersprüchlich ist. Über das Makro haben wir nicht genügend Information, z.B. in der Thermodynamik, weil wir die vielen Mikros nicht kennen. Im Mikrobereich kommen wir schnell in der Bereich der quantenphysikalischen Zufälle, die sich statistisch auf das Makro auswirken. Der reine Holismus ist so falsch wie der reine Emergentismus.

  367. @Joker @Balanus

    Ein Problem mit Erklärungsmodellen ist, dass wir für alle Empfindungen überhaupt keine konkrete Erklärung haben – wie ihre bewusste Wahrnehmung entsteht:
    Farbwahrnehmungen, Emotionen, Bewusstsein, Gefühle, Sinneswahnehmungen.

    Wer also behauptet, dass das Bewusstsein nichtphysikalischer Art ist, der behauptet somit ganz konkret, dass das gleiche auch für Zahnschmerzen gilt.

    Es ist erstaunlich, wenn man dann doch zu Zahnärzten geht um sich behandeln zu lassen und nicht zu Priestern.

  368. @Stegemann – Hoel et al

    Aber halten wir doch bitte fest, dass auch die makroskopischen Zustände, denen Hoel et al. kausale Power zuschreiben wollen, physische Zustände sind.

    Ferner finde ich es trivial, dass das Entscheidende im Hinblick auf Verhalten (und vermutlich auch Erleben) nicht die jeweiligen Bausteine sind (die im Stoffwechsel ständig ausgetauscht werden), sondern deren Organisation (z.B. i.S.v. neuronalen Erregungsmustern).

    Haben Hoel et al. also eine fünfte Wechselwirkung in der Physik entdeckt? Wohl kaum …. Ich vermute, dass Physiker den Hokuspokus hier schnell entlarven würden.

  369. @Stegemann – Hoel et al

    Ich zitiere aus dem Spektrum-Artikel:

    Ganz am anderen Ende des Begeisterungsspektrums steht Scott Aaronson, theoretischer Informatiker an der University auf Texas in Austin (USA). Aaronson sagt, die fundamentalen Annahmen der kausalen Emergenz seien überhaupt nicht radikal. Hoels Essay “Agent above, Atom below” … kommentierte Aaronson wie folgt: “Es war schwer darin irgendetwas zu finden, dem selbst der orthodoxeste Reduktionist der Welt widersprechen würde. Natürlich will man höhere Abstraktionsebenen nutzen, um Vorhersagen zu machen und um kausale Geschichten zu erzählen – der Essay zählt dafür lediglich ein paar Gründe auf.

    Genau.

  370. @Axel Krüger

    Für “Blabla” entschuldige ich mich. Philosophisch gibt es den Begriff des “bullshit” (H.G. Frankfurt) – der passt dann aber doch ganz gut: Es wird zwar in einer erklärenden Weise geredet, man gibt vor, etwas zu wissen, aber wenn man genauer und kritischer hinschaut, stellt sich heraus, das gar nichts erklärt wird – eine Art rhetorischer Trick.

    Sie verwenden den Begriff Bewusstsein als Substantiv. Sie sagen, dass es etwas hervorbringt. Also frage ich quasi nach dem Bewusstsein selbst – bevor es etwas hervorbringt. Wenn dieses Bewusstsein nicht bei Bewusstsein ist, könnte es niemals erklären, dass durch es ein Anderes zu Bewusstsein gelangt. Meines Erachtens können Sie dieses Bewusstsein gar nicht anders definieren als ein Sein, das bei Bewusstsein ist. – Gäbe es nun ein solches Bewusstsein und wäre dies die Erklärung dafür, dass ich bei Bewusstsein bin, dann wären Phasen der Bewusstlosigkeit unmöglich; denn dann wäre das Bewusstsein bewusstlos und das ist eine contradictio in adiecto. Es wäre auch unklar, wo dann neues Bewusstsein herkommen könnte, wenn das Bewusstsein selbst bewusstlos war.

  371. @Stephan

    Das Segensgebet war bewegend, ja.

    Aber wie viel mehr hat Amanda Gorman **GESTRAHLT**, die in ihrem Gedicht “The Hill We Climb” zwar einmal auf die “Scripture” rekurriert, aber ansonsten nicht von Gott spricht – sondern von der Hoffnung, die wir Menschen einander geben können, d.h. von einem nichtreligiösen Glauben, der uns motiviert.

  372. Timotheus 2,23
    “Den thörichten und meisterlosen Grübeleien aber weiche aus, im Bewußtsein, daß sie nur Streitigkeiten hervorbringen.”
    Dieses Zitat passt zu einer Diskussion.

    In der Bibel wird der Begriff Bewusstsein gebraucht im Sinne von, etwas wissen, von Etwas überzeugt sein.

    Und dass dieses Wissen direkt von Gott kommt, das sollte man auch im Hinterkopf behalten.

  373. @Stegemann
    Ich lebe in der Raumzeit, kann meine Zeit meistens selbst bestimmen. Haben Sie Zeit enträtselt?

  374. @reutlinger: versuchen Sie einmal, die mathematische Argumentation von Hoel in dem o.g. Artikel zu verstehen. Und dann äußern Sie Kritik an diesem mathematischen Beweis. Darüber können wir uns dann unterhalten. Pauschalkritik akzeptiere ich nicht, das gilt auch für @Hoppe.

  375. @hwied

    Der Begriff “Bewusstsein” wurde im Deutschen erstmals 1719 von Christian Wolff verwendet. Auch der Sache nach kommt dieser Begriff, wie wir ihn in den Neurowissenschaften und in der Philosophy of Mind diskutieren, garantiert nicht in der Bibel vor.

  376. @Stegemann

    Hat Hoel eine fünfte Wechselwirkung entdeckt, ja oder nein?

  377. @Stegemann

    Oh doch, die Kausalität auf dem “micro” level beruht ja gerade auf den vier heute bekannten Wechselwirkungen. Behauptet man nun eine eigenständige “macro” Kausalität würde man auf dieser Ebene so etwas wie eine neue, fünfte “Kraft” behaupten (“causal emergency” – bzw. emergent causality), von der die Physik bisher nichts wusste. Das wäre ein Knaller. Und es würde nicht im “Neuroscience of Consciousness”-Journal erscheinen.

  378. Niemand behautpet, dass man praktisch (!) die Chemie oder die Biologie (einschl. Psychologie) auf Physik reduzieren könnte oder gar sollte; das ist ein Missverständnis von “Reduktionismus”. Aber in diesen Fächern bestreitet niemand ernsthaft die durchgängige Gültigkeit der Physik für die beobachtbaren physischen Vorgänge.

  379. @Hoppe, ich wiederhole mich, lesen Sie den Artikel von Hoel, den ich weiter oben verlinkt habe und schieben Sie nicht irgendwelche Physiker vor, die “den Hokuspokus hier schnell entlarven würden”. Sie müssen schon selber denken, der gesunde Menschenverstand hilft hier nicht weiter. Nochmals: setzen Sie sich mit der mathematischen Argumentation Hoels auseinander, darüber können wir dann reden.

  380. Christian Hoppe,
    ……auch der Sache nach kommt der Begriff „Bewusstsein in der Bibel nicht vor.“

    Wollen wir mal sehen , ob das stimmt.
    Hier in der Diskussionsrunde sind wir schon sehr weit gekommen. Wir haben gehört, dass beim dualen Denken von Körper und Geist die Rede ist.
    Wir haben gehört, dass wir unterscheiden müssen zwischen dem Bewusstsein als Zustand und dem Bewusstsein als Substantivierung.

    Substantivierungen gaukeln uns vor, dass etwas real existiert, sprich physisch existiert.
    In der Tat spricht die Bibel nicht von Bewusstsein im Sinne von Etwas, das wir empfangen können und weitergeben können, das wird in der Bibel „Geist“ genannt. Der Hl. Geist ist eine Substantivierung, die im Pfingstereignis ganz anschaulich beschrieben wird.

    1 Korinther : So werdet nun fest, meine geliebten Brüder, unerschütterlich, unerschöpflich im Werk des Herrn allezeit, im Bewußtsein, daß eure Mühe im Herrn nicht umsonst ist.

    Bewusstsein wird in der Bibel so benützt, wie in der Umgangssprache, im Sinne von „im Bewusstsein von“ also aktiv .

    Also nichts Geheimnisvolles.

  381. @Stegemann
    Mit selbstgewählten Definitionen und Axiomen kann ich jeden Unsinn beweisen.

    Neben physikalischen Theorien gibt es eine pragmatische Nützlichkeit. Man muss die Welt nicht von ganz oben bis ganz unten durchforsten. Es gibt die Atome, die eine sehr stabile Ebene bilden. Nur selten zerfallen Atome und nur selten werden welche aus Elementarteilchen neu gebildet. Das allermeiste Weltgeschehen, die allermeisten Kausalitäten finden auf der atomaren Ebene und darüber statt. Was sich ändert, das sind die Zustände von Atomen, besonders in der Form elektromagnetisch wirksamer Ionen. Subatomare Teilchen im Sonnenwind werden von Molekülen der Atmosphäre neutralisiert. Quantenphysikalische Effekte können auf die Bahnen der Elektronen einwirken.

    Es ist trivial, dass sich Anhäufungen von Objekten anders verhalten als einzelne oder wenige Objekte, weil es Wechselwirkungen gibt zwischen den Objekten, deren Auswirkungen mit der Zahl der Objekte exponentiell anwächst. Dadurch können quantitativ neue Phänomene beobachtet werden. Das bedeutet nicht, dass es dadurch neue Qualitäten als emergente Phänomene oder Kausalitäten gibt. Auch die Quantelung von Energie kann dabei eine Rolle spielen, wiederum bei den Bahnen der Elektronen.

    Bei Anhäufungen von Objekten spielt die Statistik eine große Rolle, indem nicht das einzelne Objekt als Mikrozustand betrachtet wird, sondern nur der Makrozustand als “Summe” der Mikrozustände. Der Grund dafür ist die mangelnde Berechenbarkeit aller Mikrozustände zu einem bestimmten Zeitpunkt unter Berücksichtigung der räumlichen Verteilung aller Objekte und auch unter Berücksichtigung ihrer Wechselwirkungen mit der Umwelt.

  382. Nachtrag Christian Hoppe,
    in der Elberfelder Bibel wird nur von “wissen ” gesprochen.
    Daher, meine geliebten Brüder, seid fest, unerschütterlich, allezeit überreich in dem Werk des Herrn, da ihr wisst, dass eure Mühe im Herrn nicht vergeblich ist!

  383. Hallo, Herr Reutlinger,
    ich bin wieder präsent, bei uns hat sich etwas verschoben.
    Aus der physikalischen Diskussion halte ich mich heraus, weil sie meiner Meinung nach das Wesen des Denkens nicht erklären kann.

  384. @anton reutlinger // 21.01.2021, 10:43 Uhr

    » Es ist trivial, dass sich Anhäufungen von Objekten anders verhalten als einzelne oder wenige Objekte, weil es Wechselwirkungen gibt zwischen den Objekten, deren Auswirkungen mit der Zahl der Objekte exponentiell anwächst. «

    Das eine Anhäufung von Nervenzellen z.B. subjektiv erlebbare Farbempfindungen hervorbringen, ist ja nun alles andere als trivial.

    Wenn Hoel et al. tatsächlich einen Weg gefunden haben, wie aus dem Grundrauschen akausaler molekularer Prozesse makroskopische Kausalität mathematisch beschrieben werden kann, dann scheint mir auch das alles andere als trivial zu sein.

    (Der Begriff Emergenz dürfte dabei überflüssig sein).

  385. @hwied
    Sie vergessen, dass die Bibel mehrfach übersetzt ist und die Begriffe in den Übersetzungen daher mehr oder weniger willkürlich sind, auf jeden Fall nicht original. Niemand weiß, was die damaligen Autoren aus vergangenen Kulturen wirklich gemeint haben. Etwas zu wissen heißt Wissen, von etwas überzeugt sein kann Wissen oder Glaube sein. Bewusstsein ist etwas ganz anderes.

    Glauben Sie was Sie wollen, aber erwarten Sie nicht, dass Andere solche “Fake News” glauben.

  386. Anton Reutlinger,
    Bewusstsein ist das, wie wir den Begriff definieren.
    Die Naturwissenschaft tut das auf ihre Weise. Die Geisteswissenschaft tut das auch auf ihre Weise. Der Vorwurf von Fake News, der ist nicht angebracht.

  387. @Balanus
    Da bin ich doch verblüfft, dass Sie vom “Grundrauschen akausaler molekularer Prozesse” sprechen. Ich kann mir darunter nichts Sinnvolles vorstellen. Der Körper hat eine Temperatur von 37°C, schon dadurch entsteht ein erhebliches Wärmerauschen im gesamten Organismus, das alles andere als akausal ist. Das Rauschen bedeutet epistemische Unordnung und Information ist die Differenz zwischen Ordnung und Unordnung. Was hier “effektive Information” sein soll, erschließt sich mir nicht.

    Trivial ist, dass es eine Menge an Wechselwirkungen gibt, von denen wir abstrahieren, weil wir sie nicht beherrschen, weil wir ihre Kausalität oder Effektivität vernachlässigen können, weil wir sie apriori nicht messen oder berechnen können. Man sollte Kausalität nicht mit Effektivität verwechseln.

  388. Christian Hoppe,
    für diesen Link haben Sie einen Oskar verdient. Sogar die Qualia wurde nicht ausgespart.
    Verdict: Hier steh ich nun ich armer Tor und bin so klug als wie zuvor.

  389. @hwied
    Ich möchte zwar überzeugen, aber nicht missionieren und auch nicht missioniert werden. Religiöser Glaube gehört nicht in wissenschaftliche Diskurse, sonst müsste man alle Glaubensrichtungen zulassen. Wozu religiöser Glaube, auch christlicher Glaube, fähig ist, das sieht man gegenwärtig in aller Welt. Wissenschaftlich sind Gläubige ohnehin nicht zu überzeugen, dass sie Irrtümern anhängen.

  390. @Joker
    Die Realisten und Naturalisten (und viele andere auch) sind begeistert von der Tatsache, dass die Methoden der Wissenschaft zu immer besseren Theorien führen. Da liegt der Gedanke nahe, dass es eine ideale Theorie geben müsse, die mit der Realität bestens korrespondiert, und der wir uns annähern.

    Gerhard Vollmer beispielsweise geht sogar davon aus, „dass also das Objekt unserer Erkenntnis, die Welt, einmalig und eindeutig bestimmt“ sei, und „dass wir diese Welt erkennen können“ (“Warum wir die Welt erkennen können”, 2003, S. 15).

    Die Selbstreferentialität der Welt (Realität) spricht gegen die (naheligende) Annahme einer idealen Theorie. Das kann man schon paradox finden. Richard Rorty (“Philosophy and the Mirror of Nature, 2009, S. 298) sieht darin den Grund dafür, dass sich Hilary Putnam vom metaphysischen Realismus abgewandt und dem internen Realismus zugewandt hat (und letztlich sogar beim naiven Realismus gelandet ist, soweit ich sehen kann).

  391. @reutlinger: “Was hier “effektive Information” sein soll, erschließt sich mir nicht.”
    Hier soll niemand missioniert werden, es geht in einer solchen Diskussion um Anregungen. Effektive Information können Sie auch übersetzen mit minimaler Entropie. Die kausale Emergenztheorie steht ganz am Anfang. Aber ich kann mir vorstellen, daß man mit ihr den scheinbaren Widerspruch zwischen klassischer und Quantenphysik erklären kann. Erstere gilt für unsere medioskopische Welt, da die Objekte dort maximale effektive Information beinhalten und die klassischen Naturgesetze (die im übrigen wir geschaffen haben, nicht die Natur) die Phänomenologie optimal abbilden. Die mikroskopische Ebene der Quantenphysik können wir wahrnehmen und in Gleichungen ausdrücken, aber ihre effektive Information ist minimal, daher begreifen wir sie nicht. Dasselbe gilt für die makroskopische Ebene der Relativitätstheorie.

    Ein lebendiger Organismus zwingt ein Molekül, Dinge zu tun, die dieses in der unbelebten Natur nicht tun würde.
    Und: betrachten Sie ein Gemälde. Sie können nicht sagen, ich betrachte aber nur einen Teil dieses Gemäldes, genausowenig wie Sie sagen können, ich blende einen Teil meines Bewußtseins aus, ganz egal, welche Hirnbereiche gerade aktiv sind. Das Makro ist auf der phänomenologischen wie auf der Funktionsebene immer das regulative.
    Man kann die Welt nicht auf immer kleinere Elemente reduzieren (Reduktionismus), um ihre Funktionsweise zu begreifen.

  392. @Stephan // 19.01.2021, 11:26 Uhr

    Ich komme nochmal zurück auf das Bieri-Trilemma, weil mir scheint, dass Timm Grams Begründung, warum Naturalisten vor allem die 2. Prämisse ablehnen würden, nicht hinreichend gewürdigt wurde.

    Stephan, Du schreibst:

    » Ontologisch gesehen bestreitet der Materialismus doch vor allem die erste Prämisse, den mentalen Realismus (MR). Das, was manche “mentale Vorgänge” nennen, sind laut Materialist nichts als physikalische Vorgänge (bzw. Gehirnprozesse).«

    Und auch:

    » Lass es mich mal so sagen: Ob die drei einander ausschließen, liegt natürlich auch daran, wie man sie genau versteht. Oftmals ist es eben eine Frage der Definition.«

    Mir geht es jetzt um die Definition das ersten Lemmas:

    1 Mentale Phänomene sind nichtphysikalische Phänomene.

    Wenn mit „mental“ und „nichtphysikalisch“ die subjektiven Erscheinungen und Empfindungen (Farben, Töne, Schmerz) gemeint sind, dann kann auch ein Materialist kaum bestreiten, dass es diese subjektiven Empfindungen irgendwie „gibt“—er erlebt sie ständig selbst. Zwar beruhen diese Phänomene auf materiellen Prozessen, sind aber selbst keine. Infolgedessen können diese „mentalen“ Erscheinungen auch nicht kausal wirksam werden.

    Demnach wäre das zweite Lemma,

    2 Mentale Phänomene sind im Bereich physikalischer Phänomene kausal wirksam,

    ganz wie Timm Grams das geschrieben hat, vom Materialisten oder Naturalisten abzulehnen.

    Aber nicht nur von denen, sondern jeder vernunftbegabte Mensch, also auch Timm Grams selbst, müsste das zweite Lemma ablehnen. Warum das nicht geschieht, ist eine interessante Frage.

    Timm Grams schreibt im Kommentar vom 19.01.2021, 16:41 Uhr:

    » Zu den (reinen) Denkinhalten zählt Martin Mahner nicht nur Einhörner und dergleichen, sondern unter anderem auch mathematische Gegenstände und Strukturen. Ohne diese gäb’s keine Flugzeuge, keine der eleganten Schrägseilbrücken, eigentlich fast nichts von dem, was unser modernes Leben ausmacht. Also haben sie ganz reale Auswirkungen.«

    Ist ein reiner Denkinhalt phänomenologisch das Gleiche wie eine Sinnesempfindung? Meiner Auffassung nach, ja.

    Ist „eine Auswirkung haben“ das Gleiche wie „eine Wirkung haben“? Ich meine, nein.

    Auch ein Materialist/Naturalist würde wohl nicht bestreiten wollen, dass seine Gedanken Auswirkungen gaben.

    Ich finde, da geht Vieles durcheinander im Naturalismus-Diskurs.

  393. @anton reutlinger // 21.01.2021, 11:47 Uhr

    » Da bin ich doch verblüfft, dass Sie vom “Grundrauschen akausaler molekularer Prozesse” sprechen. Ich kann mir darunter nichts Sinnvolles vorstellen.«

    Versetzen Sie sich mal gedanklich (keine Angst, dabei kann einem nichts passieren 😉 ) in eine beliebige Körperzelle, als Beobachter in der Größe eines Kohlenstoffatoms. Sie würden jede Menge Ereignisse sehen, für die es kein verursachendes Ereignis gibt (z.B. Konformationsänderungen bei Proteinen). Insoweit würde ich Heisenberg schon zustimmen wollen, auf der atomaren/molekularen Ebene gilt kein Kausalgesetz.

  394. @Stegemann
    Die “Missionierung” beziehe ich nur auf religiöse Aspekte.

    Ein lebendiger Organismus zwingt ein Molekül, Dinge zu tun, die dieses in der unbelebten Natur nicht tun würde.

    Dieser, meines Erachtens esoterischen Behauptung möchte ich entschieden widersprechen.

    Man kann die Welt nicht auf immer kleinere Elemente reduzieren (Reduktionismus), um ihre Funktionsweise zu begreifen.

    Den Reduktionismus halte ich für notwendig, aber nicht hinreichend.

  395. @Balanus
    Sie würden jede Menge Ereignisse sehen, für die es kein verursachendes Ereignis gibt (z.B. Konformationsänderungen bei Proteinen).

    Konformationsänderungen bei Proteinen (Tertiärstruktur) beruhen auf den elektrostatischen Eigenschaften der Aminosäuren, bzw. deren Seitenketten. Metallionen können solche Veränderungen verursachen. Verschiedene chemische Kräfte, die Sie sicher kennen, sind daran beteiligt, z.B. Wasserstoffbrücken.

    Eine Konformationsänderung kann auf verschiedenen Wegen erreicht werden, zum Beispiel durch eine Änderung des pH-Werts, der Salzionen, durch eine Temperaturänderung, durch Chaperone, durch Prionen, in biologischen Systemen aber überwiegend durch die Bindung eines Liganden.

    aus dem Internet

  396. Zitat Timm Grams “ Die Selbstreferentialität der Welt (Realität) spricht gegen die (naheligende) Annahme einer idealen Theorie. “
    Die Welt des Menschen (Kunst, Literatur, Politik) enthält viele selbstreferentielle Momente mit der Folge, dass es dort in der Tat keinen Standpunkt von aussen gibt/geben kann.
    Ganz anders ist das aber für die Untersuchungsobjekte der Naturwissenschaft. Theorien über chemische Prozesse, Viren, das Wetter oder das Klima sind Theorien in denen Selbstreferentialität keine Rolle spielt. Doch selbst in diesen Fällen sucht man heute kaum noch nach der idealen Theorie, sondern vielmehr nach besseren Theorien. In weiten Teilen der Naturwissenschaften hat sich nämlich die Bayesianische Sichtweise/Denkweise durchgesetzt in der es um einen ständigen Update, eine ständige Anpassung der Annahmen aufgrund neu hinzugekommener Daten geht. Bayesianisch Denken kann man übrigens auch in den Sozial- und bis zu einem gewissen Grade in den Geisteswissenschaften.
    Das passiert übrigens bereits und erklärt Forschungsartikel wie Modeling other minds: Bayesian inference explains human choices in group decision-making
    oder Bücher wie Introduction to Bayesian Inference for Psychology

  397. @Timm Grams // 21.01.2021, 12:15 Uhr

    » Gerhard Vollmer beispielsweise geht sogar davon aus, „dass also das Objekt unserer Erkenntnis, die Welt, einmalig und eindeutig bestimmt“ sei, und „dass wir diese Welt erkennen können“ (“Warum wir die Welt erkennen können”, 2003, S. 15). «

    Genau genommen sieht Vollmer das so:

    »Wie kommt es, dass wir die Welt erkennen können?
    Wer so fragt, setzt schon einiges voraus. Er (oder sie) setzt voraus, dass es (so etwas wie) die Welt tatsächlich gibt. Der bestimmte Artikel – »die« Welt – legt zudem nahe, dass es auch nur eine solche Welt gibt, dass also das Objekt unserer Erkenntnis, die Welt, einmalig und eindeutig bestimmt sei.
    Unsere Frage setzt weiter voraus, dass wir diese Welt erkennen können – vielleicht nicht vollständig, vielleicht nicht beliebig genau, vielleicht nicht irrtumsfrei, aber eben doch einigermaßen. «
    (Kursivsetzungen im Text)

    Wo ist das Problem?

  398. @anton reutlinger // 21.01.2021, 12:44 Uhr

    » Konformationsänderungen bei Proteinen (Tertiärstruktur) beruhen auf…«

    Ich habe nicht behauptet, dass sie unerklärlich wären, sondern sage, dass völlig unbestimmt ist, wann diese zeitlich auftreten (man kann nur Wahrscheinlichkeiten angeben).

  399. @Balanus
    Enzyme wie auch Nervenverbindungen würden nicht funktionieren, wenn die Konformationsänderungen so unbestimmt wären wie Sie behaupten. Gerade die Konformationsänderungen bestimmen den Funktionszweck oder die Nichtfunktion bei Proteinen. Leben wäre sonst gar nicht möglich! Veränderungen sind die Ursachen für Veränderungen und Leben ist stetige Veränderung.

    Veränderungen von Mikroobjekten können den Anschein erwecken, das entsprechende Makroobjekt würde sich nicht verändern. Zum Beispiel ist der Austausch von Elektronen in einem Atom nicht erkennbar, außer wenn der Austausch selber beobachtet wird, als Emission und Absorption.

  400. In der Quantenphysik lassen sich keine deterministischen, genauen Voraussagen treffen, es sind nur Wahrscheinlichkeitsaussagen möglich. Dies ist … nicht durch die Unkenntnis verborgener Variablen bedingt, sondern spiegelt einen auf Quantenebene existierenden absoluten Zufall wider. Daher ist also nicht nur der Laplacesche Dämon unmöglich (…), sondern der Determinismus ist an sich falsch
    (Wikipedia)

  401. Anton Reutlinger,
    der blog nennt sich menschen-bilder
    Dabei darf nicht nur die naturwissenschaftliche Ansicht gehört werden, sondern auch die geisteswissenschaftliche.
    Nachtrag: Ich hatte nicht vor, Sie zu missionieren. Ich wollte nur verhindern, dass der blog von den Naturwissenschaftlern dominiert wird.

    Stegemann,
    mit der Meinung zum Determinismus stehen Sie nicht allein.
    “Die Welt ist viel zu komplex, als dass man sie als etwas Anderes als eine Approximation begreifen kann.” (John von Neumann)

  402. @Balanus: erste und zweite Prämisse

    Für den Materialisten reicht es nicht, dass “mentale Phänomene” irgendwie aus physischen Prozessen heraus entstehen; für ihn müssen auch die mentalen Phänomene vom Wesen her materiell sein.

    Bei mentaler Verursachung (zweite Prämisse) geht es darum, ob “mentale Phänomene” (oder wie ich lieber sage: psychische Prozesse) aufgrund ihrer spezifischen mentalen/psychischen Eigenschaften (z.B. intentionaler oder phänomenaler Gehalt) kausal wirksam werden können. Wenn du einen Kausalbegriff verwendest, der das von vorneherein ausschließt, dann schließt du das eben aus: Das ist dann aber eine Annahme deines Denkens und keine Schlussfolgerung.

    Dann bleiben die Probleme zu erklären, wie es denn sein kann, dass aus Materiellem etwas mit psychischen Eigenschaften entstehen kann; und wieso wir uns so verhalten und so handeln, wie wir uns verhalten und wie wir handeln, wenn unsere psychischen Eigenschaften dafür keine Rolle spielen: Beispielsweise ziehst du dann nicht die Hand weg von der Herdplatte, weil da Schmerz ist (ein psychischer Vorgang), sondern nur aufgrund irgendeiner zellulären Reaktion im Nervensystem; du hast dann auch nicht geheiratet (nehme ich an), weil du verliebt warst oder weil das eine soziale Norm war (psychosoziale Vorgänge), sondern weil du biologisch darauf konditioniert warst; und so weiter und so fort.

    Wir haben als Materialisten balanesischer Art also mehr offene Fragen als vorher.

  403. @Christian Hoppe 18.01. 17:07

    Aufgrund der hohen Frequenz dieses Blogs kam ich zwischendurch gar nicht ganz hinterher. Hier fällt mir noch was zu ein:

    „Ich sprach ausschließlich von den materiellen/physischen Vorgängen. Diese mögen einem psychischen Phänomen (z.B. einer Handlung) zugrunde liegen oder nicht. Das ändert nichts daran, dass alles was dort physisch geschieht in der Physik bereits seine vollständige Erklärung gefunden hat.“

    Ich glaube eben nicht, dass die Physik wirklich schon vollständig ist. Ich meine, dass eventuell die Quantenzufälle auch mal auf die Zukunft ausgerichtet sein können. Wenn ich richtig informiert bin, ist das natürlich nicht Stand der Physik, aber würde ihr zumindest auf der Ebene von physikalischen Experimenten auch nicht fundamental widersprechen. Von daher hoffe ich darauf, dass das tatsächlich so sein kann.

    Das wäre wohl auch durchaus Experimentell zu prüfen. Wenn solche Effekte wo auftreten, und in der Physis wirksam werden, ließe sich das auch feststellen. Aber klar, wer hier überhaupt keine Anhaltspunkte hat, derartiges für möglich zu halten, den wird das nicht interessieren.

    „Physische Vorgänge können Sie durch Hinweis auf mentale Phänomene genau dann überhaupt nicht mehr erklären, wenn Sie sich diese mentalen Phänomene nicht von vornherein als physisch realisiert denken.“

    Wenn unser Bewusstsein sowohl auf den Prozesszuständen unseres Gehirn beruht, aber noch zusätzlich geistige Anteile hat, die diese Bewusstseins-Zustände unterstützen, dann funktioniert es dennoch, dass unser Bewusstsein auch wieder zunächst ins ganze Gehirn zurückwirkt, und in der Folge beispielsweise auch konkrete Handlungen in Gang setzt.

    In diesem Sinne wäre

    2. Mentale Phänomene sind im Bereich physikalischer Phänomene kausal wirksam.

    erfüllt.

    „Jede psychisch bedeutsame Situation, jedes psychische Phänomen umfasst materielle/physische Vorgänge – diese sind bereits hinreichend erklärt.“

    Da gehe ich von aus, ich gehe ja normalerweise nicht auf Geistesreise, wobei ich meine Physis ganz hinter mir lasse. Aber ich glaube, das da mehr dahinter steckt.

    „Der subjektive Rest ist faszinierend, auch sicherlich noch nicht erklärt, aber er hat *als solcher* keine kausale Wirkung.“

    Ich halte mein Bewusstsein für grundsätzlich interaktiv. Ich bin nicht nur passiver Beobachter, ich kann meine innere Erlebniswelt selbst ändern, indem ich mich auf was Bestimmtes konzentriere, und ich kann mir was vornehmen, dass ich später oder sofort physisch umsetzen kann.

    „Das ist auch der Grund, warum wir dem anderen seine Qualia nicht ansehen können: *als solche* wirken sie nicht kausal auf uns ein. Wir sehen immer nur das materielle/physische Verhalten unseres Gegenübers.“

    Auch hier habe ich andere Lebenserfahrung. Ich kann, glaube ich, durchaus die Emotionen von Menschen, mit denen ich enger verbunden bin, direkt erspüren. Auch über die direkten physischen Hinweise hinaus. Dieses zählt für mich sogar zu meinen direkten spirituellen (Alltags-) Erfahrungen.

    „Die einseitige Abhängigkeit des Erlebens von hirnmateriellen Vorgängen halte ich für erwiesen.“

    Wer hat das denn bewiesen? Und wie? Das kann sein, dass das so ist, und Sie dürfen das auch gerne glauben. Ich glaube das aber nicht. Ich glaube, unser Erleben hat hier auch noch einen Geistesanteil und wirkt auch auf das Gehirn wieder zurück.

    Die zukünftige Hirnforschung könnte uns hier weiterhelfen, hoffe ich. Sowohl was Hinweise in Ihrem Sinne betrifft, als auch andersrum. Die konkreten Strukturen, die unserem Bewusstsein zugrunde liegen, die fände ich in jedem Fall hochinteressant. Auch hinsichtlich der Weiterentwicklung von KI. Ich denke, man wird daran auch in den nächsten Jahren und Jahrzehnten intensiv weiterforschen.

  404. @ Balanus 21.01.2021, 12:35 Uhr

    Zitat: „Versetzen Sie sich mal gedanklich (keine Angst, dabei kann einem nichts passieren 😉 ) in eine beliebige Körperzelle, als Beobachter in der Größe eines Kohlenstoffatoms. Sie würden jede Menge Ereignisse sehen, für die es kein verursachendes Ereignis gibt (z.B. Konformationsänderungen bei Proteinen). Insoweit würde ich Heisenberg schon zustimmen wollen, auf der atomaren/molekularen Ebene gilt kein Kausalgesetz.“

    „Absolut zufällig“ ist für mich sozusagen das Gegenteil von „absolut kausal“.

    Es gibt Konstruktionen von besonderen „elektronischen Zufallsgeneratoren“, die abhängig vom “Zyklus der Abfrage“ beiden Grenzen zumindest nahe kommen.

    Allerdings neigen Menschen dazu, wenn sie eine Kausalität (bei Prozessen) nicht erkennen oder messen können, es einfach für „Zufall“ halten, obwohl man später bei besserer Messtechnik oder neuen Erkenntnissen, sehr wohl eine Kausalität erkennt.

    Die vier grundsätzlichen Wechselwirkungen der Physik in ihrer „Stärke“ und Auswirkungen an einem bestimmten Ort des Experiments und in ihren Kombinationen, mit absoluter Genauigkeit zu bestimmen, dürfte kaum möglich sein. Ausgeklügelte Messtechnik kann die Situation möglicherweise nur „verbessern“. Daher kann man nur schwer schließen, ob etwas wirklich „absoluter Zufall“ ist, oder vielleicht auch z.B. von den Ausgangsbedingungen und dem Messzeitpunkt abhängt.

    Ein Physiker, sozusagen im Rang eines „Bischofs“, meinte einmal, die geflügelten Worte „wer misst, misst Mist“ verursachten bei ihm durchaus „Glaubenszweifel“ (an der Physik).

    Bei Konformationsänderungen bei Proteinen könnte man z.B. deswegen keine Ursachen erkennen, weil man z.B. wenige „Fremdatome“ (Verschmutzungen), oder z.B. zufällig auftretende oder veränderte z.B. elektrische oder sonstiger Felder (der vier grundsätzlichen Wechselwirkungen der Physik), realistisch gesehen, einfach nicht absolut messen oder sonst irgendwie berücksichtigen kann. Dann wären die Wirkungen zwar praktisch zufällig, aber in Wirklichkeit doch kausal….

    Welche Gesetzmäßigkeit widerspricht dieser eher „schlichten“ Sichtweise?

  405. noch eine Anmerkung: Anton Zeilinger stellte in einem Interview einmal die Frage, woher weiß ein Atom, daß es zu einem Hamburger werden soll. Einige unterstellten ihm einen esoterischen Begriff von nichtmaterieller Information, was natürlich Unsinn ist. Die Antwort findet sich im Theorem der Selbstorganisation (Manfred Eigen, Ilya Prigogine): Gerät ein Gegenstand oder System durch Ungleichgewicht, Nichtlinarität, Symmetriebruch etc. in ein neues Regime, verhält es sich zukünftig nach dessen Regeln. Das erklärt, warum ein Molekül in einem lebenden Organismus anders “arbeitet”, als in der unbelebten Natur.

  406. @Balanus 21.01.2021, 12:58 Uhr

    “Wo ist das Problem?”

    Unter anderem hier: Der Erkennende erkennt in der Welt auch sich selbst, wie er gerade die Welt erkennt. Die Rekursion ist unvermeidlich. Als Nächstes kämen dann die Fragen nach Anfang und Ende der Welt. Schon Immanuel Kant hat davon abgeraten, sich mit solchen, die Vernunft überstrapazierenden Dingen zu beschäftigen.

    Anders die Frage nach dem Woher des Bewusstseins. Sie könnte eine gute Antwort finden.

  407. @Elektroniker
    „Absolut zufällig“ ist für mich sozusagen das Gegenteil von „absolut kausal“.

    Nein, Zufälligkeit und Kausalität sind grundverschiedene Dinge. Auch absolut zufällige Ereignisse haben eine Ursache, die genauso zufällig sein kann, bei zeitlicher Zufälligkeit. Zufälligkeit hat mehr mit Indeterminismus zu tun, wenn es um zufällige Resultate oder Folgeereignisse geht. Indeterminismus darf wiederum nicht mit Akausalität verwechselt werden.

    Physikalische Zufälligkeiten sind prinzipiell auf die Unschärferelation der Quantenphysik und auf die Quantelung von Energie zurückzuführen. Das ist keine Akausalität.

    „Wir sehen, dass das Universum auf ganz großen Skalen strukturiert ist, in Galaxien zum Beispiel. Die Theorie der kosmischen Inflation führt diese Struktur zurück auf die Vakuumfluktuationen, die beim Urknall vorhanden waren. Demnach haben die Vakuumfluktuationen kurz nach dem Urknall durch ihre statistische Verteilung der Felder dem Universum eine Struktur aufgeprägt.“

    Alfred Leitensdorfer, Uni Konstanz

  408. @Stegemann
    Ein Atom oder Molekül hat immer dieselben Eigenschaften. In unterschiedlichen Umgebungen oder Verbänden kann ein Atom oder Molekül unterschiedliche Zustände einnehmen. Dann verhält es sich entsprechend, z.B. in unterschiedlichen chemischen Bindungen oder in Lösungen, bei unterschiedlicher Temperatur, in gravitativen oder in elektromagnetischen Feldern. Das ist aber völlig unabhängig von belebt oder unbelebt.

  409. @anton reutlinger // 21.01.2021, 13:26 Uhr

    » Enzyme wie auch Nervenverbindungen würden nicht funktionieren, wenn die Konformationsänderungen so unbestimmt wären wie Sie behaupten. «

    Doch, das funktioniert, wie man sehen kann, alles sehr gut. Man könnte sogar sagen, wäre es anders, gäbe es keine „lebenden“ und erst recht keine „denkenden“ Systeme.

    Vielleicht hilft es wenn man sich die Öffnungswahrscheinlichkeit eines Ionenkanals vor Augen hält, oder sich überlegt, was bei einem Pharmakon der Begriff Dissoziationskonstante bedeutet.

  410. @Timm Grams // 21.01.2021, 15:39 Uhr

    » Der Erkennende erkennt in der Welt auch sich selbst, wie er gerade die Welt erkennt. Die Rekursion ist unvermeidlich. «

    Ist aber kein großes Problem. Über sich selbst nachzudenken ist Teil des alltäglichen Lebens. Über das Denken nachdenken gehört zum philosophischen Handwerk (bzw. Geisteswerk).

    Und dass es naturbedingte Grenzen der Erkenntnis gibt—so what?

    (nicht, dass ich die Probleme kleiner reden möchte als sie sind, mir scheinen sie bloß kein spezielles Problem naturalistischer Weltauffassungen zu sein)

  411. @anton reutlinger // 21.01.2021, 12:36 Uhr

    @Stegemann schrieb:

    » Ein lebendiger Organismus zwingt ein Molekül, Dinge zu tun, die dieses in der unbelebten Natur nicht tun würde. «

    Darauf Sie:

    » Dieser, meines Erachtens esoterischen Behauptung möchte ich entschieden widersprechen. «

    Ich habe überhaupt kein Problem mit Stegemanns Äußerung. Mir scheint auf der Hand zu liegen, was damit gemeint ist:

    Es gibt kein Ding namens „Leben“, „lebend sein“ ist ein sich selbst erhaltender Prozess, in dem zum Beispiel die Transskriptasen nur solange aktiv sind, wie genügend chemische Energie zur Verfügung steht. Oder denken Sie an den Citrat-Zyklus oder an die Photosynthese, All das sind Prozesse in lebenden Systemen, in nichtlebenden Systemen kommen sie zum Erliegen.

  412. @anton reutlinger 21.01. 15:55

    „Zufälligkeit hat mehr mit Indeterminismus zu tun, wenn es um zufällige Resultate oder Folgeereignisse geht. Indeterminismus darf wiederum nicht mit Akausalität verwechselt werden.
    Physikalische Zufälligkeiten sind prinzipiell auf die Unschärferelation der Quantenphysik und auf die Quantelung von Energie zurückzuführen. Das ist keine Akausalität.“

    Vielleicht ist hier eher der Gegensatz von Regelmäßig und Unregelmäßig treffender.

    Wenn ich bei schlechten Lichtbedingungen mit der Digitalkamera ein Foto mache, hat das ein Bild des Motivs zur Folge, das mit Pixelrauschen überlagert ist. Wenn ich ein scharfes, gut belichtetes Foto habe, und ich dieses im Computer so nachbearbeiten will, dass es wie ein unterbelichtetes Foto aussieht, muss ich jeden Pixelwert des Bildes mit einer Zufallszahl addieren. Dieses bearbeitete Bild sieht dann so aus, als wenn ich es bei schlechten Lichtbedingungen fotografiert hätte.

    Wenn ich mathematisch erzeugte Pseudozufallszahlen dafür verwende, dann müssen die so gestaltet sein, dass sie keinerlei Regelmäßigkeit aufweisen. Andernfalls würden sie dem Anspruch an Zufälligkeit nicht gerecht, bzw. würde man das eventuell dann auch dem Bildergebnis ansehen.

    Wenn ich jetzt aber analoge Zufallszahlen verwende, und ich eine praktikable Quelle dafür finde, werden diese Zufallszahlen auch daran zu messen sein, dass sie keine eigene Regelmäßigkeit haben. Die Quantenzufälle, die beim Zusammenbruch von Wellenfunktionen entstehen, würden wohl dieser Anforderung theoretisch genügen können.

    Hier wäre eben der Gegensatz von Regelmäßig-Unregelmäßig entscheidend. Kausal-Akausal oder Determiniert-Indeterminiert sind hier beim Erstellen dieser nachgemacht unterbelichtet verrauschten Fotos irrelevant. Hier kommt es ganz auf den Aspekt der Unregelmäßigkeit an.

  413. @Balanus / 20.01.2021, 19:13 Uhr

    »Auch im verlinkten Artikel (Im Naturalismus gefangen), da möchte man schon nach dem fünften Satz aufhören zu lesen: …«

    Da kann ich Dir nicht ganz folgen, warum Du da aufhören möchtest zu lesen.

    »Es scheint, als habe Geert Keil (und mit ihm viele andere) übersehen, dass sich der metaphysische Naturalismus logisch selbst widerlegt.«

    Falls Geert Keil meinen sollte, man könne eine antinomienfreie Gesamtheit all dessen, ›was es gibt‹ bilden, hätte er etwas übersehen. Ob er das meint, lässt sich allerdings aus dem, was Du als Zitat anführst, nicht ersehen, und da ich diesen Text nicht kenne, kann ich das auch nicht beurteilen. (Dass viele andere seit Adam und Eva schon verdammt viel übersehen haben, halte ich indes für völlig unstrittig.)

  414. @Balanus
    All das sind Prozesse in lebenden Systemen, in nichtlebenden Systemen kommen sie zum Erliegen.

    Das sind doch keine Geheimnisse. Lebensprozesse spielen sich abseits des Gleichgewichts ab. Energie wird ständig zu- und abgeführt, mit der Nahrung, mit dem Sauerstoff der Atmung, mit der relativen Kälte der Umgebung. Das Besondere des Lebens sind die komplexen Moleküle der organischen Chemie, basierend auf den atomaren Eigenschaften des Kohlenstoffs. Es ändert aber grundsätzlich nichts an den chemischen Prozessen.
    Ich kann Ihnen da nicht folgen.

  415. Zunächst kurz zu Ethik: Der den Irrtumsvorbehalt voraussetzende, kritische wissenschaftliche Realismus scheint eine eher ungefährliche Haltung zu sein, denn es sind keine mordenden Milizen bekannt, die sich darauf berufen. Wir kennen aber Milizen, die mordend durch die Gegend ziehen und sich dabei auf eine Religion berufen. Das NS-Regime und der Stalinismus waren übrigens religionsähnliche Systeme mit irrationalen Überzeugungen und einem Oberpriester an der Spitze. Adolf Eichmanns letzte Worte waren: “Gottgläubig war ich im Leben. Gottgläubig sterbe ich.” Es lassen sich noch zahlreiche weitere empirische Belege anführen, die zeigen, dass Religionen oder religionsähnliche Überzeugungen nicht zwingend zu besserem ethischen Verhalten führen. Ich nehme der Einfachheit an, dass sich die religiöse Einstellung im Mittel nicht im ethischen Verhalten auswirkt. Gläubige werden dies öfters wütend bestreiten und für sich eine überlegene Moral reklamieren.

    Was das Verhältnis von Wissenschaft und Religionen angeht, so werden Forschungsresultate regelmäßig von religiöser Seite angegriffen, wenn sie nicht zu den jeweiligen Glaubensinhalten passen. Das war und ist bei der Evolutionstheorie so und gilt auch für die Neurologie. In den meisten Religionen gilt das Dogma vom “frei schwebenden Geist”, was anschaulich gesprochen der Annahme entspricht, die Realität sei voller Gespenster. Das wird in den Kirchen natürlich viel poetischer ausgedrückt und dort gehört es auch hin. Mit den empirischen Wissenschaften ist das aber nicht kompatibel, es existieren überbordend viele und immer detailreichere Hinweise darauf, dass Denken und Fühlen in lebenden Körpern physisch erzeugt wird. Ein Gespenst erklärt überhaupt nichts. In der neurologischen Forschung gelingt es hingegen, die Inhalte bestimmter mentaler Vorstellungen aus Hirnscans überzufällig abzulesen. Kreative gläubige Menschen haben allerdings wirksame rhetorische Strategien entwickelt, wie z.B. den Gish-Galopp, um sich gegen Kritik der Vorstellung eines absoluten Geists abzuschirmen. Der kritische Rationalist Hans Albert hat religiöse Immunisierungsstrategien ausgiebig analysiert.

    Entgegen einer der obigen Einlassungen reicht es für den Materialisten, dass “mentale Phänomene” aus, physischen Prozessen heraus entstehen. Das nennt sich emergentistischer Materialismus und wird u.a. von Mario Bunge vertreten. Ein eliminativer Materialismus braucht keineswegs vertreten zu werden. Die Ad-hoc-Idee, dass es da ein mental verursachendes Gespenst im Körper gibt, wird in der Neurologie nicht verfolgt. Ein mentales Gespenst ist ein völlig unspezifisches Konzept, erklärt daher gar nichts und ist mit Beliebigem kompatibel.

    Die behauptete Selbstwiderlegung des Naturalismus ist schlicht ein Strohmann. Das Annahme, dass reines Denken die Außenwelt praktisch nicht beeinflusst, ist mitnichten selbstwiderlegend. Die einzige Wirkung auf die Außenwelt sind sehr schwache elektromagnetische Felder. Der Monitor, der vor mir steht, wird durch meine Gedanken nicht beeinflusst, das geht nur durch eine Handlung unter Einsatz der Extremitäten. Das ist empirisch sehr gut belegt.

    Es gibt keine Hinweise darauf, dass wir in einer Art “Matrix” leben. Deutliche “Fehler in der Matrix”, die Fähigkeit sich einen bewaffneten Helikopter zu wünschen, wenn man einen braucht, oder im Cha-Cha-Rhythmus zerfallende Radium-Atome würden den Naturalismus erschüttern. Der Supranaturalismus als Gegensatz des Naturalismus ist hingegen mit beliebigen Vorkommnissen in der Welt kompatibel und daher völlig kritikimmun. Er erklärt alles und damit nichts. Kritisierbarkeit ist eine erkenntnistheoretisch wünschenswerte Eigenschaft, da kein Letztbegründungsverfahren bekannt ist.

    Zum Determinismus: Dieser muss nicht angenommen werden. Falls die Quantenmechanik vollständig ist, gibt es nämlich absoluten Zufall in der Welt. Diese folgen jedoch auch naturgesetzlichen Mustern, d.h. statischen Verteilungen, die hochgenau ausgemessen wurden.

    Der *absolut* freie Wille: Dieses Konzept erschwert nur die Zuschreibung von Entscheidungen zu einer bestimmten Person, da vorausgesetzt wird, dass eine Person immer Beliebiges entscheiden kann, was reinem Zufall entspräche. Was es gibt, ist Denk- und Handlungsfreiheit.

    Zum Begriff Welt: Umgangssprachlich ist oft die Erde gemeint, ansonsten oft das Universum. Es ist eine wissenschaftliche Standardannahme, dass es das Universum gibt. Man muss Existieren und In-der-Welt-sein nicht als äquivalent definieren, wie Gabriel das tut.

    Das ist freilich alles kontrovers. Was man tun kann, ist, die verschiedenen Annahmen einer kritischen vergleichenden Prüfung auszusetzen.

  416. @Jeckenburger
    Regelmäßigkeit ist zweifellos ein Gegensatz zur Zufälligkeit. Es gibt viele Formen von Regelmäßigkeit, die nicht immer leicht zu erkennen sind, oder gar nicht. Ein Beispiel: In Zeitschriften wie in IQ-Tests gibt es mathematische Rätsel, bei denen in einer scheinbar zufälligen Zahlenreihe Regelmäßigkeiten zu suchen sind und die Reihe entsprechend fortgesetzt werden soll. So gibt es in der Welt bestimmt auch Regelmäßigkeiten, die nicht erkannt werden und daher für Zufall gehalten werden.

    Regelmäßigkeit bedeutet Ordnung bedeutet Information. Je größer die Differenz zwischen Regelmäßigkeit und Unregelmäßigkeit desto höher ist der Informationsgehalt. Eine zufällige Zahlenfolge kann Information durch Sortieren importieren. Die Information liegt in der Vorkenntnis der Zahlen. Eine vollständig sortierte Zahlenfolge kann Information exportieren, aber nicht importieren. Das Sortieren einer vollständig sortierten Zahlenfolge bedeutet Redundanz, die an der Zahlenfolge nichts ändert.

  417. @anton reutlinger // 21.01.2021, 18:25 Uhr

    » Es ändert aber grundsätzlich nichts an den chemischen Prozessen. «

    Grundsätzlich natürlich nicht, schließlich gelten die naturgegebenen Gesetzmäßigkeiten immer und überall.

    Dann werfen wir doch mal einen Blick auf die Proteinbiosynthese. Da kann man doch durchaus (mit @Stegemann) sagen, dass der lebende Organismus Moleküle zwingt, Dinge zu tun, die sie in der unbelebten Natur nicht tun würden.

  418. @Stephan Schleim // 21.01.2021, 14:36 Uhr

    » Für den Materialisten reicht es nicht, dass “mentale Phänomene” irgendwie aus physischen Prozessen heraus entstehen; für ihn müssen auch die mentalen Phänomene vom Wesen her materiell sein. «

    Sagt wer?

    Ich habe nochmal nachgeschaut, was zum Glaubenssystem eines Materialisten gehört. Der Begriff „Wesen“ gehört glaube ich nicht dazu. Sehr wohl aber kennt der Materialist Gedanken, und die stehen, man ahnt es schon, im gleichen Verhältnis zum Gehirn wie die Galle zur Leber. Es wäre aber völlig verfehlt, nun anzunehmen, Gedanken selbst, oder mentale Phänomene, seien materielle Dinge, das wäre ja absurd.

    Der auf MENSCHEN-BILDER vielfach gescholtene bekennende und mittlerweile verstorbene Materialist Mario Bunge betrachtete mentale Phänomene, also etwa die Sinnesempfindung, die auch er nicht leugnen konnte, als eine spezielle Eigenschaft der komplex organisierten Hirnmaterie (ob er damit aus dem Schneider ist bzw. war, mögen andere entscheiden).

    »Bei mentaler Verursachung (zweite Prämisse) geht es darum, ob “mentale Phänomene” (oder wie ich lieber sage: psychische Prozesse) aufgrund ihrer spezifischen mentalen/psychischen Eigenschaften (z.B. intentionaler oder phänomenaler Gehalt) kausal wirksam werden können. «

    Ok, Du sprichst von „psychischen Prozessen“, ich aber habe von „mentalen Phänomenen“ wie etwa die Schmerzempfindung oder das Farbensehen gesprochen. Das kommt mir vor wie das Verschieben des Torpfostens.

    » Wenn du einen Kausalbegriff verwendest, der das [kausal wirksam werden mentaler Phänomene] von vorneherein ausschließt, dann schließt du das eben aus: Das ist dann aber eine Annahme deines Denkens und keine Schlussfolgerung.«

    Ich verstehe unter Kausalität das übliche Ursache-Wirkungs-Prinzip aus der klassischen Physik. Etwas anderes ergibt im Bieri-Trilemma doch überhaupt keinen Sinn.

    »Dann bleiben die Probleme zu erklären, wie es denn sein kann, dass aus Materiellem etwas mit psychischen Eigenschaften entstehen kann; …«

    Naja, was sind „psychische Eigenschaften“? Wenn man sich die Entwicklung eines Menschen, beginnend mit der befruchteten Eizelle, anschaut, dann schreibt man dem Individuum irgendwann „psychische Eigenschaften“ zu. Schätze, die sind dann nicht vom Himmel gefallen… (ist keine Erklärung, ja ich weiß…)

    Aber mit dem Bieri-Trilemma hat das ja allenfalls am Rande zu tun. Da geht es doch wohl eher um “mentale Phänomene” als da sind … (siehe oben).

    Die wirklich interessante, spannende und höchstwahrscheinlich unlösbare Frage ist, wie Nervenzellverbände subjektiv erlebbare Farben und Töne erzeugen: Warum erscheint uns eine bestimmte Wellenlänge des Lichts rot und nicht blau.

  419. @Chrys // 21.01.2021, 18:07 Uhr

    » Da kann ich Dir nicht ganz folgen, warum Du da aufhören möchtest zu lesen. «

    Also bitte, da sollen naturwissenschaftliche Disziplinen eine „vollständige“ (!) Erklärung beanspruchen, sodass damit für „Freiheit“ und „verantwortliches Handeln“ kein Platz mehr bleiben soll, weil das Gehirn alles steuere. Da geht doch alles wild durcheinander, da werden sämtliche Beschreibungsebenen zu einem unverdaulichen Brei verrührt. Was meinem derzeit empfindlichen Magen gewiss nicht zuträglich ist…

    » Falls Geert Keil meinen sollte, man könne eine antinomienfreie Gesamtheit all dessen, ›was es gibt‹ bilden, hätte er etwas übersehen. «

    Er spricht es halt nicht an. Was verwundert, da es sich ja um Kernaussagen des metaphysischen Naturalismus handeln soll, die da logisch inkonsistent sind.

    Aber es ist zugegeben ein relativ kurzes Kapitel in seinem Aufsatz, da hat er die Metaphysik sicherlich nicht erschöpfend behandelt. Neben dem metaphysischen wird noch der szientifisch-methodologische und der semantisch-analytische Naturalismus besprochen, wohlwissend, dass im Bereich des methodologischen Naturalismus „auch noch feinerere Differenzierungen [existieren]“ (huch, „existieren“, na, wenn das man gut geht…).

  420. @Balanus
    Da kann man doch durchaus (mit @Stegemann) sagen, dass der lebende Organismus Moleküle zwingt, Dinge zu tun, die sie in der unbelebten Natur nicht tun würden.

    Das ist so, als würde ich meinen Darm zwingen, Dinge zu tun, die er in der unbelebten Natur nicht tun würde.
    Viele Moleküle des lebenden Organismus gibt es in der unbelebten Natur gar nicht.

  421. @Balanus: Materialismus

    Sagt wer?

    Wenn wir uns nicht einmal darauf einigen können, dass laut Materialismus alles Materie ist, dann lassen wir es doch bleiben und nennen wir es, wieder einmal: Balanismus.

    Schön.

  422. @ Balanus 21.01.2021, 19:02 Uhr

    Zitat: „Die wirklich interessante, spannende und höchstwahrscheinlich unlösbare Frage ist, wie Nervenzellverbände subjektiv erlebbare Farben und Töne erzeugen: Warum erscheint uns eine bestimmte Wellenlänge des Lichts rot und nicht blau.“

    Unbewiesene aber naheliegende Erklärungen gibt es schon.
    Eine z.B. blau gestrichene Bank wirft Lichtstrahlen einer bestimmten modulierten Wellenlänge des Lichtes auch in Richtung der bei der Farbe blau „aktiv“ werdenden „Zapfen“ im Bereich der Netzhaut. Andere Zapfen werden bei „rot“ aktiv….

    Vermutlich „fliegen“ bei diesen sensorischen Zellen, wie bei anderen Sensorikelementen auch, Elektronen aus den Elektronenbahnen bestimmter Moleküle. Diese „emittierten“ Elektronen werden über Nervenleitungen zwecks Auswertung in das neuronale System geführt. Es wird offensichtlich auf die Elektronen der Elektronenbahnen nach quantenphysikalischen Gesetzen Einfluss genommen werden.

    Es ist eine unbeweisbare Vermutung und Behauptung, dass dabei z.B. auch die „Blauempfindung“ (direkt an den Zapfen) entsteht und ausgewertet wird. Z.B. hinsichtlich ihrer Lokalität und ihrer Nachbarschaft… so dass letztlich „Muster“ erkannt werden und weitere Auswertemöglichkeiten bestehen.

    Diese Quanteneffekte kann es natürlich auch auf der blauen Farbe der Bank geben, allerdings hat die Bank kein neuronales System um die Empfindungen selbst auszuwerten. Elektronen würden einfach vom Holz unselektiv absorbiert und erhitzen die Bank. Der modulierte Lichtstrahl ist das Transportmedium zu Mensch oder Tier soferne ein geeignetes neuronales System bereit ist, den modulierten Lichtstrahl aufzunehmen und umzusetzen.

    Danach dürfte es in mehreren Stufen, immer realisiert in geeigneten Neuronentypen, „aufspaltende- “ und danach „zusammenführende Verknüpfungen“ geben, die an Zwischenschichten (vergleichbar wie auf der Netzhaut) informelle Abbildungen (als gleichzeitig triggernde Neuronen) erzeugen die ebenfalls nach den Prinzipien Aufspaltung -Zusammenführung weiter ausgewertet werden.

    Über denkmögliche Prozesse habe ich schon mehrfach aus Sicht der Elektronik geschrieben.

    Vielleicht könnten Sie, auch möglichst sachlich begründen, warum dieses Konzept, z.B. aus der Sicht von Neurologen falsch ist.

  423. @Timm Garms

    Sie schreiben in Kommentaren:

    Der Erkennende erkennt in der Welt auch sich selbst, wie er gerade die Welt erkennt. Die Rekursion ist unvermeidlich.

    und

    Mein Punkt ist die Rekursion, auch infiniter Regress genannt.

    Probleme bei der Rekursion könnten dann auftreten, wenn immer das selbe Selbst sich erkennen müsste.

    Im Blog-Text schreiben Sie:

    Es müsste schon dieselbe Person sein. Aber nach dem ersten Durchlauf ist deren mentale Ausstattung bereits verändert, so dass sie für den folgenden Prüflauf nicht infrage kommen kann.

    Also ist es nach ihren eigenen Worten, nimmt man für das Betrachten keinen instantanen Vorgang an, immer eine andere Person (ein anderer Erkennender), die eine vergangene Person betrachtet, beim Versuch diese zu erkennen. Zum Beispiel könnte jemand, beim Versuch, über sich selbst etwas herauszufinden, seine Erinnerung an vergangene Gedanken problemlos mit der Aufzeichnung eines zeitgleich entstandenen Hirnscans abgleichen. Da droht kein infiniter Regress.

    Ein infiniter Regress droht nicht in der Natur, er droht in der Logik. Er droht, wenn man eine Person, deren Geist oder das Selbst als über die Zeit unveränderliche Instanz setzt. Nur, wer macht sowas schon, sei es auch nur gelegentlich? Sicher kein Naturalist.

    Erkenne dich selbst!

  424. Herr Grams, ich hoffe, Sie erkennen sich als Gemeinter auch bei der Anrede ‘Garms’.

    Sorry.

  425. Ein Spiel mit dem Lagrange Dämon soll zeigen, dass Kausalität nicht immer gegeben ist. Wir haben ein Glücksrad von blauen und roten Feldern, genau 64 Felder. Das erste Feld ist blau, das zweite Feld ist rot, das dritte Feld ist blau, …..das 64. Feld ist rot.

    Die Spielregel lautet, wenn man auf ein rotes Feld setzt, wird der gesetzte Betrag halbiert. Setzt man auf ein blaues Feld wird der Betrag verdreifacht, und man bekommt zusätzlich einen Euro vom Glücksrad -Besitzer .

    Beispiel: Anni setzt 30 Euro auf das rote 30. Feld. Beim nächsten Spiel hat sie nur noch 15 Euro auf dem blauen Feld 15.

    Aber beim 3. Zug darf sie ihren Einsatz verdreifachen, weil sie auf dem blauen Feld ist, also 45 Euro auf dem blauen Feld 45. Sie bekommt aber noch 1 Euro und muss vor auf das rote Feld mit 46 Euro.

    Jetzt werden Anni und dem Dämon die Augen verbunden und jeder setzt einen Betrag. Nach dem 3. Zug, oh, Wunder haben beide ihren Spieleinsatz auf dem roten 16. Feld mit je 16 Euro.

    Frage: Welchen Betrag hat Anni am Anfang gesetzt und auf welches Feld.
    Welchen Betrag hat der Dämon gesetzt und auf welches Feld. ?

    Da Feld und Betrag gleich sind dürfte es dem Dämon leicht fallen durch Rückwärtsrechnen die Lösung zu finden .

  426. @Joker 21.01.2021, 20:30 Uhr

    Sie vermischen zwei voneinander getrennte Gedankengänge, einen zur Erkennbarkeit der Welt und einen zweiten zu Willensfreiheit.

    Sie ändern die Prämissen und kommen, was kein Wunder ist, zu anderen Schlussfolgerungen als ich. Daraus kann kein Widerspruch erwachsen.

  427. Das hier klingt natürlich seltsam:
    Wer “Realität” mit “Wirklichkeit” verwechselt, der legt falsche Fährten, die subtile Falschdeutungen erzeugen.

    Realität kommt meiner Ansicht nach von “realisieren” und ist ein subjektives Szenario. Hingegen ist die Wirklichkeit das, was wirklich ist und wirkt.
    Wir selbst sind zwar auch “wirklich”, aber wir sind in unserem bewusstysein eben eingeschränkt, weil wir nur das zur Wirklichkeitsbeschreibung verwenden können, was wir auch tatsächlich realisieren und gedanklich “repräsentieren” und Kombinieren können.

    Ich glaube, das jede Sprache einst ausschliesslich aus Verben hervorging, weil das Verb eigendlich das wesendlichste und wichtigste in einfachen Sozialgemeinschaften aussagt. Was tun? Was sein? Was werden? Immer ist vor allem das Verb zentral, was vermittelt/Kommuniziert werden will, wenn Mernschen “bewusst” ihre Umwelten erkennen. Wichtig ist Bewegung und Prozess, wenn man die Umwelten begreifen und beherrschen will. “Stillleben” und tote Gegenstände sind da nur Randerscheinungen. Was sich nicht bewegt, was keinen Prozess vollzieht, das ist nebensächlich.

    Eine Ausnahme scheint hier das Lateinische zu sein. Ich bin aber … mangels dazu hinreichender “Schuldbildung (Hauptschulabschluß) kein Spezialist für Latein.

  428. @Balanus

    Sehr wohl aber kennt der Materialist Gedanken, und die stehen, man ahnt es schon, im gleichen Verhältnis zum Gehirn wie die Galle zur Leber.

    Bei diesem Gedanken kommt mir die Galle hoch.

    @Stephan Schleim

    dass laut Materialismus alles Materie ist

    Bei diesem Gedanken auch.

    (Hervorhebung jeweils im Original, Kursivsetzung durch Fettschrift ersetzt)

  429. @ anton reutlinger 21.01.2021, 15:55 Uhr

    Zitat Elektroniker: ““Absolut zufällig“ ist für mich sozusagen das Gegenteil von „absolut kausal““.

    O.K. Ich habe schlampig formuliert, der Begriff “kausal“ geht hier gar nicht.

    Formuliere jetzt anders: “Absolut zufällig“ ist für mich sozusagen das Gegenteil von „absolut nicht zufällig“

    Zufälligkeit hat in der Tat mehr mit „Indeterminismus“ zu tun, wie Sie es sehen. Absolute „nicht Zufälligkeit“ wäre demnach eher Determiniertheit, wenn es um nicht zufällige Resultate oder Folgeereignisse geht.

    Es geht mir darum, mit dem „elektronischen Zufallsgenerator Modell“ versuchen aufzuzeigen, dass es nicht um „zufällig – nicht zufällig“ („determiniert – nicht determiniert“) geht, sondern es so etwas wie einen „Grad der Zufälligkeit“, gibt.

    Ich weiß es nicht, aber vielleicht ist mit dem Begriff „Unschärfe(relation)“ ohnehin so etwas ähnliches wie ein „Grad der Zufälligkeit“ gemeint.

    Es geht mir weniger um die Entstehung von Galaxien.

    Es gibt Konstruktionen von besonderen „elektronischen Zufallsgeneratoren“, die abhängig vom “Zyklus der Abfrage“ Zufallsvariable zwischen den beiden Grenzen „generieren“ können, bzw. auch „wirkliche Zufallszahlen“, oder auch „determinierte Zahlen“ die höchstens wie Zufallszahlen scheinen, aber völlig determiniert sind.

    Menschen neigen, wenn sie eine Determiniertheit (bei Prozessen) nicht erkennen oder messen können dazu, es einfach für „Zufall“ zu halten, obwohl man später bei besserer Messtechnik oder neuen Erkenntnissen, sehr wohl eine Determiniertheit erkennt.

    Die vier grundsätzlichen Wechselwirkungen der Physik in ihrer „Stärke“ und Auswirkungen an einem bestimmten Ort des Experiments und in ihren Kombinationen, mit absoluter Genauigkeit zu bestimmen, dürfte kaum möglich sein.

    Ausgeklügelte Messtechnik kann die Situation möglicherweise nur „verbessern“. Daher kann man nur schwer schließen, ob etwas wirklich „absoluter Zufall“ ist, oder vielleicht auch z.B. von den Ausgangsbedingungen und dem Messzeitpunkt abhängt.

    Ein Physiker, sozusagen im Rang eines „Bischofs“, meinte einmal, die geflügelten Worte „wer misst, misst Mist“ verursachten bei ihm durchaus „Glaubenszweifel“ (an der Physik).

    Bei Konformationsänderungen bei Proteinen könnte man z.B. deswegen keine Ursachen erkennen, weil man z.B. wenige „Fremdatome“ (Verschmutzungen), oder z.B. zufällig auftretende oder veränderte z.B. elektrische oder sonstiger Felder (der vier grundsätzlichen Wechselwirkungen der Physik), realistisch gesehen, einfach nicht absolut messen oder sonst irgendwie berücksichtigen kann. Dann scheinen die Wirkungen zwar praktisch zufällig, aber in Wirklichkeit wären die Prozesse determiniert ….

    Welche Gesetzmäßigkeit widerspricht dieser eher „schlichten“ Sichtweise?

  430. @Joker: P.S. Materialismus

    Wird dir bei der Definition dieser Fachleute auch schlecht?

    Materialism: A metaphysical doctrine in philosophy that the world and all its entities and phenomena, including psychological phenomena, are manifestations of spatiotemporal matter.

    Bem, S. & de Jong, H. L. (2006). Theoretical Issues in Psychology: An Introduction. Sage, p. 277.

  431. @Balanus / 21.01.2021, 19:29 Uhr

    »Also bitte, da sollen naturwissenschaftliche Disziplinen eine „vollständige” (!) Erklärung beanspruchen, sodass damit für „Freiheit” und „verantwortliches Handeln” kein Platz mehr bleiben soll, weil das Gehirn alles steuere.«

    Da musst Du nicht lange suchen, um derlei Positionen schon hier im Kommentarbereich verteten zu finden.
    Siehe Christian Hoppe (18.01.2021, 13:21 Uhr) @Stephan Schleim:

    Das ändert aber nichts daran, dass die dabei stattfindenden physischen Vorgänge in der Physik bereits ihre vollständige prinzipielle Erklärung gefunden haben.

    Hinsichtlich materieller/physischer Vorgänge bleibt kein unerklärlicher Rest, der dann z.B. psychologisch erklärt werden könnte.

    »Er [Keil] spricht es halt nicht an. Was verwundert, da es sich ja um Kernaussagen des metaphysischen Naturalismus handeln soll, die da logisch inkonsistent sind.«

    Es handelt sich um die von Kant herausgestellten Antinomien bei Totalitätsbegriffen (mit denen mengentheoret. Antinomien, die man dann später gefunden hat, durchaus bereits antizipiert sind). Keil scheint mir dem Zitat zufolge diejenigen Naturalisten, die mit Totalisierung der Natur argumentieren, für besonders einfältig zu halten und vermeidet es vielleicht lieber, sich mit den Totalitätsfragen näher zu befassen, indem er sie als nicht spezifisch für Naturalismus abtut. Möglicherweise kennt er ja Varianten von metaphysischem Naturalismus, wo nicht totalisiert wird, und die dann nicht unter ontischen Monismus fallen würden. Ich kenne einstweilen keine, aber gegebenenfalls kannst Du das noch etwas aufklären.

    »„auch noch feinerere Differenzierungen [existieren]”«

    Ja, das geht hier sehr gut mit dem “existieren”; ist ja keine Metaphysik…

  432. @Timm Grams

    Sie vermischen zwei voneinander getrennte Gedankengänge, einen zur Erkennbarkeit der Welt und einen zweiten zu Willensfreiheit.

    Dass das getrennte Gedankengänge waren, ist offensichtlich. Spricht etwas dagegen, diese zu vermischen?

    Sie ändern die Prämissen und kommen, was kein Wunder ist, zu anderen Schlussfolgerungen als ich.

    An welcher Stelle ändere ich Prämissen? Die stammen alle im Original von Ihnen.

    Verstehe ich Sie dann richtig, ‘Personen’, die im Zusammenhang mit Willensfreiheit erwähnt werden, sind nach Ihrem Verständnis nicht ‘Personen’, die im Zusammenhang mit Welt- und Selbsterkenntnis stehen?

    Falls begriffliche Kohärenz kein Anspruch in Ihrer Kritik ist, sollten Sie diesen Anspruch auch nicht gegenüber den von Ihnen kritisierten Naturalisten aufrecht erhalten. Gleichheit für Alle.

    Sehen Sie meinen Kommentar einfach als Denkanstoß und Gelegenheit, selbst noch einmal über das Problem des infiniten Regresses reflektieren zu können, selbstverständlich nur mit Ihren eigenen Prämissen.

  433. @Stephan Schleim

    Wird dir bei der Definition dieser Fachleute auch schlecht?

    Materialism: A metaphysical doctrine in philosophy that the world and all its entities and phenomena, including psychological phenomena, are manifestations of spatiotemporal matter.

    Nein, mir nicht.

    Die Definition ist kritikwürdig, wie wohl alle Definitionen, man ist in solchen Fällen ja immer gezwungen, auf andere Begriffe zurückzugreifen, aber die scheint mir auf den ersten Blick ganz gut.

    Kannst Du aufgrund dieser Definition für mich (das ist die Herausforderung) schlüssig ableiten, ein Materialist müsse sagen,

    Eine Definition ist Materie
    Die Welt ist Materie
    Raum ist Materie
    Zeit ist Materie
    Psychologie ist Materie
    Ein Phänomen ist Materie
    Eine Manifestation ist Materie
    Ein Satz ist Materie
    Ein Vorgang ist Materie
    Ein Gedanke ist Materie
    Ein Gedankeninhalt ist Materie
    Eine Eigenschaft ist Materie
    Eine Gesetzmäßigkeit ist Materie
    Unfug ist Materie
    Ein Widerspruch ist Materie
    Eine Lücke zwischen materiellen Objekten ist Materie
    Eine Wissenslücke ist Materie

    oder kurz, dass alles Materie ist.

    Wie gefällt Dir die Definition von Laien:

    Der Materialismus ist eine erkenntnistheoretische und ontologische Position, die alle Vorgänge und Phänomene der Welt auf Materie und deren Gesetzmäßigkeiten und Verhältnisse zurückführt.

  434. Joker,
    Bei den meisten Beiträgen hier vermisse ich konkrete Beispiele.
    Beispiel Willensfreiheit. Wenn ich morgens zum Frühstück die Auswahl habe zwischen Tee oder Kaffee und frei entscheiden darf, dann ist das Willensfreiheit.

    Ich vermisse auch den Unterschied von Wachsein und Traum. In beiden Fällen bin ich mir bewusst, aber im ersten Fall kann ich dabei Zahnschmerzen haben. Während eines Traumes hatte ich noch nie Zahnschmerzen.

    Der beste Satz: alles ist Materie. Dann sollten wir mehr Mitleid mit der Materie haben. Manche Menschen sind in Materie verliebt, Geld, Gold , Diamanten.

    Das Wort des Tages: “spatiotemporal”. Vor diesem Wort haben die Traumtänzer Angst. Schluss, back to reality.

  435. @Joker: Laiendefinition

    Das ist eine schwächere Form… die dann aber meinem Verständnis nach z.B. einen Eigenschaftsdualismus ermöglicht, etwa in der Form: Zwar ist das grundlegende Prinzip Materie (oder Energie oder Wasauchimmerphysikalisches), doch entstehen aus bestimmten Strukturen neue Eigenschaften, die dem Wesen nach nicht-materiell sind.

    So hat man sich ja in der Analytischen Philosophie um die Jahrhundertwende (20./21. Jh.) um einen nichtreduktiven Materialismus bemüht, der nicht auf der einen Seite in einen starken Materialismus kollabiert und nicht auf der anderen Seite den Naturwissenschaften widerspricht. Der letzte Stand war, meines Wissens, dass Jaegwon Kim – ich sehe gerade, er ist im November 2019 verstorben – das Projekt als Aussichtslos aufgegeben hat (“Physicalism, or Something Near Enough”, 2005).

    Wenn man die z.B. balanesischen Aussagen genauer analysieren würde, käme man meiner Meinung nach zwigend zum Ergebnis, dass sie auf einen Eigenschaftsdualismus hinauslaufen; und dann stünde man quasi wieder am Ausgangspunkt und hätte es jetzt mit der Frage zu tun, ob sich diese Eigenschaften (1) vollständig auf Materie (bzw. Energie, etc.) reduzieren lassen oder (2) sie stark emergent sind in dem Sinne, dass selbst vollständiges Wissen der grundlegenderen (hier also: materiellen, energetischen) Ebene die emergenten Eigenschaften nicht erklären kann.

    Aber auch ein starker Materialismus, so lese ich deinen Kommentar, müsste eine Antwort auf die Frage nach der Existenz von Abstraktem, Allgemeinen geben, sprich: eine Antwort für den seit der Antike tobenden Universalienstreit geben. Er würde wahrscheinlich sagen, dass es Abstracta usw. nur im Denken der Menschen gibt (Kognitivismus) und in diesem Simme – laut Hypothese des Materialisten – nichts als Materie sind.

  436. @Grams: Woher kommt das Bewusstsein?

    Haben Sie schon mal die die Wechselwirkung und die Aussagenlogik in Deckung gebracht (Energie + Mechaninsmus)? Dann wäre der Lebensfunke der “Motor”.

  437. @Joker

    Sie verstehen mich richtig, wenn Sie schreiben: “‘Personen’, die im Zusammenhang mit Willensfreiheit erwähnt werden, sind […] nicht ‘Personen’, die im Zusammenhang mit Welt- und Selbsterkenntnis stehen.”

    Einmal betrachtet ein “Versuchsleiter” von ihm verschiedene “Versuchspersonen”. Im anderen Fall ist er selbst die “Versuchsperson”.

    Im ersten Fall entsteht kein Rekursionsproblem. Das Experiment kann durchaus in einem nicht vollständig durchschaubaren Unniversum stattfinden.

  438. @hwied
    Beispiel Willensfreiheit. Wenn ich morgens zum Frühstück die Auswahl habe zwischen Tee oder Kaffee und frei entscheiden darf, dann ist das Willensfreiheit.

    Das ist eben die große Frage, ist das wirklich Willensfreiheit? Ich behaupte nein, oder nur zum Teil. Lassen wir uns bei unseren Entscheidungen nicht von unseren Erinnerungen und unseren gegenwärtigen Wahrnehmungen leiten, z.B. ist noch genug Kaffee oder Tee vorhanden, welche Sorten? Wieviel Zeit habe ich für das Frühstück, wohin tendiert meine Neigung, und ähnlichen Fragen?

    Die philosophische Willensfreiheit ist meines Erachtens ein Trugschluss. Selbstverständlich werde ich dabei nicht von außen gezwungen, aber in subtiler Weise entscheide ich nicht willkürlich und nicht wirklich frei. Natürlich fühle ich mich dabei nicht bevormundet oder manipuliert. Was ist das bewusste “Ich” oder mein Bewusstsein anderes als die Gesamtheit meiner Gedanken, Erinnerungen, Erfahrungen, Wahrnehmungen, Gefühle, Empfindungen?

    All das sind molekularbiologische bzw. biochemische Prozesse, die zum Teil schon gut verstanden sind, besonders die sinnesphysiologischen Prozesse. Es ist kein Geheimnis, dass noch viele Lücken offen sind. Das Problem sind die Brücken zwischen der neurobiologischen Ebene und der ganzheitlich psychologischen Ebene. Zum Verständnis des Menschen ist der naturwissenschaftliche Reduktionismus als Naturalismus unumgänglich.

    Die wahren Reduktionisten sind die Antinaturalisten, weil sie die Komplexität des Nervensystems nicht wahrhaben wollen oder für grundsätzlich unerforschbar halten. Dafür suchen sie nach einfachen, holistischen Erklärungen, indem sie den phänomenalen Begriffen der Psychologie eigene Existenz und Erklärungscharakter zuweisen, z.B. dem Willen! Natürlich gibt es Zusammenhänge zwischen psychologisch phänomenalen Prozessen, die in der Psychotherapie aufgearbeitet werden können. Letztlich aber mündet diese Form von Reduktionismus nur in zirkulären Wortspielereien ohne wissenschaftlichen Erkenntnisgewinn.

  439. Ich möchte noch eine allerletzte Beitrag Idee zum besten geben. Ich denke, daß man abgrenzen muß zwischen Bewußtsein und Qualia. Bewußtsein ist für mich eine neuronale Funktion, die sich im Zuge der Evolution als Orientierungswerkzeug, als eigene Regulationsebene, entwickelt hat, hingegen die Qualia, also das persönliche Erleben eine sehr individuelle Erfahrung ist, die sich aus dem Zusammenspiel von genetischem Erbe und individueller Erfahrung ergibt, als innerster Kern, dem man einen säkularisierten Begriff von Seele zuschreiben könnte, der aber nicht objektivierbar ist und auch nicht objektivierbar zu sein braucht. Objektivierbar sind dann nur äußere phänomenologische Erscheinungen, die das Individuum äußert und die auch in gewisser Weise quantifizierbar sind. Die Qualia wäre dann vom Standpunkt des äußeren Betrachters quantifizierbar, vom Standpunkt des inneren Betrachters, also sich selbst, nur von diesem erlebbar.

  440. Christian Hoppe – gestern – 08:15 Uhr

    Bullshit ist erneut ein (höflich formuliert) unkultivierter Ausdruck – insb. i.V.m. Buddhismus. Das Eine od. Andere kann man ja durchaus kritisieren; s. ggfls. mein post vom 20.1. – 11:08

    Was sie hier zum Ausdruck bringen ist, m.A.n., wenig intelligente Wortklauberei (Oberbegriff Sophismus). Wenn das Bewusstsein nicht bei Bewusstsein ist. Buddhismus ist kein Nichtsein (Nihilismus). Ich bin nicht “erleuchtet“ – vermag Ihnen also nicht alles zu erklären. Finde den buddhistischen Denkansatz aber durchaus nützlich (wenn auch nicht einfach, da er ziemlich wider die Natur/Normalität geht). Auch Sie tun ja auch Nützliches – z.B. beruflich. Und darüber hinaus: Man kann ja auch aus Irrtümern lernen.

    Worauf Sie nicht eingegangen sind war meine Frage inwiefern Narkose ein Beispiel dafür ist, dass Materie Bewusstsein hervorbringt. Muss aber nicht sein.

  441. Anton Reutlinger,
    Wir sprechen von verschiedenen Ebenen aus oder sind wir uns einig ?
    Willensfreiheit ist real, wenn ich sie verwirklichen kann. Wenn ich weder Kaffee noch Tee habe, was nützt mir dann die „eingebildete Freiheit. „

    Mir ist es egal, ob ich wegen eines Kinheitserlebnisses Kaffe oder Tee bevorzuge. Bei Kamillentee muss ich sofort an Krankenhaus denken. Wir sind als Menschen geprägt, erzogen, sozialisiert.
    Und das ist gut so. Das jetzt als „unfrei „ zu bezeichnen, das ist „Outlawphilosophie“.

    Zurück zum Thema:
    „Was ist das bewusste “Ich” oder mein Bewusstsein anderes als die Gesamtheit meiner Gedanken, Erinnerungen, Erfahrungen, Wahrnehmungen, Gefühle, Empfindungen? „

    Sie schließen daraus, dass wir determiniert sind. O.K. Wenn aber jetzt das bewusste Ich zu der Erkenntnis kommt, dass wir in Grenzsituationen vor der Wahl stehen so oder so? Wenn wir dann sagen, egal, ich bin determiniert, dann wären wir Fatalisten. Tun wir aber nicht, wir quälen uns mit der Entscheidung.
    Anmerkung: Der Schuldbegriff setzt die freie Entscheidung voraus.

  442. @hwied
    Ein Determinismus des Willens ist auszuschließen, daran gibt es keine Zweifel. Es geht darum, ob wir ganz frei oder doch nicht ganz frei sind. Ich tendiere zum Letzteren, denn “wir quälen uns mit der Entscheidung”. Ich sehe die freie Entscheidung darin, dass ich mich selber “eigenwillig” (Achtung, doppeldeutig) entscheide, aber auch das ist eine Grauzone, denn ich höre auch auf meine Ehefrau (manchmal). Dadurch muss ich mich in der Willensfreiheit nicht eingeschränkt fühlen.

    Generell wägen wir vorher ab, welche Konsequenzen eine Willensentscheidung hat. Solche Entscheidungen bzw. Handlungen sind dann nicht mehr ganz frei. Bei den Juristen geht es um Abwägungen konkreter Situationen und deren Verallgemeinerung zur Bemessung von Schuld und Strafen.

    Die Diskussionen um Willensfreiheit sollte man nicht überbewerten. Man fällt leicht in einen endlosen Sophismus, gerade weil man keine naturwissenschaftlichen Grundlagen hat. Zum Verständnis des Bewusstseins tragen sie auch nicht Wesentliches bei.

  443. Zitat Stegemann: „Ich möchte noch eine allerletzte Beitrag Idee zum besten geben. Ich denke, daß man abgrenzen muß zwischen Bewußtsein und Qualia. Bewußtsein ist für mich eine neuronale Funktion, die sich im Zuge der Evolution als Orientierungswerkzeug, als eigene Regulationsebene, entwickelt hat, …“

    Ich sehe es auch so, allerdings sehr „Technik orientiert“.

    Qualia dürften die realen Empfindungen sein, die an den jeweiligen „inneren sensorischen Organen“ entstehen. Z.B. die „Blau“ Empfindung an den entsprechend sensitiven Zapfen an der Netzhaut im Auge. Im Ohr dürfte es für Schallwellen sensitive Zellen geben, die Hörempfindungen (ebenfalls Qualia) generieren, die weiters elektrische Ladungen emittieren und in das neuronale System zwecks Auswertung und Speicherung „einspeisen“. Die vielen an der Netzhaut anstehenden Bildpixel emergieren zu Bildern.

    Empfindungen könnten auftreten, wenn auf die Elektronenbahnen bestimmter sensorischer Moleküle Einfluss genommen wird und Elektronen aus der Bahn „geschleudert“ werden.

    Im Prinzip entsteht im neuronalen System ein „Datenstrom“, abgebildet auf den elektrischen Signalen. Die jeweilige Leitung transportiert im Prinzip zunächst einmal Information abbildende Signale vom Auge, Ohr, …. oder von bestimmten „Hirnorganen“ die z.B. die Information vom Auge und vom Ohr verknüpft haben. (Die Bilder bekommen letztlich eine sprachliche Bedeutung).

    Im Prinzip dürfte es immer die anliegende Information baumartig aufspaltende (und in weiterer Folge zusammenführende) Prozesse geben, so dass zunächst nicht nur die Farbinformation eines Bildpixels, sondern dass auf zusätzlichen, von den verknüpfenden Neuronen ausgehenden Axone ein Signal ausgesendet wird, wenn benachbarte Bildpixel die gleiche Farbe, oder neben einem blau z.B. ein rot abbildendes Bildpixel anliegt. Diese Muster dürften danach zusammengeführt werden, so dass aus den einzelnen Mustern z.B. eine Parkbank erkannt wird.

    Wesentlich im Zusammenhang mit Bewusstsein dürfte aber sein, dass die Information abbildenden elektrischen „Datenströme“ sozusagen auf flächigen Zwischenschichten (wie bei einer Mattscheibe) „Bewusstseinsabbildungen“ entstehen lassen.

    Könnte man sich vorstellen, so wie Programmierer früher bei technischen Computerprogrammen in die riesigen „Programmschlangen“ „Haltepunkte“ eingefügt haben, um das Bedienpersonal über den aktuellen Status der Prozesse oder über „Unregelmäßigkeiten“ zu informieren. Damit das Personal einen Überblick über die Situation bekommt um die nötigen Maßnahmen zu setzen, die das System nicht selbst (sozusagen „unterbewusst“) erledigen kann.

    Da werden Meldungen angezeigt wie: Job Lohnverrechnung beendet, keine Fehler. Im Drucker 3 geht das Papier aus. Drucker 4 hat eine Störung (Techniker rufen). Temperatur in einem Teilsystem bedrohlich hoch….

    Empfindungen werden nicht angezeigt und schon gar nicht wahrgenommen, weil es derzeit noch keine besondere technische Qualia Sensorik gibt.

  444. @anton reutlinger 22.01. 10:36

    “Was ist das bewusste “Ich” oder mein Bewusstsein anderes als die Gesamtheit meiner Gedanken, Erinnerungen, Erfahrungen, Wahrnehmungen, Gefühle, Empfindungen?”

    Das besondere am Bewusstsein ist doch seine Eigenschaft als Innenraum. Das Bewusstsein bietet doch einen internen Überblick über die aktuellen Wahrnehmungen, das fast komplette Wissen und das Wissen um die eigenen Strategien, die eigene Agenda, die man selbst verfolgt.

    Der interne Überblick ist doch die zentrale Eigenschaft von Bewusstsein. Und in diesem Sinne der Kern der Willensfreiheit. Wenn ich mit Drogen benebelt bin, funktioniert dieser Überblick nicht, und genau deswegen setzt dann auch die Willensfreiheit aus, was auch juristische Konsequenzen haben kann, mit Recht. Weil ich dann die aktuelle Sachlage und meine eigenen Entscheidungen eben nicht mehr mit meinem gesammeltem Weltwissen fundieren kann.

    Insbesondere geht es bei der juristischen, bzw. politischen Willensfreiheit nicht um höhere Philosophie, sondern in erster Linie um die Freiheit, gemäß der eigenen Interessen entscheiden zu dürfen.

    „Die wahren Reduktionisten sind die Antinaturalisten, weil sie die Komplexität des Nervensystems nicht wahrhaben wollen oder für grundsätzlich unerforschbar halten.“

    In der Tat gibt es diese Esoteriker, die selbst massiv reduzieren. Da verdrehe ich auch die Augen. Meine eigenen Versuche, Geisteswelten zu integrieren, sind entsprechend minimal-invasiv, eben weil ich davon ausgehe, dass wir dieses überaus komplexe Gehirn wohl auch brauchen, um klarzukommen und um menschliches Bewusstsein zu entwickeln.

    Was mir am meisten fehlt, das ist eine Idee, wie unser Innenraum geöffnet wird. Ich kenne mich etwas mit Informatik aus, und ich habe nicht die geringste Idee, wie eine materielle Informatik diese Innenräume realisieren könnte, in denen wir mental existieren.

    Das heißt natürlich nicht, dass das wirklich nicht geht. Aber diese Unklarheit motiviert mich dann doch, hier eine Beteiligung von kosmischem Geist für möglich zu halten, der eben den Aspekt eines informatischen Innenraumes grundsätzlich mitbringt, und eben auch unabhängig vom Menschen schon als solcher im Kosmos aktiv ist. Nicht in spezifisch menschlicher Form, diese ist sicherlich vom menschlichem Gehirn wesentlich mitbestimmt.

    Aber bei uns realisiert als Synthese des offenen kosmischen Innenraumes mit den Inhalten unseres individuellen menschlichen Gehirns.

    Ich denke grundsätzlich, dass die zukünftige Hirnforschung hier diese Frage klären kann. Wenn der Innenraum ganz alleine durch Nervenzellen geöffnet werden kann, wird man das ziemlich sicher feststellen können. Und damit wäre auch klar, welche Strukturen Bewusstsein bilden können. Das wäre vor allem für die KI-Forschung hochinteressant.

    Wenn aber meine Idee richtig ist, dann geht das eben nur mit geistiger Unterstützung. Aber immer noch nicht ohne Gehirn, welches in jedem Fall den wesentlichen Teil der Bewusstseinsinhalte mitbringt, auch wenn der „Container“ wirklich geistiger Art ist.

    Spirituelle Erfahrungen sind so integrierbar, was mir jetzt wichtig ist. Und die religiösen Traditionen erscheinen dann auch in einem anderem Licht, was mir jetzt nicht so wichtig ist. Insbesondere diese stark reduzierenden esoterischen Konzepte, die die wirkende Komplexität des Gehirns völlig unterschätzen, wären dann aber auch widerlegt.

  445. Jeckenburger,
    die Idee des Innenraumes. Und wie man diesen Innenraum öffnet. Fragen Sie einmal ihre Frau. Frauen warten den richtigen Augenblick ab, wenn sie in Stimmung sind. Wir sind ja auch physische Wesen, die man stimulieren kann, durch Parfum z.B. In dieser Sicht, ist es vorbei mit der Willensfreiheit.

    Was eine software betrifft, die Die Freiheit emuliert.
    Ich habe einmal ein Programm geschrieben für ein Autorennen.
    Damit das realistisch wird und spannend, habe ich nicht festgelegt, welches Auto gewinnt, sondern mit einer Subroutine und einem Zufallsgenerator die Geschwindigkeiten reguliert. Mit der Freiheit ist es ähnlich, die hängt auch vom Zufall ab. Eine Kaufentscheidung aufgeschoben kann bedeuten, dass ich 10 Minuten später eine andere Entscheidung treffe. Der Wille ist nicht determiniert, das stimmt, der hängt dann aber von Zufälligkeiten ab.

  446. @Jeckenburger
    Ihre Vorstellung von Bewusstsein als “interner Überblick” oder als mentaler Innenraum, in dem sich das Bewusstsein abspielt, ist akzeptabel. Bewusstsein ist etwas dynamisches, zwar ein mentaler Zustand, aber nicht statisch. Deshalb hätte ich besser nicht von Gedanken, Erinnerungen usw. geschrieben, sondern von den bewussten Prozessen des Denkens, Erinnerns usw. Natürlich hinterlassen diese Prozesse ihre Spuren in den plastischen Strukturen des Nervensystems.

    Mit kosmischem Geist und spirituellen Erfahrungen kann ich allerdings nichts anfangen. Vielleicht liegt es an unterschiedlichen Deutungen.

  447. Wenn mein Wille frei wäre, müsste ich ihn aus einem vorgeschalteten Willen heraus bestimmen. Wenn dieser vorgeschaltete Wille nicht frei ist, ist es auch der nachgeschaltete nicht. Ist er aber frei, geht es weiter: Ein dem vorgeschalteten Willen vorgeschalteter Wille müsste den vorgeschalteten Willen bestimmen, der dann den Willen bestimmt … usw.

    Wenn meine Gedanken frei wären, müsste ich einen Gedanken wählen können, noch bevor ich ihn denke.

    Leute – das macht alles keinen Sinn!

  448. @Stephan // 21.01.2021, 20:09 Uhr

    » Wenn wir uns nicht einmal darauf einigen können, dass laut Materialismus alles Materie ist, dann lassen wir es doch bleiben…«

    Hier die Materialismus-„Definition“ von Ch. Wolff (1679-1754):

    »Materialisten werden Philosophen genannt, die nur die Existenz von materiellen Dingen und Körpern zugeben.«

    Ganz offenkundig hat sich da in über 200 Jahren nicht viel dran geändert, wenn man das mit der von Dir zitierten Definition von Bem & de Jong (2006) vergleicht.

    Sinnesempfindungen können gemäß der aktuelleren Definition als Manifestationen raumzeitlicher Materie aufgefasst werden. Der von mir erwähnte Mario Bunge sprach in diesem Zusammenhang von Eigenschaften.

    @22.01.2021, 09:59 Uhr

    »Wenn man die z.B. balanesischen Aussagen genauer analysieren würde, käme man meiner Meinung nach zwigend zum Ergebnis, dass sie auf einen Eigenschaftsdualismus hinauslaufen; …«

    Ach was, meine Aussagen laufen schlicht darauf hinaus, dass materielle Dinge und Körper notwendigerweise Eigenschaften oder Systemeigenschaften haben, je nachdem. Hätten sie keine, könnten wir sie als Körper überhaupt nicht erkennen.

    » …und dann stünde man quasi wieder am Ausgangspunkt…«

    Nun ja, wenn der Ausgangspunkt die gängige Materialismus-Definition war, dann hat sich ja nichts geändert.

  449. Ch. Hoppe,
    Unser Rechtsystem beruht auf dem freien Willen, sonst gäbe es keine Schuld.
    Nur in Extremfällen gesteht man dem Angeklagten mildernde Umstände zu.

    Es gibt eigenwillige Kinder, wo sich die Eltern fragen, woher hat er das. Eines unserer Nachbarkinder hat so stark getrotzt, dass es den Kopf gegen die Wand geschlagen hat. Die Stimme war so schrill , dass man damit hätte Stahl zerschneiden können. Zum Glück ließ sich die Mutter nicht aus der Ruhe bringen und hat auch dieses Kind gebändigt.

    Zu den Gedanken. Eine gewagte Vorstellung. Die Gedanken umkreisen uns , um im günstigen Eigenblick in unserem Hirn Gehör finden. Gedanken sind elektromagnetische Wellen. Ich und meine Frau, wir haben oft im gleichen Augenblick den gleichen Gedanken. Meistens ist meine Frau aber schneller, die spricht meinen Gedanken aus, bevor ich ihn zu Ende gedacht habe.
    Merke: Denke nie gedacht zu haben, denn das Denken der Gedanken ist gedankenloses Denken. Aber denken tust du nie.

  450. @anton reutlinger 22.01. 14:17

    „Natürlich hinterlassen diese Prozesse ihre Spuren in den plastischen Strukturen des Nervensystems.“

    Das ist erstmal wichtig.

    @Christian Hoppe 22.01. 14:43

    „Wenn meine Gedanken frei wären, müsste ich einen Gedanken wählen können, noch bevor ich ihn denke.“

    Können wir uns vielleicht darauf einigen, dass es bei wichtigen Entscheidungen einen internen Auswahlprozess gibt? Der teils bewusst, teils unbewusst ist und sich meistens an den eigenen persönlichen Zielen orientiert?

  451. @Christian Hoppe

    Leute – das macht alles keinen Sinn!

    Sie sollten vielleicht besser sagen, (speziell) das ergibt keinen Sinn.

    Auf das Problem des infiniten Regresses hatte im Übrigen ja auch schon Timm Grams aufmerksam gemacht. Allerdings bisher nur in einem von der Willensfreiheit getrenntem Gedankdengang.

    Ihre Anmerkung halte ich daher an dieser Stelle für hilfreich.

  452. Aufgrund absurder Aussagen von Timm Grams zum Naturalismus möchte ich hier kurz kommentieren.

    Zunächst danke ich @Joker (17.01.2021, 22:18 Uhr) für seinen Hinweis zum wohlmeinenden Textverständnis. Dieses scheint bei Grams leider nicht gegeben zu sein. Die Situation ist sogar schlimmer. So schreibt Grams: „Vor allem soll diese Welt – die Realität – „außerhalb unseres Denkens“ angesiedelt sein (Mahner, 2018, S. 46)‟. [Hervorhebung von mir]

    Wenn man sich nun die Mühe macht, auf S. 46 des zitierten Buches nachzuschauen, so wird man dort im Rahmen der Analyse der ontologischen Voraussetzungen von realwissenschaftlichen Experimenten und deren Unterscheidung von Gedankenexperimenten folgenden Satz finden: „Wir nehmen also (in aller Regel ganz stillschweigend) die Existenz einer Welt außerhalb unseres Denkens an.‟ Man beachte den unbestimmten Artikel „einer‟. Hier steht nichts davon, dass DIE (ganze) Welt außerhalb unseres Denkens existiert, etwa in dem Sinne, dass unser Denken nicht zur realen Welt gehörte. Genau so dreht aber Grams die Behauptung durch verkürztes Zitieren hin.

    Welchem Zweck dient also das verkürzte Zitieren? Ist es lediglich das Resultat mangelnden Textverständnisses oder wird absichtlich so verkürzt zitiert, weil nur so überhaupt ein Angriffspunkt im Sinne eines Strohmanns zu bekommen ist, den man dann „zerpflücken‟ kann?

    Jedenfalls sind weder Gerhard Vollmer noch ich so blöd zu behaupten, unser Denken gehöre nicht zur Welt. Die Formulierung „außerhalb unseres Denkens‟ dient üblicherweise lediglich der Abgrenzung von solipsistischen und idealistischen Auffassungen. Als ontologische Naturalisten sagen wir vielmehr, dass Denken und Bewusstsein keine immateriellen, eigenständig existierenden Objekte sind, sondern (emergente) Eigenschaften neuronaler Systeme, die dann im Sinne eines Innenaspekts auftreten, wenn diese Systeme bestimmte hochspezifische Prozesse durchmachen. Herauszufinden welche das genau sind, ist eine Aufgabe für die Neurobiologie.

  453. @Chrys // 22.01.2021, 00:15 Uhr

    Christian Hoppe behauptet nicht, dass bestimmte Naturwissenschaften eine „vollständige“ Erklärung beanspruchen, sondern er sagt, dass bestimmte physische Vorgänge in der Physik bereits ihre vollständige prinzipielle Erklärung gefunden haben. Und, dass „hinsichtlich materieller/physischer Vorgänge [] kein unerklärlicher Rest [bleibt], der dann z.B. psychologisch erklärt werden könnte“.

    Erkennst Du den kleinen, aber wesentlichen Unterschied?

    »Keil scheint mir dem Zitat zufolge diejenigen Naturalisten, die mit Totalisierung der Natur argumentieren, für besonders einfältig zu halten…«

    Da liest Du wohl zu viel hinein. Wegen der fehlenden „kritische[n] Pointe“ (Keil) scheint die Sache für ihn relativ uninteressant zu sein. Eine Philosophie, die sich auf die „Natur“, also auf „alles was ist“, stützt, was soll man damit anfangen? Es fehlt eben das, was nicht Natur ist. In Keils Worten:

    Der metaphysische Naturalismus muß irgendwelche Merkmale angeben können, die eine Entität als Naturgegenstand disqualifizieren, denn auch für ihn gilt: Omnis determinatio est negatio.

    Und das fehlt offenbar.

    Der Materialismus behauptet wenigstens, es gäbe keine nichtmateriellen Körper und Dinge, was in bestimmten Kreisen Anlass gibt zu heftigem Widerspruch.

  454. Martin Mahner spricht in seinem sehr persönlich gehaltenen Plädoyer Punkte an, die im Laufe dieser ziemlich ausgiebigen Diskussion bereits abgearbeitet worden sind. Ich werde mich nicht wiederholen. Neuem widme ich mich gern.

  455. @Mahner: Emergenz & Naturalismus

    Schön, dass Sie hier mitdiskutieren.

    Emergenz wird aber doch (stärker noch als Supervenienz) in aller Regel so verstanden, dass das emergente Phänomen, die emergente Eigenschaft oder der emergente Prozess nicht reduktiv erklärbar ist. Wie kann man da noch einen Naturalismus vertreten?

  456. @Stephan Schleim

    Ich habe leider keine Zeit zum Mitdiskutieren, sorry. Daher nur schnell zu Ihrer Frage. Der Einfachheit halber zitiere ich aus einem früheren Opus (Bunge & Mahner 2004, Über die Natur der Dinge):

    „DEFINITION 2.6. Es stehe P für eine Eigenschaft eines Dings b. Dann ist P eine emergente Eigenschaft von b genau dann, wenn entweder
    (i) b ein komplexes Ding (ein System) ist, dessen Komponenten P nicht besitzen, oder
    (ii) b ein Ding ist, welches P dadurch erworben hat, dass es Teil eines (anderen) Systems geworden ist (d.h., b besäße P nicht, wenn es ein unabhängiges oder isoliertes Ding wäre).‟ (S. 79)
    ……
    Der aufmerksame Leser wird bemerkt haben, dass wir hier der verbreiteten Definition einer emergenten Eigenschaft als einer, die nicht aus der Kenntnis der Eigenschaften der Teile vorhergesagt bzw. erklärt werden kann, nicht gefolgt sind. Der Grund ist einfach: Emergenz hat etwas mit der realen Welt zu tun, nicht mit unserem Wissen von ihr: Der Emergenzbegriff ist ein ontologischer, kein erkenntnistheoretischer. Für die Neuheit einer Eigenschaft eines Systems kann es keine Rolle spielen, ob wir sie voraussagen oder erklären können oder nicht: Qualitative Neuheit bleibt – wenn es sie gibt – ontisch qualitative Neuheit, ob sie erkannt wird oder nicht. Mit anderen Worten: Erklärte Neuheit ist nicht weniger neu als unerklärte, und vorausgesagte Neuheit ist ontisch genauso neu wie nicht vorhergesagte oder gar unvorhersagbare Neuheit. Eine ontologische Kategorie mithilfe erkenntnistheoretischer Begriffe zu definieren, ist ein Kategorienfehler. Die populäre, wiewohl inadäquate erkenntnistheoretische Definition von “Emergenz” dürfte auch der Grund sein, weshalb viele Philosophen den Emergenzbegriff bis heute für mysteriös oder irrational halten und ihm daher mit Misstrauen gegenüberstehen. Tatsächlich lassen sich viele emergente Eigenschaften durch Kenntnis der Zusammensetzung und Struktur von Systemen erklären, wenn auch vielleicht nicht immer voraussagen (Vollmer 1995, Kap. 4). Ob dies für alle emergenten Eigenschaften gilt, wissen wir nicht, ist aber für die ontologische Frage ohnehin irrelevant.‟ (S. 82f).

    Ich denke, das beantwortet die Frage nach dem Naturalismus: Hier ist nichts Nichtnatürliches…

  457. @Elektroniker // 21.01.2021, 20:12 Uhr

    » Vielleicht könnten Sie, auch möglichst sachlich begründen, warum dieses Konzept, z.B. aus der Sicht von Neurologen falsch ist. «

    Was ich in meinem Kommentar angesprochen habe, nämlich warum uns eine bestimmte Wellenlänge des Lichts rot und nicht blau erscheint, zielt auf die Zuordnung einer Farbqualität zu einem bestimmten Nervenreiz. Hätte der durch 700 nm Wellenlänge angeregte Rezeptor einen blauen Farbeindruck zur Folge, dann würde dieser Rezeptor, den wir aktuell Rotrezeptor nennen, dann eben ein Blaurezeptor sein—obwohl sein Empfindlichkeitsspektrum unverändert ist.

    Im Laufe der Evolution haben es Organismen irgendwie geschafft, die unterschiedlichen Wellenlängen des sichtbaren Lichts zu unterscheiden, indem, zumindest bei höheren Wirbeltieren, bestimmte Farbqualitäten wahrgenommen werden. Warum bei 700 nm ausgerechnet ein roter Farbeindruck entsteht und kein blauer, ist vermutlich nicht naturgesetzlich zu erklären.

  458. @Balanus

    Warum bei 700 nm ausgerechnet ein roter Farbeindruck entsteht und kein blauer, ist vermutlich nicht naturgesetzlich zu erklären.

    Wir einigen uns, wenn uns 700 nm-Wellen in die Augen fallen, den Farbeindruck als rot zu bezeichnen. Ob die Eindrücke von Einem zum Anderen gleich sind, haben wir meines Wissens nach noch gar nicht klären können. Einiges spricht dagegen, z.B. das Phänomen Synästhesie. Vielleicht entspricht Dein rot-Eindruck eher meinem blau-Eindruck.

    Näheres wird in meinem Aufsatz zu finden sein, “What Is It Like to Be a Joker?” (im Entstehen).

  459. @anton reutlinger 22.01. 14:17

    „Mit kosmischem Geist und spirituellen Erfahrungen kann ich allerdings nichts anfangen. Vielleicht liegt es an unterschiedlichen Deutungen.“

    Da habe ich kein Problem mit. Es hat nicht jeder spirituelle Erfahrungen, zumal diese meistens eher sporadisch auftauchen. Ich denke, dass liegt einfach daran, dass wir diese im Alltag meistens nicht brauchen. Aber wenn man welche hat, will man sie auch gerne in sein Weltbild integrieren. Das geht mir jedenfalls so.

    Dass ich vermute, dass Geisteswelten beim Bewusstsein eine regelmäßige Rolle spielen, liegt daran, dass ich mir nicht vorstellen kann, wie das die Nervenzellen alleine hinbekommen sollen. Hier wäre eben die theoretische Lage so, dass der kosmische Geist sich schon deswegen beteiligen muss, weil es eben anders nicht geht, und das Projekt Bewusstes Leben nicht anders zu realisieren ist.

    Aber das ist nur eine Vermutung, die wirklich niemand teilen muss. Ich weiß es ja selber nicht, sondern habe da nur einen Verdacht. Aber vielleicht erlebe ich das ja noch, dass die Hirnforscher mit einer funktionierenden Simulation eines Mäusekonnektoms fertig werden. Das könnte dann Antworten liefern, wie das alles funktioniert. Ob nun mit oder ohne geistige Unterstützung.

    Was Religion angeht, so denke ich, dass das zunächst mal was anderes ist, als originale spirituelle Erfahrungen. Und wenn man sich einer Religion zuwenden will, muss man auch erstmal entscheiden, welche überhaupt. Ich habe insofern vielleicht Glück gehabt, dass ich meine Kindheit und Jugend religionslos erleben durfte. Jedenfalls habe ich bis heute kein wirkliches Bedürfnis nach einer Glaubensgemeinschaft.

  460. @Joker // 23.01.2021, 00:43 Uhr

    »Ob die Eindrücke von Einem zum Anderen gleich sind, haben wir meines Wissens nach noch gar nicht klären können.«

    Darauf kommt es m. E. nicht. Das Entscheidende ist, dass überhaupt Unterscheidungen getroffen werden können (und vielleicht auch, dass eine elektromagnetische Welle etwas fundamental anderes ist als der von ihr bedingte Sinneseindruck).

    Das Leben ist voller Wunder…

    Gut’s Nächtle ; )

  461. @Mahner
    Im Grunde versuchen wir alle der Ambivalenz zu ‘entfliehen’.
    Dieses Phänomen werden wir vermutlich auch dann nicht lösen,wenn wir für genau einen Zustand oder eine Eigenschaft ein Wort/Begriff haben (Gödel Unvollständigkeit).
    Worüber sollen wir uns denn sonst ‘streiten’ und Versöhnen.

  462. @Jeckenburger
    Dass ich vermute, dass Geisteswelten beim Bewusstsein eine regelmäßige Rolle spielen, liegt daran, dass ich mir nicht vorstellen kann, wie das die Nervenzellen alleine hinbekommen sollen.

    Auch ich kann mir viele Dinge nicht vorstellen, die bisher noch Rätsel sind. Aber deswegen glaube ich noch lange nicht an Geister, Gespenster oder Götter. Erstens brauche ich sie nicht und zweitens gibt es keinerlei Hinweise dafür, noch viel weniger als für die fehlenden naturwissenschaftlichen Erkenntnisse.

    Die Naturwissenschaft gewinnt laufend neue Erkenntnisse hinzu. Beim Geisterglauben wachsen nur die Phantasien, die immer abstruser werden. Viele Geistergläubige verharren meilenweit hinter dem Stand der Naturwissenschaft, weil der Geisterglaube viel bequemer, verständlicher und unterhaltsamer ist. Die Menschen brauchen Unterhaltung, deshalb blüht die Unterhaltungsindustrie. Je abwegiger und unwahrscheinlicher die Inhalte sind, desto höher ist der Nachrichtenwert, desto größer ist das Interesse dafür und die Befriedigung.

    Auch den Götterglauben sollte man unter diesem Aspekt sehen, obwohl der Klerus alles tut, den Unterhaltungswert zu minimieren. Man denke an die Götter der Antike.

  463. Balanus,
    Sinneseindruck und elektromagnetische Welle. Die beiden Begriffe lassen sich verbinden mit dem physikalischen Begriff der Wechselwirkung.
    Im Grunde haben wir schon alles zusammen, was das Denken erklärt, das Bewusstsein, das Ich und die Außenwelt.
    Es fehlt nur noch der Zusammenhang und dafür hat die Naturwissenschaft eine gute Vorarbeit geleistet mit den Begriffen Energie und Wechselwirkung.

    Wie wechselwirkt ein reales Objekt mit einem anderen realen Objekt. Über elektromagnetische Strahlung. Das wärmere Objekt gibt Wärmestrahlung an das kältere Objekt ab.

    Der Mensch wechselwirkt mit einem anderen Menschen auch über elektromagnetische Strahlung. Aber nicht nur. Er wechselwirkt auch über Schallwellen mit dem anderen Menschen, das nennt man Sprache, und wenn es hauptsächlich um Gefühle geht, dann benützt er die Musik.

    Unsere Sinnesorgane nehmen die Schallwellen auf, z.B. das Ohr und leitet sie in einen Konverter, der Teil des Gehirnes ist um in Ionen. Die Ionen wandern durch die Nervenleitung in eine Gehirnregion, die Nervenreize verarbeitet. Diese Verarbeitung ist eine Form des Denkens. Schließlich erreicht der Nervenreiz unser „Ich“ , das eine Einheit bildet, so wie der Bordcomputer im Auto.
    Und jetzt verstehen wir auch warum ein “Wort” eine physikalische Wirkung zeigt, weil es einen Reiz verursacht.
    Das Wort hat wie die Musik eine Wirkung. Bei dem Wort Gewinn ist die Wirkung positiv, beimWort Verlust ist sie negativ. Man könnte auch sagen, Verlust ist disharmonisch.

    Unser aufgeschrecktes Ich nennen wir dann Bewusstsein. Denkansatz 7

  464. Apropos Kategorienfehler: Ich habe noch keine Ontologie gesehen, die nicht von Erkenntnis abgeleitet worden ist. Im Naturalismus beispielsweise passt sich das (ontologische) Postulat der Kausalität ständig neuen Erkenntnissen an: erst klassische Mechanik, dann Quantenmechanik, und dann was auch immer.

    Das kann meines Erachtens gar nicht anders sein, da wir keinen erfahrungsunabhängigen Zugang zum Reich der “Dinge an sich” haben.

    Da hier vom Kategorienfehler bezüglich Ontologie und Erkenntnis die Rede ist, muss ich mich irren. Wer hilft?

  465. Timm Grams,
    Ontologie beschäftigt sich mit allem was es gibt. (Wikipedia) Eine ontologische Kategorie ist nichts anderes als eine Einteilung der Objekte durch Oberbegriffe, Guppen, Klassen usw,.so wie man es bei Tieren und Pflanzen macht.
    Aber was gibt es alles? Sind abstrakte Objekte, gedachte Objekte auch etwas? Man denke an die Begriffe in der Physik, z.B. Energie, Kraft, man denke an die Begriffe in der Mathematik. Gibt es Zahlen ? Gibt es Bewusstsein ?
    Und hier tut sich schon ein Abgrund auf.

    Diesen Abgrund überbrückt die analytische Erkenntnistheorie, indem sie die gleichen Methoden anwendet, wie die Ontologie, indem sie Wissen als wahre begründete Meinung definiert.
    Obwohl Erkenntnistheorie ursprünglich einer anderen Kategorie angehört als die Erkenntnistheorie kommt sie dennoch zu dem gleichen Ergebnis= Wissen wie die Ontologie und der Unterschied ist aufgehoben.

  466. @hwied

    Und hier tut sich schon ein Abgrund auf. […]

    Obwohl Erkenntnistheorie ursprünglich einer anderen Kategorie angehört als die Erkenntnistheorie […]

    der Unterschied ist aufgehoben

    Sehe ich ganuso.

  467. @ Balanus 22.01.2021, 22:09 Uhr

    Danke für Ihre Antwort.

    Ich meine, es ist durchaus schwer (oder unmöglich) zu verstehen, warum überhaupt „Empfindungen“ entstehen. Klar dürfte sein dass sie entstehen. Vermutlich (und eigentlich offensichtlich), entstehen sie in für Empfindungen sensitiven Molekülverbänden.

    Die Zuordnung, dass bestimmten „Farbqualitäten“ zu einem ganz bestimmten dynamischen Nervenreiz gehören, eine bestimmte „Wellenlänge“ einen z.B. „Blau“ genannten Farbeindruck auslöst und eine andere Wellenlänge „Rot“ auslöst, ist z.B. besonders anschaulich für Radiotechniker erklärbar.

    Es kommt einfach auf die für eine bestimmte „Inputfrequenz“ erforderliche „Selektivität“ des „Umsetzungselementes“ (z.B. Filter) an. Es wurden z.B. „Schwingkreise“ gebaut die (grob gesagt) für die Frequenz von Radio Luxenburg, oder auch den Freiheitssender 904 selektiv „abgestimmt“ werden konnten.

    Die Evolution hat Lebewesen eben „molekulare Filter“ eingebaut, die nicht nur vom Input abhängig Ladungsträger emittieren, sondern auch das „Empfindungsphänomen“ generieren.

    Die klassische Physik geht eher davon aus, dass sich gleiche Strukturen in gleicher Umgebung auch gleich verhalten.

    Natürlich kann man nicht ausschließen, allerdings würde ich es eher für wenig wahrscheinlich halten, dass die Frequenz – Empfindungszuordnung etwas (leicht) individuell ist.

    Also jemand lernt dass man die Farbe eines blauen Bildes „Blau“ nennt. Aber in der Verarbeitungskette der Signale beim Empfänger könnten so etwas wie „Überlagerungen von Frequenzen“ auftreten (Stichwort: „Überlagerungsempfänger, Nebenempfangsstellen“), wobei übertrieben (anschaulich aber nicht korrekt) formuliert, blau als rot erscheint und der Empfänger sieht zwar „rot“, lernt aber dass es „blau“ ist.

    Bastarde die mitunter, (kurz gesagt) 2 „verschiedene Augen“ haben, nehmen tatsächlich die Farbeindrücke auf dem jeweiligen Auge etwas unterschiedlich war.

  468. @hwied 23.01.2021, 11:25 Uhr

    “Obwohl Erkenntnistheorie ursprünglich einer anderen Kategorie angehört als die Erkenntnistheorie [Ontologie?] kommt sie dennoch zu dem gleichen Ergebnis= Wissen wie die Ontologie und der Unterschied ist aufgehoben.”

    Danke. Ist mein Korrekturvorschlag korrekt?

  469. Ontologie, Erkenntnis
    Kant hat es deutlich zum Ausdruck gebracht:

    unsere Vorstellung der Dinge, wie sie uns gegeben werden, richte sich nicht nach diesen als Dingen an sich selbst, sondern diese Gegenstände vielmehr als Erscheinungen richten sich nach unserer Vorstellungsart.

    Das Nervensystem konstruiert eine Vorstellung, in der die Welt repräsentiert wird. Die Evolution hat dafür gesorgt, dass die Repräsentation geeignet ist, mit der Welt erfolgreich zu interagieren, um das Überleben zu sichern. Mehr ist eigentlich nicht notwendig. Das menschliche Bewusstsein ist ein Luxus, der uns offensichtlich dazu befähigt, die Realität zu ignorieren oder zu manipulieren und damit unsere Lebenswelt zu vernichten. Dann müssen wir uns über Kategorienfehler keine Gedanken mehr machen.

    Die Differenz zwischen Ontologie und Erkenntnis zeigt sich ganz praktisch in dieser Viruspandemie. Viele Leute haben die Realität der Gefahr noch immer nicht begriffen, obwohl sie mitten darin sind und täglich mit Informationen über die Realität überschwemmt werden! Ist die objektive Realität beliebig geworden und nur noch für Historiker von Bedeutung? Was ist los mit dem Bewusstsein, wo ist es geblieben?

  470. Timm Grams,
    Danke für den Korrekturvorschlag.
    Es sollte heißen “Obwohl die Erkenntnistheorie einer anderen Kategorie angehört als die Ontologie, kommt sie dennoch zu dem gleichen Ergebnis wie die Ontologie und der Unterschied ist aufgehoben.
    Nachtrag:
    Der Unterschied ist tatsächlich aufgehoben, weil die moderne Physik z.B. die Quantenmechanik gar nicht mehr durch ontologische Begriffe erklärt werden kann. Die Verschränkung des Lichtes z.B ist nur noch mathematisch erklärbar.

  471. Anton Reutlinger,
    das öffentliche Bewusstsein hinkt dem Wissensstand hinterher.
    Deswegen sind naturwissenschaftliche Zeitschriften wichtig.
    Deswegen ist dieser blog wichtig, weil man dabei gezwungen wird über Zusammenhänge wie Denken und Wirklichkeit nachzudenken.

    Viele Leute haben die Gefahr noch nicht begriffen…..Zustimmung.
    Heute Morgen ist mir nur eine Frau mit Mundschutz begegnet.
    Man sollte im Rundfunk neben den Verkehrsmeldungen jedesmal eine Warnung aussprechen :”Corona ist überall, leg deinen Mundschutz an!”

  472. Timm Grams, Mussi
    Jetzt werden auch Sie für den alternativen Oskar vorgeschlagen.
    Die Naturwissenschaft wollte ja die gesamte Wissenschaft dominieren.
    Jetzt drehen wir den Spieß um und erklären die Naturwissenschaft als Teildisziplin des Naturalismus. (Kritik daran ist ausdrücklich erwünscht)

  473. @Balanus / 22.01.2021, 19:49 Uhr

    »Erkennst Du den kleinen, aber wesentlichen Unterschied?«

    Nein. Mir ist keine menschliche Handlung vorstellbar, welche nicht auch die Form der Referenz als physischer Vorgang erlauben würde. Und falls für diesen Vorgang dann eine vollständige Erklärung durch die Physik gegeben wäre, was für Christian Hoppe zumindest prinzipiell immer erfüllt ist, dann bliebe da nichts weiter zu erklären, sondern höchstens noch etwas zu übsersetzen, von der Physik beispielsweise in die Psychologie, wo der in Rede stehende physische Vorgang dann eben nur als menschliche Handlung interpretiert wird.

    »Es fehlt eben das, was nicht Natur ist. In Keils Worten:

    Der metaphysische Naturalismus muß irgendwelche Merkmale angeben können, die eine Entität als Naturgegenstand disqualifizieren, denn auch für ihn gilt: Omnis determinatio est negatio.

    Und das fehlt offenbar.«

    Es ist nicht sonderlich schwierig, Entitäten zu benennen, die in der Natur nicht vorfindlich sind. Primzahlen etwa. Oder auch allgemeiner: Natürliche Zahlen sind keine Naturgegenstände und in diesem Sinne eben nicht natürlich.

    Wenn Keil erwartet, dass metphysische Naturalisten in Schwierigkeiten kommen mit der semantischen Begriffsbestimmung von “Natur” oder “natürlich”, nun, das ist ganz gewiss so. Diese Schwierigkeiten haben wir aber alle, wenn wir diese Begriffe verwenden. Dennoch erachten wir sie gemeinhin als sinnvoll und wollen sie nicht aus unserem Vokabular verbannen. Wenn das alles ist, was Keil gegen metaphysischen Naturalismus einzuwenden hat, dann wird er schwerlich jemanden damit beeindrucken können, und ganz bestimmt keinen metaphysischen Naturalisten.

    Das Auftreten von Antinomien bei der Bildung totaler Gesamtheiten ist ein syntaktisches und kein semantisches Problem. Es betrifft jede Form von ontischem Monismus, sei es metaphysischer Naturalismus, Materialismus, Physikalismus, Szientismus etc. Die sind alle nach dem gleichen Denkmuster gestrickt und zeigen alle die gleiche logische Inkonsistenz. Kann schon sein, dass Keil da etwas verpasst hat, doch Gabriel hat es immerhin erkannt, das muss man ihm lassen.

    Im übrigen hat der metaphysische Naturalismus durchaus eine Pointe: Wenn nur Natürliches existiert, dann ist alles Übernatürliche kategorisch aus der Welt ausgeschlossen, und das dürfte vielen als weltanschauliche Leitidee doch ausgesprochen attraktiv erscheinen.

  474. @Timm Grams
    Naturalismus=Naturwissenschaft.

    Die Ironie ist, dass die Naturwissenschaft zunehmend immaterieller wird und die Psychologie sowie die Philosophie als Wissenschaft des Geistes zunehmend materieller wird.

    Die Physiker arbeiten inzwischen mit Laserblitzen im Bereich von Femtosekunden, also billiardstel Sekunden. Damit werden sogar die Photonen der Vakuumfluktuationen detektierbar. Diese Auflösung ermöglicht auch für die Chemie und die Neurobiologie neue Anwendungen. Die praktische Seite ist, dass damit optische Quantencomputer realisierbar werden, die um Vielfache schneller sind als die “elektrisch betriebenen” Computer. Damit wiederum können komplexere und schnellere Prozesse der Natur simuliert werden.

  475. @Mahner: Emergenz & Naturalismus

    Ja, ich verstehe, dass nicht jeder hier mitdiskutieren kann oder will; danke, dass Sie sich doch einen Moment genommen haben.

    Ich stimme völlig mit Ihnen überein, dass die Vermischung von ontologischen und epistemischen Aussagen in der Leibe-Seele- und ähnlichen Diskussionen ein Ärgernis ist, das zu großer Verwirrung führt.

    Ich denke, das beantwortet die Frage nach dem Naturalismus: Hier ist nichts Nichtnatürliches…

    Gut. Ihr Standpunkt ist klar. Auf dem Papier können Sie das natürlich so definieren. Wie weit uns diese Definitionen in der Praxis helfen (und damit: inwieweit der Naturalismus zutrifft), ist aber eine offene Frage. Reduktionen sind in der Wissenschaftsgeschichte viel seltener, als Reduktionisten es gerne darstellen, sogar in der Physik; man weiß ja noch nicht einmal mit Sicherheit, warum Eis überhaupt glatt ist.

    P.S. Mein “Doktorvater” promovierte meines Wissens übrigens bei Vollmehr; aber wir gingen dann besser getrennte Wege.

  476. @anton reutlinger 23.01. 10:16

    „Beim Geisterglauben wachsen nur die Phantasien, die immer abstruser werden. Viele Geistergläubige verharren meilenweit hinter dem Stand der Naturwissenschaft, weil der Geisterglaube viel bequemer, verständlicher und unterhaltsamer ist.“

    Nicht jede als spirituell eingestufte Erfahrung ist auch eine. Hier können sich auch Fantasien, Einbildungen und Halluzinationen untermischen. Angesichts dessen glaube ich zwar an eine Art kosmischen Geist und an eine geistige Beteiligung am menschlichen Bewusstsein, aber nicht an eine wie auch immer geartete Bevölkerung der Geisteswelten mit menschenartigen Geistern, Göttern und Dämonen.

    Angesichts des überaus komplexen Gehirns, dass immer noch die Grundlage jeglichen Seelenlebens ist, gehe ich davon aus, das ohne ein funktionierendes Gehirn keine Geister herumgeistern können. Das wäre unsinnig, wenn die Lebenden ein Gehirn bräuchten, dass ganze aber auch rein immateriell funktionieren kann. Entsprechend halte ich auch nichts vom Leben nach Tod.

    Eine Bevölkerung der Geisteswelten mit sich tummelnden Gespenstern wäre auch keinesfalls eine Vereinfachung, sondern eine Komplikation, würde ich sagen. Das hilft wirklich nichts.

    Dagegen wäre eine mögliche Beteiligung von Geist beim Bewusstsein tatsächlich eine Vereinfachung, weil dann das Gehirn nicht alles alleine verwirklichen muss. Wenn wir mal unser Konnektom entschlüsselt haben, und die neuronale Basis unseres Bewusstsein kennen, dann könnten wir auch eine entsprechende KI bauen, die die gleichen Funktionen implementiert, aber eben im Betrieb auch noch mit geistiger Hilfe richtig lebendig werden kann.

    Entsprechend würde auch für eine solche KI gelten können, dass Bewusstseinsfunktionen durch einen gewissen geistigen Service in diesem Kosmos unterstützt werden.

  477. T. Jeckenburger,
    Deutschland macht mit seinen 80 Mio. Einwohnern gerade mal 1% der Weltbevölkerung aus. Europa mit seinen 500 Mio. Einwohner hat einen Anteil von 16 %.
    Vergessen wir die Animisten nicht, die in Afrika, Asien , Australien und Südamerika leben. Da ist Geisterglauben real.
    Geisterglauben kann man nur mit Schulen eindämmen. Warten wir ab, welche Defizite sich bei unseren Schülern einstellen, dem homeschooling geschuldet.
    Unser Enkel weiß schon gar nicht mehr wie die Schule von innen aussieht.
    Und schauen wir uns die Medien an, da wimmelt es nur von Untoten, Leuten die Geister sehen und Menschen mit Superkräften.
    Da sage ich nur Gute Nacht und kein Bette.

  478. @hwied
    Geisterglauben kann man nur mit Schulen eindämmen.

    Götterglaube ist doch nichts anderes als Geisterglaube. Warum wird er in der Schule nicht eingedämmt? Dafür sorgt allerdings der Klerus selber. Die Gottesgläubigen werden weniger, andererseits, das muss man zugeben, die Geister werden bunter und die Gläubigen werden fanatischer. Die Naturwissenschaft wird daran wohl nichts ändern, die Psychologie vermutlich auch nicht. Es ist vielleicht die Ohnmacht gegenüber dem Schicksal.

  479. Joker 23.01.2021, 00:43 Uhr:
    “What Is It Like to Be a Joker?” YMMD 🙂

    Timm Grams, 23.01.2021, 12:44 Uhr:
    “Naturalismus = Naturwissenschaft.” Das ist falsch. Naturalismus ist eine Voraussetzung von Naturwissenschaft. Naturalismus postuliert z.B. die Abwesenheit von Gespenstern und anderen übernatürlichen Entitäten im Universum, die naturgesetzinkompatible Ereignisse auslösen könnten. Naturalismus verstehe ich so: “Im Universum gibt es ausschließlich naturgesetzliche Ereignisse.” Das ist ein enger Naturalismusbegriff. Es gibt auch erweiterte Natualismen. Leider ist es für mich oft schwer herauszubekommen, welchen Aspekt Timm Grams jeweils genau kritisiert.

    anton reutlinger, 23.01.2021, 13:21 Uhr:
    Eine “immaterielle Naturwissenschaft” müsste eine Wissenschaft sein, die ohne den physikalischen Energiebegriff auskommt. Davon kann ich nichts sehen. Am LHC werden hochenergetische Protonen aufeinander geschossen. Photonen (Lichtquanten) haben auch Energie. Im modernen Materiebegriff wird alles als materiell bezeichnet, das die Eigenschaft Energie aufweist.

  480. Anton Reutlinger,
    “Dafür sorgt allerdings der Klerus selber. ”
    Das ist nicht so. Ich kenne sehr viele Geistliche, bis zum Bischof von Würzburg.
    Das, was diese Leute auszeichnet ist Intelligenz und Gottvertrauen.
    Gottvertrauen hat überhaupt nichts mit Geistern und Göttern zu tun.

    Gottvertrauen ist auch nicht übersinnlich, das ist ein Lebensgefühl .
    Lesen Sie doch einmal ein theologisches Werk.
    Der ehemalige Bischof von Würzburg hat ein Buch geschrieben “Ich sehe den Himmel offen”. Darin ist auch ein Foto von mir über eine Kirche in Würzburg.
    Urteilen Sie nicht voreilig, der Mann hat vernünftige Ansichten über das Leben, ich bin überzeugt, Sie können dem auch zustimmen.

  481. @Feodor 23.01.2021, 17:06 Uhr

    Das hatten wir doch alles schon im Hoppla!-Blog. Ich schlage vor, das hier nicht noch einmal auszurollen. Die immer wiederkehrenden Argumentationmuster habe ich im Artikel Täuschende Argumente besprochen.

  482. @Feodor: Naturgesetze

    Im Universum gibt es ausschließlich naturgesetzliche Ereignisse.

    Aber die Naturgesetze werden doch vom Menschen formuliert: Und wenn der etwas entdeckt, das er vorher für (naturgesetzlich) unmöglich hielt, dann passt er das Gesetz an oder formuliert ein neues. So sehen wir es doch immer und immer wieder in der Wissenschaftsgeschichte.

  483. @Reutlinger
    Hut ab,sehr gut erkannt!
    Ohnmacht als ‘verschwiegene’ Rolle und umredetes Thema.

  484. Stephan Schleim 23.01.2021, 18:32 Uhr:
    “Aber die Naturgesetze werden doch vom Menschen formuliert: Und wenn der etwas entdeckt, das er vorher für (naturgesetzlich) unmöglich hielt, dann passt er das Gesetz an oder formuliert ein neues. So sehen wir es doch immer und immer wieder in der Wissenschaftsgeschichte.”

    Das ist ein üblicher Einwand gegen die Voraussetzung der Naturgesetzlichkeit, der nahe zu legen scheint, dass sich beliebige Ereignisse unter den Begriff der Naturgesetzlichkeit bringen ließen. Und natürlich muss zwischen den Mustern im Verhalten der Dinge und den falliblen Naturgesetzaussagen unterschieden werden.

    Ich zitiere mich selbst (feodor.de/node/25):

    “Naturgesetzlichkeit lässt sich auch nicht einfach dadurch mit beliebigen Vorkommnissen in Einklang bringen, dass es möglich ist, jede beliebige Ereignisfolge im Nachhinein irgendwie formal zu beschreiben. Es kommt vielmehr darauf an, dass sich aus Naturgesetzen mögliche und unmögliche zukünftige Ereignisse ableiten lassen. Wenn letzteres nicht (mehr) gelingt, scheitert mit dem Konzept der Naturgesetzlichkeit auch die Wissenschaft, die dieses Konzept voraussetzt.”

    Die Voraussetzung der Naturgesetzlichkeit kann prinzipiell scheitern, wenn starke Phänomene auftauchen, die sich naturgesetzlich nicht fassen lassen. Oben schrieb ich bereits: “die Fähigkeit sich [plötzlich] einen bewaffneten Helikopter zu wünschen, wenn man einen braucht, oder [plötzlich in allen Laboren] im Cha-Cha-Rhythmus zerfallende Radium-Atome würden den Naturalismus erschüttern. Der Supranaturalismus als Gegensatz des Naturalismus ist hingegen mit beliebigen Vorkommnissen in der Welt kompatibel und daher völlig kritikimmun. Er erklärt alles und damit nichts. Kritisierbarkeit ist eine erkenntnistheoretisch wünschenswerte Eigenschaft, da kein Letztbegründungsverfahren bekannt ist.”

    Aus der Astrophysik sind Belege dafür bekannt, dass physikalische Naturgesetze seit Milliarden Jahren konstant sind (da uns die Stahlung zeitverzögert erreicht). Weitere Einsichten liefert das Korrespondenzprinzip der Theorienhierarchie: de.wikipedia.org/wiki/Korrespondenzprinzip#Das_Korrespondenzprinzip_als_Konzept_zur_Theorienhierarchie

  485. @ Stephan Schleim 23.01.2021, 18:32 Uhr

    Zitat: „Aber die Naturgesetze werden doch vom Menschen formuliert: Und wenn der etwas entdeckt, das er vorher für (naturgesetzlich) unmöglich hielt, dann passt er das Gesetz an oder formuliert ein neues. So sehen wir es doch immer und immer wieder in der Wissenschaftsgeschichte.“

    Ich meine schon, dass es die „Naturgesetze“ wirklich gibt, wir kennen sie halt nicht vollständig.

    Naturgesetze sind so etwas „wie implizites Wissen“ dass latent existiert, aber nur zum Teil nachvollziehbar und „explizit gespeichert“ ist. Wie viel wir eigentlich nicht wissen, sehen wir jetzt in Pandemiezeiten.

    Auch ist es eine Frage, wie, sozusagen in welcher „Sprache“, man „Naturgesetze“ bestmöglich formuliert.

    Könnte mir eine Art „Kunstsprache“, ähnlich wie Programmiersprachen, die viele mathematische Begriffe enthalten und die für KI Anwendungen geeignet sein sollte, vorstellen.

    Was wir (noch) nicht wissen, gilt eben als „transzendent“, wie in der Mathematik.

  486. @Feodor
    Es hat sich schon bis zu mir herumgesprochen, dass Materie und Energie äquivalent sind, aber nicht identisch. Deshalb ist es nach wie vor berechtigt, Materie und Energie zu unterscheiden. Zunehmend nimmt die Energie eine stärkere Rolle ein in der Physik, man sieht es deutlich an den Energien, die in den Teilchenbeschleunigern benötigt werden. In der Psychologie nimmt dagegen die Genetik mit den materiellen Molekülen der DNA, mit Neurotransmittern und Hormonen einen immer breiteren Raum ein.

    Zur Zeit hat das Immunsystem mit Immunglobulinen als Antikörper Hochkonjunktur. Tausende demonstrieren wieder gegen die Realität der Materie und gegen den Geist der Vernunft, weil man die Natur zu beherrschen meint.

    Bekanntlich sind es besonders die Esoteriker, die gerne und leichtfertig mit abstrakten, immateriellen Dingen wie Energie und Information umgehen, unbedarft von Fachkenntnissen und Widersprüchen zur Wissenschaft. An Materie könnte man sich den Kopf stoßen, an Phantasien nicht. Herr Mahner könnte davon ein Lied singen, wahrscheinlich ein ganzes Potpourri..

  487. @Elektroniker
    Was wir (noch) nicht wissen, gilt eben als „transzendent“, wie in der Mathematik.

    Da muss ich Sie korrigieren. Transzendent bedeutet in der Philosophie (nach Kant) nicht “unbekannt”, sondern “unerkennbar”. Um erkennen zu können, müssen a priori gewisse Bedingungen erfüllt sein, bloße Sinneswahrnehmung allein reicht nicht aus. Zur Anschauung gehört der Begriff dazu. Folglich ist der Verstand das Subjekt von Erkenntnis.

    Das Problem ist, woher kommt der Begriff für das Angeschaute, woher wissen wir, dass ein Haus ein Haus ist? Es bedarf des Wissens vor der Erfahrung, der “reinen Vernunft” bei Kant. Das ist ein Stück notwendige Metaphysik. Natürlich wissen wir, dass wir als Kinder von den Vorfahren lernen, aber woher hatten die VorVorfahren ihr Wissen? Deshalb bedarf es gewisser grundlegender, angeborener Ideen, von Kant als Kategorien bezeichnet, z.B. Raum und Zeit. Kant hatte allerdings noch kein Wissen von der Evolution.

    Die Naturgesetze gibt es nicht wirklich. Es sind unsere Konstrukte, um die Welt strukturieren und verstehen zu können. Manche Naturgesetze sind sicher so gut wie “ewig”, andere unterliegen dem Fortschritt der Erkenntnis, Beispiele sind wohlbekannt. Naturgesetze sagen nicht die Wahrheit, sondern wie wir mit der Welt umgehen können, z.B. zum Mond fliegen oder gegen Viren schützen.

  488. @Feodor: Vorhersagekraft

    Mit Verlaub, doch das halte ich für kein gutes Kriterium:

    So sagt man etwa über die Evolutionstheorie (konkret: im Sinne der Synthetischen Theorie), dass diese zwar rückwirkend viel erklärt, wir daraus aber z.B. nicht ableiten können, welche Spezies es in fünf, fünfzig, fünfhundert… Jahren einmal geben wird. (Das dürfte schon aus dem theoretischen Grund [zumindest so gut wie] unmöglich sein, dass a) genetische Mutationen zufällig sind und b) die zukünftige Umwelt, in der die Lebewesen dem Selektionsdruck unterliegen, chaotisch und in diesem Sinne unvorhersehbar ist; fangen wir doch einmal mit dem Wetter von übermorgen an.)

    Außerdem sind nach meinem Wissen viele bahnbrechende Entdeckungen “Zufallsfunde” und zeigen sie gerade etwas, was sich vorher niemand vorstellen konnte oder was vielleicht sogar als unmöglich ausgeschlossen wurde: Denken wir beispielsweise an die Epigenetik oder auch an so etwas wie die Entdeckung der “Spiegelneurone”.

    In der populärwissenschaftlichen Darstellung vom wissenschaftlichen Fortschritt wird oft so getan, als bauten die Entdeckungen logisch aufeinander auf. Wissenschaftshistoriker (spätestens seit Kuhn) haben aber immer wieder gezeigt, dass dem nicht so ist: eben trial and error, Versuch und Irrtum.

    Ich glaube vielmehr, dass Sie sich ein paar einfache Beispiele aussuchen, die zu Ihren Ansichten passen, aber im Wesentlichen hier ideologisch auftreten, weil Sie vor allem bestimmte religiöse Ansichten als “naturwissenschaftlich unmöglich” ausschließen wollen, weil Ihnen die nicht gefallen. Nochmals: Naturalismus ist eine philosophische, vielleicht sogar ideologische, nicht aber (natur-) wissenschaftliche Position.

  489. @ anton reutlinger 23.01.2021, 20:39 Uhr

    Die Transzendenzfrage war nicht meine Kernaussage, ich würde mich jedenfalls auf die Mathematik beziehen. Da sind transzendente Objekte (pi, e) jedenfalls nicht vollständig bekannt, können sie nicht sein. Zur „Unerkennbarkeit“ mache ich, vorsichtiger Weise, keine Aussage.

    Zitat Wikipedia: „Als Naturgesetz wird in der Wissenschaftstheorie eine Regelmäßigkeit von Vorgängen in der Natur bezeichnet. Die Pluralform „Naturgesetze“ bezeichnet darüber hinaus die Gesamtheit dieser Regelmäßigkeiten, einschließlich solcher, die noch nicht entdeckt oder formuliert wurden, unabhängig von ihrer spezifischen Formulierung. Von anderen Gesetzen unterscheiden sich Naturgesetze darin, dass sie nicht von Menschen nach deren Belieben in Kraft oder außer Kraft gesetzt werden können. Eine genaue, einheitliche abschließende Definition des Begriffs existiert derzeit nicht.“

    Zitat Reutlinger: „Die Naturgesetze gibt es nicht wirklich.“

    Diese Formulierung ist eher „daneben“. Viele Gesetzmäßigkeiten auf die sich z.B. die Physiker geeinigt haben und die im Experiment bestätigt sind, dürften korrekt und bedeutsam sein. Fehler kann man nie ausschließen.

    Es kann im Prinzip nicht falsch sein, von einem „Ideal“ auszugehen, dem man sich in der Realität nur annähern kann.

    So etwas „wie implizites Wissen“ (über „Naturgesetze“) dass latent existiert, aber nur zum Teil „explizit gespeichert“ ist, gibt es offenbar auch. Sonst könnte nicht fast täglich neues Wissen „geschürft“ werden. Vermutlich können KI Systeme künftig beim „Schürfen nach Wissen behilflich sein. (Das sage ich, obwohl ich mich vor einigen Stunden über die „Dummheit“ eines KI Systems geärgert habe.)

    Auch ist es eine Frage, wie, sozusagen in welcher „Sprache“, man „Naturgesetze“ bestmöglich formuliert.

    Könnte mir eine Art „Kunstsprache“, ähnlich wie Programmiersprachen, die viele mathematische Begriffe enthalten und die für KI Anwendungen geeignet sein sollte, vorstellen.

    Dass es auch andere nützliche Gesetzmäßigkeiten gibt die das Leben erleichtern, ist mir klar.

  490. ´Tranzendenzerfahrungen´ lassen sich einfach mit Drogen erzeugen:

    http://www.sciencedaily.com/releases/2019/04/190423145511.htm
    ´Experiences of ´ultimate reality´ or ´god´ confer lasting benefits to mental health´

    DOI: 10.1371/journal.pone.0214377 ´Survey of subjective ´God encounter experiences´: Comparison among naturally occurring experiences and those occasioned by the classic psychedelics psilocybin, LSD, Ayahuasca or DMT´

    Für Naturalisten: diese Erfahrungen lassen sich mit den Fachbegriffen ´predictive coding theory + state dependent retrieval´ komplett als Ergebnis eines simplen Erinnerungsvorgangs beschreiben/erklären.

  491. @Chrys // 23.01.2021, 13:00 Uhr

    »Nein.«

    Hmm, das wundert mich.

    Meinem Eindruck nach reden Christian Hoppe und Alexander Riebel von unterschiedlichen Dingen. Und vor allem implizieren Hoppes Äußerungen nicht, dass bei menschlichen Handlungen eine außerphysische Instanz im Spiel sein könnte, wie das bei Riebel der Fall ist.

    »… dann bliebe da nichts weiter zu erklären, sondern höchstens noch etwas zu übsersetzen, von der Physik beispielsweise in die Psychologie, …«

    Selbst wenn das gelänge, damit wäre überhaupt nichts gewonnen oder gar (wissenschaftlich) erklärt, sondern es würden nur neue Fragen aufgeworfen.

    Aber gibt da nichts zu „übersetzen“, schon allein deshalb nicht, weil das intentionale Vokabular völlig unbestimmt ist.

    » Die [diversen ontischen Monismen] sind alle nach dem gleichen Denkmuster gestrickt und zeigen alle die gleiche logische Inkonsistenz.«

    Mag schon sein. Aber was lehrt uns das? Etwa über die menschliche Logik oder die Denkmöglichkeiten? Wäre der ontische Pluralismus die Lösung?

    Ich frage mich, inwieweit ein ontischer Monismus überhaupt konstitutiv für den Naturalismus ist. Zumal es _den_ Naturalismus ohnehin nicht gibt. Und es scheint in der Tat so zu sein, dass heutzutage diese metaphysische Spielart in der Fachwelt kaum eine Rolle spielt. Und dort, wo er behandelt wird, geht es, soweit ich das überblicke, nicht um (vermeintliche?) logische Inkonsistenzen.

    Und nachdem wir ja nun den ganzen Satz aus Mahners Buch kennen, der da lautet: „Wir nehmen also (in aller Regel ganz stillschweigend) die Existenz einer Welt außerhalb unseres Denkens an‟, dürfte sich zumindest an dieser Stelle der Vorwurf der logischen Inkonsistenz erledigt haben.

    Abschließend ein Zitat aus der Einleitung zu dem von Geert Keil und Herbert Schnädelbach herausgegebenen Band über den Naturalismus (Suhrkamp 2000):

    Man tut gut daran, sich zunächst vor Augen zu führen, daß »Naturalisrnus« ein Ausdruck der philosophischen Fachsprache ist. Das bedeutet nicht, daß der Begriff wohldefiniert wäre, es bedeutet nur, daß Nichtphilosophen in der Regel nicht wissen, was Philosophen mit »Naturalismus« meinen.

  492. @Feodor // 23.01.2021, 17:06 Uhr

    » Naturalismus ist eine Voraussetzung von Naturwissenschaft. «

    Das ist eine problematische Aussage. Ist es nicht vielmehr so, dass sich der (philosophische) Naturalismus auf die Naturwissenschaften beruft?

    In der Einleitung zu dem von Geert Keil und Herbert Schnädelbach herausgegebenen Band über den Naturalismus (Suhrkamp 2000) findet sich dieser Abschnitt, der das Problem treffender und prägnanter beschreibt, als ich es könnte:

    Tatsächlich trifft man gelegentlich auf missionarisch veranlagte Naturalisten, die bei allen, die sich nicht zum Naturalismus bekennen, Wunderglauben, Obskurantismus oder Okkultismus vermuten. Damit unterschätzen Naturalisten aber nicht nur ihre Gegner, sie machen auch ihre eigene Position undeutlich. Mit der Antithese »Natur vs. Übernatur« läßt sich heute keine interessante Position mehr markieren. Wenn Naturalismus nichts anderes als die Annahme wäre, daß alles in der Welt mit rechten Dingen zugeht, würden ihm alle ernstzunehmenden Opponenten abhanden kommen, und eine Position ohne Gegenposition ist keine Position mehr; wenn man überall steht, steht man nirgends. Sellars’ Feststellung, daß »we are all naturalists now«,5 spricht nicht für den Naturalismus, sondern gegen eine Begriffsbestimmung, die diese Konsequenz hat.

    Aus nichtphilosophischer Sicht hätte ich nichts dagegen, in diesem Sinne „überall zu stehen“, aber das nur nebenbei bemerkt.

  493. Balanus,
    es läuft immer auf das Gleiche hinaus, welches Etikett klebe ich auf die Flasche.

  494. @Stephan Schleim

    Das ist eine schwächere Form…

    Nicht unbedingt. Deine Verkürzung, alles in der Welt sei Materie, beruht nur auf einer ‘äußerst starken’ Interpretation dessen, was unter ‘manifestations’ zu verstehen sein soll. Da die von Dir in die Diskussion eingebrachte Definition eine solche Interpretation möglich macht, könnte es sich also sogar um die eigentlich ungenauere, mithin schwächere Form handeln. Deine Interpretation gibt zumindest nicht die Weltsicht wieder, die auch nur ein mir bekannter Materialist vertreten würde (siehe meine Beispiele).

    Eigenschaftsdualismus

    Warum so sparsam? Man könnte ja auch versuchen, den Sinnfeldbegriff von Markus Gabriel produktiv zu nutzen, um mit ihm einen Eigenschaftspluralisums zu begründen. Zusammen mit etwas Biosemantik (Ruth Millikan) könnte ein neuer philosophischer Gedanke emergieren. Ich halte Fortschritt auch in der Philosophie noch für möglich.

    Aber auch ein starker Materialismus, so lese ich deinen Kommentar, müsste eine Antwort auf die Frage nach der Existenz von Abstraktem, Allgemeinen geben

    Eine kurze Gegenfrage, länger als die Antwort, ist Dir die wirklich nicht bekannt?

    Universalienstreit

    Der Materialismus muss sicher nicht alle Fragen beantworten können. Insbesondere kann er nur Fragen innerhalb seiner Weltanschauung beantworten. So wird er keinen Streit zwischen verschiedenen Weltanschauungen auflösen können. Und auch innerhalb des Materialismus können sich jederzeit weitere Subkulturen herausbilden, die weder logisch noch empirisch zu befrieden sind.

    dass es Abstracta usw. nur im Denken […] gibt und in diesem [Sinne] nichts als Materie sind.

    Im Denken ist keine Materie, aber es gibt Bezüge.

    PS.
    Zu deiner in einem Kommentar weiter oben an alle “Möchtegernnaturalisten” gestellten Hausaufgabe.

    Ich möchte mich hier soweit herausreden, diese Hausaufgabe nicht gemacht zu haben, weil ich einfach nicht die Zeit dazu hatte (ich war spazieren im Park und viel im Internet, man könnte auch sagen in sehr unterschiedlichen Welten Sinnfeldern aktiv).

  495. @Joker: Diskussion

    Eine kurze Gegenfrage, länger als die Antwort, ist Dir die wirklich nicht bekannt?

    Was bringt uns diese Gegenfrage? Spielst du hier auf die Träger/Inhalt-Unterscheidung an? Wenn ja, dann verlagern wir das Problem auf die nächste Ebene: Was ist Inhalt und wie ließe sich dieser materialistisch fassen? Ich kann deine Gedanken nicht lesen; und bezweifle auch, dass es nur “die” eine Antwortmöglichkeit gibt.

    Im Denken ist keine Materie, aber es gibt Bezüge.

    Es ging doch um eine Diskussion aus materialistischer Sicht… und es gibt genügend Hirnforscher (und Philosophen), die in den letzten Jahren wortwörtlich gesagt haben, Gedankenvorgänge seien nichts als neurophysiologische Vorgänge: Du bist dein Gehirn!

  496. Stephan Schleim
    „Du bist dein Gehirn“, diese plakative Aussage verschweigt, dass wir nicht allein auf dieser Erde sind. Kein Mensch kann allein existieren. Er hat eine Mutter, die ihn geboren hat, er hat einen Arbeitsplatz, er hat Freunde und er hat Bücher , die er liest. Das Du ist auch ein Wir.
    Wer wir sind ist dynamisch zu denken. Ich bin morgen nicht mehr derselbe wie heute, wenn ein entscheidendes Ereignis eintritt, das nicht aus unserem Gehirn kommt.

    Und wenn wir dann noch ein Schichtenmodell verwenden mit ES, ICH, ÜBERICH, dann stellt sich mir die Frage, Wer bin Ich ?
    Und wenn ich mir dann noch am Sonntagmorgen das EWIGE LICHT in unserer Kirche anschaue und merke wie es wirkt, wie mich die Freude überkommt, dann ist die Vorstellung vom Gehirn als Ursache meines Ich nicht mehr überzeugend.

    Wir sollten eher vom Phänomen „Ich „ reden. Oder gleich, wie es die Christen machen, die reden von der Seele.

  497. @ Stephan Schleim 24.01.2021, 14:40 Uhr

    Zitat: „…. genügend Hirnforscher (und Philosophen), die in den letzten Jahren wortwörtlich gesagt haben, Gedankenvorgänge seien nichts als neurophysiologische Vorgänge: Du bist dein Gehirn!“

    Gedankenvorgänge sind neurophysiologische Vorgänge, das wäre aus technischer Sicht naheliegend.

    „Du bist dein Gehirn!“ scheint für einen „Normalo“ deswegen nicht korrekt, weil von außen betrachtet, ein Mensch wesentlich mehr als sein Gehirn ist, funktionelle und auch strukturelle Unterschiede bestehen.

    Allenfalls wäre eine Gemeinsamkeit, dass sozusagen beides „Fleisch“ ist, der Kategorie Fleisch angehört, für Philosophen eher eine „lächerliche Aussage“, höchstens für Metzger von Bedeutung.

    Warum verwendet die Philosophie immer wieder derartige „Gleichsetzungen“ wenn genau genommen diese Gleichheit z.B. wegen unterschiedlicher Kategorien gar nicht gegeben ist?

    In der Informatik wäre derartiges wegen der Widersprüche die sich ergeben können völlig tabu?

  498. @Stephan Schleim
    Zitat: “Nochmals: Naturalismus ist eine philosophische, vielleicht sogar ideologische, nicht aber (natur-) wissenschaftliche Position.”

    Philosophisch ja, ideologisch eher nein. Genau das versuche ich ja in dem von Grams zitierten Buch darzulegen: der ontologische Naturalismus als “Bedingung der Möglichkeit” natur- bzw. realwissenschaftlicher Erkenntnis. Also ontologischer Naturalismus als Voraussetzungsmetaphysik. Das klingt nach Kant, gibt aber halt keine idealistische Antwort. Daher kann der metaphysische Naturalist auch kein radikaler erkenntnistheoretischer Naturalist sein, der behauptet, Realwissenschaft sei philosophiefrei zu betreiben (abgesehen von etwas Logik vielleicht). Der metaphysische Naturalist muss nicht jeden anderen Naturalismus unterschreiben!

  499. N sei im Folgenden die Annahme, dass es im Universum ausschließlich naturgesetzlich zugeht. Die logische Negation dieser Aussage ist S: Im Universum sind auch Ereignisse möglich, die sich nicht naturgesetzlich fassen lassen.

    Zu Balanus, 24.01.2021, 11:52 Uhr:

    Keil und Schnädelbach sprechen gleich zu Beginn von “Sympathie”, was m.E. unangemessen ist. Brauche ich Sympathie für die Gravitation? Recht haben die Autoren im Folgenden allerdings, dass der Begriff “Naturalismus” mit unterschiedlichen Bedeutungen belegt und oft schlecht erläutert wird. Ich verwende daher einen engen Begriffsumfang, d.h. Naturalismus bedeutet bei mir N. S als logischer Gegensatz ist identisch mit dem Supranaturalismus. Der Einwand der Autoren gegen einen engen spezifischen Naturalismusbegriff ist völlig haltlos. Zum einen kann ein Begriff auch dann deskriptiv hilfreich sein, wenn niemand das logische Gegenteil verträte. Zum anderen behaupten die Autoren, dass Gruppen, die die N bestreiten, nicht ernst zu nehmen seien. Das erscheint mir zweifelhaft. Es gibt immer noch heftigen Widerstand gegen die Evolutionstheorie, etwa in der USA und in der Türkei. Eine Argumentation für S gibt es z.B. bei den Philosophen Michael Dummet und Alvin Plantinga, die “Gottesbeweise” entwarfen. 2019 fand die von der “Internationalen Akademie für Philosophie” veranstaltete Tagung „Free Will and Divine Action“ statt, auf der ebenfalls von professoraler Seite gegen N argumentiert wurde. Wann ist etwas ernst zu nehmen und wann nicht?

    Stephan Schleim 23.01.2021, 22:52 Uhr:

    Stephan Schleim benennt ein ganzes Bündel von Argumenten. Ich fühle mich genötigt darauf hinzuweisen, dass es sowohl in den Einzelwissenschaften als auch im metatheoretischen Bereich üblich ist, verschiedene Lösungsalternativen einer vergleichenden Analyse zu unterziehen. Die gegenwärtige Diskussion scheint ein weiterer Beleg dafür zu sein, dass N immer noch umstritten ist. Die Argumente im Einzelnen:

    1. Quantenzufall und deterministisches Chaos verhinderten die Vorhersage von Ereignissen, die nicht passieren dürfen, wenn die Annahme der Naturgesetzlichkeit des Universums richtig ist. Die Evolutionstheorie sage vieles nicht voraus.

    Das scheint der klassische Erklärunglückeneinwand zu sein, der aber ein Argumentum ad ignorantiam darstellt. Des weiteren ist es auch kein Einwand gegen die Kritisierbarkeit von N und die Kritikimmunität von S. Kapriziöse Abweichungen bei der Statistik von Quantenereignissen lassen sich gut messen. Auch im Falle des deterministischen Chaos in einem System lassen sich Zustände angeben, die naturgesetzlich ausgeschlossen sind. Was die Evolution angeht, verweise ich auf das üppige “Counter-Creationism Handbook” von Mark Isaak.

    2. Die Entdeckung der Epigenetik und der “Spiegelneurone” zeige, dass es in der Vergangenheit zu überraschenden Erkenntnissen gekommen sei.

    Weder Epigenetik noch “Spiegelneurone” sind inkompatibel mit den heute bekannten Naturgesetzen.

    3. Wissenschaftliche Entdeckungen bauten nicht logisch aufeinander auf. Die wissenschaftliche Entwicklung verlaufe sprunghaft.

    Das ist kein Einwand gegen die Kritisierbarkeit von N. Die Entwicklung der allgemeinen Relativitätstheorie (ART) hat die Gravitationsverhältnisse auf der Erde nicht verändert. Für kleine Massen und Relativgeschwindikeiten konvergiert die ART gegen die newtonschen Formeln (Stichwort Korrespondenzprinzip).

    4. Die Beispiele für Ereignisse, die es nicht geben sollte, wenn die Naturgesetzlichkeitsannahme gilt, seien zu einfach.

    Dass leicht verständliche Beispiele präsentiert werden, invalidiert die Kritisierbarkeit von N nicht. Es lassen sich auch schwer verständliche Beispiele aus der Quantenphysik aufstellen.

    5. N sei eine reine Ideologie.

    Dieser Einwand hat keine Substanz.

    6. Mir würde einfach nur etwas nicht gefallen.

    Das ist eine Ad-hominem-Einlassung, die irrelevant ist. Es ist schlicht egal, was mir gefällt oder auch nicht.

    7. Stephan Schleim scheint vielleicht auch nahe legen zu wollen, dass Tatsachenaussagen, wenn sie aus dem Bereich einer Religion kommen, nicht kritisiert werden dürften.

    Wenn dem so wäre: Es gilt die Freiheit der Wissenschaft. Religiöse Aussagen dürfen kritisiert werden, vor allem dann, wenn sie Tatsachen betreffen.

    8. Naturalismus sei eine philosophische, aber keine (natur-)wissenschaftliche Position.

    Ich stimme zu, dass N eine philosophische metatheoretische Annahme aus dem Bereich der Ontologie darstellt. Ich sehe nicht, wie das die Kritisierbarkeit von N und die Kritikimmunität von S widerlegen soll. Die Kritisierbarkeit und Kritikimmunität ergibt sich jeweils aus der logischen Struktur der Aussagen. Eine Allaussage ist kritisierbar, eine unspezifische Existenzaussage ist nicht kritisierbar.

  500. @Feodor: Konsistenz

    Sie haben doch formuliert:

    Es kommt vielmehr darauf an, dass sich aus Naturgesetzen mögliche und unmögliche zukünftige Ereignisse ableiten lassen.

    Wenn Sie nicht interessiert, dass das für wesentliche naturwissenschaftliche Theorien nicht gilt, dann ist das ein Problem für Ihre Position, nicht für meine. Jedenfalls haben Sie gut an meinem Einwand und Beispiel vorbeigeredet.

    Und von dem, was Sie mir zur Religion in den Mund legen, kann ich mich nur distanzieren: Weder habe ich so etwas jemals behauptet, noch gemeint.

  501. @Stephan Schleim

    Aber auch ein starker Materialismus, so lese ich deinen Kommentar, müsste eine Antwort auf die Frage nach der Existenz von Abstraktem, Allgemeinen geben

    Die Antwort, das war ich noch schuldig geblieben, die uns der Materialismus gibt, lautet – Trommelwirbel:

    Nein.

    Im Materialismus existiert nichts Abstraktes, nur Konkretes; nichts Allgemeines, höchstens Gemeines. Im Universalienstreit ist der Materialismus nicht Schlichter, sondern Partei.

    Was ist Inhalt und wie ließe sich dieser materialistisch fassen?

    Bei der Phrase ‘im Denken’ hat das ‘im’ nicht den räumlich-zeitlichen Aspekt, den es bei Materie hat. Inhalt ist daher auch eher metaphorisch zu verstehen, es gibt Ähnlichkeiten, aber es ist nicht alles so wie bei einer Mülltonne.

    Gedankenvorgänge seien nichts als neurophysiologische Vorgänge

    Das Denken selbst ist materiell. Das ist durch die Dynamik neurophysiologischer Hirnprozesse gegeben. Die Inhalte der Gedanken sind nicht materiell.

    Etwas Abstraktes lässt sich verstehen als etwas, was in allen Hirnen einen korrespondierenden Prozess hervorruft. Nehmen wir die Zahl 2. Egal ob ein denkendes Wesen nun zwei oder 2 liest, oder ‘zwei’ hört, stellt sich ein mit 2 assoziierter Prozess ein.

    Wenn das Wahrnehmungssystem ein Paar Schafe identifiziert, dann wird, neben einem Prozess der ‘Schaf’ aktiviert, ebenfalls genau diese dynamische Hirnkonfiguration aktiviert, die “2” repräsentiert und die Hirnprozesse konvergieren zu einem Gedanken mit dem Inhalt ‘Da stehen zwei Schafe.´

    ‘Da’ und ‘stehen’ sind ebenfalls zu lernende Abstraktionen, ‘Schaf’ repräsentiert etwas Existierendes.

    Eine Schwierigkeit sehe ich nicht beim Abstrakten, knifflig wird es bei der Qualia, dem ‘hard problem’.

    @Balanus

    Darauf kommt es m. E. nicht [an].

    Meines Erachtens nach ist es das Entscheidende.

  502. @Schleim
    Wenn ich die Naturwissenschaften =Naturalismus richtig verstehe,dann sucht er zu ergründen.
    Sie sagen:so ‘ist’ es.
    Obwohl Sie sich als Skeptizistiker geben.
    Das passt nicht ‘zusammen’.

  503. @Joker // 24.01.2021, 19:12 Uhr

    »Darauf kommt es m. E. nicht [an].

    Meines Erachtens nach ist es das Entscheidende. «

    Es hat bei meiner Entgegnung scheint’s mehr gefehlt als das Wörtchen ‚an‘. Nämlich eine kurze Begründung, warum es mir Zusammenhang mit den Punkt, den ich machen wollte, nicht wichtig erschien. Ansonsten d’accord…

  504. Realität, Fiktion, Emergenz

    Realität ist die Summe der materiellen Dinge. Fiktion sind die Inhalte unserer Gedanken oder verallgemeinert, das was informationsverarbeitende Maschinen an inneren Repräsentationen erzeugen. Information verarbeitende Maschinen selbst sind real, sie lassen sich auf physikalische Komponenten und Prozesse zurückführen. Die Fiktionen haben ebenfalls eine reale Grundlage, nämlich die informationsverarbeitenden Maschinen ohne die sie nicht erzeugt werden können. Doch die Bedeutung der Fiktionen existiert nur für die informationsverarbeitenden Maschinen. Mathematische Abstraktionen sind ein Beispiel für solche Fiktionen. Ihre Bedeutung erschliesst sich nur den Mathematikern, die diese Fiktionen erfunden haben. Und zu einem geringeren Grad auch den Anwendern, die nicht unbedingt Mathematiker sein müssen.

    Emergenz bedeutet in der physikalischen Welt, dass ein Ding Eigenschaften besitzt, die keines seiner Bestandteile hat. Es bedeutet nicht, dass sich die neue Eigenschaft nicht erklären liesse, nur dass es keine einfache Erklärung gibt. In der Physik taucht Emergenz schon bei ganz grundlegenden Dingen auf. So haben Protonen und Neutronen die Fähigkeit/Neigung sich zu Atomkernen zusammenzulagern, doch die Bestandteile von Neutronen und Protonen, nämlich die Quarks besitzen diese Eigenschaft nicht – im Gegenteil: Es werden sich nie 10 Quarks zusammenballen. Das können erst die Nukleonen, also Neutronen und Protonen. Die „Verrücktheiten“ unserer realen physikalischen Welt lassen sich nur durch solche emergente Phänomene erklären wie im Beispiel gerade von mir genannt. Ein Physiker, der ein Proton oder einen Atomkern beschreiben will, also auf seine Bestandteile und ihre Interaktionen zurückführen will, muss viele Jahre seines Lebens darin investieren das zu tun – oder er greift auf die Arbeiten seiner Vorgänger zurück, die das schon getan haben.

    Ich bin versucht, auch Fiktionen, die wir in unserem Geist erzeugen als etwas Emergentes zu betrachten, bin mir aber nicht ganz sicher wo die Fallstricke bei diesem Gedanken sind.

  505. @Joker: Danke

    Das ist hilfreich…

    …ich verstehe aber nicht, wieso phänomenaler Gehalt (Erleben) für den Materialisten ein größeres Problem wäre als intentionaler Gehalt (Inhalt). Ob es sich für uns irgendwie anfühlt, einen bestimmten Bewusstseinsprozess zu haben, ist dann völlig irrelevant. Im Prinzip wären dann Chalmers’ Zombies möglich; und wir wüssten nicht, ob unser subjektives Erleben eine Einbildung ist – oder ein Epiphänomen.

    P.S. Über Qualia spreche ich lieber nicht, wegen der Reifizierung.

  506. @Balanus / 24.01.2021, 11:25 Uhr / Zur Vollständigkeit

    Wenn Christian Hoppe dabei auf das »vollständig« verzichtet hätte, dann wäre mir immerhin ein Unterschied erkennbar.

    Mich würde noch interessieren, wie Stephan das verstanden hat, und ob er den Unterschied sieht, den Du mir nahezubringen versuchst. An ihn war dieser Kommentar von Christian Hoppe schliesslich gerichtet. Und vielleicht liegt’s ja nur an meinem Sprachverständnis…

    Noch so als gedankliche Anregung: Wenn sich schon arithmetische Sachverhalte feststellen lassen, die mit den Regeln der Arithmetik nicht erklärbar sind, dann wäre doch die Frage zu stellen, ob sich nicht prinzipiell auch physikal. Sachverhalte finden lassen, die mit den Regeln der Physik nicht erklärbar sind. (Denk’ mal wie Hofstadter, oder versuch’ es vielleicht wenigstens mal.)

  507. Die bisherige Diskussion zeigt, dass man das eigentliche Problem nicht verstanden hat.
    Ob es nichtphysikalische Phänomene gibt oder nicht – dazu kann man nur eine persönliche Glaubensäußerung abgeben:
    Entweder man glaubt daran oder man glaubt nicht daran.

    Sinnvolle Argumente/Belege dazu/dagegen gibt es nicht und kann es auch nicht geben

  508. KRichard,
    ob es nichtphysikalische Probleme gibt oder nicht, das wird schon in der Sprache entschieden. Die Erkenntnistheorie untersucht, unter welchen Umständen Erkenntnis=Wissen zustande kommt. Für einen Materialisten ist die Abstraktion nicht real. Aber das führt zu dem Widerspruch, dass wir das Reale, das wir sehen , erkennen nur über Begriffe erkennen, die Abstraktionen sind.
    Wenn wir keine abstrakte Vorstellung über die Elektrizität hätten, dann würden wir Blitz und Donner immer noch für das Wirken des Donnergottes halten.

    Der Siegeszug der Physik wurde dadurch eingeleitet, dass abstrakte Begriffe die phänomenale Naturbeschreibung ersetzt haben. Aus Naturbeschreibung wurde die Naturwissenschaft. Das geht nur über Abstraktionen. Eine mathematische Beschreibung eines Naturvorganges ist eine Abstraktion.

    Beim analytisch deduktivem Denken denken wir abstrakt. Nur beim assoziativen Denken, da können wir beim bildhaften Denken bleiben.
    Und jetzt zu der Kernfrage: Ist abstraktes Denken physikalisch ?
    Ist es nicht, weil man sonst eine Formel dafür aufstellen könnte.
    Der weitgehendste Versuch “Denken” wissenschaftlich einzuordnen, das sind die Kategorien von E. Kant. .
    Verdict: Denken ist ein nichtphysikalisches Phänomen.

  509. hwied
    zum Denken braucht man ein Gehirn – also konkrete physikalische Grundlagen.

  510. @KRichard, hwied (Zitat): Wie Martin Mahner in seinem Kommentar vom 24.01.2021, 16:22 Uhr geschrieben hat, ist ontologischer Naturalismus Voraussetzungsmetaphysik für Naturwissenschaftler und für Menschen, die daran glauben, dass Naturwissenschaftler unsere gemeinsame Welt erforschen.

    Klar ist das eine Glaubensfrage. Aber anders als viele meinen, ist Glauben oft unvermeidbar und wer das „Falsche“ glaubt, kann in die Irre gehen.

    Gemäss dem ontologischen Naturalismus/Materialismus gründet die Realität aller Dinge in der Natur, in der Physik. Auch unsere Gedanken und Abstraktionen haben keine Eigenexistenz, sondern werden in unseren Köpfen geboren. Für einen ontologischen Materialisten gibt es keine Mathematik ohne Mathematiker und keine Gottesvorstellung ohne Leute, die daran glauben und denken.

    Und fast noch wichtiger: Was in unseren Köpfen passiert sind letztlich physikalische Vorgänge und gehorcht physikalischen Gesetzen. Das bedeutet konkret: Es kann von vornherein keine Dinge wie Telepathie geben und wenn doch, dann müsste die Telepathie physikalisch erklärbar sein. Ferner: die Erkenntnisse zu denen wir kommen entstehen in unserem Kopf, sie werden uns nicht von Gott oder einem anderen Wesen eingeflösst.
    Alles geht also immer mit rechten Dingen zu wenn wir sind und etwa denken.

    Dieser Glaube wir seien selbst Dinge in der uns umgebenden physikalischen Welt hat grosse Konsequenzen. Er bedeutet implizit auch, dass wir Menschen auch Naturdinge sind und uns niemand geschaffen haben muss, genau so wie auch niemand die Tiere und alles andere geschaffen hat. Für uns gelten genauso die Naturgesetze wie für alle anderen und selbst für alle Dinge. Wenn also die Naturwissenschaft sagt, alle materielllen Dinge unterliegen der Schwerkraft, dann unterliegen auch wir der Schwerkraft. Und selbst die Fähigkeit zu denken setzt physikalische Prozesse voraus und – welch Wunder – verbraucht Energie.

  511. @Martin Holzherr
    Wie Sie schreiben, “ist ontologischer Naturalismus Voraussetzungsmetaphysik für Naturwissenschaftler”.

    Das klingt nach einer Petitio Principii: Der ontologische Naturalismus ist “Bedingung der Möglichkeit” natur- bzw. realwissenschaftlicher Erkenntnis. Da wir gut bestätigte und bislang unwiderlegte Erkenntnis haben, gilt der ontologjsche Naturalismus. Damit ist der logische Zirkel geschlossen.

    Ich meine, der ontologische Naturalismus hängt in der Luft, er ist untauglich, Wissen zu begründen.

  512. Holzherr
    “dass wir Menschen auch Naturdinge sind und uns niemand geschaffen haben muss, ”
    Wenn die Natur so ein komplexes Gebilde wie den Menschen hervorbringen kann, warum dann nicht eine Rolex-Uhr, die wir versteinert im Gebirge finden könnten.?

    Das , was der Mensch kann, das kann die Evolution nicht, der Mensch kann abstrahieren. Ein Mensch ist schöpferisch, er kann Dinge machen, die es im ganzen Universum noch nicht gab.
    Deshalb gehört die Kategorie Denken einer anderen Kategorie an wie Gehirn.
    Ich weiß, alles schon gehört.

  513. Was ist Bewusstsein ? 8. Denkansatz

    Mal ganz praktisch. Ein Fahranfänger der muss lernen, den Motor anlassen, die Kupplung treten, den Gang einlegen, Gas geben und dabei die Kupplung langsam kommen lassen. Das muss der Fahranfänger wissen.
    Jedesmal, wenn er losfährt muss er sich konzentrieren , denken und sich den Ablauf vorstellen. Er macht das bei vollem Bewusstsein.
    Irgendwann braucht er das nicht mehr zu denken, der Ablauf hat sich verinnerlicht, er läuft automatisch ab. Er muss auch nicht darüber im Gedächtnis suchen.
    Wo steckt das Wissen über den Ablauf? Es steckt im Bewusstsein.
    Bewusstsein bedeutet in diesem Zusammenhang = Verinnerlichung
    Und wenn man jetzt sagt : ” Im Bewusstsein , dass Widerworte den Streit nur noch weiter eskalieren lassen”, dann meint man damit die verinnerlichte Einsicht den Mund zu halten, wenn man den Streit vermeiden will.

  514. @Chrys // 25.01.2021, 00:46 Uhr

    Hoppe schrieb im Zusammenhang mit physischen Vorgängen von einer vollständigen „prinzipielle[n]“ Erklärung und bestreitet an keiner Stelle, dass Menschen für ihr Handeln verantwortlich sind.

    Riebel hingegen … aber gut, vielleicht habe ja ich da etwas missverstanden und er meint—mit anderen Worten—im Grunde das Gleiche wie Hoppe.

    Für gedankliche Anregungen bin ich immer dankbar. Also: Sind physikalische Sachverhalte denkbar, die sich prinzipiell nicht „mit den Regeln der Physik“ erklären lassen? Ja, sicher, die Welt (der SF) ist voll von solchen Vorstellungen. Weiterhin gibt es Bereiche, die der Physik messtechnisch prinzipiell verschlossen bleiben.

    Aber daraus würde ich nicht schließen, dass wir die Idee von der prinzipiellen Erklärbarkeit aller physikalischen Phänomene aufgeben sollten. Das wäre schlecht für die Forschung.

  515. @hwied (Zitat):

    “ Wenn die Natur so ein komplexes Gebilde wie den Menschen hervorbringen kann, warum dann nicht eine Rolex-Uhr, die wir versteinert im Gebirge finden könnten.?“

    Antwort: Das ist letztlich eine praktische Frage, die Frage nämlich, wie etwas zustande kommt. Rolexuhren und Menschen sind Dinge der natürlichen Welt und aus unserer Kenntnis der Naturgesetze wissen wir, dass Dinge eine Geschichte haben, dass es Kräfte und Prozesse gibt, die sie formen. Heute ist die Naturwissenschaft ja so weit, dass sie die materielle Welt bis zum Urknall erklären kann.
    Und es gibt sogar eine neue Art Geschichte zu betreiben, Big History genannt. Big History will den Geschichtsunterricht so abändern, dass nicht nur die Geschichte des Menschen, sondern die des gesamten Universums erklärt wird.

  516. @ hwied 25.01.2021, 08:52 Uhr

    Zitat: „Und jetzt zu der Kernfrage: Ist abstraktes Denken physikalisch ?
    Ist es nicht, weil man sonst eine Formel dafür aufstellen könnte.
    Der weitgehendste Versuch “Denken” wissenschaftlich einzuordnen, das sind die Kategorien von E. Kant. .
    Verdict: Denken ist ein nichtphysikalisches Phänomen.“

    Dass Denken ein nichtphysikalisches Phänomen wäre, könnte man praktisch nur behaupten, wenn das „Empfindungsphänomen“ außerhalb der Gesetzmäßigkeiten der Physik „entstehen“ würde.

    Die Informationsverarbeitung in neuronalen Systemen (zumindest außerhalb der Empfindungen) dürfte physikalisch erklärt werden können, ähnlich wie es in der Informatik absolut selbstverständlich ist. Natürlich können logische Zusammenhänge, anders als in der Informatik, nur statistisch gesehen werden.

    Die Sensortechnik, ein heutzutage extrem wichtiges Fachgebiet, befasst sich mit Prozessen die im Grenzbereich zur Quantenphysik stattfinden. Es wäre auch denkmöglich, dass dabei unter bestimmten Umständen, auch „Empfindungsphänomene“ auftreten.

    Kenne jedenfalls keine Erkenntnisse die derartiges ausschließen würden?

    Was meinen Sie dazu?

  517. @hwied (Zitat): “ Deshalb gehört die Kategorie Denken einer anderen Kategorie an wie Gehirn.“
    Das gilt doch nicht nur für das Denken. Auch biologisches Leben gehört einer anderen Kategorie an als die unbelebte Natur. Das aber ist alles innerhalb unseres physikalischen Universums möglich. Wer unserem Universum nur zutraut, dass es Dinge neu mischen, neu komponieren kann ohne wirklich Neues zu schaffen, der hat einfach ein falsches Bild von der Natur.
    Eine wichtige Konsequenz hat der Glaube, dass auch der Mensch ein physikalisches Ding ist. Es bedeutet, dass kaum etwas exklusiv nur für den Menschen gilt. Der Mensch ist ein Objekt und ein Produkt dieser Welt und er gehorcht Gesetzen, die wir mit der Naturwissenschaft erforschen und mit der Technik nachbilden können.

  518. @Timm Grams (Zitat):

    Das klingt nach einer Petitio Principii: Der ontologische Naturalismus ist “Bedingung der Möglichkeit” natur- bzw. realwissenschaftlicher Erkenntnis. Da wir gut bestätigte und bislang unwiderlegte Erkenntnis haben, gilt der ontologjsche Naturalismus. Damit ist der logische Zirkel geschlossen.

    Antwort: Der (Selbstzitat) „ontologische Naturalismus als Voraussetzungsmetaphysik für Naturwissenschaftler“ ist doch das Resultat eines Denkprozesses von Philosophen. Und dieser Denkprozess abstrahiert das, was Naturwissenschaftler tun, ihre Arbeits- und Denkweise und die wichtigsten Resultate, die sie hervorgebracht haben. Die Philosophen dahinter haben also etwas herausgeschält, sie haben versucht aus der naturwissenschaftlichen Vorgehensweise und den Resultaten etwas herauszuschälen, was die Philosophie eine Ontologie nennt.
    Konkret: Im Buch „Über die Natur der Dinge“ von Mario Bunge und Martin Mahner, wird beispielsweise der physikalische Begriff der Energie, die Frage nämlich, was Energie ist, zu einer ontologischen Frage erklärt. Die Autoren haben hier also eine Verbindung zwischen einem Begriff der Physik und einem Begriff der Philosophie geschaffen.
    Mario Bunge und Martin Mahner haben also etwas Ähnliches getan wie Kant. Nur hat Kant eben etwas aus seinen Gedanken, Vorstellungen und seinem damaligen Wissen über die Welt herausfiltriert, während Bunge und Mahner als Quelle die heutigen Naturwissenschaften verwenden.
    Natürlich ist das – wie bei Kant – nur ein Versuch, ein Versuch aber mit Schlussfolgerungen, von denen die Autoren glauben, dass sie sie mit überzeugenden Argumenten verteidigen können.

  519. Martin Holzherr,
    ein kluger Satz “Auch biologisches Leben gehört einer anderen Kategorie an als die unbelebte Natur.”

    Das Geheimnis des Lebens ist auch ein nichtphysikalisches Phänomen.
    Ich meine jetzt nicht die biologischen Abläufe, und die Chemie des Biologischen. Es geht um das “Leben”.
    Mit nur einem Wort ist es nicht erklärt. Warum lebt etwas ???

  520. @ Martin Holzherr 25.01.2021, 10:08 Uhr

    Zitat: „Auch unsere Gedanken und Abstraktionen haben keine Eigenexistenz, sondern werden in unseren Köpfen geboren. Für einen ontologischen Materialisten gibt es keine Mathematik ohne Mathematiker und keine Gottesvorstellung ohne Leute, die daran glauben und denken.“

    Ich unterstelle einmal, den „Hardcore Materialisten“ geht es darum, alles was auch nur „winzigst“ auf so etwas wie „Gott“ auch nur entfernt hindeuten könnte, wütend zu bekämpfen und nicht wirklich nach „Wahrheit“ zu suchen.

    Gedanken, Abstraktionen, „informelle Objekte der Mathematik/Informatik“ sind praktisch „Information“ mehr oder weniger korrekte „Abbildungen“ alles in der Natur auch nur „Möglichen“ oder auch „Existenten“. Es kann sogar absolut nicht Existentes und Unmögliches „informell abgebildet“ werden.

    Bei technischen Information verarbeitenden Prozessen gibt es „jede Menge“ künstliche , abstrakte, vom Programmierer „geschaffene“, und mitunter nur Millisekunden im Prozessor bei der Programmausführung real existierende „informelle Objekte“.

    Diese Objekte können und haben meistens (praktisch immer) auch einen nützlichen Grund, aber ein anderer, sagen wir einmal „erfahrenerer Programmierer“, würde z.B. wesentlich zweckmäßigere „Objekte“ deklarieren und verwenden.

    So braucht man sich nicht zu wundern, dass auch im normalen Leben bei der Kommunikation von Menschen untereinander, viele „Geisterobjekte“ existieren, über deren Nützlichkeit und Zweckmäßigkeit man sich wundert, oder auch nicht. Manche könnten existentiell wichtig sein, andere scheinen entbehrlich.

    Man kann das alles ganz „cool“ sehen, oder auch mit Schaum vorm Mund….

  521. @hwied (Zitat): “ Warum lebt etwas ??“ Diese Frage haben sich auch Naturwissenschaftler gestellt – und sie haben sie beantwortet.

    Kleiner Seitenschwenk: Stephen Hawking sagt in seinem letzten Buch über die Frage ob Viren bereits Lebewesen seien: Viren hätten zwar keinen eigenen Stoffwechsel und lebten prinzipiell immer parasitisch. Das aber gelte auch für uns Menschen, beuteten wir doch die Natur aus wie es nur die schlimmsten Parasiten tun.

    Das ist ein typisches Beispiel für den Hawking‘schen Humor, den er nicht nur in diesem Buch verbreitet.

  522. Ich rate zum Nachdenken in Bescheidenheit!

    Dass sehr viele US-Amerikaner trotz ausgezählter Stimmergebnisse immer noch ernsthaft glauben, dass Trump die Präsidentenwahl gewonnen hat – sagt viel mehr über die menschliche Intelligenz, Dummheit bzw. Erkenntnisfähigkeit aus, als uns lieb sein dürfte.

  523. @Elektroniker (Zitat): “ So braucht man sich nicht zu wundern, dass auch im normalen Leben bei der Kommunikation von Menschen untereinander, viele „Geisterobjekte“ existieren, über deren Nützlichkeit und Zweckmäßigkeit man sich wundert, oder auch nicht. Manche könnten existentiell wichtig sein, andere scheinen entbehrlich.“
    In der Tat können uns Denkkonstrukte so wichtig erscheinen, dass wir glauben nicht darauf verzichten zu können.

    Bis uns dann jemand einen Weg zeigt wie wir auch ohne diese Hilfskonstrukte auskommen. Lukrez und vorher Epikur haben uns beispielsweise einen Weg gezeigt wie man ohne den Glauben an über- und aussernatürliche Dinge existieren kann. In diesem Fall taxierten sie bestimmte gedankliche Hilfskonstrukte als Täuschungen und falsche Weltsichten ein.
    In vielen andern Fällen wird ein Denkkonstrukt/Hilfskonstrukt einfach durch ein anderes ersetzt. Oft zählt ja vor allem das Resultat und es führen viele Wege nach Rom.

  524. @Martin Holzherr 24.01. 21:43 / 25.01. 12:17

    „Ich bin versucht, auch Fiktionen, die wir in unserem Geist erzeugen als etwas Emergentes zu betrachten, bin mir aber nicht ganz sicher wo die Fallstricke bei diesem Gedanken sind.“

    Ist ja auch etwas mehr als Emergenz. Ich würde es organisierte Emergenz nennen. Schon in der Zellchemie haben wir eine Organisation durch die DNA im Zellkern, deswegen nennen wir das ja auch Leben, abgegrenzt von der restlichen unbelebten Natur. Dort gibt es die einfache Emergenz, etwa wie im Zusammenspiel von Wind und Wasser sich Wellen auf dem Ozean entwickeln, die man selbst wiederum für sich mathematisch beschreiben kann.

    Da ist die Emergenz der Zellchemie einfach Kategorien komplexer, deshalb nennt man das ja auch Leben, und die Biologie als Wissenschaft ist auch ein eigenes Fachgebiet.

    Wenn wir jetzt Gedanken betrachten, so müssen wir feststellen, dass diese noch mal komplexer sind, als nur die einzelnen Nervenzellen. Ganze Zellverbände sind dann auch noch adaptiv den abzubildenden Informationen aus der Außenwelt gegenüber, die über die Sinne hereinkommen. Und die Ebene der Gedanken umfasst nochmal das Gesamtzusammenspiel des ganzen Gehirns, dessen konkrete Ausgestaltung und Verschaltung noch gar nicht erforscht ist.

    So wenig erforscht und bekannt, dass ich sogar auf die Idee komme, dass hier auch noch Geisteswelten im Spiel sein können, die der aktuellen Physik bisher entgangen sind, weil sie sich erst im Zusammenhang mit ganzen Gehirnen regelmäßig zeigen.

    „Eine wichtige Konsequenz hat der Glaube, dass auch der Mensch ein physikalisches Ding ist. Es bedeutet, dass kaum etwas exklusiv nur für den Menschen gilt. Der Mensch ist ein Objekt und ein Produkt dieser Welt und er gehorcht Gesetzen, die wir mit der Naturwissenschaft erforschen und mit der Technik nachbilden können.“

    Der Mensch ist eben ein Subjekt und kein Objekt. Er ist bestimmt von Zielen, weniger von Ursachen. Er hat eine eigene innere Repräsentation seiner unmittelbaren Außenwelt, die in sich selbst eine Welt ausmacht und einen Mesokosmos des Mentalen bildet. Hier gibt es entsprechend dann auch das Fachgebiet der Psychologie, die sich mit diesem Mesokosmos des Mentalen beschäftigt.

    Ich glaube, wir müssen hier zukünftiger Forschung nicht unbedingt vorgreifen. Noch haben wir kein Konnektom entschlüsselt. Wenn wir das mal haben, wird sich erst zeigen, ob unserer Subjektsein nicht sogar prinzipiell neue physikalischen Aspekte zeigt, und so noch zu physikalischen Neuentdeckungen führt.

  525. @Balanus / 24.01.2021, 11:25 Uhr / Zum Naturalismus

    »Ich frage mich, inwieweit ein ontischer Monismus überhaupt konstitutiv für den Naturalismus ist. Zumal es _den_ Naturalismus ohnehin nicht gibt. Und es scheint in der Tat so zu sein, dass heutzutage diese metaphysische Spielart in der Fachwelt kaum eine Rolle spielt.«

    Historisch gesehen sind seit mindestens dem 19. Jhdt diverse philos. Positionen als `naturalistisch’ bezeichnet worden, die in der Tat nicht alle miteinander verträglich unter einen Deckel passen. T.R. Baldwin (1993, Two Types of Naturalism) hat zwei Grundtypen als die gegenwärtige Diskussion dominierend ausgemacht, nämlich eine metaphysische (ontologische) und eine methodologische (epistemologische) Variante. Das kann man wohl so gelten lassen; Noam Chomsky orientiert sich beispielsweise an dieser Einteilung, von dem hab’ ich das jetzt auch so übernommen. Dabei ist die letztgenannte Variante im wesentlichen durch W.V.O. Quine repräsentiert, den sich Chomsky in seiner Naturalismus-Kritik vorknöpft.

    Quines Naturalismus ist andererseits auch Angriffsziel für metaphysische Naturalisten, wenn auch aus anderen Gründen als denen, die Chomsky anführt. Quine war insbesondere ein Feindbild von Bunge, der seinerseits mit dem, was er `Scientific Materialism’ genannt hat, für metaphysischen Naturalismus steht, und dessen Position einen idealtypischen Fall von ontischem Monismus darstellt.

    Über Mahner kann ich momentan nicht mehr sagen, als dass ich ihn quasi als Bunges Botschafter im Deutschen wahrgenommen habe, wo Scientific Materalism dann Ontologischer Naturalismus heisst, wodurch inhaltlich aber nichts verändert ist.

    25.01.2021, 11:56 Uhr

    Du musst für die Physik unterscheiden zwischen einem Phänomen, etwa einem Federpendel, und seiner theoretischen Modellierung, bei diesem Beispiel die math. Gleichung für den harmonischen Oszillator. Die sog. Naturgesetze sind typischerweise solche Gleichungen, die uns nicht etwa sagen, wie die “Natur” (was immer das ist) ein Phänomen hervorbringt, sondern nur, wie wir uns mit unseren theoretischen Begriffen in einer mehr oder minder formalen Sprache modellhaft vorstellen können, wie das in Betracht stehende Phänomen entsteht.

    Indem nun die theoret. Physik sich der Mathematik als Sprache bedient und diese Sprache im Gödelschen Sinne unvollständig ist, vererbt sich das an die Physik. Das Problem tritt auch in der Mathematik nicht auf, so lange man nicht reflexive Strukturen betrachtet wie insbesondere jene, die Hofstadter “Strange Loop” genannt hat. Die Grenzen der naturgesetzl. Erklärbarkeit in der Physik sind also dort zu erwarten, wo bei der Modellierung ein “Strange Loop” als strukturelles Merkmal auftritt. Und Hofstadter erwartet das eben konkret in Zusammenhang mit dem durch das Wörtchen “ich” ausgedrückten Selbstbezug, d.h. mit konkretem Bezug auf die Frage nach ich-Bewusstsein — “I am a Strange Loop“.

    Ich erwarte allerdings nicht, dass ich seine Message hier besser vemitteln kann, als Hofstadter es selbst vermag, und er hat ja selbst schon enttäuscht feststellen müssen, dass bei den meisten seiner Leser so gut nichts davon angekommen ist.

  526. Kategorien bestimmen wie wir über etwas denken und reden können.

    Wenn Bung&Mahner physikalische Energie als eine Eigenschaft bezeichnen, die alle materiellen/realen Dinge besitzen, so sagen sie damit was ein reales Ding ist. Es ist etwas, was Energie besitzt. Die Physik sagt uns nun, dass alle Veränderungen mit Energieaustausch einhergehen und dass materielle Objekte veränderbar sind.
    Damit wird nun der Satz sinnvoll: „Der Tee kühlt sich ab“. Nicht sinnvoll dagegen ist der Satz: „Die Zahl 13 kühlt sich ab“. Denn die Zahl 13 ist nicht veränderbar und hat damit keine Energie. Ohne die Eigenschaft Energie aber ist “13“ kein physikalisches Objekt.

    Was nun ist die philosophische Leistung von Bunge&Mahner?
    Nun, in meinen Augen ist es vor allem die, zu zeigen wie wir über Dinge reden können ohne inkompatibel mit den Erkenntnissen der Naturwissenschaften zu werden. Und zwar in der Alltagssprache, ohne Verwendung von Mathematik. Die Sprache der Physik ist zwar die Mathematik, aber man kann über physikalische Dinge auch in Alltagssprache sprechen, zumal die meisten unserer Erfahrungen im Kontakt mit der physikalischen Welt zustande kommen. Zudem zeigen Bunge&Mahner was der Zusammenhang zwischen realen Dingen und Fiktionen ist und wie man reale Dinge von Fiktionen unterscheiden kann.

    Wenn jemand beispielsweise glaubt Gott existiere, sei aber unveränderlich und besitze keine physikalische Energie, dann spricht er/sie von einer Fiktion.
    Zeus mit seinen vielen Kindern, Missetaten und Verstrickungen in irdische Dinge kommt dagegen eher als etwas recht Reales rüber.

  527. Martin Holzherr 25.01.2021, 12:48 Uhr

    All das ist doch Erkenntnis und Kategorisierung von Erkenntnissen und nicht Voraussetzung von Erkenntnis. Irgendwie muss ich auf dem falschen Dampfer sein.

  528. @Tobias Jeckenburger (Zitat): “ Der Mensch ist eben ein Subjekt und kein Objekt.“
    Antwort: Der Mensch ist gemäss ontologischem Naturalismus ein Objekt und gleichzeitig ein Subjekt sein kann. Naheliegend scheint mir es, ein Objekt, welches sich selbst als Teil der Welt erkennt, welches denken und handeln kann, als Subjekt zu bezeichnen.

    Kurzum: Ein Objekt, ein Ding, kann auch ein Subjekt sein. Und sogar: Ein Subjekt ist immer ein Ding, ein Objekt.

  529. @Timm Grams (Zitat): “ All das ist doch Erkenntnis und Kategorisierung von Erkenntnissen und nicht Voraussetzung von Erkenntnis. “
    Wenn sie in einer naturalistisch bestimmten Welt zu Erkenntnis kommen wollen, müssen sie den Regeln folgen in der man in der naturalistischen Welt zu Erkenntnis kommt.
    Beispiel: Um zu erforschen wie man hört oder Gedanken mitteilt, müssen sie davon ausgehen, dass diese Prozesse in einer materiellen Welt stattfinden. Sogar Denken findet in einer materiellen Welt statt mit der Folge, dass es etwa schnelle und langsame Denker gibt und dass die Lichtgeschwindigkeit sogar beim Denken eine Rolle spielt.

  530. Elektroniker
    Denken und Software,
    Der Vergleich ist richtig und der Gedanke von informellen Objekten geradezu genial. Erlaubt er doch etwas anzunehmen was nur temporär von Bedeutung ist, als Bezeichnung von nur Gedachtem , das aber eine (Gedanken)Lösung erleichtert.
    Gott ist z.B. kein Begriff, er ist nur eine Name für etwas, das außerhalb unseres Begreifens steht. Das wär dann ihr informelles Objekt.

  531. @ Krichard 25.01.2021, 13:25 Uhr

    Zitat: „Dass sehr viele US-Amerikaner trotz ausgezählter Stimmergebnisse immer noch ernsthaft glauben, dass Trump die Präsidentenwahl gewonnen hat – sagt viel mehr über die menschliche Intelligenz, Dummheit bzw. Erkenntnisfähigkeit aus, als uns lieb sein dürfte.“

    Da machen Sie es sich zu leicht. Wir wissen einfach nicht, wie weit bei den Briefwahlstimmen manipuliert wurde, eine auffällige Ungleichheit gegenüber den Ergebnissen in den Wahllokalen gibt es jedenfalls. Auch ist es eine Frage wo legale „Wahlhilfe“ endet und „Wahlbetrug“ anfängt.

    Ich sehe jedenfalls die Berichterstattung der Medien kritisch.

    Das erkläre ich an folgenden Beispielen.

    Zu Zeiten des Vietnamkrieges gab es bei uns, frei nach meiner Erinnerung folgende Meldungen in Zeitungen zum gleichen Ereignis.

    In der westlich orientierten Zeitung: Bei Angriffen gegen die Vietcongterroristen in Vietnam ist angeblich ein Gebäude leicht beschädigt worden. Dabei könnten es einige Opfer unter Jugendlichen gegeben haben.

    In der Kommunistischen Zeitung: Amerikanische Luftpiraten haben bei ihrem Bombenterror gegen die wehrlose Zivilbevölkerung in Vietnam einen Kindergarten hinterhältig überfallen und hunderte kleine Kinder grausam ermordet.

    Sachlich kann beides wahr sein …..

  532. @Timm Grams
    Ich meine, der ontologische Naturalismus hängt in der Luft, er ist untauglich, Wissen zu begründen.

    Der ontologische Naturalismus ist wissenschaftlich nicht notwendig. Es ist nicht seine Aufgabe, Wissen zu begründen. Er ergibt sich von ganz allein aus der Geschichte des Menschen und der Naturwissenschaft, indem es weder Beobachtungen noch logische Argumente gibt, die ihn widerlegen würden.

    Seit Jahrtausenden beobachten Menschen das natürliche Geschehen. Vieles erscheint unerklärlich oder als Wunder, allen voran das Leben. Die Naturwissenschaft hat dafür Kausalbeziehungen, strukturelle Regelmäßigkeiten und Abfolgen, sowie Entwicklungsprozesse gefunden, die solche Phänomene verstehen und zuverlässig erklären können.

    Diese Welt ist vollständig und systematisch aus einfachsten Bausteinen aufgebaut, soweit die Physik erkannt hat. Daher ist die Hypothese begründet, dass die Welt physikalisch geschlossen ist, dass es keine anderen Bausteine mit inkommensurablen Eigenschaften gibt, die dieses wissenschaftlich stabile Gebäude erschüttern könnten. Dass es noch nicht vollständig ist, stimmt natürlich, ist aber unerheblich, solange keine essentiellen Widersprüche auftreten.

    Die Naturwissenschaft ist ein endloser, fortlaufender Prozess mit Modifikationen und Korrekturen. Das ist keine Begründung für Zweifel oder Ablehnung. Sie führt allmählich zu einem einheitlichen Netzwerk von Theorien und Naturgesetzen, das dadurch immer dichter und zuverlässiger wird.

    Ob der ontologische Naturalismus als metaphysisch oder ideologisch bezeichnet wird, spielt keine Rolle für die Bedeutung des naturwissenschaftlichen Weltbildes. Dieses Weltbild bewährt sich seit Jahrtausenden für den Umgang des Menschen mit seiner Lebenswelt. Es gibt keine objektiven Hinweise oder Begründungen für die Annahme der Existenz gegensätzlicher Alternativen. Die Zweifler und die Gegner des Naturalismus müssen sich ihrerseits dieser Prüfung, der Kritik und dem Zweifel stellen!

    Kritik ist berechtigt, wo der ontologische Naturalismus zur Ideologie gemacht oder zur Wahrheit erhoben wird.

  533. @ hwied 25.01.2021, 14:36 Uhr

    Zitat: „… Der Vergleich ist richtig und der Gedanke von informellen Objekten geradezu genial. Erlaubt er doch etwas anzunehmen was nur temporär von Bedeutung ist, als Bezeichnung von nur Gedachtem , das aber eine (Gedanken)Lösung erleichtert.
    Gott ist z.B. kein Begriff, er ist nur eine Name für etwas, das außerhalb unseres Begreifens steht. Das wär dann ihr informelles Objekt.“

    Ersteres habe ich in meiner Informatik Weiterbildung (vor rund 50 Jahren) vom Informatiklehrer so erfahren. Praktisch alle meine Aussagen zur Philosophie und Logik stammen letztlich von ihm. Es ist normaler Lehrstoff in der etwas gehobenen Informatikausbildung.

    Dass „Gott“ genau in dieses „Denkschema“ passt, habe ich von einem Bischof, auch schon vor Jahrzehnten, ich glaube anlässlich seiner Pfingstpredigt, zufällig im Grenzstau im Autoradio gehört.

    Er hat, nach meiner Erinnerung, „einen transzendenten Gott“ sozusagen „mathematisiert“. Der „Bezeichner“ steht genau für alle real existierenden Sachverhalte der Weltentstehung und jeglicher Steuerung, alle Prozesse, jegliche Information …. alles ganz genau so wie es wirklich ist, eine nicht widerlegbare logische Tautologie.

  534. Martin Holzherr 25.01.2021, 14:31 Uhr

    “Um zu erforschen wie man hört oder Gedanken mitteilt, müssen sie davon ausgehen, dass diese Prozesse in einer materiellen Welt stattfinden.”

    Muss ich wirklich? Der ontologische Naturalismus ist übrigens eine viel stärkere Voraussetzung als die “einer materiellen Welt”. Deswegen musste er auch immer wieder einmal revidiert werden seit Lukrez.

  535. elektroniker,
    eine nicht widerlegbare Tautologie. Für einen Außenstehenden ist das logisch.
    Für einen Gläubigen wird der transzendente Gott zu einem persönlichen Gott zu dem er beten kann, mit dem er reden kann und er bekommt eine Antwort von ihm.
    Und wenn man noch in Betracht zieht, dass der Anteil der Gläubigen in der Welt in die Milliarden geht, dann kann man Glauben als Realität ansehen, der nicht erst bewiesen werden muss.

  536. @anton reutlinger 25.01.2021, 15:10 Uhr

    Weitgehende Zustimmung.

    Besonders Ihr letzter Satz beschreibt die Motivation meiner Aktivitäten gegenüber gewissen Naturalisten sehr gut: “Kritik ist berechtigt, wo der ontologische Naturalismus zur Ideologie gemacht oder zur Wahrheit erhoben wird.”

  537. Stephan Schleim, 24.01.2021, 18:44 Uhr

    Lieber Herr Schleim,

    vielleicht haben Sie in der Eile die wichtigen Vokabeln “vielleicht” und “wäre” (Konjunktiv II) übersehen, die die Unsicherheit meines Rekonstruktionsversuchs eigentlich hätten deutlich machen sollen. Eventuell interpretiere ich auch Ihre Wendung “in den Mund legen” falsch. Ich halte daher nur fest, dass ich Ihre religionsbezogene Aussage nicht rekonstruieren kann. Dabei könnte man es belassen.

    Ihre andere Aussage ist:

    “[D]ass sich aus Naturgesetzen mögliche und unmögliche zukünftige Ereignisse ableiten lassen […][, gilt] für wesentliche naturwissenschaftliche Theorien nicht”.

    Weiter oben war in diesem Zusammenhang von der Evolutionstheorie die Rede. Ich verwies diesbezüglich auf das Counter-Creationism Handbook. Ich sehe gerade, dass es das auch online gibt. Relevant ist in unserem Kontext die Kategorie CA200 (Theory of Science), die 13 Einwände enthält. (Ich behaupte *keineswegs*, dass sie kreationistische Thesen vertreten! Sie haben nur ein Argument benutzt, dass auch Kreationisten benutzen.)

    http://www.talkorigins.org/indexcc/list.html#CA

    Von Belang sind dort die Einwände:

    CA210. Evolution does not make predictions.
    CA211. Evolution can not be falsified.
    CA211.1. Karl Popper said Darwinism is not testable.

    Aus Gründen der Ökonomie verweise ich auf die Webseite und ergänze das noch um zwei Beispiele:

    Für SARS-CoV-2 wurden Immune-escape-Mutationen vorhergesagt, die nun leider in Südafrika und Brasilien auftauchten. Bei der Züchtung von Hunden wird das evolutionäre Prinzip Mutation und Selektion in höchstem Maße praktisch erfolgreich, wenn auch ethisch fragwürdig angewendet.

    Zum Wetter: Dies ist zwar ein chaotischer Prozess, kurzfristige Vorhersagen sind dennoch möglich und es ist sehr gut verstanden, warum langfristige Wettervorhersagen nicht möglich sind. Dennoch lassen sich leicht Wetterereignisse postulieren, die es aus physikalischen Gründen nicht geben darf. Es sieht aber auch so aus, als ob längerfristige Vorhersagen von Klimatendenzen möglich seien. Die Funktion von Treibhausgasen ist sehr gut belegt.

    Abgesehen davon gib es *zahlreiche* Effekte, die es nicht geben darf, wenn das Postulat der Naturgesetzlichkeit gültig ist. Das betrifft insbesondere auch die ganze Palette para-psychologischer Effekte wie Prägognition, Telekinese, Telepathie etc. Dass sich derartige Effekte bislang trotz gründlicher Suche nicht nachweisen ließen, neutralisiert keineswegs ihr Kritikpotenzial gegenüber dem Naturgesetzlichkeitspostulat.

    Wie üblich ist alles umstritten. Man kann aber die verschiedenen Konzeptionen vergleichend evaluieren. http://www.talkorigins.org weist diesbezüglich in die richtige Richtung. Ansonsten gilt bislang der Irrtumsvorbehalt.

  538. Der erbitterte Kampf zwischen Naturalisten und Antinaturalisten ist eigentlich unnötig. Aber wie bei Evolutionstheorie und Kreationismus gibt es auf beiden Seiten erbitterte Missionare um das “richtige Denken”. Es hat wohl psychologische Gründe, die auch bei Xenophobie und Homophobie zu beobachten sind. Man fühlt sich wohler und sicherer im eigenen Milieu der Ideen und Überzeugungen. Es erspart das Umdenken, das Zweifeln, die Diskussionen, die Last des Lernens und Verstehens.

    Die Naturwissenschaft hat große Erfolge zu verzeichnen, deren Anwendungen wir alle täglich nutzen, gerade auch jetzt in der Pandemie und allgemein in der Medizin, nicht nur in der Technik zur Erleichterung des Lebens. Es wäre wichtig, auch über ein weltweit humanes Zusammenleben nachzudenken. Dazu bedarf es keiner transzendenten Wesenheiten, die objektiv nicht erfahrbar sind und höchst wahrscheinlich nur fiktiv existieren. Gerade deswegen führen ihre Gläubigen unter sich einen erbitterten Kampf um Vorherrschaft, weil sie die Konkurrenz um die angebliche Wahrheit ihrer Einbildungen nicht ertragen.

  539. @Feodor: Vorhersage & Parapsychologie

    Danke für die Verweise. Die Artikel machen mehr oder weniger die Vagheit Ihres Arguments deutlich. Und inwiefern braucht man nun wirklich eine Evolutionstheorie (etwa im Sinne der Synthetischen Theorie), um Resistenzenbildung durch Antibiotika “vorherzusagen”? Wenn ich etwa beim Würfeln (Analogie zu: Zufallsprozesse bei Mutationen) alle geraden Zahlen heraushole (Analogie zu: Abtötung durch Antibiotikum), dann ist es schlicht trivial, dass nur ungerade Zahlen übrig bleiben, falls überhaupt noch gewürfelt wird.

    Ich stimme mit Ihnen überein, dass es keine harten Belege für parapsychologische Phänomene gibt. (Bei uns am Institut wurde und wird das übrigens erforscht.) Aber Sie müssten erst einmal zeigen, welches Naturgesetz das konkret ausschließt.

    Denken wir uns einen Radiologen, der aufgrund einer Abbildung (etwa: Röntgenaufnahme) die Zukunft vorhersagen will (etwa: ob ein Tumor gefährlich ist und operiert werden muss). Nun gibt es solche Vorhersagen mit überzufälliger Wahrscheinlichkeit. Wir nennen dieses Phänomen in der Psychologie bzw. biomedizinischen Forschung nun “Prognose” oder “Expertenwissen” und versuchen, es wissenschaftlich zu erklären (was übrigens nur ansatzweise gelingt).

    Was zeigt uns das Beispiel? Aus welchem Naturgesetz hätte man jetzt ableiten sollen, dass es etwas wie Expertenwissen, ein Phänomen unserer Welt, geben muss? Oder welches Gesetz sollte zeigen, dass andere Vorhersagen unmöglich sind? Sobald das Phänomen wiederholt auftritt, versucht man, es wissenschaftlich zu erklären. So funktioniert eben Wissenschaft.

    Ihr Hinweis darauf, dass Kreationisten ein ähnliches Argument verwenden, ist für die Diskussion völlig irrelevant und bestätigt einmal mehr meinen Verdacht, dass Sie hier vor allem ideologisch unterwegs sind. Hitler war Vegetarier. Ja und? Ist Vegetarismus darum ethisch schlechter? So führt man keine Diskussionen, sondern Meinungskampf/Propaganda, in dem man andere Anschwärzt und aufgrund einer konstruierten Nähe zu Problemgruppen mundtot machen will. Für sowas ist bei MENSCHEN-BILDER kein Platz!

  540. Naturgesetze als Gesetzmäßigkeiten von Ereignissen in der Natur machen Prognosen möglich. Die meisten Alltagsphänomene sind jedoch so komplex, dass sie mehreren überlagernden Naturgesetzen unterliegen und Prognosen daher nur eingeschränkt möglich sind, oder nur statistisch bzw. probabilistisch vorhersagbar sind. Für bestimmte Prozesse und begrenzte Zeiträume können heuristische Prognosen möglich sein, z.B. für Krankheitsverläufe oder für das Wetter, auf Grund von Wissen und Erfahrung. Für parapsychologische, spirituelle oder ähnliche Phänomene sind Prognosen definitionsgemäß nicht möglich. Auch für Träume sind Prognosen nicht möglich, was aber nicht bedeutet, dass sie parapsychologischer Natur wären!

  541. @Chrys // 25.01.2021, 14:06 Uhr

    »… eine metaphysische (ontologische) und eine methodologische (epistemologische) Variante. […] Noam Chomsky orientiert sich beispielsweise an dieser Einteilung, von dem hab’ ich das jetzt auch so übernommen. Dabei ist die letztgenannte Variante im wesentlichen durch W.V.O. Quine repräsentiert, «

    Stimmt diese Reihenfolge?

    Ich habe gelesen, dass Chomsky selbst einen methodologischen Naturalismus vertritt—bzw. ihn zumindest für ganz brauchbar hält.

    Vielleicht gibt es ja noch einige Feinheiten, die ich jetzt nicht so im Blick habe… (etwa das Epistemologische).

    Bei der Erwähnung Hofstaders habe ich zwar spontan an dessen „seltsame Schleifen“ gedacht, war mir aber nicht sicher, ob Du darauf anspieltest. Zumal es hier im Blog ja auch um das Hirn-Geist-Problem geht, insbesondere, worauf „Denken“ beruht.

    Was das angeht, da scheinen Hofstader keine großen Zweifel zu plagen, was für was ursächlich ist (S. 361):

    Bis jetzt hat dieser Ansatz [die Hirnforschung ist gemeint, B.] noch nicht viel Licht darauf geworfen, wie das Gehirn mit Begriffen umgeht, aber es gibt uns eine gewisse Vorstellung von den biologischen Mechanismen, auf denen die Manipulation des Denkens beruht.

    Und weiter (S. 729):

    Sowohl Maschinen wie auch Menschen bestehen aus Hardware, die nach den physikalischen Gesetzen von allein abläuft.

    Wer wollte da widersprechen?

    »…Grenzen der naturgesetzl. Erklärbarkeit in der Physik…Selbstbezug…«

    Das sehe ich soweit ein. Allerdings scheint mir diese „Erklärbarkeit“ nicht ganz deckungsgleich zu sein mit der prinzipiellen wissenschaftlichen Erklärbarkeit von Naturvorgängen, die im Rahmen empirischer Forschung angenommen oder auch behauptet wird.

  542. @Timm Grams

    Ich meine, der ontologische Naturalismus hängt in der Luft, er ist untauglich, Wissen zu begründen.

    Ja, das ist eine treffende Charakterisierung des Naturalismus. Aber es ist keine abwertende Kritik speziell an ihm. In der Luft hängt doch jeder ‘-ismus’ der Philosophie, und keiner kann Wissen begründen. Oder wissen Sie mehr?

    Ignorabimus, wenn es doch nur das gewesen wäre, was Sie uns sagen wollten. Nur ihr Anspruch war ja ein anderer. Sie wollten in ihrem Text einen inneren Widerspruch in den Annahmen des Naturalismus aufzeigen.

    Er [der Naturalist] scheitert schon auf der Ebene der Logik

    Das zu zeigen, da sind doch eher Sie gescheitert. Zeit für ein Fazit.

    Ihre Auslegung von “außerhalb” und “unabhängig” entsprechen nicht der in den Zitaten von Mahner und Vollmer intendierten. Da darf ich Sie selbst zitieren:

    Sie ändern die Prämissen und kommen, was kein Wunder ist, zu anderen Schlussfolgerungen […]. Daraus kann kein Widerspruch erwachsen.

    Beim Bieri-Trilemma vermuten Sie, ein Naturalist müsse These 2 ablehnen. Damit stehen Sie allein auf weiter Flur. Es handelt sich um einen Folgefehler aus ihren anfänglichen Annahmen.

    Ihre Kritik an Schmidt-Salomon halten sie zwar für psychologisch nachvollziehbar, er dürfe sich nicht wundern über deren Art und Weise, allerdings dass sie logisch nicht gerechtfertigt ist, was ich begründet hatte, dem haben Sie nie widersprochen.

    Danke, dass Sie sich der Diskussion gestellt haben.

    Alles wie es war:

    Das Bewusstsein bleibt rätselhaft.

  543. Stephan Schleim, 25.01.2021, 17:51 Uhr

    Lieber Herr Schleim,

    Ich schrieb:

    “Ich behaupte *keineswegs*, dass sie kreationistische Thesen vertreten!
    Sie haben nur ein Argument benutzt, dass auch Kreationisten benutzen.”

    Das sollte eigentlich *verhindern*, dass Sie sich ärgern, nachdem ich Texte von der Counter-Creationism-Site zitierte, die Fragen behandeln, die Sie gestellt hatten. Ich hatte hierbei nämlich (wie Sie auch) im Sinn, dass es “für die Diskussion völlig irrelevant” ist, “dass Kreationisten ein ähnliches Argument verwenden” (Ihre Formulierungen)! Es ist mir ein Rätsel, wie der Disclaimer als “Anschwärzen” interpretiert werden konnte.

    Vielleicht hat im 2. Satz auch nur die Vokabel “zufällig” gefehlt? Nochmal die Aussagen:

    “Ich behaupte *keineswegs*, dass sie kreationistische Thesen vertreten!
    Sie haben nur [zufällig] ein Argument benutzt, dass auch Kreationisten benutzen.”

    Es sei zur Sicherheit redundant hinzugefügt: Das sollte eine präventive Entschuldigung, d.h. eine Höflichkeitsgeste sein. Weder sollte mundtot gemacht werden, noch ängeschwärzt oder ethisch in Frage gestellt werden. Statt einer “Nähe zu Problemgruppen” war das *genaue Gegenteil* gemeint! Schlechter kann Kommunikation wohl kaum gelingen. Meine obigen Anmerkungen zu Ethik sollten nur die Annahme plausibilisieren, dass sich nicht ohne weiteres Gruppen ausmachen lassen, die ein im Mittel ethisch überlegeneres Verhalten praktizieren.

    Nun bin ich gespannt, ob ich mich verständlich machen konnte.

    Was die Bemerkungen zu meiner Person angeht, verzichte ich ganz bewusst auf eine scharfe Gegenreaktion, weil ich das nicht für hilfreich halte. Ich bin kritischer Rationalist, da mir kein Verfahren der (Letzt)Begründung bekannt ist. Begründungen hängen in der Luft, wie kürzlich hier formuliert wurde. Bewährung und Kritik ist hingegen möglich. Ich sammle daher Kritik. Aussagen über die Welt müssen sich an ihrer Kritik bewähren. Daher war und bin ich daran interessiert, welche Kritik an der Bewährung und der Kritisierbarkeit der Naturgesetzlichkeitsannahme geübt wird.

    Mich wundert die *heftig scheinende* Ablehnung speziell der (vorläufigen hypothetischen Annahme der) Naturgesetzlichkeit. Oben wird öfters ein Materialismus vertreten (was ich *nicht* kritisiere!), der eher noch stärkere Annahmen macht. Die interessanten Fragen sind ja: Wie ist Wissenschaft ohne Naturgesetzlichkeit möglich? Welche Wissenschaft kommt ohne Naturgesetzlichkeit aus und warum?

    Aus gegebenem Anlass verweise ich auch auf meinen Prolog:
    https://feodor.de/node/13

  544. Geist-Materie—Bewusstsein Denkansatz 9

    Wer Kleinkinder hat, der weiß es, kleine Kinder entwickeln eine Privatsprache. Nur ihre Geschwister können sie verstehen, und sogar die Mutter muss nachfragen, was hat der Christian gesagt ?

    Was er akustisch gesagt hat, das weiß die Mutter, aber sie weiß nicht, was es bedeutet.
    Jetzt stellen wir uns ein Schreckensszenario vor, dieses Kleinkind und eines der Geschwister sind die Einzigen Überlebenden einer Naturkatastrophe. Und da es auf der Insel genug Nahrung gibt, überleben beide.

    Zwanzig Jahre später findet man sie auf der Insel. Sie haben es sich zurecht gemacht und auf einer Felswand findet man seltsame Zeichen. Und sie sprechen eine unverständliche Sprache und können sich nur über Gesten mitteilen.

    Leider hat die Schiffsbesatzung auch eine Infektionskrankheit eingeschleppt, an der die beiden sterben. An den Schriftzeichen haben sich die größten Gelehrten die Köpfe zerbrochen, man konnte nicht herausfinden, was sie bedeuten.

    Warum nicht, weil die Bedeutung der Schriftzeichen der Kategorie Geist zuzuordnen ist. Die Schriftzeichen sind materiell, ihre Bedeutung ist geistiger Natur.

    Wie die Wissenschaftler so sind, einer kam auf die aberwitzige Idee, die Schriftzeichen durch einen Versuch herauszufinden. Er kaufte sich in einem Land, dessen Name hier verschwiegen wird zwei Kleinkinder und zog mit ihnen auf die Insel. Er überließ die Kinder sich selbst, (bitte jetzt keine bösen Zuschriften) und beobachtete was die Kinder machten.
    Sie malten die Zeichnungen an der Wand ab und ……sie malten neue Zeichen dazu.
    „Im Bewusstsein der bestehenden Zeichen(Anmerkung des Verfassers)- Und als der Wissenschaftler seine Kinder fragte, was das heißt, dann erklärten sie ihm die Bedeutung.

    Mehr sage ich nicht.

  545. @feodor: Diskussion

    Sie haben mir erst unterstellt, ich würde keine Kritik an Religionen zulassen (das Gegenteil ist wahr); danach brachten Sie meine Argumentation ohne Notwendigkeit in Zusammenhang zu Kreationisten.

    Wenn Sie sich als kritischen Rationalisten sehen (ich mich übrigens auch), dann konzentrieren Sie sich halt mal auf die Sache.

    Hierzu ist alles gesagt. Ich arbeite sowieso am nächsten Artikel.

    P.S. Das Thema könnte Sie interessieren. Versuchen Sie es gerne noch einmal, dann besser ohne Unterstellungen.

  546. Für wen ist der ontologische Naturalismus?

    Physiker und andere Naturwissenschaftler brauchen den ontologischen Naturalismus nicht, denn sie brauchen keine Welterklärung, die Begriffe aus der Philosophie verwendet. Naturwissenschaft kam schon immer ohne Philosophie aus, denn Naturwissenschaft beschäftigt sich immer mit konkreten Problemstellungen und nicht mit der Welterkenntnis an und für sich wie das die Philosophie macht.

    Viele Physiker, sogar heutige, die sich im Jahr 2020 ausbilden lassen, orientieren sich in ihrer Einstellung zur Welt eher an Texten von Richard Feynman, denn Feynman ist nicht nur ein hochrangiger Physiker, sondern er hat sich auch intensiv mit dem wissenschaftlichen Erkenntnisprozess auseinandergesetzt – das aber immer auf der Basis seiner eigenen Erfahrungen als Physiker (und Hobby-Biologe). The Feynman Principles geben einen Eindruck in Feynmans erkenntnistheoretische Einsichten.

    Ein Buch wie „Über die Natur der Dinge“ von Mario Bunge und Martin Mahner ist für Leute, die etwa bereits mit in der Philosophie gebräuchlichen Begriffen vertraut sind und die wissen wollen, wie die in der Naturwissenschaft dominierende naturalistische/materialistische Weltsicht in eine Gesamtsicht der Welt münden kann und wie man das in philosophische Begriffe und Denkschablonen fassen muss. Ich selbst hatte im Gymnasium Philosophieunterricht, lernte Plato, Aristoteles, die klassische Logik, Popper, Kant und vielleicht noch als Exoten Maturana und später die Konstruktivisten kennen. Doch mir schien all das was ich da gelernt hatte, inkompatibel mit dem, was man in einer naturwissenschaftlich/technisch orientierten Welt über die Welt denkt.

    Erst das Buch „Über die Welt der Dinge“ überwand diesen Graben zwischen der Weltsicht, die sich einem Naturwissenschaftler anbietet und der Begriffswelt der Philosophie. Ich habe das Buch übrigens vor etwa 10 Jahren auch meinem Sohn zum Lesen gegeben und der war ganz empört darüber: das seien ja lauter Trivialitäten, die da stünden. Niemand könne irgend etwas davon bezweifeln. Nun vielleicht liegt das daran, dass er Physik studiert hat, vielleicht aber auch daran, dass wir heute in einer anderen Zeit leben.

    Und ja, es scheint auch heute noch Leute zu geben, die Aussagen aus „Über die Natur der Dinge“ bezweifeln – Leute wie Timm Grams.

  547. @Martin Holzherr 26.01.2021, 08:57 Uhr

    Und ja, es scheint auch heute noch Leute zu geben, die Aussagen aus „Über die Natur der Dinge“ bezweifeln – Leute wie Timm Grams.

    Um die Rolle einer
    Voraussetzungsmetaphysik ging es. Zu den wissenschaftlichden Aussagen habe ich mich nicht geäußert, soweit ich mich erinnere.

  548. @hwied
    Mehr sage ich nicht.

    Gut so, denn Sie sind auf dem Holzweg. Die Bedeutung von Zeichen ist subjektiv oder konventional. Dieselben Zeichen, die wir auf dem Bildschirm sehen, können von jedem anders gedeutet werden. Es gibt keine Möglichkeit zu erkennen, ob die Kinder die Zeichen im Sinne der Erzeuger gedeutet haben. Mit dem Geist bzw. Gehirn der Zeichenerzeuger ist auch die Bedeutung der Zeichen im Sinne ihrer Erzeuger futsch. Man kann dann nur raten, so wie die Archäologen oder Historiker.

    Ihre Behauptungen sind in keiner Weise ein Argument für einen unabhängigen Geist. Sie verkennen immer wieder, dass alles was wir denken neurophysiologische Vorgänge im Gehirn sind. Da gibt es keine Ausnahme. Zusätzlich werden im Gehirn (Hypophyse) Hormone produziert und sezerniert, was wir dann als Gefühle deuten. Das Gehirn bekommt Signale aus der Außenwelt wie auch aus allen Körperteilen (außer dem Gehirn selber).

    Geben Sie es auf!

  549. @Joker 26.01.2021, 00:31 Uhr

    Das ist Ihr Blick auf einen ziemlich komplexen Diskussionsverlauf. Ich halte einige Ihrer Anmerkungen für unangemessen. Nur ein Beispiel. Sie schreiben:

    Beim Bieri-Trilemma vermuten Sie, ein Naturalist müsse These 2 ablehnen. Damit stehen Sie allein auf weiter Flur.

    In Antworten an Stephan Schleim (19.01.2021, 12:10 Uhr) und Sie (19.01.2021, 10:37 Uhr) habe ich meine Aussage präzisiert, da sie wohl nicht für alle Naturalisten gilt. Ich habe also dazugelernt.

    Meine Beurteilung fand hier durchaus auch Zustimmung. Das müssen Sie übersehen haben.

    Zu meinem Urteil bin ich im Laufe einer Diskussion mit einem dualistisch gesonnenen Naturalisten gekommen. Seine Auffassung finden Sie in der Diskussion zum selben Thema im Hoppla!-Blog wieder.

    Interpretationen eines Diskussionsverlaufs zum Besten zu geben, führt doch nur zu endlosen Wiederholungen. Wir sollten das unterlassen.

  550. @Balanus / 25.01.2021, 21:41 Uhr

    »Stimmt diese Reihenfolge?«

    Du meinst, ich hätte das “methodologisch” falsch zugeordnet? Das will ich insofern nicht ausschliessen, als ich das hier nur aus dem Gedächtnis rekapituliert hatte (momentan komme ich an meine digitale Sammlung philosoph. Texte nicht heran, technisch bedingt). Inwiefern Baldwins Typisierung als allgemein akzeptiert gelten kann, weiss ich im zudem auch nicht; andere Autoren mögen da andere Bezeichnungen und eigene Schemata der Einteilung verwenden.

    Noch zur “Erklärbarkeit”. Was in einem Modell wahr ist, sollte mit dem Phänomen zumindest observationell kompatibel sein, andernfalls wäre das Modell falsifiziert und als unbrauchbar zu verwerfen. Was dann für das Phänomen nomologisch (naturgesetzlich) erklärbar ist, entspricht im Modell dem, was dort formal beweisbar ist. Unvollständigkeit bedeutet, dass nicht alles, was im Modell wahr ist, dort auch beweisbar ist.

    Während in der Wissenschaftstheorie die Konsequenzen von Gödel längst angekommen sind — namentlich bei Stegmüller findet man viel dazu — ist das an gemeinen Naturforschern weitgehend komplett vorbeigerauscht. Klar, die haben solche Probleme normalerweise auch nicht, aber wenn sie etwa bei Hirnsimulationen beteiligt sind, dann kommen da Weisheiten heraus wie “Das Gehirn ist ein Objekt endlicher Grösse, man kann alles darüber rauskriegen” (Markus Diessmann). Unfortunately, size does not always matter…

  551. @Joker / 26.01.2021, 00:31 Uhr

    Der Bruch mit der modernen Logik ist von Bunge ja erklärtermassen vollzogen worden, indem er unverdrossen gegen den Existenzquantor gefochten hat wie Don Quixote gegen die Windmühle. Da muss man jemandem wie Bunge gar nichts erst nachweisen, das hat er schon selber besorgt: Frege, Russell, Tarski — alles nur “logische Imperialisten”, die nix kapiert haben . So geht’s halt zu im “Scientific Materialsm”…

  552. Ich möchte versuchen, das was hwied meint, aus technischer Sicht zu erklären.

    Es geht um die „Synchronisierung“ von Prozessen, die besonders in der Elektronik, aber wie ich vermute auch in der Biologie, von Bedeutung sind. Das möchte ich aus originellen Beispielen aus dem Leben erläutern.

    Vor vielen Jahren ist auf meinem (längeren) Schulweg immer an einer Straßenkreuzung, jeden Tag ein hübsches Mädchen, ungefähr 100 m vor mir, auf meinem Schulweg abgebogen.

    Das gab natürlich Stoff für „philosophisches Anbandeln“. Die Erklärung war später ganz einfach.

    Wir hörten beide eine damals bei den Jugendlichen extrem beliebte, rund 5 Minuten dauernde Sendung im Radio. Danach drehten viele Tausende Schüler das Radio ab und machten sich auf dem Schulweg mit, physikalisch gesehen, immer der gleichen Geschwindigkeit auf. Dann ist das Zusammentreffen pure Physik, die Sendung im Radio hat das Geschehen „synchronisiert“.

    Aber es gibt auch andere originelle Muster für “synchronisierende Muster”.

    Beispiel eine Busfahrt in Griechenland: Dort standen nach unübersichtlichen Kurven so etwas wie „Bildstöcke“ in der Landschaft. Einer Frau im Bus ist dieses „Muster“ frühzeitig aufgefallen und sie hat, obwohl sie noch nie dort war, zum Gelächter der im Bus mitfahrenden Reisenden, immer „vorhergesagt“, wann wieder so ein Bildstock zu erwarten ist. Hat fast immer gestimmt.

    Sie hat einfach instinktiv das Muster wie die Bildstöcke angeordnet sind, erkannt. Es war keine Wiedergeburt, was manche im Bus und auch die „hellseherische Frau“ selbst vermuteten, kein Hellsehen, keine heimlichen Botschaften…… Es war pure Musterverarbeitung.

    Hwied geht offenbar davon aus, dass „gleichartige Kinder“ die aus ihrem natürlichen Instinkt heraus die ursprünglichen Muster geschaffen haben, auch besser imstande sind diese Muster nachzuvollziehen und weiterentwickeln zu können, weil sie auf annähernd dem gleichen Entwicklungsstatus scheinen.

    Es sind abstrakte „Muster“ ohne Materie, die allerdings auf Materie „abgebildet“ sind. Eine „reife Leistung“ für die Psychologie wäre es schon, könnte man aus den Kindern wie es hwied meint, tatsächlich Informationen „herauslocken“.

  553. Ergänzte Kopie des vorherigen Beitrages.
    @ anton reutlinger 26.01.2021, 10:33 Uhr

    Ich möchte versuchen, das was hwied meint, aus technischer Sicht zu erklären.

    Es geht um die „Synchronisierung“ von Prozessen, die besonders in der Elektronik, aber wie ich vermute auch in der Biologie, von Bedeutung sind. Das möchte ich aus originellen Beispielen aus dem Leben erläutern.

    Vor vielen Jahren ist auf meinem (längeren) Schulweg immer an einer Straßenkreuzung, jeden Tag ein hübsches Mädchen, ungefähr 100 m vor mir, auf meinem Schulweg abgebogen.

    Das gab natürlich Stoff für „philosophisches Anbandeln“. Die Erklärung war später ganz einfach.

    Wir hörten beide eine damals bei den Jugendlichen extrem beliebte, rund 5 Minuten dauernde Sendung im Radio. Danach drehten viele Tausende Schüler das Radio ab und machten sich auf dem Schulweg mit, physikalisch gesehen, immer der gleichen Geschwindigkeit auf. Dann ist das Zusammentreffen pure Physik, die Sendung im Radio hat das Geschehen „synchronisiert“.

    Aber es gibt auch andere originelle Muster für “synchronisierende Muster”.

    Beispiel eine Busfahrt in Griechenland: Dort standen nach unübersichtlichen Kurven so etwas wie „Bildstöcke“ in der Landschaft. Einer Frau im Bus ist dieses „Muster“ frühzeitig aufgefallen und sie hat, obwohl sie noch nie dort war, zum Gelächter der im Bus mitfahrenden Reisenden, immer „vorhergesagt“, wann wieder so ein Bildstock zu erwarten ist. Hat fast immer gestimmt.

    Sie hat einfach instinktiv das Muster wie die Bildstöcke angeordnet sind, erkannt. Es war keine Wiedergeburt, was manche im Bus und auch die „hellseherische Frau“ selbst vermuteten, kein Hellsehen, keine heimlichen Botschaften…… Es war pure Musterverarbeitung.

    Hwied geht offenbar davon aus, dass „gleichartige Kinder“ die aus ihrem natürlichen Instinkt heraus die ursprünglichen Muster geschaffen haben, auch besser imstande sind diese Muster nachzuvollziehen und weiterentwickeln zu können, weil sie auf annähernd dem gleichen Entwicklungsstatus scheinen.

    Es sind abstrakte „Muster“ ohne Materie, die allerdings auf Materie „abgebildet“ sind. Eine „reife Leistung“ für die Psychologie wäre es schon, könnte man aus den Kindern wie es hwied meint, tatsächlich Informationen „herauslocken“.

  554. @Timm Grams (26.01.2021, 10:59 Uhr)

    Interpretationen eines Diskussionsverlaufs zum Besten zu geben, führt doch nur zu endlosen Wiederholungen. Wir sollten das unterlassen.

    Mir scheint es gerade in etwas verfahrenen Diskussionen wichtig zu sein, zwischenzeitlich Pausen einzulegen und das bis dahin gemeinsam Erarbeitete festzuhalten. So lassen sich noch bestehende Unterschiede erkennen, die man dann akzeptieren und stehen lassen kann, oder, wenn noch Interesse besteht, sich im Detail noch einmal anschaut.

    Meine Beurteilung [ein Naturalist müsse These 2 ablehnen] fand hier durchaus auch Zustimmung. Das müssen Sie übersehen haben.

    Oder Sie bilden sich das nur ein.

    Können Sie mir sagen, von wem da Zustimmung gekommen sein soll, oder wenigstens in welchem Zusammenhang? Ich habe nochmal Quergelesen (nicht Quergedacht) und nichts gefunden. Zumindest nichts, was ihre Allaussage unterstützt. Mit ‘der’ oder ‘ein’ Naturalist meinten sie zumindest im Text ja noch, jeder Naturalist.

    ich [habe] meine Aussage präzisiert, da sie wohl nicht für alle Naturalisten gilt. Ich habe also dazugelernt.

    Falls das bedeutet, Sie halten jetzt die von Mahner, Vollmer und Schmidt-Salomon vertretenen Ansichten nicht mehr für Selbstwidersprüchlich, fände ich das beruhigend.

    Dass Sie gezeigt haben könnten, eine bestimmte Spielart des Naturalismus sei selbstwidersprüchlich, will ich nicht ausschließen. Um den Vorwurf zu entgehen, Sie würden da nur einen Strohmann abfackeln, sollten Sie besser noch einen denkenden Vertreter dieser Spielart nennen.

  555. @ Chrys, 26.01.2021, 13:30 Uhr

    Der Bruch mit der modernen Logik ist von Bunge ja erklärtermassen vollzogen worden, indem er unverdrossen gegen den Existenzquantor gefochten hat wie Don Quixote gegen die Windmühle. Da muss man jemandem wie Bunge gar nichts erst nachweisen, das hat er schon selber besorgt: Frege, Russell, Tarski — alles nur “logische Imperialisten”, die nix kapiert haben . So geht’s halt zu im “Scientific Materialsm”…

    Das ist erläuterungsbedürftig. Bunge verwendet den Existenzquantor, unterscheidet aber konzeptionelle und reale Existenz. An welcher Literaturstelle geht es um Tarski? Semantics II, S.101ff? Bunge widerspricht dort einer allgemeinen platonistschen Wahrheitstheorie. Abstakte (mathematische) Wahrheit wird von realistischer Wahrheit unterschieden.

    Ich würde das nur gerne nachvollziehen können.

  556. @Joker 26.01.2021, 14:41 Uhr

    Der Skeptiker ist kein Rudeltier. Kampfbündnisse sind nicht mein Ding. Deshalb möchte ich auch niemanden für meine Ansichten in die Pflicht nehmen. In diesem Gesprächsfaden taucht aber tatsächlich die Bemerkung auf, dass meine Auffassung in dem angesprochenen Fall nicht hinreichend gewürdigt worden sei.

    Meine Kritik am Naturalismus betrifft mehrere Punkte. Der wesentlichste ist der Anspruch, eine Voraussetzungsmetaphysik zu sein. Das verpflichtet mich aber nicht, alles abzulehnen. Viel von dem, was beispielsweise Mahner schreibt, findet auch meine Zustimmung. Das muss aber nicht eigens erwähnt werden.

    In manchen Punkten besteht eben Klärungsbedarf. Dafür sind doch Foren wie dieses hier da.

  557. Aufräumarbeit
    Die mir bekannten Naturalisten kennen die Drei-Welten-Lehre des Karl Raimund Popper. Welt 1 beinhaltet das Materielle und wird in meiner Tabelle durch die obere, dunkle Hälfte beschrieben. Welt 2 beherbergt das Geistige, in meiner Tabelle die untere, helle Hälfte. Dazu kommt bei Popper die Welt 3 der kulturellen Erzeugnisse. Für mich ist die Welt 3 nur ein kollektiver Speicher zu Welt 2, der in Welt 1 realisiert ist.
    Wir habe es so gesehen mit dem klassischen Dualismus zu tun, und stehen damit vor der Frage, wie Welt 1 und Welt 2 zusammenhängen. Für Popper gibt es Wirkungen in beide Richtungen; er behauptet, „ungefähr wie ein naiver Realist, es gebe physikalische Welten und eine Welt der Bewusstseinszustände, die aufeinander wirken“ (Objektive Erkenntnis, 1973, S. 124).
    Die Rede, dass die Realität außerhalb unseres Denkens angesiedelt und in ihrer Existenz und ihren Eigenschaften unabhängig von unserem Bewusstsein sei, lässt die Vermutung zu, dass es so etwas wie die Welt 2 geben müsse, wenn auch – anders als bei Popper – die Wirkung nur von Welt 1 hin zur Welt 2 gerichtet ist. Dieser Eindruck wird bestärkt durch den Hinweis auf die Illusionen, die der Körper für uns bereithält.
    Daraus sehe ich, dass Naturalisten durchaus an eine Welt 2 denken und dass diese keinerlei Auswirkungen auf die physikalische Welt 1 hat. Besonders nachdrücklich wird dieser Standpunkt von Rainer Wolf vertreten. Dabei kann ich ihm sogar folgen: Wenn für ihn die Welt 2 nur die Bilder enthält, die von der Welt 1 produziert werden, dann ist eine Rückwirkung der Bilder auf den Maler ausgeschlossen. Aber er hat es mit zwei Welten zu tun, von der nur eine real ist. Ich halte es für eine berechtigte Frage, was mit der anderen ist. So gesehen muss der „schein-dualistische“ Naturalist die zweite Aussage des Trilemmas von Bieri verneinen.
    Andererseits legen die Naturalisten Wert darauf, ein monistisches Weltbild zu vertreten; sie werden deshalb mit der ersten Aussage nichts anfangen können.
    Der Naturalist sieht dieses Problem und modifiziert seine Ausgangsstatements. Martin Mahner wird die Welt 2 mit seinem Fiktionalismus los und Gerhard Vollmer bringt zu diesem Zweck die Identitätstheorie ins Spiel. Die Welt 2 wird in beiden Fällen sozusagen in die Welt 1 hineingeklappt. Beide Lösungen stehen im Widerspruch zu der anfänglichen Charakterisierung des Realismus, nach der die Realität unabhängig von unserem Bewusstsein sei. Man hätte diese Charakterisierung als erste Näherungen und als vorläufig kennzeichnen können; dann hätte sich eine Debatte darüber möglicherweise erübrigt.

  558. Elektroniker,
    Kompliment, besser hätte ich meine Geschichte nicht erklären können.

    Reutlinger,
    Sie wollen einen Beweis, sie bekommen einen Beweis. Geduld, bis morgen.

  559. @Stephan Schleim, 26.01.2021, 08:30 Uhr:

    Um Missverständnissen vorzubeugen, möchte ich festhalten, dass ein Dissens hinsichtlich der Interpretation der Fehlkommunikation bestehen bleibt. Auch nach genauer Reanalyse der Texte und ihrer Reihenfolge kann ich den Anschuldigungen gegen meine Person nicht zustimmen. Das Resümee muss also “agree to disagree” heißen. Letzteres ist in diesem Bereich aber überhaupt nichts außergewöhnliches.

    Ich bedanke mich für den Hinweis auf einen neuen Artikel. Ich bin gespannt und werde den Text genau lesen.

  560. Martin Holzherr, 26.01.2021, 08:57 Uhr

    Dem kann ich ganz zustimmen.

    Meiner Erfahrung nach gibt es gegen diese “Trivialitäten” oft ganz erbitterten Widerstand – auch bei hoch gebildeten Menschen. Ich bin diesbezüglich extrem rauhe Attacken gewohnt. Damit verglichen ist der Diskurs hier im Kommentarbereich äußerst harmonisch. Diese ruhigere Atmosphäre ist aus meiner Sicht sehr begrüßenswert.

  561. @Feodor: Fakten

    Ich weiß nicht, wie Sie darum herumreden wollen, dass Sie schrieben (1 und 2):

    7. Stephan Schleim scheint vielleicht auch nahe legen zu wollen, dass Tatsachenaussagen, wenn sie aus dem Bereich einer Religion kommen, nicht kritisiert werden dürften.

    Sie haben nur ein Argument benutzt, dass auch Kreationisten benutzen.

    Den ersten Punkt hätten Sie wenigstens begründen und nicht nur so in den Raum stellen können. Wie gesagt: er ist völlig absurd. Im Übrigen ist es schon sehr schräg, hier in meinen Blog zu kommen, an der Diskussion teilzunehmen und dann so in der dritten Person über mich zu reden. Zum zweiten Punkt habe ich das Nötige gesagt.

  562. Warum viele die fiktionale Welt für die wirkliche Welt halten
    Zitat Timm Grams: „ Martin Mahner wird die Welt 2 mit seinem Fiktionalismus los“
    Jedes Wort, jeder Satz in diesem Blog entspringt der fiktionalen Welt, also der Welt, die in unserem Kopf erst entsteht. Wer ständig mit Sprache arbeitet und vielleicht sogar Denken für „stummes Sprechen“ hält [Anmerkung: so wie Platon], der kann bald einmal auf die Idee kommen, das Geistige als Heimatort der Sprache sei die wirkliche Welt. Konstruktivisten, besonders die radikalen unter ihnen, gehen gar so weit, zu bestreiten, das es eine objektiv erkennbare Aussenwelt gäbe. Sie vertrauen nur noch der Logik ihrer Gedanken, glauben aber diese Logik könne nichts gültiges über die Aussenwelt aussagen, weil diese Aussenwelt, falls sie überhaupt existiert, von unseren Sinnen beliebig verfälscht werden kann. Konstruktivisten halten wissenschaftliche Erkenntnisse deshalb für Konsens, nicht für Wahrheit. Kaum ein Naturwissenschaft kann dem wohl zustimmen, fast alle streben nämlich nach der Wahrheit und nicht nur nach dem Konsens.

    Die Welt der Fiktion kennt fast unbeschränkte Möglichkeiten. Und vielen davon sind wir schon begegnet. Wenn Chrys hier von reflexiven Strukturen berichtet, “wie insbesondere jene, die Hofstadter “Strange Loop” genannt hat“, so ist das in der Welt der Fiktion und überhaupt der Welt der Informationsverarbeitung gar nichts besonderes. Schon Programmiersprachen, die die vom Programmier definierten Datenstrukturen analysieren, kategorisieren und verändern können sind reflexiv.

    Ein wichtiger Teil des Humors und der tiefer gehenden Scherze kommt durch „gebrochene Fiktionen“ zustande. Beispiel: eben habe ich Haruki Murakamis Roman Pinball 1973 gelesen. Die (fiktive) männliche Hauptgestalt in diesem Roman lebt in Japan und führt scheinbar ein reales Leben dort. Doch eines Morgens wacht er mit 2 Zwillingen im Bett auf, von der die eine ein T-Shirt mit der Nummer 208, die andere eines mit der Nummer 209 trägt. Die begleiten ihn nun, verwöhnen ihn ohne selbst mehr als Kaffe und Kuchen zu konsumieren und verschwinden irgendwann wieder. Ich selbst bin zum Schluss gekommen, dass 208 und 209 Fiktionen der Hauptfigur sind. Dass sie nur halb real sind.
    Wir haben in Pinball also folgende Situation: Die Hauptfigur des Romans ist eine Fiktion von Murakami und die Zwillinge sind eine Fiktion sowohl der Hauptfigur als auch von Murakami.
    Ja, man kann im Traum träumen.

    Frage: ist die fiktionale Welt die bessere Welt oder ist gar die fiktionale Welt die einzige Welt, die wir haben?

  563. Wenn ich die einzelnen Diskussionsbeiträge lese, wird mir immer klarer, dass es eine der Hauptbeschäftigungen der Philosophen ist, die Termini des einen in die eines anderen zu übersetzen, ohne dass sich der Inhalt verändert 😉

    Um Missverständnisse zu vermeiden versuche ich, umstrittene Aussagen möglichst präzise zu formulieren:

    (1) Der Geist ist eine nichtphysische (also immaterielle) Entität.
    (2) Der Körper ist eine physische (materielle) Entität.
    (3) Körper und Geist (physische und nichtphysische Entitäten) können miteinander interagieren.

    Im Gehirn laufen materielle Prozesse ab, die wir subjektiv, aus der Erste-Person-Perspektive heraus, als Denken oder andere Wahrnehmungsinhalte erleben.
    (3) kann wohl nur als eine Halbwahrheit betrachtet werden, denn die Kausalität wirkt offensichtlich nur einseitig: Gezielte Eingriffe in das physische (neuronale) Geschehen können dramatische psychische Folgen haben. Sie können aber auch ganz spezifische Gedächtnisinhalte ins Bewusstsein rufen, die von den Versuchspersonen als spontan auftauchende Erinnerungen geschildert werden, obwohl sie ohne ihr Wissen durch elektrische Reize künstlich ausgelöst waren (Wilder Penfield, zitiert in R. Wolf, Naturwiss. Rundschau 66:154-156, 2013).
    Zwar erleben wir aus der Erste-Person-Perspektive auch das Umgekehrte: dass beispielsweise unser Willensentscheid – eine geistige, immaterielle Entität – eine Handlung auslöst. Aber dies ist wohl eine der vielen Illusionen, von denen wir uns grundsätzlich nicht befreien können. Alle neurobiologischen Befunde sprechen dafür, dass die materielle Welt kausal in sich geschlossenen ist. Was bei einem Willensentscheid die gewollte Handlung auslöst, ist demnach einzig und allein dessen materielles, neurobiologisches Korrelat.

    Ich fasse die Thesen zusammen:

    Denken und Bewusstsein sind nicht unabhängig von der materiellen Welt, sondern vielmehr vollständig von ihren neurobiologischen Korrelaten abhängig; sie können ohne dieses nicht existieren. Dass es sich auf eine ganz bestimmte Weise anfühlt, ein lebendes neuronales System zu sein, schließt die Behauptung aus, dieses Gefühl existiere überhaupt nicht. Damit sehe ich in dem extremen Eliminativen Materialismus keine sinnvolle Alternative.

    Unsere Bewusstseinsinhalte, also alles was wir aus der Erste-Person-Perspektive heraus erleben, sind immaterielle Entitäten. Sie sind vollständig abhängig von ihrem jeweiligen materiellen, neurobiologischen Korrelat in unserem Gehirn und können ohne dessen Präsenz nicht existieren.

    Außer dem Wunsch, einer der vielen „Kränkungen“ des Menschen infolge zunehmender wissenschaftlicher Erkenntnis entgehen zu wollen, sehe ich keinen Grund zu bezweifeln, dass allein der materielle Aspekt eines Willensentscheids, also sein neurobiologisches Korrelat, das materielle Geschehen in der Realen Welt bestimmt und nicht der immaterielle, geistige Aspekt, den wir aus der Erste-Person-Perspektive als unsere Entscheidung erleben.

    Es gibt jedenfalls bisher keine belastbaren Hinweise darauf, dass immaterielle, geistige Entitäten einen kausalen Einfluss haben auf das materielle Geschehen im Gehirn. Gäbe es sie, läge aus heutiger Sicht ein paranormales Phänomen vor mit dem Potenzial, den Naturalismus zu falsifizieren. Für den ersten Existenznachweis eines paranormalen Phänomens hat die James Randi Educational Foundation seit Jahrzehnten 1 Million Dollar ausgelobt. Auch bei den wissenschaftlichen Psi-Tests der GWUP, die wir seit 2004 am Würzburger Biozentrum jährlich durchführten, konnte bisher noch kein Bewerber paranormale Fähigkeiten glaubhaft demonstrieren.

    Mir sind die Fallstricke der „nothing-buttery“ wohlbekannt. Aber hier scheint sie mir angebracht zu sein: Unser Bewusstsein ist nichts anderes als wie es sich ANFÜHLT, ein wahrnehmendes, denkendes und handelndes neuronales System zu SEIN! Die Frage aber, warum sich Qualia wie beispielsweise die Farbe Rot so und nicht anders anfühlen – das „hard problem“ von David Chalmers – ist vermutlich ein (unlösbares) Scheinproblem.

    Obwohl für uns noch viele „Welträtsel“ ungelöst sind, sehe ich keine belastbaren Argumente, die gegen den Naturalismus sprechen. Es versteht sich von selbst, dass wir nicht absolut sicher sein können, dass er die Reale Welt angemessen beschreibt, dass also alles materielle Geschehen „ehernen“ Naturgesetzen genügt und es immer und überall „mit rechten Dingen zugeht“. Aber ich gehe mit höchstem Einsatz die Wette ein, dass dies zutrifft.

    Der Behauptung von Timm Grams, die Thesen des Naturalismus enthielten logische Widersprüche, kann ich also nicht folgen. Den kritischen Beiträgen von Feodor dagegen stimme ich uneingeschränkt zu. Tendenzen zu ideologischem oder gar dogmatischem Denken – ein Vorwurf von Stephan Schleim – kann ich bei ihm nirgendwo entdecken.

    Als Nachtrag noch ein kurzer Kommentar zu Poppers Drei-Welten-Theorie, die ich gern nutze, obwohl sie von anderen oft dualistisch fehlgedeutet wird wegen unserer hoffnungslos dualistisch verseuchten Sprache.

    Die 3-Welten-Theorie von Karl Popper ordnet alles, was es in unserer Welt gibt, einer von drei grundverschiedenen Kategorien zu:

    Welt 1 (materiell): Die gesamte „Welt an sich“ mitsamt den objektiven
    Gesetzmäßigkeiten, denen sie genügt
    Welt 2 (immateriell): Alles, was subjektiv erlebt wird: Qualia und andere
    Wahrnehmungen, Gefühle, Gedanken…
    Welt 3 (immateriell): Alle gewonnenen Erkenntnisse, mit wissenschaftlichen
    Methoden erschlossene Naturgesetze, erdachte Theorien
    und Ideen, also sowohl richtige als auch falsche
    Hypothesen, die geistigen Aspekte von Kunst, kurz: die
    Welt der „Meme“
    Popper begeht – wie viele andere Dualisten – einen Kategorienfehler, indem er annimmt, dass die immateriellen Welten 2 und 3 kausal auf die materielle Welt 1 einwirken.
    Rainer Wolf

  564. @Dr. habil. Rainer Wolf

    Was verstehen Sie denn unter einem belastbaren Hinweis?
    Was verstehen Sie denn unter einem belastbaren Argument?

    Der Behauptung von Timm Grams, die Thesen des Naturalismus enthielten logische Widersprüche, kann ich also nicht folgen.

    Sie können keine Argument widerlegen, indem Sie Ihre Sicht der Dinge einfach dagegenhalten.

    Popper begeht – wie viele andere Dualisten – einen Kategorienfehler

    Popper sagt – wie viele andere Dualisten – das Gegenteil. Und jetzt?

    So wie Sie das beschreiben wird der Naturalismus tatsächlich zur Ideologie.

  565. @Timm Grams / 26.01.2021, 17:55 Uhr

    Der Versuch, aufzuräumen, ist löblich. Manchmal wird dabei aber auch etwas Staub aufgewirbelt. Ich versuche, einen kleinen Beitrag zu leisten.

    Staubwolke (1)
    » Die Rede, dass die Realität außerhalb unseres Denkens angesiedelt und in ihrer Existenz und ihren Eigenschaften unabhängig von unserem Bewusstsein sei, lässt die Vermutung zu, dass es so etwas wie die Welt 2 geben müsse […] Daraus sehe ich, dass Naturalisten durchaus an eine Welt 2 denken und dass diese keinerlei Auswirkungen auf die physikalische Welt 1 hat. «

    Das klingt schräg.

    Wenn der Staub sich gelegt hat, sehen wir: Diese sogenannte „Welt 2“ steht für die meisten Naturalisten für die „Welt“ der Gedanken und Empfindungen, die notwendigerweise Teil der realen Welt 1 ist.

    Damit ist im Grunde alles Wesentliche zu den beiden Welten gesagt.

    Staubwolke (2)
    »Ich halte es für eine berechtigte Frage, was mit der anderen ist [Welt 2 ist gemeint, B.] «

    Es hat sich nichts geändert, die Frage erübrigt sich, auch hier gilt: „Welt 2“ ist notwendigerweise Teil der realen Welt 1.

    Staubwolke (3)
    » Andererseits legen die Naturalisten Wert darauf, ein monistisches Weltbild zu vertreten; sie werden deshalb mit der ersten Aussage [im Bieri-Trilemma; B.] nichts anfangen können. «

    Es gibt an der Stelle kein „anderseits“, nach wie vor gilt: „Welt 2“ ist notwendigerweise Teil der realen Welt 1.

    Staubwolke (4)
    » Der Naturalist sieht dieses Problem und modifiziert seine Ausgangsstatements. «

    Das „Ausgangsstatement“ lautet unverändert: „Welt 2“ ist notwendigerweise Teil der realen Welt 1.

    Ich hoffe, dass nun, nachdem der Staub sich gelegt hat, alle etwas klarer sehen: Wir sind ein winziger Teil der Welt 1, wir blicken hinaus in die uns umgebende Welt 1 und erleben und erfahren sie, wenn man so will, in „Welt 2“.

    Die Beschaffenheit von dem, was wir erblicken, ist von den subjektiven Sinneseindrücken unabhängig.

  566. Für Balanus‘ Gedanke, dass die Welt der Gedanken in die Welt, die uns umgibt, in der wir leben, eingebettet ist, gab es schon immer gute Gründe. Heute gibt es aber noch mehr Gründe dafür. Denn heute wissen wir viel mehr über die nahe Verwandtschaft des Menschen mit anderen Tieren und viele Beobachtungen weisen darauf hin, dass auch Tiere relativ komplexe Gedanken haben – und wohl auch Qualia.
    Inzwischen können selbst vom Menschen geschaffene Maschinen bis zu einem gewissen Grad denken. Alles was es dazu braucht ist Elektronik und Strom.
    Ein Dualist, ja die meisten Nicht-Naturalisten müssen das aber wohl ablehnen. Dass Maschinen je denken könnten, nämlich. Tatsächlich sagte kürzlich in einem Interview Markus Gabriel, der mit dem Neuen Realismus, er sehe in der künstlichen Intelligenz nur eine Gefahr: dass uns Konzerne wie Google damit kontrollieren, nicht aber dass KI je denken könne.

  567. @Martin Holzherr 26.01. 19:56

    „Frage: ist die fiktionale Welt die bessere Welt oder ist gar die fiktionale Welt die einzige Welt, die wir haben?“

    Wenn ich am Strand sitze, und den Sonnenuntergang bewundere, habe ich ein richtiges Naturerlebnis. Das hier alles über meine Sinne hereinkommt, tut dem überhaupt keinen Abbruch, im Moment des Erlebens kann ich mein inneres Bild sogar als in der Außenwelt existierend, mich selbst ebenso im Außen lokalisiert sehen.

    Was tut das zur Sache, wenn ich denke, das sind nur Außenwahrnehmungen? Und was ist, wenn ich tatsächlich einen inneren direkten Zugang zu meiner aktuellen Umgebung habe, der noch zusätzlich zu meinen Sinneseindrücken stattfindet?

    Gerade wenn es um Naturwahrnehmungen geht, oder wenn ich mit geliebten Menschen zusammen bin, dann fühle ich doch den anderen mehr, in einer Art Telepathie, als nur meine Sinne liefern können. Ähnliches passiert, wenn ich wirklich verstehe, was andere meinen, oder wenn ich wirklich verstanden werde.

    In einer Welt, wo Innenwelt und Außenwelt in beide Richtungen wechselwirken, taucht das Problem, welche Welt jetzt echter ist, doch überhaupt nicht mehr auf.

    Auch wenn die Parapsychologen noch keine Nachweise haben, unser teils mystisches Erleben und unsere Erlebnisse sind einfach als Evidenz wirksam, uns selbst gegenüber. Wir können das aber offenbar Menschen nicht begreifbar machen, die als Naturalisten ihre Standpunkte haben.

    Aber über alle Differenzen hinweg, kann man wohl sagen, das unsere Fiktionen durchaus eine ordentliche Schnittmenge mit der wirklichen Welt da draußen haben. So dämlich sind wir jetzt alle nicht, wir kommen ja offenbar in der Welt ganz gut zurecht.

  568. Reutlinger,
    Sie kennen das Bild von Miro aus dem Jahre 1926 „Hund bellt den Mond an“-
    Dieses Motiv wurde von vielen anderen Künstlern aufgegriffen und gemalt.
    Ich hatte so ein Bild mit einem großen gelben Mond und am unteren Rand des Mondes war ein schwarzer Punkt.
    Meine Nichte , 2 Jahre alt, sah das Bild, zeigte mit dem Finger auf den schwarzen Punkt und sagte „Wau-Wau“. Ich war platt, für mich war das nur ein Schwarzer Punkt. Dann erst las ich den Titel des Bildes „Hund bellt den Mond an“

    Wie lässt sich das erklären ? Was macht den schwarzen Punkt zu einem Hund ?

    Hier geht es um das Phänomen Kunst und was die Kunst ausmacht. Der Künstler drückt einen Gedanken in einem Bild aus. Nicht jeder kann diesen Gedanken heraussehen, Das kleine Mädchen konnte das. Und der Beweis war der Titel des Bildes.
    Mit Mustererkennung hat das nichts zu tun. Das ist das Geheimnis eines Kunstwerkes. Deswegen ist Kunst teuer. Das Bewusstsein des Malers ist in diesem Gemälde. Das Gemälde ist materiell, der Gedanke darin, der ist es nicht. (sonst könnte ihn jeder erkennen)

  569. @Balanus 26.01.2021, 21:56 Uhr

    Sie schreiben:

    Wir sind ein winziger Teil der Welt 1, wir blicken hinaus in die uns umgebende Welt 1 und erleben und erfahren sie, wenn man so will, in „Welt 2“.

    Da es voraussetzungsgemäß keine Rückwirkung der Welt 2 auf Welt 1 gibt, ist das Erleben wie beim Kinobesuch. Die Filmhandlung ist für den Besucher unabänderlich. Das Bewusstsein spielt keine aktive Rolle. Demnach ist es für das Überleben nicht notwendig. Weshalb sollte die Evolution dann so etwas hervorbringen? Nur Kosten und kein Nutzen? Im Artikel schreibe ich:

    Wenn es nur eine Welt gibt, die es zu erkennen gilt, wo finden die Gedanken über diese Welt dann ihren Platz? In der Welt kann ihr Platz nicht sein, denn die Welt soll ja unabhängig vom Bewusstsein und den darin aufgehobenen Gedanken existieren. Irgendwo anders ist auch kein Platz, da es ja nur diese eine Welt gibt.

    Wir haben nach langem Hin und Her jetzt einen Platz gefunden, wo der Naturalist das Bewusstsein unterbringen könnte: Die Welt 2 wird zur Realität erklärt und ergänzt sozusagen die Welt 1 der rein materiellen Vorgänge um die mentalen. Die von Naturalisten postulierte einseitige Abhängigkeit sagt aber, dass dieses so verortete Bewusstsein nutzlos ist. Das sieht nach eliminativem Materialismus aus, auch wenn Feodor das mit den Worten “Ein eliminativer Materialismus braucht keineswegs vertreten zu werden” abstreitet, ohne das zu belegen.

  570. Was ist Bewusstsein? Versuch 10 Formel und Bewusstsein

    Diese Formel kennt fast jeder E = m mal c²

    Sie stammt von Albert Einstein und besagt, die Masse (Materie) multipliziert mit der Lichtgeschwindigkeit im Quadrat ergibt die Energie. Dabei bleibt die Energie eine black box, ein Begriff, der noch leer ist. Wir wissen aber von der Energie , dass sie weder erzeugt noch vernichtet werden kann, man kann die Energieformen nur untereinander umwandeln. Die Energie selbst ist ewig.
    Liebe Geisteswissenschaftler da sind uns doch die Materialisten überlegen. Die behaupten frech, Energie ist ewig.

    Noch einmal , was ist Energie ? Ohne eine Erklärung ist das nur eine Leerformel.
    Was braucht man für eine Erklärung ? Na ? Geist. Ohne Geist verstehen wir die Formel nicht.
    Wenn es also Materie gibt, dann gibt es auch Energie. Energie ergib erst einen Sinn, wenn es einen Geist gibt, der Energie denken kann.

    Und jetzt kommt der zweite Mann zu Wort, der sich über die Welt Gedanken gemacht hat, Moses. Er war Anführer der Israeliten und hat der Welt ein Gesetzeswerk hinterlassen, das heute noch Vorbild ist für die Gesetzbücher dieser Welt. Darin steht: Du sollst nicht töten, du sollst nicht stehlen, Du sollst Vater und Mutter ehren,…….Insgesamt stehn in diesem Gesezbuch nur 10 Gesetze. Deswegen heißen die, die 10 Gebote.
    Dieser praktisch denkende Moses hat uns aber auch noch eine bemerkenswerte Formel hinter lassen
    Und sie bezieht sich auf das Verhältnis von Materie zu Geist.
    Sie lautet: Im Anfang war das Wort und das Wort war bei Gott und Gott war das Wort.

    Achten Sie auf den Anfang: „ Im Anfang“. Das bedeutet, dass der Anfang der Welt nicht nur zeitlich zu denken ist, sondern auch räumlich. Das ist die Beschreibung des Urknalls.
    Und er spricht nicht von materiellen Dingen , er spricht vom „Wort“ –

    Und in diesem Bewusstsein sollten wir die Entstehung und Entwicklung der Welt betrachten.

  571. @hwied
    Ihre Beispiele sind zum Schmunzeln. Kannten die ersten Menschen schon Energie, oder hatten sie noch kein Bewusstsein?

    In dem Gemälde von Miro erkenne ich eindeutig einen “Wauwau”. Das kann eine Zweijährige auch. Richtig ist, dass auch Kleinkinder schon über Geist verfügen, manchmal in erstaunlicher Weise. Trotzdem unterliegen sie einer geistigen Entwicklung, ihr Geist ist nicht fertig vorhanden.

    Wenn der Geist unabhängig vom Gehirn wäre, warum ist er dann so individuell, geprägt von sinnlichen Wahrnehmungen und körperlichen Erfahrungen des Individiuums? Die Dualisten haben keinerlei sinnvolle oder fundierte Erklärungen, außer weiteren adhoc-Phantasien.

  572. Anton Reutlinger,
    Danke für Ihre Hartnäckigkeit. Der Geist ist nicht unabhängig von der Materie, sprich uns, das ist doch die Krux.
    Wir als biologisches Wesen sind mit Geist und Urteilskraft ausgestattet. Und wir sind untrennbar miteinander verbunden.

    Mir geht es doch darum, dass man aus der Abhängigkeit des Geistes nicht folgern soll, es gibt keinen Gott.

  573. Zitat Timm Grams:

    Da es voraussetzungsgemäß keine Rückwirkung der Welt 2 auf Welt 1 gibt, ist das Erleben wie beim Kinobesuch. Die Filmhandlung ist für den Besucher unabänderlich. Das Bewusstsein spielt keine aktive Rolle. Demnach ist es für das Überleben nicht notwendig.

    Von diesem Gedanken ist es nicht mehr weit zu folgender Idee der Dualisten: Es könnte Menschen ohne Bewusstsein und ohne Qualia geben und die wären äusserlich und von ihrem Verhalten her nicht unterscheidbar von Menschen mit Bewusstsein oder wie man in der Wikipedia unter Philosophischer Zombie liest:

    Ein philosophischer Zombie oder P-Zombie in der Philosophie des Geistes und der Erkenntnistheorie ist ein hypothetisches Wesen, das von außen nicht von einem normalen Menschen zu unterscheiden ist, aber keine bewusste Erfahrung oder Empfindungen (Qualia) erlebt.[1] Beispielsweise empfindet ein philosophischer Zombie, der mit einem spitzen Gegenstand gestoßen wird, keinen Schmerz, er verhält sich aber genau so, als ob er Schmerzen empfände.

    Mein Argument dagegen: Dass Qualia atomar und ohne Bedeutung für unsere Erinnerungen und Handlungen seien ist schlicht und einfach falsch.

    Wer von einer roten Weihnachtskugel am hell erleuchteten Weihnachtsbaum berichtet, hat in Wirklichkeit nie eine (rein) rote Weihnachtskugel gesehen, sondern eine Weihnachtskugel in der sich die Lichter der Weihnachtskerzen spiegeln und wo man alle möglichen Farben und Spiegelbilder sieht. Das heisst: die „rote Weihnachtskugel“ ist bereits eine Abstraktion. Die Farben über die wir berichten sind nicht die wahrgenommenen Farben, es sind die „gewussten“ Farben. Wenn wir diese Abstraktionsfähigkeit nicht besitzen würden und also nicht in der Lage wären, einen Gegenstand als „rot“ zu erkennen, dann wären wir evolutiv schwer benachteiligt, weil wir dann den gleichen Gegenstand in einem anderen Ambiente gar nicht wiedererkennen könnten.
    Folgerung: Qualia sind für unseren Erfolg in der Welt in der wir leben sehr wichtig. Es sind keinesfalls reine Erlebnisprodukte ohne irgendeinen Wert für unser Überleben.

  574. Nachtrag Reutlinger,
    ….hatten die noch keine Energie….jetzt begreife ich erst.
    Nein, wir hatten auch keine Energie im physikalischen Sinne .
    Bis 1807 Thomas Young diesen Begriff in die Mechanik eingeführt hat.
    Der philosophische Begriff Energie von Aristoteles bedeutet “Wirkkraft”, das ist aber etwas anderes, weil der neuzeitliche Energiebegriff die Ruheenergie mit einschließt. Sie merken, dass unsere Vorstellung=Bewusstsein von den Begriffen abhängt, die wir verwenden. Wenn wir noch keinen Begriff haben, dann haben wir auch keine Vorstellung von einer Sache.

  575. Zusatz zum Vorgängerkommentar vom 27.01.2021, 12:07 Uhr:
    Wer aber als Dualist behauptet, es sei ein Zombie denkbar, der zwar die Farbe rot erkennt, dabei aber überhaupt nichts empfindet, nichts erlebt, der hat ein falsches Bild vom Menschen: unsere Emfindungen moderieren unsere Gedanken, sie bestimmen was wichtig oder unwichtig ist, sie lenken unsere Aufmerksamkeit.
    Ohne selektive Aufmerksamkeit aber sind wir stark eingeschränkt. Das zeigen sogat die neuesten Entwicklungen in der künstlichen Intelligenz und beispielsweise das Papier Attention is All You Need

  576. @Timm Grams / 27.01.2021, 10:32 Uhr

    » Da es voraussetzungsgemäß keine Rückwirkung der Welt 2 auf Welt 1 gibt, ist das Erleben wie beim Kinobesuch. Die Filmhandlung ist für den Besucher unabänderlich. Das Bewusstsein spielt keine aktive Rolle.«
    (Fettung von mir)

    Genau so verhält es sich meiner Auffassung nach. Es handelt sich schlicht um eine physikalische Unmöglichkeit.

    » Demnach ist es [das Bewusstsein] für das Überleben nicht notwendig. Weshalb sollte die Evolution dann so etwas hervorbringen?«

    Dass das Bewusstsein, also das, was wir uns darunter so vorstellen, für lebende Systeme nicht überlebenswichtig ist, ist ja offenkundig.

    Umso drängender mag sich die Frage stellen, wie diese Form und Qualität der Bewusstheit beim Menschen überhaupt entstehen konnte. Es muss im Zusammenhang mit diesem bewussten Erleben also etwas geben, wo die Evolution ansetzen kann, und da bietet sich als Einziges die genetische Information an. Diese steuert die Entwicklung der Individuen inklusive der Nervensysteme mit ihren sensorischen Apparaten, ohne die sich Organismen in ihrer Umwelt nicht frei bewegen könnten. Im zentralen Nervensystem müssen sich Strukturen ausgebildet haben, die die hierfür notwendigen Unterscheidungen treffen können (hinsichtlich des elektromagnetischen Spektrums, der Luftdruckschwankungen, bei chemischen Substanzen, etc.). Welche Rolle bei diesen neuronalen Prozessen das subjektive Erleben spielt, ist eine schwierige Frage, aber es scheint zu helfen, wenn Sinneseindrücke irgendwie erlebt werden, wenn Bilder und Töne (also gewissermaßen die Unterscheidungen) im Kopf entstehen (aber es ist kompliziert, auch nicht bewusst werdende sensorische Reize können adäquat verarbeitet werden).

    Insgesamt scheinen die neuronalen Prozesse, die mit dem bewussten Erleben im Zusammenhang stehen, also von evolutionärem Vorteil gewesen zu sein. Gerade auch im Zusammenspiel mit dem Leben in sozialen Verbänden und den spezifischen kulturellen Fähigkeiten (Sprache, Werkzeugherstellung).

    Langer Rede kurzer Sinn: Es sind nicht die erlebten Bilder, an denen evolutionäre Prozesse angreifen und die handlungswirksam werden, sondern, wie schon oft gesagt, die den Bildern und Gedanken unterliegenden physischen Strukturen. Im Ergebnis ist es anscheinend so, als ob unser bewusstes Sein der Hauptakteur wäre. Letztlich kann es aber nur das Sein sein.

    Vielleicht ist ja dieses ‚als ob‘ gemeint, wenn in diesem Zusammenhang gelegentlich von ‚Illusion‘ die Rede ist.

  577. @hwied: das Wort Begriff kommt von begreifen und das hat wohl etwas zu tun mit „greifen“. Mit Begriffen bekommen wir etwas in die Hand, etwas, was vorher diffus war ist nun greifbar.

    Folgerung: Wer etwas greifen kann ist im Vorteil gegenüber jemanden, der etwas nicht greifen kann, weil es zu diffus für ihn ist, weil er es nicht fassen kann.
    Begriffsfähigkeit ist also ein Überlebensvorteil!

  578. Holzherr,
    genau so ist es. Wir begreifen etwas , wenn wir es anfassen können.
    Bei abstrakten Begriffen behelfen wir uns mit Vorstellungsmodellen oder wir weichen auf die Mathematik aus.
    Um Begriffe wieder begreifen zu können braucht man geistige Fähigkeiten.
    Wie man es dreht und wendet, ohne Geist geht es nicht.

  579. @hwied (Zitat): “ Wie man es dreht und wendet, ohne Geist geht es nicht.“
    Klar. Geist bedeutet – unter anderem – dass wir uns ein inneres, strukturiertes Bild der Aussenwelt und eventuell auch der Innenwelt machen. Informationsverarbeitende Maschinen machen das häufig, sie schaffen Zwischenrepräsentationen, zerlegen etwa das Bild eines Hammers innerlich in einen Hammergriff und das Hammereisen. Dabei benutzen sie auch ihr Wissen und vielleicht sogar die konkrete Erinnerung, haben sie doch vielleicht einmal einen Hammer zusammengesetzt.
    Das machen aber nicht nur Menschen. Das könnte eventuell auch ein Affe machen oder ein Roboter.

  580. Balanus,
    das Bewusstsein entscheidet sogar bei einem Hund, ob er sie als Freund oder Feind sieht. Im Bewusstsein angegriffen zu werden schießt der Soldat zurück.
    Im Bewusstsein von Freunden besucht zu werden, legt der Soldat die Waffe weg.
    Bewusstsein = Grundeinstellung

  581. Philosophischer Zombie

    Aus neurobiologischer Sicht kann es diesen Zombie nicht geben. Die neuronalen Verbände, die das sogenannte „Bewusstsein“ implizieren, sind auch verhaltensrelevant. Wenn diese ausgeschaltet werden, erlischt neben dem Bewusstsein auch das von diesen neuronalen Verbänden abhängige Verhalten.

  582. @hwied

    Wie man es dreht und wendet, ohne Geist geht es nicht.

    Was halten Sie von der Theorie, die ihrer widerspricht, dass manche Kommentare auch ohne Beteiligung von Geist verfasst wurden? Die geistlosen.

    Es gibt Kommentare ohne jegliche Substanz.

  583. @hwied // 27.01.2021, 13:15 Uhr

    » Balanus,
    das Bewusstsein entscheidet sogar bei einem Hund, …
    «

    Ein Hund ist eben kein philosophischer Zombie. 😉

  584. @ hwied 27.01.2021, 09:03 Uhr

    Zitat: „….das Bild von Miro aus dem Jahre 1926 „Hund bellt den Mond an“-
    Dieses Motiv wurde von vielen anderen Künstlern aufgegriffen und gemalt.
    Ich hatte so ein Bild mit einem großen gelben Mond und am unteren Rand des Mondes war ein schwarzer Punkt.
    Meine Nichte , 2 Jahre alt, sah das Bild, zeigte mit dem Finger auf den schwarzen Punkt und sagte „Wau-Wau“. Ich war platt, für mich war das nur ein Schwarzer Punkt. Dann erst las ich den Titel des Bildes „Hund bellt den Mond an“

    Wie lässt sich das erklären ? Was macht den schwarzen Punkt zu einem Hund ?“

    Dazu hätte ich ein weiteres Beispiel aus dem Leben. Eine Mutter die mit ihrem kleinen Jungen im Auto unterwegs war, hielt an einer Kreuzung bei „rot“ an. Ihr Junge zeigte auf die Ampel und meinte: „Wenn das „Lichti“ ist „Wiese“, dann kannst du weiterfahren“.

    Es scheint klar, die „Begriffswelt“ entwickelt sich langsam. Aber Kleinkinder sind sehr früh in der Lage, auch abstrakte Zusammenhänge zu erkennen, was sie ja aus Lerngründen immerzu tun müssen.

    Interessant wäre, ob das kleine Mädchen von dem Sie berichtet haben, mit einen eher kleinen schwarzen Hund in der näheren Umgebung gelebt hat, womöglich gemeinsam aufgewachsen ist, zumindest aber einen schwarzen Hund gekannt hat?

    Tiere sind bedeutsam für die kindliche Entwicklung. Es können sich aber auch Traumata entwickeln, wenn man z.B. von einem Hund gebissen wurde.

    Statt die korrekten Begriffe zu verwenden, verwenden Kinder Assoziationen, z.B. „Licht“ statt „Ampel“, „Wiese“ statt „grün“, allenfalls auch einen Hund, bzw. dessen „Sprache“, als Assoziation für den „schwarzen Punkt“.

    Beim Erwachsenen wäre, wegen seiner weitaus größeren „Begriffswelt“, ein schwarzer Punkt mit weitaus mehr Assoziationen verknüpft, so kommt er nicht auf den Gedanken dass es ein Hund sein könnte.

    Vielleicht haben Künstler absichtlich „kindliche Assoziationen“ als Vorbild für ihre Werke benutzt?

    Für Psychologen sind die o.a. Zusammenhänge völlig naheliegend.

  585. Joker,
    na ja, also, wenn die Rechtschreibung stimmt, dann ist schon mal Entspannung angesagt. Wenn einer in Spiegelschrift schreibt, dann hat der ein höheres Bewusstsein. Versteht man eine Antwort nicht, dann ist sie nicht geistlos. Da ist ein höherer Geist am Werke gewesen. Sind Antworten banal, dann zeigt das, das der Absender genau auf dem gleichen Level liegt, weil man ja alles schon weiß.
    Merke: Wenn du denkst, denkst du nur du denkst, aber denken tust du nie.

    Damit man nicht langweilig wird sollte man seinen Gesprächspartner entweder loben oder ihn beschimpfen. Nichts ist schlimmer, als ein emotionsloser Kommentar. (Ich habe mir erlaubt, geistlos durch emotionslos zu ersetzen.)

  586. Elektroniker
    die Nichte ist mit einem großen dunkelbrauen Hund zusammen aufgewachsen. Wenn sie ein Eis in der Hand hatte und wir nicht aufpassten , dann trieb der Hund, der größer war, die Nichte in eine Ecke, und schnappte sich das Eis.
    So habe ich das noch gar nicht betrachtet.

  587. @Martin Holzherr 26.01. 22:29

    „Ein Dualist, ja die meisten Nicht-Naturalisten müssen das aber wohl ablehnen. Dass Maschinen je denken könnten, nämlich. Tatsächlich sagte kürzlich in einem Interview Markus Gabriel, der mit dem Neuen Realismus, er sehe in der künstlichen Intelligenz nur eine Gefahr: dass uns Konzerne wie Google damit kontrollieren, nicht aber dass KI je denken könne.“

    Das passt aus meiner Sicht nicht zusammen, obwohl ich kein Naturalist im herkömmlichem Sinne bin. Wenn das mit dem Bewusstsein bei Lebewesen funktioniert, müsste das doch auch funktionieren, wenn man es beim Vorbild genau genug untersucht und entschlüsselt hat, und dann entsprechend funktional in Maschinen einbaut.

    Das würde eben sogar dann noch funktionieren können, wenn Bewusstsein geistige Anteile hat, die über gezielte Quantenzufälle als Schnittstelle mitspielen. Diese Schnittstellen könnten bei technischen Systemen einfach analoge Zufallszahlen sein, wenn man sie an den richtigen Stellen in die Funktionalität integriert. Unsere Maschinen existieren eben im selben Kosmos wie wir, und alle Effekte, die uns ausmachen, müssten zumindest theoretisch nachbaubar sein.

    Dann hätten wir sogar etwas, das mit künstlichem „Seelenleben“ zu tun hat. Das ist hier nicht die reine Utopie: Wenn wir das Gehirn genau genug erforschen, werden wir in diese Bereich vordringen, und die Fakten erkennen können, wie immer das auch in der Natur implementiert ist.

    Hier stellt sich höchstens die Frage, ob man Maschinen mit Bewusstsein braucht. Wenn Bewusstsein nicht nur zum Vergnügen da ist, und seine Funktionalität wirklich lebenstüchtiger macht, dann wäre es wohl auch als Maschine zu gebrauchen. Nicht für alle Anwendungen, aber hier und da auf jeden Fall. Z.B als Besatzung von interstellaren Raumschiffen.

    Und natürlich für Systeme, die beliebig skalierbar sind, einmal in ihrer Größe, und auch in ihrer Lebenszeit. Davor könnte man dann vielleicht wirklich Angst haben. Vielleicht. Wenn sie gleichzeitig Seele haben, könnten sie vielleicht auch vertrauenswürdig bleiben, egal wie groß sie skaliert werden. Aber das einzuschätzen könnte für uns schwierig sein.

  588. @Stephan Schleim

    …ich verstehe aber nicht, wieso phänomenaler Gehalt (Erleben) für den Materialisten ein größeres Problem wäre als intentionaler Gehalt (Inhalt).

    Ich vermute das liegt daran, dass man sich über intentionalen Gehalt meistens intersubjektiv verständigen kann, wie am Beispiel der Zahlen demonstriert. Es gibt im semiotischen Dreieck eine Ecke, die im Zugriff für alle Beteiligten liegt. Nicht zwingend ein reales Ding, aber so etwas wie einen ‘common ground’. Beim phänomenalen Gehalt scheint mir das nicht so zu sein.

    Ob es sich für uns irgendwie anfühlt, einen bestimmten Bewusstseinsprozess zu haben, ist dann völlig irrelevant.

    Sicher ist das in vielerlei Hinsicht irrelevant, aber doch nicht völlig.

    Timm Grams sieht das ähnlich: es “spitzt sich alles auf die Frage zu, ob mein Nachbar die Farbe Rot genauso empfindet wie ich, oder ob sich sein Erleben derselben Situation von dem meinen unterscheidet.”

    Im Prinzip wären dann Chalmers’ Zombies möglich; und wir wüssten nicht, ob unser subjektives Erleben eine Einbildung ist – oder ein Epiphänomen.

    Zombies sind möglich. Dass subjektives Erleben keine Einbildung ist, darüber bin ich mir nur in einem Fall gewiss, und das auch nicht absolut.

    ‘Ein Zweifel, also bin ich?’

  589. @Timm Grams / 27.01.2021, 10:32 Uhr

    » Das [die von Naturalisten postulierte einseitige Abhängigkeit der „Welt 2“ von Welt 1] sieht nach eliminativem Materialismus aus, auch wenn Feodor das mit den Worten “Ein eliminativer Materialismus braucht keineswegs vertreten zu werden” abstreitet, ohne das zu belegen. « .

    Als Nichtphilosoph bin ich mir nicht sicher, was genau da „eliminiert“ werden soll. Die sogenannten Bewusstseinsphänomene, die Sinnesempfindungen und Traumbilder, sind ja Fakt, die lassen sich nicht philosophisch eliminieren. Oder anders formuliert: Die Systemeigenschaften von Teilen des Gehirns kann man nicht einfach wegdiskutieren.

    „Eliminieren“ könnte man höchstens die Vorstellung von der kausalen Wirksamkeit der phänomenalen Empfindungen, die mit bestimmten neuralen Aktivitäten einhergehen.

    Mit den diversen philosophischen Kategorien ist es im Grunde wie mit den Bewusstseinsphänomenen: sie haben auf das neurobiologische Geschehen keinerlei Einfluss.

  590. @Joker / 27.01.2021, 14:56 Uhr

    » Dass subjektives Erleben keine Einbildung ist, darüber bin ich mir nur in einem Fall gewiss, und das auch nicht absolut. « .

    Das könn(t)en alle von sich sagen.

    Ein Zombie sähe sich niemals (auf Grund von Zahnschmerzen) veranlasst, zum Zahnarzt zu gehen. Das wäre ein Beispiel, wo das Verhalten eines Zombies vom Nichtzombie zwangsläufig abweicht. Woraus folgt: Zombies sind nicht möglich.

  591. @Tobias Jeckenburger (Zitat):

    “Wenn das mit dem Bewusstsein bei Lebewesen funktioniert, müsste das doch auch funktionieren, wenn man es beim Vorbild genau genug untersucht und entschlüsselt hat, und dann entsprechend funktional in Maschinen einbaut.“

    Mit dieser Aussage sind sie eindeutig kein (klassischer) Dualist. Klassische Dualisten nehmen an, die geistige Welt sei nichtphysikalisch und könne deshalb in unserer physikalisch/technischen Welt nicht nachgebaut werden

    Und richtig: Auch wer Quantenphänomene für das Bewusstsein verantwortlich macht ist kein klassischer Dualist, denn Quantenphänomene sind physikalische Phänomene und damit von dieser Welt. Sollten allerdings Quantenphänomene beim Bewusstsein eine Rolle spielen könnte man das Bewusstsein technisch nur dadurch nachbilden, dass man entweder einen gewaltigen, klassischen Supercomputer einsetzt oder aber einen Quantencomputer benutzt.

  592. Der Äther ist nach der Meinung von Einstein leer. Andererseits nimmt man Quantenfluktuationen an. Was gilt jetzt ?
    Unser Bewusstsein. Ist das physikalisch leer, oder zeichnet es sich durch Quantenfluktuationen aus, den Intelligenzquanten ?

  593. @Balanus

    Ein Zombie sähe sich niemals (auf Grund von Zahnschmerzen) veranlasst, zum Zahnarzt zu gehen.

    Einem Zombie würde die gleiche nervöse Rückmeldung auf dem Zahnbereich reichen, um gleiches Verhalten auszulösen, wie bei dir – Moment, erkenntnistheoretisch besser abgesichert sollte ich wohl sagen, wie bei mir, Gesicht verziehen und zum Zahnarzt gehen.

  594. @Balanus

    Das könn(t)en alle von sich sagen.

    Das sagen ja auch alle von sich, aber woher will ich wissen, dass die anderen nicht lügen? Zombies sind nicht ehrlich – aus Gründen.

  595. @Joker

    „Nervöse Rückmeldung“? So wie beim Kniesehnen- oder Lidschlussreflex?

    Beim Zurückziehen der Hand von der heißen Herdplatte könnte ich mir so etwas zur Not noch vorstellen.

    Aber das dürfte sich leicht überprüfen lassen. Man braucht ja nur im Hirn die Schmerzempfindung auszuschalten, dann wird man sehen, welche „nervöse Rückmeldungen“ noch für Reaktionen verbleiben.

    » Zombies sind nicht ehrlich – aus Gründen. «

    Oh! Hast Du gelogen? Wenn ja, warum? Sei ehrlich!

  596. @hwied
    Quantenfluktuationen gibt es im materiell leeren Raum. Bekanntlich existiert im Kosmos eine Hintergrundstrahlug von durchschnittlich 2,7K, also Wärmeenergie. Die Teilchen der Quantenfluktuationen haben eine Lebensdauer von einigen Femtosekunden. Daher sind sie physikalisch nicht wirksam, im Gegensatz zur Paarbildung von Teilchen und Antiteilchen.

    Solche Spekulationen am Rande der Physik führen bezüglich des Bewusstseins nicht weiter und sind nicht besser als die Annahme eines immateriellen Geistes. Von Quantenintelligenz spricht man bei einem IQ < 1.

  597. @anton reutlinger

    IQ < 1

    Genauer 0 < IQ < 1

    IQ-positiv sind auch die Quantenintelligenten.

  598. @Feodor / 26.01.2021, 14:56 Uhr

    Bunge hat verschiedentlich gegen den Existenzquantor argumentiert, u.a, in Doing Science In the Light of Philosophy (World Scientific, 2017). Im wesentlichen beschwert er sich, dass mit dem Existenzquantor, als Negation der Allquantifizierung “for all”, nur ein “not for all” = “for some” ohne jeglichen ontologischen Gehalt ausgedrückt werde, weshalb man dann auch den “existence quantifier” in “someness quantifier” umbenennen solle.

    Das wird man aber gewiss nicht tun, denn die moderne Logik folgt Kant dahingehend, dass Existenz kein logischen Prädikat ist, womit der Existenzbegriff hier auch eine andere Bedeutung hat als für Bunge.

    Die traditionelle Auffassung ist, dass “realen” Dingen die (essentielle) Eigenschaft zukommt, aus sich selbst heraus zu existieren, und diesen Dingen können dann weitere (akzidentelle) Eigenschaften anhaften. Die Extension dieses Existenzprädikats ist die “Welt”, und seine Intension ist, nach Aristoteles, die metaphysische “Substanz”. Bunge spricht nun von Materie/Energie wie Aristoteles von Substanz, und Bunges Wandelbarkeit realer Dinge entspricht bei Aristoteles einer Änderung der dem Ding anhaftenden Akzidenzen.

    Nach Kants Auffassung sehen wir hingegen keineswegs “reale” Dinge, denen diese oder jene Akzidenzen anhaften, sondern wir stellen zunächst ein Erfülltsein von Prädikaten fest — wie “ist farblos”, “ist flüssig”, “ist elektrisch geladen” etc. — woraus dann erst ein empirisches Ding als solches in der Vorstellung konstituiert wird. Ein Ding wird demnach durch gewisse feststellbare Eigenschaften bestimmt, und zu existieren bedeutet hier, feststellbare Eigenschaften zu haben resp. Prädikate zu erfüllen.

    Vor diesem Hintergrund ist dann in der modernen (im Unterschied zur traditionellen) Logik ein Existenzsatz eine Aussage über die Erfüllbarkeit von Prädikaten bezüglich einer Klasse von Subjektvariablen. So wird mit ∃x∈K:Px ausgesagt, dass das Prädikat P bezüglich der Klasse K erfüllbar ist (sofern die Formel wahr ist). Oder äquivalent: “Es existiert ein x∈K mit der Eigenschaft P.” Das ist dann völlig in Ordnung mit Hinblick auf den Existenzbegriff von Kant, darüber gibt es nichts zu lamentieren.

    Im übrigen sind in der modernen Physik, speziell in der QFT, Vorstellungen über eine traditionell substialistisch gedachte Beschaffenheit von Materie schon ziemlich überholt. In diesem Zusammenhang sei auch an den SdW Artikel Was ist real? von Meinard Kuhlmann erinnert:

    Die Welt der klassischen Physik besteht aus Teilchen, die unter dem Einfluss von Kraftfeldern bestimmte Bahnen ziehen. Doch in der Quantenfeldtheorie ist nicht mehr klar, was Teilchen und Felder überhaupt sind. Die fundamentalsten Objekte lassen sich nicht wie Alltagsdinge beschreiben, sondern als Bündel von Eigenschaften wie Form und Farbe, Masse und Ladung.

  599. @hwied, anton reutlinger: Quantenfluktuationen, also kleinste Energieschwankungen von Punkt zu Punkt gibt es auch ohne kosmische Hintergrundstrahlung, also im „nackten“ Vakkum. Allein schon deswegen weil es gemäss Quantenfeldtheorie keine zu jedem Zeitpunkt feldfreien Vakuen geben kann und weil Heisenbergs Unschärferelation Energieschwankungen zulässt bezugsweise sogar erzwingt. Eine der Konsequenzen der Quantenfeldtheorie ist übrigens, dass immer wenn die Bedingungen für die Existenz eines Teilchens/Teilchenpaares erfüllt sind, diese Teilchen auch spontan entstehen. In der Wikipedia liest man dazu (übersetzt aus dem Englischen):

    Vakuumfluktuationen treten als virtuelle Teilchen auf, die immer in Teilchen-Antiteilchen-Paaren entstehen.[4] Da sie spontan und ohne Energiequelle entstehen, wird von Vakuumfluktuationen und virtuellen Teilchen gesagt, dass sie die Energieerhaltung verletzen. Dies ist theoretisch zulässig, da sich die Teilchen innerhalb eines durch die Unschärferelation bestimmten Zeitlimits gegenseitig annihilieren und somit nicht direkt beobachtbar sind.

    Der Casimir-Effekt ist eine der Konsequenzen von Vakuumquantenfluktuationen.
    Einschub: Der Casimir-Effekt bewirkt, dass zwei parallele, sich fast berührende Platten im Vakuum sich gegenseitig anziehen.

    Fazit: Selbst ein noch so leeres Vakuum ist von physikalischen Effekten erfüllt. Es gibt in diesem und jedem anderen Universum mit denselben physikalischen Gesetzen keinen Ort ohne physikalische Wechselwirkungen.

    You are never alone

  600. Zitat reutlinger:

    Quantenfluktuationen gibt es im materiell leeren Raum. Bekanntlich existiert im Kosmos eine Hintergrundstrahlug von durchschnittlich 2,7K, also Wärmeenergie.

    Bemerkung 1) dazu: Ein Vakuum erfüllt mit Wärmestrahlung ist nicht leer.
    Bemerkung 2) dazu: Bunge&Mahner betrachten auch Strahlung als materiell. Mit andern Worten: Wenn Bunge und Mahner von materiell sprechen, so meinen sie physikalisch.

  601. @Martin Holzherr 27.01. 15:33

    „…denn Quantenphänomene sind physikalische Phänomene und damit von dieser Welt.“

    Wenn es sie denn gibt, wäre die Möglichkeit von gezielten Quantenzufällen durchaus im Bereich der Naturgesetze, im Sinne von gesetzlich geregelten Ausnahmen, aber von der Idee her sind sie hier vor allem die Schnittstelle zu Geisteswelten, und dort speziell in der Richtung vom Geiste zur Materie.

    „…könnte man das Bewusstsein technisch nur dadurch nachbilden, dass man entweder einen gewaltigen, klassischen Supercomputer einsetzt oder aber einen Quantencomputer benutzt.“

    Ich glaube nicht, das unser Gehirn Effekte wie die von heutigen Quantencomputern nutzt. Entsprechend wären für den Nachbau eines Gehirns im Computer nur analoge Zufallszahlen nötig, die die Schnittstelle von der Informatik mit den Geisteswelten herstellen. Zur Produktion von analogen Zufallszahlen kann eine analoge Fernsehkarte dienen, oder eine abgedunkelte Webcam. Diese kann man dann auf jedem normalem Computer in die jeweiligen Programme einbinden.

    Um ein menschliches Gehirn nachzubauen, bräuchten wir dann aber einen gewaltigen Supercomputer, allein um die ganzen neuronalen Netze nachzubilden. Hier müsste man wohl extra ganz neue Hardware konzipieren, die von sich aus schon so ähnlich arbeitet, wie es Nervenzellen tun. Oder man begnügt sich erstmal mit Mäusekonnektomen.

    Sollte man allerdings die grundsätzlichen Strukturen identifiziert haben, die die materielle Seite unseres Bewusstseins ausmachen, kann man dann das Bewusstseinsprinzip auf künstlichen Systemen aller Art implementieren, zu ganz verschiedenen Zwecken und auch auf ganz verschiedener Hardware.

    Das wäre aber nur die materielle Seite des Bewusstseins, die Geisteswelten müssten dann das Angebot annehmen und dazu kommen. Ob sie das tun, wird man dann sehen. Wenn nicht, funktioniert das so nicht. Aber wenn ja, hätte man dann künstliches, beseeltes Leben konstruiert, was man dann zu Zwecken einsetzen kann, wo das Sinn macht.

  602. @ Martin Holzherr 27.01.2021, 15:33 Uhr

    Zitat: „Sollten allerdings Quantenphänomene beim Bewusstsein eine Rolle spielen könnte man das Bewusstsein technisch nur dadurch nachbilden, dass man entweder einen gewaltigen, klassischen Supercomputer einsetzt oder aber einen Quantencomputer benutzt.“

    Ich würde vermuten, dass Quantenphänomene zwar beim Bewusstsein eine Rolle spielen könnten, aber ich glaube eher nicht, dass diese Phänomene in elektronischen Supercomputern tatsächlich realisiert werden können. Außer man gestaltet die Systeme „hybrid“. Würde bedeuten, man verknüpft über biologisch/elektrische Schnittstellen genau die sensorischen Zellverbände, die Empfindungen realisieren können, mit den elektrischen Schnittstellen.

    Bei Quantencomputern käme es darauf an, die sensorischen Zellverbände perfekt, sozusagen wie ein „Modell“ (eher keine Simulation) in die „Quantencomputerwelt“ einzubinden. Eine „Simulation“ scheint eher keine „Realisation“.

  603. Balanus
    27.01.2021, 15:11 Uhr

    Wenn ich die Philosophen des eliminativen Materialismus richtig verstehe, gehen sie davon aus, dass mentale Zustände Produkte einer Alltagsphilosophie sind und dass diese Theorie durch eine bessere ersetzt werden könne, beispielsweise durch eine, die auf neurowissenschaftlichen Erkenntnissen fußt. Die Volkspsychologie und damit das Mentale werden sozusagen eliminiert.

  604. @Elektroniker (Zitat):

    Ich würde vermuten, dass Quantenphänomene zwar beim Bewusstsein eine Rolle spielen könnten, aber ich glaube eher nicht, dass diese Phänomene in elektronischen Supercomputern tatsächlich realisiert werden können.

    Alles was ein Quantencomputer berechnen kann, kann auch ein klassischer Computer berechnen. Richard Feynman erkannte allerdings, dass beispielsweise die Berechnung von Elektronenkonfigurationen, wie sie in Molekülen vorkommen, mit der Anzahl der zu berücksichtigenden Elektronen exponentiell wächst.
    Mit andern Worten: Irgendwann kommt ein klassischer Computer nicht mehr mit.

    Dass Quantenphänomene beim Denken eine Rolle spielen ist auf jeden Fall sehr unwahrscheinlich, denn Quantenprozesse laufen nur in isolierten Systemen zuverlässig ab.

  605. @Timm Grams / 27.01.2021, 18:22 Uhr

    » Die Volkspsychologie und damit das Mentale werden sozusagen eliminiert. «

    Ok, dagegen wäre dann ja nichts einzuwenden. Überholte Vorstellungen und Postulate ohne jegliche empirische Basis können problemlos aufgegeben werden.

  606. @Martin Holzherr // 27.01.2021, 19:02 Uhr

    » Dass Quantenphänomene beim Denken eine Rolle spielen ist auf jeden Fall sehr unwahrscheinlich, denn Quantenprozesse laufen nur in isolierten Systemen zuverlässig ab. «

    Vorsicht, Quantenphänomene spielen mit Sicherheit ein essentielle Rolle bei den Eigenschaften der Nervenzellmembranen.

  607. @Tobias Jeckenburger (Zitat):

    Wenn es sie denn gibt [Quantenphänomene], wäre die Möglichkeit von gezielten Quantenzufällen durchaus im Bereich der Naturgesetze, im Sinne von gesetzlich geregelten Ausnahmen, aber von der Idee her sind sie hier vor allem die Schnittstelle zu Geisteswelten, und dort speziell in der Richtung vom Geiste zur Materie.

    Alles was im Kleinsten, im Submikroskopischen passiert wird durch die Quantentheorie beschrieben. Quantenphänomene gibt es also – vor allem im Bereich des Allerkleinsten aber.

    Dass Quantenphänomene die Schnittstelle zu Geisteswelten sind – wie sie schreiben -, glauben wohl nur ganz wenige Physiker.

  608. @Balanus (Zitat):

    “ Vorsicht, Quantenphänomene spielen mit Sicherheit ein essentielle Rolle bei den Eigenschaften der Nervenzellmembranen.“

    Schon möglich. Mit Denken meine ich aber Vorgänge, bei denen Nervenzellen und eventuell sogar verschiedene Hirnteile miteinander kommunizieren. Dass dabei Quantenphänomene eine Rolle spielen, ist sehr unwahrscheinlich.
    Wenn es um Eigenschaften von chemischen Stoffen oder beispielsweise von Nervenzellmembranen geht, dann hat das nichts mit Denkprozessen zu tun, sondern nur mit Baumaterialien. Jeder Transistor aber ist ohne Quantenvorgänge nicht erklärbar. Doch ein Transistor ist ähnlich wie eine Nervenzelle nur ein Bauelement. Mit dem Transistor allein oder mit der Nervenzelle allein ist der Computer bezugsweise das Denken noch lange nicht erklärt.

  609. Man sollte nicht vergessen, dass es um Naturalismus geht, also um das Geschehen in der Natur, nicht im Labor eines Physikers. Da gibt es Unterschiede, spezielle Konstellationen, wie sie in der Natur nicht vorkommen. Das Gehirn hat 100 milliarden Nervenzellen, Quanteneffekte gehen da in der Statistik unter, zumal es ohnehin statistische bzw. probabilistische Effekte sind. Für die Erforschung des Bewusstseins ist das eine Sackgasse und für die Philosophie unnütze Spekulation.

  610. Physikalisch wirksam, das ist ihr Kriterium für Materie.
    Physikalisch wirksam ist auch ein Kriterium für geistige Phänomene.
    Dazu gehören Einsichten, Meinungen, Erkenntnisse.

    Einen Selbstmörder kann man durch gutes Reden vom Sprung vom Balkon abzuhalten.
    Das Buch Die Leiden des jungen Werther von Goethe hat eine Selbstmordwelle ausgelöst.
    Ein liebes Wort kann aus einem traurigen Menschen einen glücklichen Menschen machen.
    Anmerkung: Das gesprochene Wort ist physikalisch, die Bedeutung des Wortes ist geistig.

    Das hatten wir alles schon.
    Zum Schluss: Kann man etwas zeichnen, was es nicht gibt ?

  611. Martin Holzherr // 27.01.2021, 19:23 Uhr

    »Wenn es um Eigenschaften von chemischen Stoffen oder beispielsweise von Nervenzellmembranen geht, dann hat das nichts mit Denkprozessen zu tun, sondern nur mit Baumaterialien.«

    Das ist aber doch die Frage, inwieweit die (dynamischen) Eigenschaften der Neuronen für deren Zusammenspiel in Gesamtsystem Hirn eine Rolle spielen.

    Ich sage ja nicht, dass Quanteneffekte Denkprozesse erklären, aber ich habe Grund zu der Vermutung, dass es ohne Quanteneffekte im Oberstübchen verdammt still wäre—zu still für irgendeinen beliebigen Gedanken.

  612. @ Martin Holzherr 27.01.2021, 19:23 Uhr

    Zitat: „Schon möglich. Mit Denken meine ich aber Vorgänge, bei denen Nervenzellen und eventuell sogar verschiedene Hirnteile miteinander kommunizieren. Dass dabei Quantenphänomene eine Rolle spielen, ist sehr unwahrscheinlich.
    Wenn es um Eigenschaften von chemischen Stoffen oder beispielsweise von Nervenzellmembranen geht, dann hat das nichts mit Denkprozessen zu tun, sondern nur mit Baumaterialien. Jeder Transistor aber ist ohne Quantenvorgänge nicht erklärbar. Doch ein Transistor ist ähnlich wie eine Nervenzelle nur ein Bauelement. Mit dem Transistor allein oder mit der Nervenzelle allein ist der Computer bezugsweise das Denken noch lange nicht erklärt.“

    Die Informationsverarbeitung im Gehirn dürfte mit dem Gatterkonzept von W. McCulloch, auch ohne Quantenphysik erklärt werden können. Jedenfalls wenn man voraussetzt, was auch gemessen werden kann, dass Neuronen hauptsächlich dann Triggern, wenn ausreichend viele Ladungsträger gleichzeitig über möglichst viele Dendriten zum Neuronenkörper gelangen. Das verhält sich annähernd vergleichbar wie in der Gatterlogik der Elektroniker.

    Die Verknüpfungen bestimmen, wie die Signalverzweigungen zu erfolgen haben.

    Das „Wissen“ wird in den Verzweigungen „abgebildet“. Bedeutet, es werden nicht „Daten“ wie in der Informatik als „Schaltzustände“ abgebildet, die auf die in der Informatik bekannten Art (mittels „Programm“) verknüpft werden. Sondern, In den Synaptischen Verknüpfungen, den „Verzweigungen“, werden sozusagen gleichzeitig die „Daten und das Programm“ (jeweils ein winziger Teil davon) abgebildet.

    Im Sinne der Informatik nennt man derartiges „Objekt“, eigentlich besser „Muster“. Es gibt keine Trennung in Daten und Programme, wie in der „alten“ Informatik. Die „Muster“ enthalten sozusagen beides. Das Gehirn ist eine Musterverarbeitungsmaschine.

    Auch deswegen ist das Geschehen im Gehirn nicht mit dem Geschehen in den bekannten Mikroprozessoren vergleichbar, sehr wohl aber mit der früher in der Industrie gebräuchlichen „Gatterelektronik“.

    E. Kandel hat sozusagen bestens erklärt, was im Gehirn bei der automatischen Synapsenbildung geschieht, was früher die Elektroniker gemäß ihres Wissens gemacht haben, wenn sie die Verdrahtung (der Gatter) mittels Lötpunkten realisiert haben.

    Die Quantenphysik sollte allerdings erklären können, wie es genau zur Ansammlung von Elektronen, letztlich den sensorischen Effekten in biologischen Zellen überhaupt kommen kann. Dass dabei womöglich auch Empfindungen wie Qualia, aber auch Lust und Schmerz entstehen.

    Dass es sich so ähnlich verhalten dürfte wie Balanus meint, dass Quantenphänomene eine essentielle Rolle bei den Eigenschaften der Nervenzellmembranen spielen, scheint naheliegend.

  613. @ Chrys, 27.01.2021, 17:32 Uhr:

    Schönen Dank für die Info! Bunge hat sehr viel geschrieben, so dass die Stellen nicht einfach zu finden sind. Ähnliches schreibt er auch im Aufsatz “The alleged ontological commitment of the existential quantifier”, den ich habe.

    Nach Kant leben wir ja ein einer Art “Matrix”, denn die “echte Außenwelt” (Ding an sich) ist nach ihm absolut tranzendent. “Empirisch real” bedeutet bei Kant “mental vorgestellt”. Bei Berkeley werden die Vorstellungen von Gott gegeben, bei Kant stammt der Input (als gedachte unschematisierte Ursache) vom transzendenten “Ding an sich”, wobei noch die subjektive Überformung hinzukommt. An dem Problem haben sich schon viele Kant-Forscher mit jeweils unterschiedlichen Ergebnissen abgearbeitet.

    (Scherz)
    Ehemann: “Bist du real?”
    Ehefrau: “Wieviel Rotwein hast du denn getrunken?”
    (/Scherz)

    Sehr seltsam ist es, wie auf kantianischem Weg eine Kommunikation über ein okkultes Medium hinweg möglich sein soll.

  614. @Balanus

    Wie wär’s mit Hypothese?

    In Gedanken werde ich die Annahme von Kausalität jetzt immer als ‘die ewige Hypothese’ bezeichnen, sowohl der Erfahrung als auch der Physik. Wenn ich selbst von metaphysischen Annahmen oder Spekulationen spreche, dann soll sie und ähnliche Hypothesen aber immer gleich mitgemeint sein (generisches Metaphysisches).

    @Elektroniker, @Balanus

    Dass es sich so ähnlich verhalten dürfte wie Balanus meint, dass Quantenphänomene eine essentielle Rolle bei den Eigenschaften der Nervenzellmembranen spielen, scheint naheliegend.

    Mittlerweile bin ich fest drauf konditioniert, ich kann nicht mehr anders, mit einer solchen Aussage assoziiere ich immer wieder den bekannte Deppen-Syllogismus:

    Ich verstehe nichts von der Seele und ich verstehe nichts von Quantenmechanik, also müssen beide was miteinander zu tun haben.

    (Lars Fischer, Quelle)

    Warum auch sollte man solch metaphysischen Spekulationen (siehe oben) alleine den Religiösen überlassen? Oder handelt es sich dabei um eine falsifizierbare Aussage? Um das zu überprüfen, würde es ja reichen, ein Gehirn nachzubauen, bei dem solche Quantenphänomene unterdrückt werden, wo immer sie auftauchen.

    Falls jemand sagt, das Falsifizieren wäre im Prinzip möglich, dann würde ich gerne näher erläutert haben, welches Prinzip gemeint ist.

  615. Man könnte es geahnt haben, es ging mir um diese Aussage (Hervorhebungen durch mich)
    :

    Die Quantenphysik sollte allerdings erklären können, wie es genau zur Ansammlung von Elektronen, letztlich den sensorischen Effekten in biologischen Zellen überhaupt kommen kann. Dass dabei womöglich auch Empfindungen wie Qualia, aber auch Lust und Schmerz entstehen.

  616. Hinweise dafür, dass unser Denkprozess eine physikalische Basis hat
    Zitat aus dem Beitrag: “ Mentale Phänomene sind nichtphysikalische Phänomene.“
    Zitat Timm Grams: „ Die Volkspsychologie und damit das Mentale werden sozusagen eliminiert.“

    Antwort: Ich zähle mich auch zum Volk und erlebe mentale Prozesse als in den physikalischen Alltag eingebettete Prozesse. Nicht-Physikalisch ist lediglich das was ich denke, also der Denkinhalt. Man muss den Denkprozess vom Denkinhalt unterscheiden und den Denkinhalt als Konstrukt erkennen und erleben. Zudem kann sich der Denkinhalt auf die Aussenwelt beziehen, muss es aber nicht. Ich behaupte sogar: Wer Mühe hat Konstrukte mit und ohne Aussenbezug zu unterscheiden bewegt sich in Richtung Einbildung/psychische Störung.

    Folgendes spricht dafür, dass Denkprozesse physikalische Prozesse sind:
    1. Denken benötigt Zeit, hat einen Beginn und ein Ende
    2. Denken kann durch Störungen von aussen und durch Störungen von innen wie Fieber und diverse Krankheiten beeinträchtigt werden
    3. Der Spruch „ Mens sana in corpore sano“ legt nahe, dass der Geist wie eine Maschine/ein Gerät gepflegt werden muss.

    Fazit: Wir alle erleben mentale Prozesse als eingebettet in unseren physikalischen Alltag und beeinflusst von unserem Körper.
    Alle mental Gesunden wissen gleichzeitig, dass das, an was sie denken aber etwas völlig anderes ist, als die physikalischen Gegenstände aus ihrem Alltag. Ein Gedanke hat mit einem Wasserkrug wenig bis gar nichts zu tun. Der Prozess aber, der den Gedanken hervorbringt, der hat gefühlsmässig einige Ähnlichkeiten mit anderen Prozessen aus unserem Alltag und es macht durchaus Sinn zu sagen: „ Heute habe ich so viel nachgedacht wie selten in diesem Jahr. Jetzt fühle ich mich total erschöpft“ Diese Aussage hat doch Ähnlichkeiten etwa mit der Aussage: „Heute hab ich 8 Stunden lang Trauben gepresst und man sieht es der Traubenpresse an“

  617. Korrektur zu meiner Nachricht vom 27.01.2021, 18:22 Uhr

    Anstelle von “Alltagsphilosophie” hätte stehen sollen “Alltagspsychologie” oder “Volkspsychologie”.

  618. @hwied
    Physikalisch wirksam ist auch ein Kriterium für geistige Phänomene.

    Es ist das Problem aller Dualisten: sie drehen sich ewig im Kreis und bohren dabei nur Löcher in den Boden.

    Das gesprochene Wort ist physikalisch, die Bedeutung des Wortes ist geistig.

    Auch das Hören ist physikalisch, das ist längst bekannt. Die Bedeutung ist in der Erinnerung, sie ist erlernt. Wo ist sie gespeichert? Auch die Bedeutung ist nun mal physikalisch, oder besser neurobiologisch in den Strukturen des Nervensystems gespeichert. Hormone sind es, die unsere Gefühle und Körperfunktionen steuern, z.B. Serotonin oder “Glückshormon”. Auch sie, jedenfalls einige davon, werden vom Gehirn gesteuert, insbesondere in der Amygdala, im Hypothalamus und in der Hypophyse, wo sie erzeugt und sezerniert werden.

    Ich frage mich immer wieder, warum sich die Dualisten nicht intensiver mit Neurobiologie beschäftigen, statt nur unsinnige Behauptungen aufzustellen! Natürlich kann die Neurobiologie nicht alle Fragen beantworten, aber es gibt keinerlei Hinweise auf alternative Erklärungen für die großen Fragen.

  619. Vom richtigen Bewusstsein,

    Ist es sinnvoll von falschem und richtigem Bewusstsein zu sprechen.
    Ohne eine moralische Komponente können wir auf den Begriff Bewusstsein verzichten.

    Einleitung : das Ich und unser Körper.
    Es gibt Fälle , wo das Ich im falschen Körper steckt. Es gibt Transvestiten, da ist der Körper ein Mann und das Ich eine Frau.
    Solche tragischen Fälle enden manchmal in einem Selbstmord, heute kann man eine Geschlechtsumwandlung durchführen.
    Dabei wird aus dem körperlichen Mann eine Frau. Das Ich bleibt unverändert.

    Was viel häufiger vorkommt, in einem Adoniskörper steckt ein kleines Ego. Der Mann sieht aus wie ein Bär , aber er ist sensibel und nicht seelisch belastbar.
    Umgekehrt gibt es kleine Frauen mit einem eisernen Willen, die ihre Umgebung tyrannisieren.

    Was wären jetzt die Eigenschaften eines idealen Bewusstseins ?
    (Sie merken ich benütze Ich und Bewusstsein synonym)

    1. Harmonie mit dem eigenen Körper.
    2. Harmonie mit den Mitmenschen
    3. Harmonie mit sich selbst

    Die Harmonie mit sich selbst ist nicht selbstverständlich. Ehrgeizige Menschen sind selten zufrieden zustellen.

    Im Alter (wir werden alle alt) leidet die Harmonie mit dem Körper.
    Im Alter leidet die Harmonie mit den Mitmenschen.
    Im Alter leidet die Harmonie mit sich selbst, wir werden unzufriedener.

    Mit diesem Wissen (in diesem Bewusstsein) ende ich vorläufig.

  620. @hwied
    Statt Transvestiten war sicherlich Transgender gemeint!

    Mit Ihren Ausführungen geben Sie selber zu, dass Körper und Geist nicht unabhängig voneinander sind. Von hier ist es nur noch ein kleines Schrittchen zum Monismus. Haben Sie Mut!

    Aufklärung ist der Ausgang des Menschen aus seiner selbst verschuldeten Unmündigkeit. Unmündigkeit ist das Unvermögen, sich seines Verstandes ohne Leitung eines andern zu bedienen. Selbst verschuldet ist diese Unmündigkeit, wenn die Ursache derselben nicht am Mangel des Verstandes, sondern der Entschließung und des Mutes liegt. Es ist so bequem, unmündig zu sein. Habe ich ein Buch, das für mich Verstand hat, einen Seelsorger, der für mich Gewissen hat, so brauche ich mich ja nicht selbst zu bemühen.

    I.Kant: Was ist Aufklärung, 1784

  621. @hwied (Zitat): „ Im Alter (wir werden alle alt) leidet die Harmonie mit dem Körper.
    Im Alter leidet die Harmonie mit den Mitmenschen.
    Im Alter leidet die Harmonie mit sich selbst, wir werden unzufriedener.“

    Diese persönlichen Feststellungen sind mir selbst momentan eher fremd. Und das ist für mich ein interessantes Phänomen. Wir Menschen können zwar mit jemandem mitfühlen dessen Schicksal und dessen Gedanken wir kennen und mitgeteilt bekommen, aber mir etwa fällt es schon schwer meinen eigenen mentalen Wandel im Laufe des eigenen Lebens zu diagnostizieren. Und ich scheine da nicht allein zu sein. Von einigen älteren Menschen habe ich gehört, sie fühlten sich immer noch gleich und als Gleiche wie in der Jugend.
    Es scheint mir äusserst schwierig sich selbst von aussen zu sehen. Und es gibt einige, die auch ihre auf Tonaufnahmen gespeicherten Stimmen oder bestimmte Fotos von ihnen selbst als Stimmen und Fotos von Fremden empfinden.

  622. @ Joker 27.01.2021, 23:44 Uhr

    Zitat: „Dass es sich so ähnlich verhalten dürfte wie Balanus meint, dass Quantenphänomene eine essentielle Rolle bei den Eigenschaften der Nervenzellmembranen spielen, scheint naheliegend.

    Mittlerweile bin ich fest drauf konditioniert, ich kann nicht mehr anders, mit einer solchen Aussage assoziiere ich immer wieder den bekannte Deppen-Syllogismus:

    Ich verstehe nichts von der Seele und ich verstehe nichts von Quantenmechanik, also müssen beide was miteinander zu tun haben.“

    Was Sie da so scharfsichtig durchschaut und erkannt haben wollen, ist sicherlich wieder ein typischer Höhepunkt philosophischer Betrachtungen und Schlussfolgerungen. Sicherlich trägt er zur gegenseitigen traditionellen „besonders hohen Wertschätzung“ zwischen „Philosophen“ und Fachleuten aus dem Gebiet „Elektrotechnik“ bei.

    Ausgangspunkt der (technischen) Überlegungen ist der „photoelektrische Effekt“ (auch lichtelektrischer Effekt oder kurz Photoeffekt) als Beispiel einer Schnittstelle zwischen Physik und Quantenphysik. Es geht grundsätzlich um nahe verwandte, aber unterschiedliche Prozesse der Wechselwirkung von Photonen mit Materie.

    Im Prinzip wird ein Elektron aus einer „Bindung“ gelöst, indem es ein Photon absorbiert. Dieser Effekt wurde von Albert Einstein erstmals gedeutet, wobei er den Begriff des „Lichtquants“ einführte.

    Ich glaube mich zu erinnern, dass ich von einem denkmöglichen Zusammenhang zwischen der „Loslösung eines Elektrons aus einer Bindung und dem Auftreten von Qualia“, erstmals etwas vor vielen Jahrzehnten im Chemieunterricht so nebenbei vom Chemielehrer erfahren habe. Der dürfte im Krieg in der Forschung gearbeitet haben.

    Philosophische Spekulationen reichen eher nicht auszuschließen, was technisch orientierten Fächern möglich erscheint, was offenbar real existiert und auch eine reale Ursache haben sollte.

    Fast alles was die Technik, besonders die Elektrotechnik realisiert hat, geht auf Grundlagenforschung der Physik zurück….

    Information verarbeitende Maschinen werden „nachgebaut“ und Qualiaeffekte treten eher nicht auf, weil die nur in bestimmten Zellverbänden und in einer besonderen „Auwerteumgebung“ (neuronales Netz) auftreten dürften .

  623. Martin Holzherr
    …….alte Menschen fühlen sich immer noch gleich.
    Ich meinte jetzt nicht das Selbstbewusstsein. Ich habe gerade einen Hexenschuss und kann nicht joggen. Das fühlt sich anders an als vorher.

    A. Reutlinger,

    Geist und Körper sind untrennbar miteinander verbunden. Dem stimme ich zu.
    Daher kommen doch die Probleme mit dem Bewusstsein.
    Um zu erkennen , dass der Körper den Geist beeinflusst, und dass der Wille den Körper beeinflusst, dazu braucht es keinen Mut.
    Und wenn Sie mir unterstellen ich kennte (Konjunktiv?) die Aufklärung nicht, dann unterstelle ich mal Ihnen , Sie haben nicht den Mut einen Gott anzuerkennen.
    Sie merken, trotz meines Hexenschusses funktioniert mein Geist noch ausgezeichnet (er ist vielleicht ein wenig gereizt)

  624. @hwied
    Dann wünsche ich Ihnen erst mal gute Besserung. Ich wollte Ihnen nicht unterstellen, dass Sie die Aufklärung nicht kennen, trotzdem darf man gelegentlich daran erinnern. Nach meiner katholischen Erziehung habe ich den Mut gefunden, mich von den Gottesmärchen und historischen Legenden zu befreien. Es ist sinnvoller, vernünftiger und befriedigender, sich mit Wissenschaften und mit Kunst zu beschäftigen.

  625. anton reutlinger,

    Danke für den Zuspruch.
    Bei mir war es gerade umgekehrt. Ich weiß nicht , ob ich getauft bin. In der frühen Nachkriegszeit, da gingen viele Unterlagen verloren.

    Erst mit 60 Jahren habe ich mein Interesse für Religion entdeckt. Jetzt fotografiere ich Kirchen und unterhalte zwei Webseiten http://www.kleinekirchen.de und http://www.kleinekirchen2.de. Der Nebeneffekt, man kommt mit vielen Menschen in Kontakt. Buchverlage fragen an, ob sie Fotos verwenden dürfen. Letztes Jahr kam ein E-mail aus USA wo ein Amerikaner auf der Suche nach seinen Vorfahren war und die Kirche seines Urgroßvaters , der dort Pfarrer war, auf meiner webseite gefunden hatte.
    Wissenschaft und Kunst kommen bei mir mit der Religion nicht in Widerspruch.

  626. @Joker // 27.01.2021, 23:44 Uhr

    » In Gedanken werde ich die Annahme von Kausalität jetzt immer als ‘die ewige Hypothese’ bezeichnen, …«

    Ohne jetzt genau zu wissen, wie das mit der „ewigen Hypothese“ zu verstehen ist, aber irgendwie passt dazu, dass vor exakt 100 Jahren Sewall Wright seinen Aufsatz „Correlation and Causation“ veröffentlicht hat. Ein Thema, das immer noch aktuell ist und wohl immer aktuell bleiben wird, zumindest solange Lebensformen sich ihrer Aktionen bewusst sind.

    Apropos Quanten:

    Gegen den Naturalismus sprechen aber auch naturwissenschaftliche Gründe, und zwar insbesondere Einsichten der Quantenphysik, also Erkenntnisse über genau jene Elementarteilchen, die für den Naturalismus die grundlegende und alles andere tragende Ebene der Wirklichkeit bilden.

    (Jens Halfwassen, Marsilius-Kolleg 2014/2015)

    Und nicht dass Dir jetzt wieder der Deppen-Syllogismus in den Sinn kommt, diese Auffassung basiert auf Gesprächen mit einem theoretischen Physiker, für den die Quantenfeldtheorie mehr ist als bloß ein Wort mit 18 Buchstaben.

  627. @Joker 27.01. 23:44

    „…assoziiere ich immer wieder den bekannte Deppen-Syllogismus:
    Ich verstehe nichts von der Seele und ich verstehe nichts von Quantenmechanik, also müssen beide was miteinander zu tun haben.“

    Deppen, die so denken, gibt es wohl. Wenn man aber aus „ich verstehe nichts von“ ein „die Wissenschaft hat keine Ahnung, wie das funktioniert“ macht, kann ein Schuh draus werden. Da Wissenslücken z.B. aus beschränkten Verfahren resultieren können, können sie auch mal gemeinsam die Ursache dafür sein, wenn hier die Wissenschaft nicht recht vorwärts kommt.

    Wenn man immer nur Regelmäßigkeiten sucht, übersieht man eben die Unregelmäßigkeiten. Wenn man hier die Mathematik überstrapaziert, wo die Effekte doch einfach nur individuell einmalig sind, kommt man eben auch nicht vorwärts.

    In diesem Sinne: Wenn Quantenzufälligkeiten eine Schnittstelle von Geist und Materie liefern, hat man hier eine durchaus sinnvoll anmutende Verbindung der Geheimnisse des Bewusstseins mit einer anderen Interpretation des Quantenkosmos gefunden.

  628. @Elektroniker // 28.01.2021, 11:05 Uhr

    Bei den von mir erwähnten Quantenphänomenen ging es nicht um eine mögliche Erklärung der „Qualia“, sondern um grundlegende Eigenschaften biologischer Moleküle.

    Bekanntlich lässt sich nicht voraus berechnen, welchen Weg ein frei beweglicher Organismus nimmt. Der Grund dafür dürfte in den quantenmechanisch bedingten Zufallsereignissen auf molekularer Ebene liegen.

    Für solche Zufallsereignisse im Gehirn gibt es jede Menge Belege. Sie könnten bei willentlichen Entscheidungen hin und wieder den Ausschlag geben. Aber es gibt nicht den geringsten Hinweis auf einen direkten (kausalen) Zusammenhang mit phänomenalen Phänomenen (wenn Sie verstehen, was
    ich meine ; ).

  629. @Balanus

    Jens Halfwassen

    Interessanter Mann, wie man dem Nachruf auf ihn, verfasst von Markus Gabriel, in der FAZ entnehmen kann. Ich höre zum ersten Mal von ihm, meine ich.

    In seinem Artikel, aus dem Du zitierst, hier als pdf, “Warum der reduktionistische Naturalismus unmöglich richtig sein kann”, spricht er sich gegen eine ganz bestimmte Variante des Naturalismus aus, die er am Anfang näher bestimmt.

    diese Auffassung basiert auf Gesprächen mit einem theoretischen Physiker, für den die Quantenfeldtheorie mehr ist als bloß ein Wort mit 18 Buchstaben.

    Ich würde auch Penrose zugestehen, dass er von dieser Theorie einiges versteht (obwohl er möglicherweise die Anzahl der Buchstaben des zugehörigen deutschen Wortes auch erst umständlich ermitteln müsste). Wenn der nun Argumente für den Naturalismus liefert (er selbst sieht das wohl differenzierter), dann lässt sich die Quantenfeldtheorie ja für beide Seiten, also für und wider, gut gebrauchen.

    Das ist schön.

  630. Was das Bewusstsein prägt

    1. Emotionale Erlebnisse
    2. Rationale Einsichten

    Kleinkinder werden durch die Liebe der Mutter geprägt. Jugendliche werden durch den Umgang mit anderen Menschen geprägt. Erwachsene werden im Berufsleben geprägt.

    Wird in der Prägephase das Ego beschädigt, dann hat das Folgen. Die Reife wird unterbrochen. Wird das Ego gestärkt, dann wächst die Persönlichkeit.

    Der Schule kommt hierbei eine wichtige Rolle zu. Sie soll die Schüler sozialisieren und gleichzeitig bilden.

    Der Vorgang der Sozialisierung und die Reife, die lässt sich naturwissenschaftlich nicht erfassen. Dafür sind die Sozialwissenschaften zuständig.

    Was sich bei den Menschen in Bezug auf Religion abspielt, das ist auch von den Sozialwissenschaften nicht erfassbar. Der Bezug des Menschen zu Gott ist individuell so verschieden, dass sich keine Regel ableiten lässt.

  631. Wenn man über Naturalismus diskutieren will, muss man auch über Realismus diskutieren. Diese Welt ist aus kleinsten Bausteinen zusammengesetzt, wie die Physik erkannt hat. Die Atome bilden eine sehr stabile Zwischenebene für die materielle Welt. Es sind sonst keine Zwischenstufen erkennbar.

    Die Welt ist in ständiger Entwicklung und Bewegung. Was wir wahrnehmen, ist ein winziger Ausschnitt, sowohl zeitlich als auch räumlich und gegenständlich. Die Realität, die wir zu erkennen glauben, ist also sehr flüchtig. Dasselbe gilt für den Menschen selber. Man kann also nicht von der gegenwärtig wahrnehmbaren Welt auf die wahre Konstitution und Realität dieser Welt, einschließlich des Menschen, schließen.

    Die Welt ist nicht so, wie wir sie wahrnehmen und erfahren. Zwar hat der Mensch objektiv erfahrbare mentale und emotionale Zustände, sie sind real, aber wir wissen nicht, was ihre wahre Realität ist. Wir dürfen sie nicht einfach als gegeben hinnehmen. Die Existenz von Bezeichnern ist kein Beweis für die Existenz des Bezeichneten.

  632. @Tobias Jeckenburger

    Wenn man hier die Mathematik überstrapaziert, wo die Effekte doch einfach nur individuell einmalig sind, kommt man eben auch nicht vorwärts.

    Ja, die Wissenschaft ist nicht geeignet, alle einmalig auftretenden Phänomene zu untersuchen. Das erlaubt selbst Wissenschaftlern, ohne sich in Widersprüche zu verwickeln, an Wunder zu glauben.

    Das führt zur Frage, wie im Text von Timm Grams auch schon angemerkt,

    Jedoch lässt sich keine Versuchsanordnung denken, mit der die Entscheidung einer Person durch Herstellen identischen Bedingungen überprüft werden könnte.

    welches Phänomen tritt denn überhaupt mehrfach auf?

  633. @Joker / 28.01.2021, 16:43 Uhr

    »Interessanter Mann,… «

    Ich hatte nicht nachgeschaut, was über Halfwassen im Netz zu finden ist, sondern nur, was es mit dem „Marsilius-Kolleg“ auf sich hat. Früh verstorben, der Mann…

    Und ja, es geht (ging) Halfwassen um das Weltbild des reduktionistischen Naturalismus (Gabriel hat noch das Attribut „naiv“ vorangestellt, sicherlich im Sinne von „gänzlich im Einklang mit der Wirklichkeit/Natur stehend“).

    Leider kann ich Halfwassens Argumentation nicht so recht nachvollziehen. Ob’s an meiner Voreingenommenheit oder am defizitären Textverständnis liegt, sei mal dahingestellt. Mir kommt es jedenfalls so vor, als würde Halfwassen zumindest an dieser Stelle das Ignoranzargument spielen:

    Dieses Erklärungsprogramm [bottom-up, vom Elementarsten zum „Geist“; B.] ist bisher allerdings nicht mehr als ein unerfülltes Versprechen. Bisher ist es nicht einmal im Ansatz gelungen, die Entstehung des Lebens aus der Physik und Chemie der Elementarteilchen zu erklären, noch die Entstehung des Bewusstseins aus der biologischen Evolution.

    Das ist in anderen Worten ziemlich genau das, was Tobias Jeckenburger immer wieder ins Feld führt, und auch Stephan argumentiert ja gerne damit, dass es an wissenschaftlich fundierten Erklärungen fehlt, etwa bei der Entstehung des Lebens oder bei den Funktionen des Gehirns.

    Ich bin da ziemlich ratlos. Auf welche Art von Erklärungen wird denn da gewartet? Man braucht sich doch nur die Entwicklung eines Menschen von der befruchteten Eizelle zum Kita-Kind (oder reifen Erwachsenen) anzuschauen: Entweder man postuliert eine Art Pfingstwunder, das da über das Kind kommt, oder man betrachtet „geistige“ Fähigkeiten als das Ergebnis selbsttätig ablaufender, natürlicher Entwicklungsprozesse.

    Was liegt näher, was klingt plausibler?

  634. Balanus,
    ….Entweder man postuliert eine Art Pfingstwunder, …..
    soweit muss man gar nicht gehen….es sind natürlich ablaufende Entwicklungsprozesse, was denn sonst. Glauben Sie, Gott ist ein Magier ?
    Gott bedient sich der Evolution. Das Geheimnis steckt in der Tatsache, dass es Sie gibt und dass Sie darüber nachdenken können.
    Gott hat das Universum erschaffen, und nach der Genesis mit den Worten: “Es werde Licht” und es ward Licht.

    Anmerkung , alles, was hier diskutiert wird ist natürlich und steht nicht im Widerspruch zur Bibel.
    Das Einzige aber Entscheidende ist, dass die Welt durch das Wort erschaffen wurde.

  635. Die Naturwissenschaften sind erst wenige Jahrhunderte, zum Teil erst Jahrzehnte alt, da glauben manche Leute, die Wissenschaft wäre schon am Ende angelangt. Was für ein Unsinn. Die Motivation dafür ist wohl, dass sie der Wissenschaft genau das zutrauen, wovor sie die größte Angst haben: die Offenlegung der Falschheit ihrer alten Weltbilder und ihrer Glaubensinhalte, somit die Schwächung ihres Selbstverständnisses und ihres gesellschaftlichen Ansehens und Einflusses.

    An vielen Stellen ist diese Entwicklung zu beobachten, die Antwort darauf ist oftmals Intoleranz und Aggression. Man sieht es am Islamismus, man sieht es an den Evangelikalen in den USA, sowie am allmählichen Übergang des moderaten Konservativismus in den radikalen Populismus.

  636. A.Reutlinger,
    Die alten Weltbilder sind am vergehen. Sie müssen jetzt noch sagen, welches Weltbild. Die ev. Kirche z.B. erkennt die Wissenschaft ausdrücklich an und ihr Weltbild steht nicht im Widerspruch zur Wissenschaft. Nennen Sie uns ein Beispiel.
    Was jetzt die USA betrifft , das ist ein eigenes Kapitel. Darauf haben wir keinen Einfluss , außer durch ein Bekenntnis zur Kirche. Nur so kann man Einfluss üben.

    Was den Islam angeht, der wird instrumentalisiert, damit die Machteliten eine Legitimation haben. Mit dem Wesen des Islam hat das nur noch wenig zu tun.

  637. @hwied / 28.01.2021, 20:11 Uhr

    »Glauben Sie, Gott ist ein Magier ?«

    » Gott bedient sich der Evolution. «

    » Gott hat das Universum erschaffen,… «

    Was glauben Sie?

  638. @Balanus

    [Pfingstwunder oder selbsttätig ablaufende Entwicklungsprozesse]
    Was liegt näher, was klingt plausibler?

    Mich musst Du da nicht fragen. Man verheddert sich allerdings selbst bei der Antwort ‘Pfingstwunder’ nicht zwangsläufig in Widersprüche. Dualisten, ich kenne einige, die noch andere Möglichkeiten ins Spiel bringen, sind durchaus logisch denkende Menschen mit, so sehe ich das zumindest, ebenfalls gesundem Menschenverstand.

    Es ist doch in der Physik eigentlich auch nicht gänzlich anders. Als man auf Phänomene stieß, die sich nicht gut erklären ließen, hat man einfach neue Kräfte postuliert oder die Ontologie nach Bedarf verändert. Trotz dessen, ganz zufrieden wirken die Physiker immer noch nicht. Demnach kann auch von dieser Seite noch was kommen.

    Insbesondere da es noch keine auch nur halbwegs befriedigende naturwissenschaftliche Antwort gibt zur Frage, wie Bewusstsein entsteht, ich hoffe darüber sind wir uns einig, verstehe ich die ganze Aufregung manchmal nicht.

    Es ist umgekehrt ja auch so, dass Dualisten ebenfalls nicht auf alle Fragen eine Antwort hätten. Die sind sich auch untereinander alles andere als einig. Da musst Du mal zuhören.

    Ich kann nur für mich sagen, deren Probleme möchte ich nicht haben.

    Auf welche Art von Erklärungen wird denn da gewartet?

    Das ist die Frage die schon Timm Grams im Text aufgeworfen hat. Korrelationen zwischen Hirnzuständen und Empfindungen anzugeben, das ist auf jeden Fall zu wenig. Was würdest denn Du als Erklärung akzeptieren, welche Form könnte die haben?

    Eine rudimentäre Erklärung aus philosophischer Sicht, die meiner Meinung nach bis jetzt am nächsten dran gekommen ist, obwohl sie auch schon wieder älter ist, stammt von Stephan Schleims Doktorvater: Thomas Metzinger, ‘Subjekt und Selbstmodell’. Online hier verfügbar.

    @Elektroniker, Quantenmechanik wird darin erwähnt, aber nicht gebraucht.

    @hwied, bevor Sie jetzt auch wieder diesen Thread für ihr Anliegen kapern, lesen Sie da mal rein. Es geht um das fünfte Welträtsel, Bewusstsein:

    ‘Die Perspektivität phänomenalen Bewußtseins vor dem Hintergrund einer naturalistischen Theorie mentaler Repräsentation’

  639. @Balanus / 28.01.2021, 18:40 Uhr

    »… oder man betrachtet „geistige” Fähigkeiten als das Ergebnis selbsttätig ablaufender, natürlicher Entwicklungsprozesse.«

    Kannst Du ein wenig präzisieren, was Du mit “Ergebnis selbsttätig ablaufender, natürlicher Entwicklungsprozesse” meinst? Beinhaltet das jetzt den Anspruch auf eine “prinzipiell vollständige naturgesetzliche Erklärbarkeit”? Oder anders gefragt, welche Kriterien sollten aus Deiner Sicht denn mindestens erfüllt sein, um etwas als ein “Ergebnis” solcher Prozesse zu beurteilen?

  640. Denkansatz 12
    Öffentliches Bewusstsein
    Im Bewußtsein seiner Verantwortung vor Gott und den Menschen, von dem Willen beseelt, als gleichberechtigtes Glied in einem vereinten Europa dem Frieden der Welt zu dienen, hat sich das Deutsche Volk kraft seiner verfassungsgebenden Gewalt dieses Grundgesetz gegeben.

    So steht es im deutschen Grundgesetz.
    Das Grundgesetz gibt den Rahmen vor für das öffentliche Bewusstsein.
    Die ersten 19 Artikel beinhalten die Grundrechte, Die sind bis auf Artikel 5 und 10 alle geisteswissenschaftlicher Natur.
    Explizit auf die Naturwissenschaften nimmt Artikel 10 Bezug, das sich auf die Informationstechnologie bezieht. Artikel 5 , das die Freiheit von Wissenschaft und Lehre garantiert, unterscheidet nicht zwischen Geisteswissenschaften und Naturwissenschaften.

    Aus der Sicht des Grundgesetzes gibt es keinen Grund anzunehmen, dass die Naturwissenschaften das Primat über die Wissenschaft hat. Eher umgekehrt.

    Interessant ist, dass das 6. Wort in der Präambel „Gott“ ist.

    Das bedeutet, dass die Religion in das öffentliche Bewusstsein eingebunden ist, ja man kann sagen, das öffentliche Bewusstsein ist auch ein religiöses Bekenntnis.
    Der Kalender ins seinem Jahreslauf bestimmt mit dem Sonntag und den Feiertagen das Bewusstsein.

    Tatsächlich ist der Staat mit der Kirche eine Verbindung eingegangen, wobei den Kirchen die sozialen Aufgaben zufallen.

    In diesem Bewusstsein können wir schon einen Überblick geben über Das Verhältnis von Wissenschaft zur Religion.

    1. Wenn Religion, Naturwissenschaft und Geisteswissenschaft gleichberechtig nebeneinander existieren, da ist die Freiheit garantiert.
    2. Wenn Religion allein das Primat hat, da wird die Naturwissenschaft in eine Nebenrolle gezwängt und die Menschenrechte sind in Gefahr.
    3. Wenn die Geisteswissensschaft das Primat hat, dann sprechen wir von einer Ideologie, als Beispiel sei der Kommunismus genannt, der eine Weltanschauung ist und keine Wissensschaft.

  641. Balanus,
    ….was glauben Sie…..
    Du sollst die kein Abbild von Gott machen, so steht es im Dekalog (altes Testament) Daran halte ich mich.
    Gott selbst tritt in der Bibel nur zweimal auf.
    1. Er sagt von sich selbst : ” Ich bin, der da ist”
    2. In Bezug auf Jesus sagt er :”Dies ist mein geliebter Sohn, an dem ich mein Wohlgefallen habe.”

    Alles was man sonst über Gott zu wissen glaubt ist Menschenwerk.

  642. @hwied

    » Alles was man sonst über Gott zu wissen glaubt ist Menschenwerk. «

    Also auch, dass er sich der Evolution bedient und das Universum erschaffen hat.

  643. @hwied
    Alles Wissen ist Menschenwerk. Alle Schriften sind Menschenwerk. Der Mensch ist ein Naturwesen, ein zufälliges Produkt der Evolution des Lebens. Er ist so vergänglich wie alle Lebewesen. Das Bewusstsein ist ein Irrweg der Evolution, der unausweichlich in Zerstörung der eigenen Lebenswelt mündet.

  644. @Joker, @Chrys, @all

    Erklärbarkeit, vollständige

    Vielleicht mal was Grundsätzliches aus meiner Sicht der Dinge.

    Meines Wissens tut sich der Physiker prinzipiell schwer damit, die Eigenschaften eines Systems aus den Eigenschaften seiner Komponenten herzuleiten. Man denke zum Beispiel an das aus Wasserstoff und Sauerstoff bestehende Wassermolekül. Dennoch hat der naturalistisch gesinnte Physiker keinen Zweifel daran, dass die Eigenschaften des Moleküls H2O allein von dessen Komponenten abhängen: Denn wird das O etwa durch S ersetzt, ändern sich die Eigenschaften des resultierenden Moleküls dramatisch.

    Das heißt, trotz der unvollständigen Erklärung sieht er keinen Grund zu der Annahme, dass es verborgene Variablen oder Kräfte gibt, mit denen sich die Systemeigenschaften basierend auf den Komponenteneigenschaften besser (oder gar vollständig) erklären ließen (zumindest habe ich dergleichen noch nicht in der Fachliteratur gelesen).

    Entsprechend verhält es sich meiner Auffassung nach auch bei höheren Systemebenen, also etwa bei der Bildung von Organen aus Zellen oder, grob gesprochen, Organismen aus Organen.

    Auf der Ebene der Quanten, also beim quantenmechanischen Zufall, schließt der Physiker ausdrücklich die Existenz verborgener Variablen aus. Warum meint er, als antinaturalistisch gesinnter Physiker, sie auf der Ebene der Organismen wieder einführen zu müssen?

    Das ergibt in meinen Augen keinen Sinn. Was aber, zugegeben, nicht bedeuten muss, dass es dennoch vernünftig sein könnte, unbekannte Variablen ins Spiel zu bringen. Und sei es auch nur, weil man sonst im Kopf keine kohärente, in sich schlüssige Vorstellung von den Naturvorgängen zustande bringt.

    Kurzum, ich benötige keinen bestimmten neurobiologischen Befund, der mir bestätigt, dass das phänomenale Erleben durch die Aktivität bestimmter Neuronenverbände zustande kommt, um endgültig verborgene Variablen oder Kräfte ausschließen zu können.

    Oder anders herum: Wenn man schlüssig zeigen könnte, dass das subjektive, phänomenale Erleben physikalisch wirksam ist, dann wäre ich wohl oder übel gezwungen, das anzuerkennen, dann hätten wir die andere Physik oder Naturwissenschaft, von der Stephan tausend Kommentare weiter oben sprach.

    (Danke für Ihre Aufmerksamkeit!)

  645. @anton reutlinger

    » Das Bewusstsein ist ein Irrweg der Evolution, der unausweichlich in Zerstörung der eigenen Lebenswelt mündet.«

    Die Evolution kennt keine „Irrwege“.

    (ich weiß, was Sie meinen, also keine Erklärung erforderlich)

  646. Balanus, Reutlinger,
    philosophische Antwort:
    Ein geschlossenes System, das Universum als Ganzes ist ein solches, wird zumindest so gedacht, kann sich aus sich selbst heraus nicht erklären.
    Deswegen scheitert die Naturwissenschaft, bei dem Versuch einen Gott zu widerlegen.
    Dass sich Gott der Evolution bedient, das ist eine logische Schlussfolgerung.
    Und die Logik, der Geist, gehört einer anderen Kategorie an als das Universum.
    Der Geist, die Logik, die kann das Universum erklären.
    Merke: ein logischer Gedanke ist mächtiger als das gesamte Universum.
    10 hoch 120 g, mit diesen vier Worten ist das gesamte Universum als Masse beschrieben. Läuft es dir da nicht kalt über den Rücken, bei der Fähigkeit zu denken.
    Nachtrag: Auch das Universum hat ein Bewusstsein . Wo? In jedem Menschen ?
    Was die Gehirnzelle für den Menschen ist, das ist das Hirn der Menschen für das Universum.
    Reutlinger
    Ein zufälliges Produkt ? Ich dachte in der materiellen Welt ist alles determiniert.

  647. @hwied

    » philosophische Antwort: «

    Mir scheint, hier zeigt sich ein Großteil des Problems mit der Philosophie.

  648. @hwied
    Sorry, aber das ist nichts als sinnleeres blabla. Man kann beliebige Wörter erfinden und ihnen eine beliebige Bedeutung geben. Oder man kann beliebige Informationen erfinden, es gibt immer Leute, die daran glauben.

  649. @Chrys // 28.01.2021, 23:36 Uhr

    »Kannst Du ein wenig präzisieren, was Du mit “Ergebnis selbsttätig ablaufender, natürlicher Entwicklungsprozesse” meinst? «

    Um mich nicht dem Vorwurf auszusetzen, ich sei der Frage ausgewichen:

    Das entscheidende Kriterium, das erfüllt sein muss, um etwas als ein „Ergebnis“ natürlicher, selbsttätig ablaufender Prozesse zu beurteilen, ist, dass man an keiner Stelle einen Hinweis auf verborgene Parameter findet.

    Würde zum Beispiel aus einem befruchteten Hühnerei ein Gänseküken schlüpfen, obwohl kein Gänserich Zugang zu der Henne hatte, dann bestünde Anlass, an irgendeiner Stelle einen künstlichen Eingriff zu vermuten. Wenn sich das aber nicht bestätigen sollte, dann wäre davon auszugehen, dass wir in der Biologie etwas Fundamentales noch nicht verstanden haben.

  650. Anton Reutlinger,
    …beliebige Wörter,
    Stellen Sie sich ein Plakat vor. Darauf steht etwas. Sie schauen sich das Plakat an und denken sich nichts dabei, weil sie den Sinn des Plakates nicht verstehen.
    Ein anderer Mensch sieht das Plakat und rennt wie in Panik davon.
    Ist Ihnen einmal aufgefallen, dass ich das Wort “glauben” nur sehr selten oder gar nicht verwende.

    Balanus
    geringschätzen Sie die Philosophie nicht.
    Sie ist der Ausgangspunkt auch der Naturwissenschaften.

  651. @ Balanus 29.01.2021, 11:33 Uhr

    Ich vermute zunächst einmal, es geht um die „Zuständigkeiten“ der Fachgebiete Physik, Quantenphysik, Chemie. Die „Quantenphysik“ wäre eigentlich das verbindende Element.

    Sie beruht auf dem Konzept der „Quantelung“, dass bestimmte Größen nur feste, diskrete Werte annehmen können. Sie „beginnt“ sozusagen damit, dass Elektronen in Atomen und Molekülen auch um so etwas wie „gemeinsame Elektronenbahnen fliegen“. Die chemischen Prozesse verlaufen im Bereich dieser (äußersten) „Elektronenhüllen“ gemäß den wirksamen „Kräftewirkungen“. („Spukhafte“ Fernwirkungen z.B. sind „Sensationen“ aber nicht das Alltagsgeschäft der Beteiligten).

    Die Chemie beschäftigt sich mit den chemischen Prozessen, deren Steuerung und die Auswirkungen. Die Quantenphysik mit den Kräften und den „tieferen Ursachen“.

    Dies war notwendig geworden, weil die klassische Physik z. B. bei der Beschreibung des Lichts oder des Aufbaus der Materie an ihre Grenzen gestoßen war.

    Die chemischen oder physikalischen Eigenschaften verschiedener Stoffe (Farbe, Ferromagnetismus, elektrische Leitfähigkeit ….) lassen sich nur quantenphysikalisch verstehen. Der „photoelektrische Effekt“ wäre eine Schnittstelle zwischen Physik und Quantenphysik. Es geht auch um Prozesse der Wechselwirkung von „Teilchen“ mit Materie, womöglich sogar in biologischen Körperzellen.

    Es scheint mir jedenfalls nicht ausgeschlossen, dass dabei z.B. „Farbempfindungseffekte“ entstehen könnten. („Irgendwo“ sollten sie jedenfalls entstehen, zumal es sie zweifellos „wirklich“ gibt, da Menschen oder auch Tiere auf Farben zuverlässig reagieren können….).

    Man könnte davon ausgehen, dass die verschiedenen physikalischen Kraftwirkungen in Wechselwirkungen mit „Teilchen“ treten könnten und dabei die vielfältigsten Effekte, besonders auch der (Bio)Chemie auftreten können.

    Zitat: „Oder anders herum: Wenn man schlüssig zeigen könnte, dass das subjektive, phänomenale Erleben physikalisch wirksam ist, dann wäre ich wohl oder übel gezwungen, das anzuerkennen, dann hätten wir die andere Physik oder Naturwissenschaft, von der Stephan tausend Kommentare weiter oben sprach.“

    Ich nehme an, Fachwissenschaftler sind an der „Sache dran“. Es ist höchstens die Frage ob „Normalos“ außerhalb der „Spezialisten“ darüber erfahren, bevor mit den Erkenntnissen fest „Kohle gemacht“ wurde. (Z.B. künstliche Lebensmittel mit beliebigen Geschmacksstoffen…).

  652. Elektroniker,
    Wechselwirkung ist das Schlüsselwort.
    Die Austauschteilchen sind namentlich benannt, gesehen haben wir noch nicht alle, und sie erklären wie etwas wirkt.
    Und es ist bekannt, dass das gesprochene Wort eine Wirkung zeigt.
    Subjektives phänomenales Erleben wäre damit wenigstens begrifflich zu verstehen. Also ein Gemälde, das muss nicht sprechen, allein das Sehen des Gemäldes bewirkt etwas.

  653. @Balanus / 29.01.2021, 11:33 Uhr

    Hinsichtlich wissenschaftl. Erklärbarkeit sollte man einen Aspekt nicht vergessen. Es besteht ein wesentlicher Unterschied zwischen einem observablen Phänomen, wie etwa einem vom Baum fallenden Apfel, und einer nomologischen (d.h., naturgesetzlichen) Erklärung, wie sie für den fallenden Apfel Isaac Newton sehr überzeugend gegeben hat. Der Apfel braucht keine Sprache, um vom Baum zu fallen, Newtons nomologische Erklärung erfordert indessen eine in einer mathemat. Sprache verfasste Modellierung des Vorgangs, um die in Gestalt von Gleichungen formulierten Gesetzmässigkeiten überhaupt in Anschlag bringen zu können.

    Newton hat damit letztlich das methodische Schema vorgegeben, nach welchem die Physik als idealtypische Naturwissenschaft an Phänomene zu deren Erklärung herangeht, was dann in vielen Fällen ja auch tatsächlich sehr gut funktioniert. Wer aber daraus zu schliessen geneigt ist, dass alle physischen Phänomene prinzipiell nach diesem Schema erklärbar sein müssten, der unterstellt implizit auch, dass die Unvollständigkeit eines zur Modellierung verwendeten mathemat. Sprachkalküls dabei niemals von Belang sein wird.

    Damit sind wir wieder bei Hofstadter und beim Thema Bewusstssein: Bei Vorgängen, die wir gemeinhin als eine unter massgeblicher Beteiligung von ich-Bewusstsein erfolgende Handlung interpretieren, könnte Hofstadter zufolge diese Unvollständigkeit der sprachlichen Mittel durchaus von Belang sein, wenn versucht wird, das Geschehen als rein naturgesetzlich erfolgenden Ablauf zu erklären.

    Auf Grenzen naturgesetzlicher Erklärbarkeit zu stossen ist demnach zu erwarten bei phänomenalen Mustern, die sich in einer formalen Modellierung zwar konsistent nachbilden lassen, aber nur durch Ausdrücke, die sich aus den konstitutiven Regeln des Modells nicht mehr herleiten lassen. Übernaturliches muss dazu nicht beschworen werden, da sind nur die “naturlichen” Grenzen unserer sprachlichen Mittel.

    29.01.2021, 12:32 Uhr

    Bezogen auf Dein Beispiel (»Entweder man postuliert eine Art Pfingstwunder, das da über das Kind kommt, oder man betrachtet „geistige” Fähigkeiten als das Ergebnis selbsttätig ablaufender, natürlicher Entwicklungsprozesse.«) würde Stephan vermutlich einwenden wollen, dass für das Ergebnis, also den Erwerb “geistiger” Fähigkeiten, die kulturelle Umgebung eine eminent wichtige Rolle spielt. Die soziale Erziehung des Kindes und alle daran Beteiligten müsstest Du schon für den Begriff “selbsttätig ablaufender, natürlicher Entwicklungsprozesse” irgendwie einbeziehen. Das wird noch recht schwierig, fürchte ich.

  654. Chrys,
    Hofstadter zu zitieren, das qualifiziert sie.
    Ja, die Unvollständigkeit der Sprache ist ein Argument für “Übernaturliches muss dazu nicht beschworen werden”.

    Es gibt nichts Übernatürliches, wenn man die Bibeltexte als einzige Metapher ansieht.

    Was immer noch unbeantwortet bleibt, das Universum selbst .
    Einfach anzunehmen, es ist einfach da, das ist der Taschenspielertrick der Physiker. (Wer hat es aus dem Hut gezaubert und warum)

  655. Naturwissenschaftliche Experimente (Informatik, Elektronik, Physik….) werden normalerweise mit Sorgfalt geplant. Fragen formuliert, eine passende „Umgebung“ für alle Prozesse, für die Messtechnik inkl. Software, allenfalls neue sprachliche und mathematischen Konstrukte geschaffen, um die zu erwartenden Sachverhalte geeignet beschreiben und auswerten zu können.

    Meistens wird auf frühere Experimente zurückgegriffen, Fehlermöglichkeiten analysiert und die Methodik verbessert.

    Besonders z.B. beim „Bewusstsein“ beginnt das Problem damit, dass man sich auf Begriffe einigen muss und vorher eine passende Struktur der Begriffe schafft.

    Die Begriffe müssen besonders „deklariert“ werden und wenn möglich sollte auf bestehende allgemeine philosophische Begriffe Rücksicht genommen werden, bzw. müssen diese besonders „deklariert“ werden.

    Das erfordert immer wieder Anpassungsprozesse, weil auf Erkenntnisse aus früheren Experimenten Rücksicht genommen werden muss. Angeblich werden mitunter Linguisten eingebunden.

    Auftretende unerklärliche Phänomene sollten aufgeklärt werden und möglichst korrekt interpretiert werden.

    Die Gehirnentwicklung von Kindern kann angeblich besonders gut nachvollzogen werden, wenn man die (im hohem Maße vorhandenen) psychologischen Erkenntnisse mit neueren technisch gewonnen) Erkenntnissen verknüpft.

  656. @Chrys
    Das Kind lebt in seiner individuellen Lebenswelt, so wie wir alle. Es nimmt diese Welt wahr, es kommuniziert und interagiert mit dieser Welt. Die Prozesse dazu finden im Nervensystem des Kindes statt, so dass sich das Kind dadurch selbstständig und selbsttätig zum erwachsenen Individuum entwickelt. Natürlich reagiert die Lebenswelt ihrerseits auf die Anwesenheit und das Verhalten des Kindes und versucht, darauf erzieherisch Einfluss zu nehmen, manchmal mit Erfolg.

    Aber letztlich bleibt es bei der selbsttätigen Entwicklung des Kindes. Niemand kann unmittelbar das Nervensystem des Kindes manipulieren, außer durch Manipulationen an der “Benutzeroberfläche” des Kindes, d.h. durch psychologisch-kommunikative Spielchen wie Drohungen und Versprechungen oder durch körperliche Gewalt.

    Als Monist bin ich überzeugt, dass jeder psychische oder geistige Prozess Entsprechungen im Nervensystem hat, d.h. es sind im Kern Prozesse im Nervensystem mit Auswirkungen auf die Peripherie. Manche Prozesse davon sind bereits erforscht: man weiß wie sinnliche Wahrnehmungen funktionieren, man weiß zumindest zum Teil wie Sprache erzeugt wird, wie Muskelbewegungen entstehen, man kennt die Funktionen von Hormonen. Auch das Problem der Funktionsweise des Bewusstseins ist lösbar. All das berechtigt zur Akzeptanz des Monismus.

  657. @hwied
    Was immer noch unbeantwortet bleibt, das Universum selbst .

    Ja, das bleibt unbeantwortet, höchst wahrscheinlich für immer, außer es findet sich noch ein übernatürliches Genie. Aber was soll’s, ändert es etwas an unserem Leben? Muss man unbedingt eine Antwort darauf haben, oder erfinden? Nützen willkürliche, irrationale Fiktionen oder Phantasien, oder altertümliche Legenden etwas? In Wirklichkeit lenken sie nur vom tatsächlichen Leben ab.

  658. Anton Reutlinger,
    ” In Wirklichkeit lenken sie nur vom tatsächlichen Leben ab.”
    Dazu eine Episode aus dem Leben.

    Auf dem Friedhof. Die Großmutter war gestorben. Der Enkel, 4 Jahre alt ,stand weinend herum.
    Da fragte der Enkel seine Mutter : Wo ist die Oma ? Die Mutter: Die Oma ist unter der Erde.
    Das Kind heulte weiter und ging zu einer Tante: Kind: “Stimmt das, die Oma ist unter der Erde?“
    Tante (schaute hoch zum Himmel zu einer Wolke). „ Die Oma ist im Himmel und schaut auf dich herab“
    Da ging ein Lächeln über das Gesicht des Kindes und es schmiegte sich an seine Tante.
    Ohne weiteren Kommentar!

  659. @Balanus / 29.01.2021, 11:33 Uhr / Zusatz

    »Auf der Ebene der Quanten, also beim quantenmechanischen Zufall, schließt der Physiker ausdrücklich die Existenz verborgener Variablen aus.«

    Das stimmt so nicht ganz und wird anscheinend häufiger missverstanden. Bell’s Theorem schliesst Lokalität aus und mithin die Möglichkeit, durch die Annahme verborgener Variablen Lokalität irgendwie erreichen zu können. Die Bohmsche Mechanik ist eine Theorie mit verborgenen Variablen, aber sie ist auch nicht-lokal und mit Bell wunderbar verträglich.

    Ausserdem ist sie deterministisch — Einstein ist also mit seinem Diktum “Der Alte würfelt nicht” noch immer im Rennen.

  660. @Chrys / 29.01.2021, 16:52 Uhr

    »Die soziale Erziehung des Kindes und alle daran Beteiligten müsstest Du schon für den Begriff “selbsttätig ablaufender, natürlicher Entwicklungsprozesse” irgendwie einbeziehen. Das wird noch recht schwierig, fürchte ich.«

    Die Schwierigkeit besteht vor allem darin, den an sich simplen Sachverhalt so zu vermitteln, dass er für jeden einsichtig wird.

    Im Grunde geht es darum, dass ein Organismus—physikalisch gesehen—ein gegenüber der Umwelt weitgehend abgeschlossenes System darstellt, mit Lunge, Verdauungstrakt und Haut als selektive Grenzflächen zur Außenwelt.

    Das Gehirn erhält seinen Input allein über periphere Nervenendigungen und diverse Sensoren (Blutversorgung und Hormone lassen wir mal außen vor).

    Es ist gewissermaßen Teil des genetischen Programms, dass die Selbstorganisation des Gehirns auf sensorischen Input angewiesen ist, Input, den das Nervensystem sich aus der Umwelt holen muss, also aktiv aufnehmen muss, es gibt keine andere Möglichkeit. Wesentlicher Teil dieser Umwelt ist das soziale (kulturelle) Umfeld, damit das Gehirn seine Funktion als Sozialorgan anlagengemäß ausbilden kann (Spracherwerb gehört z. B. dazu).

    Alles in allem also ganz einfach und einleuchtend, nicht wahr?

    (Näheres findet sich in der einschlägigen Fachliteratur)

  661. @Chrys / 29.01.2021, 23:52 Uhr

    »Ausserdem ist sie [die Bohmsche Mechanik] deterministisch — Einstein ist also mit seinem Diktum “Der Alte würfelt nicht” noch immer im Rennen.«

    Aber das macht die Rede vom quantenmechanischen Zufall nicht falsch, hoffe ich. Denn dass man den exakten Zeitpunkt eines bestimmten Quantenereignisses nicht vorausberechnen kann, ist von elementarer Bedeutung für viel biologische Phänomene.

  662. @Balanus

    Die soziale Erziehung des Kindes und alle daran Beteiligten müsstest Du schon für den Begriff “selbsttätig ablaufender, natürlicher Entwicklungsprozesse” irgendwie einbeziehen. Das wird noch recht schwierig, fürchte ich. (@Chrys)

    Mach dir keine Gedanken, ist nicht so schwer.

    Wenn die Fähigkeiten des Protogeists (des ersten und damit bis dahin einzigen über Bewusstsein verfügenden Wesens) das Ergebnis selbsttätig ablaufender, natürlicher Entwicklungsprozesse waren, dann folgt das für soziale Wesen wie uns durch Vererbungsgesetze und vollständiger Induktion. Das wird durch das ganze gerade herrschende Gewusel, das Soziale, nur etwas verschleiert.

    Falls an der Stelle allerdings ein Wunder geschehen ist … und “Wunder gibt es immer wieder” (Katja Ebstein, Quelle) – gut, also vielleicht doch schwer.

  663. Balanus
    …genetisches Programm…..
    dieses Programm hat durch das Neue Testament ein update bekommen, wo so etwas “Widersinniges”, wie , Liebe deine Feinde, verlangt wird.

    Ich denke, dafür gibt es keine Erklärung im Sinne der Evolution.
    Die religiösen Texte stellen etwas Neues dar, das dem Selbsterhaltungstrieb geradezu entgegenläuft.

  664. Das stimmt so nicht ganz und wird anscheinend häufiger missverstanden. Bell’s Theorem schliesst Lokalität aus und mithin die Möglichkeit, durch die Annahme verborgener Variablen Lokalität irgendwie erreichen zu können. [“Chrys”]

    Dies hier :
    -> https://de.wikipedia.org/wiki/Bellsche_Ungleichung

    …habe ich nie verstanden.

    Unbestimmte, verborgene Variablen können bei der sog. Verschränkung nie ausgeschlossen werden.

    MFG
    Dr. Webbaer

  665. Alles Wissen ist Menschenwerk. Alle Schriften sind Menschenwerk. Der Mensch ist ein Naturwesen, ein zufälliges Produkt der Evolution des Lebens. Er ist so vergänglich wie alle Lebewesen. Das Bewusstsein ist ein Irrweg der Evolution, der unausweichlich in Zerstörung der eigenen Lebenswelt mündet.

    Klingt nicht schlecht, Herr Reutlinger, das Bewusstsein kann allerdings kein ‘Irrweg’ sein, wenn es so beschreibt.

  666. @ Kommentatorenfreund ‘Joker’ (a.k.a. ‘Jolly”) und hierzu kurz :

    Wenn die Fähigkeiten des Protogeists (des ersten und damit bis dahin einzigen über Bewusstsein verfügenden Wesens) das Ergebnis selbsttätig ablaufender, natürlicher Entwicklungsprozesse waren, dann folgt das für soziale Wesen wie uns durch Vererbungsgesetze und vollständiger Induktion. Das wird durch das ganze gerade herrschende Gewusel, das Soziale, nur etwas verschleiert.

    Ist der Geist erst einmal entstanden, ist er erst einmal nicht mehr so leicht in die “Flasche” zu kriegen.
    Sicherlich ist Kommentatorenfreund “Bal” indolent-dull.

    MFG
    Wb

  667. @ Kommentatorenfreund ‘Joker’ und hierzu kurz :

    Insbesondere da es noch keine auch nur halbwegs befriedigende naturwissenschaftliche Antwort gibt zur Frage, wie Bewusstsein entsteht, ich hoffe darüber sind wir uns einig, verstehe ich die ganze Aufregung manchmal nicht.

    Dr. Webbaer, bereits in sehr jungen Jahren SciFi-erfahren geworden, weiß genau, was ‘Bewusstsein’ beideutet, nämlich genau das, was erkennende Subjekte meinen, wenn sie derart begrifflich bilden, die Rekursion ist sicherlich bemerkt worden, “Opi” ist ja Konstruktivist.

    In den letzten Jahrzehnten hat Dr. Webbaer “nur sehr selten” zur SciFi gegriffen, er war dann meist bei Philip K. Dick und Terry Pratchett (gilt als Fantasy, die Unterscheidung zur SciFi bleibt m.E. unklar, wenn ein derart gebildeter und netter Mensch (vs. Bär) derart theoretisiert).

    Der Geist ist kein Gegenstand der Naturwissenschaften.

    MFG
    Dr. Webbaer

  668. @anton reutlinger, Balanus / Kindererziehung

    Rückblende: Friedrich II. (1194–1250), Kaiser des HRR, soll dereinst eine Studie beauftragt haben, bei der Kinder aufgezogen wurden, mit denen kein Wort je gesprochen wurde. Man wollte so herausfinden, was die gottgegebene Sprache des Menschen sei, ob Latein oder etwas anderes. Den gelehrten Herren jener Tage war völlig klar, dass die schon irgendwie sprechen würden, denn das kann ja gar nicht anders sein, dafür hat der Herrgott schon gesorgt. Die Ergebnisse der Studie wurden meines Wissens jedoch nie publiziert…

    Gegenwart: Einigen gelehrten Herren dieser Tage scheint es völlig klar zu sein, dass Kinder auch dann geistige Fähigkeiten, Bewusstsein und Verstand entwickeln würden, wenn sie gleichsam in totaler Isolationshaft und nur mit dem physisch unbedingt Lebensnotwendigen versorgt aufwachsen würden. Denn das kann ja gar nicht anders sein, dafür hat die Natur schon gesorgt. Entsprechende Studien durchzuführen verbietet sich wohl aus ethischen Erwägungen…

    Bleibt nur das Gedankenexperiment. Ich habe zwar so meine Vorstellungen, was als Ergebnis dabei herauskommen würde, aber ich da ja kein Experte. Das Thema würde ich lieber dem Blogherrn überlassen, der ist schliesslich hier der theoret. Psychologe. — Balanus, hast Du bei Stephan nicht noch einen Themenwunsch frei? Wär’ das nicht was?

    Zitat Balanus: »Im Grunde geht es darum, dass ein Organismus—physikalisch gesehen—ein gegenüber der Umwelt weitgehend abgeschlossenes System darstellt, mit Lunge, Verdauungstrakt und Haut als selektive Grenzflächen zur Außenwelt.«

    Vermittelt e.g. Ilya Prigogine da nicht eher gerade den gegenteiligen Eindruck, wenn er vitale Organismen mit “dissipativen Strukturen” in Verbindung bringt? Siehe etwa auch

    Goldbeter, Albert (2018) Dissipative structures in biological systems: bistability, oscillations, spatial patterns and waves. Phil. Trans. R. Soc. A. 376:20170376. DOI: 10.1098/rsta.2017.0376

  669. @chrys
    Zur Kindesentwicklung.

    Die “Probanden” von Friedrich II. sind meines Wissens an dem Experiment gestorben. Wenn ich mich recht erinnere, gab es im Altertum noch ein zweites (bzw. erstes) solches Experiment, von Pharao Psammetich I, berichtet von Herodot.

    Aber es gibt auch neuere Beobachtungen dazu, bei den sogenannten “Wolfskindern” (Kaspar-Hauser-Syndrom) und bei Kindern in Waisenhäusern in den 1940er Jahren, die von Psychologen (René Spitz) später untersucht wurden, als Nachwirkungen von Hospitalisierung bzw. als Hospitalismus.

  670. Rainer Wolf: “Der Behauptung von Timm Grams, die Thesen des Naturalismus enthielten logische Widersprüche, kann ich also nicht folgen.”
    Joker: “Sie können keine Argument widerlegen, indem Sie Ihre Sicht der Dinge einfach dagegenhalten.”

    Der angebliche logische Widerspruch wurde von Martin Mahner, Martin Holzherr und mir als Strohmann entlarvt.

    Rainer Wolf: “Popper begeht – wie viele andere Dualisten – einen Kategorienfehler”
    Joker: “Popper sagt – wie viele andere Dualisten – das Gegenteil. Und jetzt?”

    Wenn eine per Definition immaterielle Entität (“Geist”) mit materiellen Objekten wechselwirken können soll, dann kann die immaterielle Entität nicht vollständig immateriell sein, denn sie weist zumindest eine essentielle Eigenschaft materieller Objekte auf, nämlich die Fähigkeit zur physischen Wechselwirkung. Das scheint mir doch ein Fehler zu sein.

    Joker: “So wie Sie das beschreiben wird der Naturalismus tatsächlich zur Ideologie.”

    Ich erkenne an, dass das gemäßigter formuliert ist als die weiter oben direkt gegen meine Person gerichtete Anschuldigung. Da steht immerhin nicht, dass die Person P ideologisch unterwegs sei, sondern lediglich dass eine These N durch eine bestimmte Argumentationsweise in eine Ideologie verwandelt würde.

    Ein Ideologievorwurf beendet üblicherweise eine rationale Diskussion. Warum ist das so? “Ideologie” wird sofort mit unhaltbaren Aussagesystemen assoziiert, die zur Ermordung Andersdenkender führten, wie im Falle der Inquisition, des Stalinismus etc. Das soll zunächst nur den stark pejorativen Charakter der Vokabel “Ideologie” erläutern. Als Beleg für diesen Begriffsgebrauch sei auch Wikipedia zitiert: “Der Vorwurf einer durch Ideologie bestimmten Argumentation findet sich häufig im politischen Diskurs. Damit wird unterstellt, dass ein Standpunkt deswegen nicht stichhaltig sei, weil er auf einer politischen Ideologie basiere. Der eigene Standpunkt wird demgegenüber implizit oder explizit so dargestellt, dass er auf einer nüchternen Analyse der Wahrheit, dem gesunden Menschenverstand oder auf einer nicht in Frage zu stellende Ethik beruhen würde.”

    Ein neutraler Begriffsinhalt spielt für die Vokabel “Ideologie” daher praktisch kaum eine Rolle. Wenn er dennoch intendiert sein sollte, müsste das explizit kenntlich gemacht werden.

    Ein Charakteristikum von Ideologien im negativen Sinne besteht meist in der Abschottung gegenüber Kritik. Mit beliebigen Vorkommnissen kompatible Aussagen sind kritikimmun gegen jegliche Erfahrung. Bei Popper heißt es, diese Aussagen seien nicht falisfizierbar. Mit einem immateriellen Geist führt Popper allerdings ein kritikimmunes Konzept ein. Das steht in einer starken Spannung zu seinem kritischen Rationalismus.

    Immaterielle Entitäten sind definitionsgemäß nicht an naturgesetzliches Verhalten gebunden. Daher ist ihre Existenz mit beliebigen Ereignissen kompatibel. Es kann nichts passieren, das ihre Existenz in Frage stellen könnte, woraus die Kritikimmunität folgt.

    Materie hingegen muss sich definitionsgemäß naturgesetzlich verhalten. Über festgestellte Ereignisse, die sich nicht naturgesetzlich fassen lassen, lässt sich dieses Konzept kritisieren. Starke Aussagen, die sich daher der Kritik aussetzen, fallen nicht unter den Begriff einer abgeschotteten Ideologie.

    Und genau so hat Rainer Wolf auch argumentiert: “Es gibt jedenfalls bisher keine belastbaren Hinweise darauf, dass immaterielle, geistige Entitäten einen kausalen Einfluss haben auf das materielle Geschehen im Gehirn. Gäbe es sie, läge aus heutiger Sicht ein paranormales Phänomen vor mit dem Potenzial, den Naturalismus zu falsifizieren. Für den ersten Existenznachweis eines paranormalen Phänomens hat die James Randi Educational Foundation seit Jahrzehnten 1 Million Dollar ausgelobt. Auch bei den wissenschaftlichen Psi-Tests der GWUP, die wir seit 2004 am Würzburger Biozentrum jährlich durchführten, konnte bisher noch kein Bewerber paranormale Fähigkeiten glaubhaft demonstrieren.”

    Ich halte es daher für unangemessen oder zumindest extrem unverständlich, dafür die Vokabel “Ideologie” zu gebrauchen.

    Das bedeutet nun nicht, lieber Joker, dass Sie ihre Position ändern müssten. Das braucht niemand. Es herrscht Meinungsfreiheit. Abgesehen davon ist die Vielfalt bekannter Positionen eine Ressource, denn gute Theorien sollen sich an Kritik bewähren.

    Noch eine allgemeine spieltheoretische Bemerkung: Die Länge dieses Postings im Vergleich zum vorangegangenen zeigt, dass Kritikeinwürfe deutlich geringere Kosten verursachen als ihre Widerlegung. Damit entsteht ein Denial-of-Service-Problem mit dem Potenzial, den Diskurs lahm zu legen. Verschärft wird dieses Problem durch die zahlreichen Wiederholungen. Ein möglicher Ausweg aus diesem Deadlock sind Argumentsammlungen, wie z.B. hier. Das schafft Übersicht und ist ein guter Anfang, löst das Problem aber noch nicht ganz, denn die gleichen Argumente kommen auch in Verkleidung.

    Aus gegebenem Anlass verweise ich auch auf meinen Prolog.

  671. @Chrys

    29.01.2021, 16:52 Uhr

    »… unter massgeblicher Beteiligung von ich-Bewusstsein erfolgende Handlung…«

    Worin könnte die „maßgebliche“ Beteiligung eines Ich-Bewusstseins wohl bestehen?

    Ich kenne keine neurobiologische Bewusstseins-Theorie, die eine maßgebliche Beteiligung des Ich-Bewusstseins im Sinne von wirksam werdenden Bewusstseinsinhalten beinhalten würde.

    Neurobiologisch gesehen findet alles phänomenale Erleben auf der Ebene der Neuronen statt, es bilden sich keine „Seltsamen Schleifen“ (Hofstadter). Diese tauchen erst auf, wenn man wie Hofstadter von unterschiedlichen „Stufen“ ausgeht, wobei die oberste auf die unterste zurückgreift und auf sie einwirkt (!), und zugleich die unterste durch die oberste bestimmt ist.

    » Auf Grenzen naturgesetzlicher Erklärbarkeit zu stossen ist demnach zu erwarten bei phänomenalen Mustern,.. «

    Es steht m. E. außer Frage, dass man spezifische Sinnesempfindungen nicht naturgesetzlich herleiten kann. Da besteht überhaupt klein Dissens. Naturgesetzlich laufen allein die neuronalen Prozesse ab, die von Sinnesempfindungen begleitet sind.

    30.01.2021, 12:30 Uhr

    »Rückblende: Friedrich II. (1194–1250), «

    Diese (unsichere) Geschichte habe ich ganz bewusst nicht aufgeführt als wissenschaftlichen Beleg für meine Ausführungen. Aber es klingt plausibel, dass ein derartiges „Experiment“ so wie erzählt enden würde.

    » Einigen gelehrten Herren dieser Tage…«

    Kann man Näheres zu diesen Herren erfahren, kannst Du Quellen benennen? Das sind ja recht abenteuerliche Thesen, die da vertreten werden.

    » Vermittelt e.g. Ilya Prigogine da nicht eher gerade den gegenteiligen Eindruck, wenn er vitale Organismen mit “dissipativen Strukturen” in Verbindung bringt? «

    Nun, das käme darauf an, ob er im Rahmen naturwissenschaftlicher Erkenntnisse bleibt oder nicht. Dem einleitenden ersten Satz im Abstract der von Dir verlinkten Studie nach scheint aber diesbezüglich alles in Ordnung zu sein (meine Fettung): .

    …how relevant is the concept of dissipative structure for understanding the dynamical bases of non-equilibrium self-organization in biological systems,…

  672. @Feodor 30.01.2021, 14:08 Uhr

    Du schreibst:

    Der angebliche logische Widerspruch [die Thesen des Naturalismus enthielten logische Widersprüche] wurde von Martin Mahner, Martin Holzherr und mir als Strohmann entlarvt.

    Nichts wurde hier “entlarvt”. Mir hat man hin und wieder widersprochen. Das ist in einer Debatte durchaus üblich; “entlarven” ist in dem Zusammenhang eine unpassende Vokabel. Ich finde es hingegen ausgesprochen unfair, einzelne Widersprüche aus einem über 700 Einträge langen Diskussionsfaden herauszupicken und den sich jeweils daran anschließenden Fortgang der Debatte auszublenden.

    Auf den Strohmann-Vorwurf, der auch von Joker angesprochen wurde, bin ich eingegangen und habe ihn meines Erachtens vollständig entkräftet, und zwar in meinen Posts vom 26.01.2021, 17:55 Uhr und vom 27.01.2021, 10:32 Uhr.

  673. @ Balanus 30.01.2021, 14:44 Uhr

    Zitat: „Neurobiologisch gesehen findet alles phänomenale Erleben auf der Ebene der Neuronen statt, es bilden sich keine „Seltsamen Schleifen“ (Hofstadter). ….

    Es steht m. E. außer Frage, dass man spezifische Sinnesempfindungen nicht naturgesetzlich herleiten kann. Da besteht überhaupt klein Dissens. Naturgesetzlich laufen allein die neuronalen Prozesse ab, die von Sinnesempfindungen begleitet sind.“

    Ich würde Sie um ihre (eher „neurologische“) Sichtweise zu den folgenden Aussagen bitten:

    Könnten unter „Seltsamen Schleifen“ auch so etwas wie elektrische „Rückkopplungsschleifen“ wie sie z.B. bei Epilepsie, Tremor …. auftreten verstanden werden? Die werden z.B. mit elektrischer Reizen eines Hirnschrittmachers „außer Tritt“ gebracht.

    Wären für Sie spezifische Sinnesempfindungen „naturgesetzlich hergeleitet“ wenn es gelänge, nehmen wir einmal an, z.B. bestimmte typische „Verhaltensmuster“ an jeweils bestimmten biologischen molekularen Strukturen im Detail nachzuweisen, die mit hoher Wahrscheinlichkeit bei mehreren Tierarten, bei einem offensichtlichen, „fast sicheren“, Rot-, Blau- oder z.B. auch einem „Orgasmusempfinden“ auftreten? Das Empfindungsphänomen sozusagen „zwingend auftretende eindeutige Begleitmusik“ bei jeweils ganz bestimmten chemischen bzw. quantenphysikalischen Prozessen ist?

  674. @Feodor 30.01. 14:08

    „Wenn eine per Definition immaterielle Entität (“Geist”) mit materiellen Objekten wechselwirken können soll, dann kann die immaterielle Entität nicht vollständig immateriell sein, denn sie weist zumindest eine essentielle Eigenschaft materieller Objekte auf, nämlich die Fähigkeit zur physischen Wechselwirkung. Das scheint mir doch ein Fehler zu sein.“

    Ich sehen da keinen Fehler. Die immaterielle Entität muss ja auch nicht vollständig immateriell sein, wenn sie für unser Bewusstsein eine wesentlich Rolle spielt. Ein Nachweis der Wirkung der immateriellen Entität auf das Nervensystem würde genügen, und sich im weiteren die Gelegenheit bieten, hier die Theorien der Physik zu erweitern, sofern das dann wieder gelingt.

    Das wäre dann auch wieder eine ganz natürliche Angelegenheit, die auch Naturalisten theoretisch problemlos integrieren könnten. So weit sind wir aber noch nicht.

    Die Frage ist eben: gibt es im Gehirn den Effekt, dass ein zumindest teilweise geistiges Bewusstsein wieder auf die Nervenzellen zurückwirkt oder nicht. Die Antwort steht noch aus, aber die zukünftige Hirnforschung wird diese Frage beantworten können. Das bleibt eben leider abzuwarten.

    Sie und andere Kommentatoren kommen immer wieder darauf zurück, dass ihre rein materielle Weltvorstellung auch zukünftigen Forschungen standhalten wird, obwohl Sie dieses nicht wissen können, aber gerne glauben dürfen. Falsifizierbar heißt eben auch, dass es tatsächlich falsifiziert werden kann. Warten wir es ab.

    Andere glauben aber eben auch mal andere Sachen.

    „Mit einem immateriellen Geist führt Popper allerdings ein kritikimmunes Konzept ein.“

    Wo ist das denn kritikimmun? Wenn wir unser Konnektom eingescannt, verstanden und simulierbar gemacht haben, kommt doch die Entscheidung. Wird das ohne jede Geistesbeteiligung laufen, in der Simulation also ohne analoge Zufallszahlen, wäre das Konzept einer Geistesbeteiligung eindeutig widerlegt. Und damit bewiesen, dass unsere Nervenzellen ganz alleine Bewusstsein hervorbringen können.

    Also wäre damit die Wirkung von immateriellem Geist zumindest im Gehirn falsifiziert. Und wenn es im Gehirn unwirksam ist, macht das ganze Konzept vom immateriellem Geist keinen Sinn mehr.

    Und da werden wir vermutlich so lange gar nicht mehr drauf warten müssen. Ein Mäusekonnektom müsste reichen.

    „Immaterielle Entitäten sind definitionsgemäß nicht an naturgesetzliches Verhalten gebunden.“

    Das ist eine Einschränkung, die willkürlich und unnötig ist. Geist, der völlig unabhängig von der Welt existiert, wäre vollkommen uninteressant.

  675. @Elektroniker / 30.01.2021, 17:21 Uhr

    »Wären für Sie spezifische Sinnesempfindungen „naturgesetzlich hergeleitet“ wenn es gelänge,… «

    Nein. Hierfür müsste gezeigt werden, warum bei einer bestimmten physischen Gegebenheit z. B. stets nur Rot und niemals Blau empfunden werden kann. Doch dazu müssten physikalische Grenzen überwunden werden. Wenn wir aber jenseits dieser Grenzen sind, dann ist es vorbei mit dem Naturgesetzlichen. So sehe ich das.

    Wenn ich das mit den „Seltsamen Schleifen“ von Hofstadter richtig verstanden habe, dann sind damit keine Rückkopplungen oder sonst welche Regelkreise gemeint. Diese Schleifen sind ja gerade deshalb „seltsam“, weil sie sonst nirgendwo vorkommen bzw. beschrieben wurden, weder in der Physik, Chemie noch Biologie.

  676. @Jeckenburger
    Was Sie meinen, das sind Schnittstellen zwischen unterschiedlichen Systemen, so dass solche Systeme miteinander kommunizieren oder interagieren können. Die Sinneszellen sind solche Schnittstellen, indem sie Signale der Außenwelt, wie Licht und Schall, in Nervensignale übersetzen.

    Zwischen immateriellen und materiellen Systemen liegen die Unterschiede nicht im physikalischen, sondern im philosophischen Bereich, weil “immateriell” nicht definiert und nicht definierbar ist. Oder können Sie es? Wie könnten also Schnittstellen vom und zum immateriellen System aussehen?

    Es ist immer wieder erstaunlich, wie naiv manche Leute trotz technischer oder naturwissenschaftlicher Bildung über immaterielle Dinge reden, als hätten sie eine genaue Vorstellung davon, als gäbe es objektive Kriterien dafür, außer Worten und Phrasen. Welche Eigenschaften, welche Funktionen, welche Wechselwirkungen zeigen immaterielle Dinge? Gibt es irgend etwas, über das sie mit materiellen Dingen kommunizieren oder interagieren könnten?

    Die Philosophen des Immaterialismus drehen sich im Kreis, im Kreis, im Kreis …
    Es gibt keine Möglichkeit der direkten Interaktion zwischen materiellen und immateriellen Dingen, es gibt keine Schnittstellen zur Übersetzung von Signalen. Dass Gedanken und Emotionen Verhalten hervorrufen, ist eine Folge ihrer Materialität, ihres Wesens als Nervensignale.

  677. T.Jeckenburger
    materiell, immateriell

    Wenn alles nur materiell wäre, wo bliebe dann die Phantasie. Wir können uns Dinge ausdenken, die es vorher nicht gab.
    Das Wesen der Kunst ist die menschliche Vorstellungskraft verknüpft mit einem schöpferischen Geist.

    Hier wird von Emotionen geredet, die man mit chemischen Substanzen beeinflussen kann. Das wussten schon die Griechen. Aber die chemischen Substanzen können nicht den Inhalt der Gedanken steuern. Isaak Newton war ein Genie, weil er Geist hatte, mehr Geist als andere. Solche Wissenschaftler gibt es in einem Jahrhundert nur einmal. Geist auf Bestellung, das geht nicht.
    Die Grenzen der Menschheit sind biologische Grenzen, aber nicht nur.
    Ein Zirkuspferd muss immer im Kreis laufen. Ein Mensch schneidet das Seil durch.

  678. Bin zufällig nochmal auf die Seite gestolpert und muss sagen, es juckt mich in den Fingern.
    Wir (Materialisten) sind uns einig, dass alles, was in diesem Universum geschieht, materieller Natur ist. Ob es regnet oder Neuronen feuern, alles ist materiell. Wenn ich Schmerz spüre, ist auch das materiell, was sollte es auch sonst sein. Jetzt kommt aber das Entscheidende: 1. Sobald ich diesem Schmerz eine (sprachliche) Bedeutung zuordne, ist diese selbstverständlich nicht mehr Gegenstand der Physik und daher ist es völliger Unsinn, hier einen Dualismus sehen zu wollen. Es ist ein rein sprachlicher Dualismus, denn ich bewege mich in zwei unterschiedlichen terminologischen Systemen. Selbst der physikalische Begriff ‚Masse‘ ist nicht physikalisch, sondern eben sprachlich. Das zweite ist die ständige Vermischung zwischen Physik und Qualia. Der physikalische Umstand, der bei mir Schmerz auslöst, lässt sich empirisch feststellen, etwa durch Hirnscans in Verbindung mit einer entsprechenden Äußerung des Probanden. Mein persönliches Schmerzempfinden aber ist wiederum etwas anderes als das physikalische Korrelat, bewegt sich ebenfalls auf einer anderen Beschreibungsebene als die der Physik. Das also, was wir Qualia nennen, kann nicht physikalisch untersucht werden.
    Und nun nochmals zum Bewußtsein. Wenn wir Bewußtsein definieren als Gesamtheit aller kognitiven (Psychologie) bzw. zentralnervös-neuronalen (Neurophysiologie) Vorgänge, dann dürfte diese Eigenschaft allen Lebewesen ab einer bestimmten Entwicklungsstufe zukommen. Die kognitiven Fähigkeiten wären dann von der Neuronenzahl, aber vor allem auch von der Zahl der Verknüpfungen und deren Verbindung mit verschiedenen Hirnarealen abhängig. Wie die Lebewesen einschließlch uns Menschen verschiedene Zustände selbst wahrnehmen bzw. empfinden, ist eine ganz andere Frage und kann, wie oben gesagt, nicht naturwissenschaftlich beantwortet werden.
    Zum Schluss möchte ich nochmal auf die Methodik zu sprechen kommen. Wir werden das, was wir Bewusstsein nennen, nicht dadurch erklären können, daß wir untersuchen, wie sich einzelne Atome im Gehirn verhalten, ebenso wenig, wie wir ein Gesellschaftssystem beschreiben können, indem wir einzelne Individuen beschreiben. Das heißt nicht, dass die mikroskopische Betrachtung obsolet wäre, im Gegenteil, aber wir müssen sie ergänzen.
    Übrigens, genauso wie eine Gesellschaft Regeln aufstellt, welche die Handlungen ihrer Mitglieder determiniert, verhält es sich mit allen komplexen Systemen. Und dafür braucht es keine zusätzliche Kraft.
    Und nun verabschiede ich mich endgültig.

  679. @anton reutlinger 30.01. 00:20

    „Welche Eigenschaften, welche Funktionen, welche Wechselwirkungen zeigen immaterielle Dinge? Gibt es irgend etwas, über das sie mit materiellen Dingen kommunizieren oder interagieren könnten?“

    Geisteswelten könnten über gezielte Zufälle in der Quantenwelt mit der materiellen Welt interagieren. Die dazu nötige Vorausplanung und überhaupt die existierende und tätige Intelligenz der Geisteswelten wäre dann darüber hinaus nicht mehr zu beobachten. Das kann man sich vielleicht wie eine riesige Informatik vorstellen, die zu den verborgenen Eigenschaften des Vakuums gehört – aber das eben ist eine eigene Welt, von der man wirklich nicht wissen kann, wie sie funktioniert.

    Aber unser Bewusstsein ist eben Teil davon, indem eben eine Synthese dieses kosmisch Geistes mit unserem lokalem Gehirn als Prozess stattfindet. Hier in diesem Bewusstseinsprozess kann man dann die Wirkungen der Geisteswelten am ehesten beobachten. Bei vielen Menschen liegen persönliche spirituelle Erfahrungen vor, die können sich dann eben diese Beteiligung von Geisteswelten gut vorstellen. Eine Integration dieser Effekte im persönlichem Weltmodell ist dann attraktiv und hilfreich.

    Für Menschen ohne eigene spirituelle Erfahrungen ist die ganze Geschichte derzeit eigentlich gegenstandslos. Wenn die zukünftige Hirnforschung hier die Geistesfrage beantwortet, werden wir eine ganz neue Situation haben: findet man die Geisteseffekte, werden wohl auch Menschen ohne eigene entsprechende Erfahrungen sich für das Thema interessieren, ist das Ergebnis andersrum, und nachgewiesen, dass unsere Nervenzellen das Bewusstsein alleine hinbekommen, dann müssen wir unsere spirituellen Erfahrungen wohl noch mal überdenken. Derzeit gibt es dafür aber wenig Anlass.

    „Die Philosophen des Immaterialismus drehen sich im Kreis, im Kreis, im Kreis“

    Nur diese Diskussion dreht sich im Kreis. Wohl solange, bis alle kapiert haben, das wir hier derzeit nur über Glaubensfragen reden können.

  680. Mein Dank an Feodor für die Klarstellung, wie der von mir bemängelte Kategorienfehler von Raimund Popper zu Fehlschlüssen führen kann!
    Der von Timm Grams ins Feld geführte angebliche innere Widerspruch des Naturalismus löst sich auf, wenn man anstelle eines eliminativen materialistischen Monismus der Identitätstheorie folgt, und zwar, in Anlehnung an Michael Pauen, in Form eines „monistischen Aspektdualismus“: Demnach gibt es nur eine einzige Welt, und alle Bewusstseinsinhalte wären identisch mit ihrem jeweiligen materiellen (bei Tieren neurobiologischen) Korrelat. In dieser Metapher verhielten sie sich ähnlich wie die beiden Seiten einer einzigen Münze, die man aus zwei unterschiedlichen Perspektiven – der Erste- und der Dritte-Person-Perspektive – betrachten kann.

    Zu Einsteins Dictum „Der Alte würfelt nicht“:
    Das Standardmodell der heutigen Physik kennt keine „verborgenen Variablen“, mit deren Hilfe man versuchen könnte, die Quantenphysik in eine streng deterministische Theorie umzuwandeln.
    Demnach wird bei einzelnen quantenphysikalischen Prozessen doch „gewürfelt“. Eine Video-Installation zeigt anschaulich, wie aus (absolut zufälligem) Quantenrauschen dank einer rekursiven Schleife sich aus dem „Nichts“ heraus ständig neue, unvorhersehbare Strukturen entwickeln (https://vimeo.com/276942254). Das Kunstwerk trägt den passenden Titel „…und Gott würfelt doch!“
    Rainer Wolf

  681. @Feodor: Denkfehler (wieder einmal)

    Wenn eine per Definition immaterielle Entität (“Geist”) mit materiellen Objekten wechselwirken können soll, dann kann die immaterielle Entität nicht vollständig immateriell sein, denn sie weist zumindest eine essentielle Eigenschaft materieller Objekte auf, nämlich die Fähigkeit zur physischen Wechselwirkung. Das scheint mir doch ein Fehler zu sein.

    Sie sehen immer nur die eine Richtung, daher fallen Ihnen mindestens drei Denkfehler nicht auf:

    Erstens lässt sich das ebensogut umdrehen und wäre eine materielle Entität immateriell in dem Sinne, dass sie mit Immateriellem wechselwirken kann.

    Zweitens müsste das erst noch einmal gezeigt werden, ob die Eigenschaft, mit (im)materiellem wechselzuwirken, eine Entität mit dieser Eigenschaft (im)materiell macht. Zum Vergleich: Farben sind sekundäre Eigenschaften von materiellen Dingen; materielle Dinge haben nur Reflexionseigenschaften. Nach Ihrem Argument müssten materielle Dinge immateriell sein, weil sie immaterielle (sekundäre) Eigenschaften haben, nämlich Farben. Hiermit hätten Sie also gerade den Materialismus widerlegt, Ihren eigenen Standpunkt. Glückwunsch!

    Drittens müssten Sie erst einmal klar machen, welchen Kausalitätsbegriff sie zugrunde legen. Nach Eccles, mit dem Popper bekanntlich seinen Dualismus entwickelt hat, kommt es an den Dendriten an (bisher nicht nachgewiesenen) “Psychonen” zu Quanteneffekten. Es ist überhaupt nicht klar, welcher Art diese Wechselwirkung sein soll. Daher zielt Ihr Argument hier ins Leere.

    Ihre zahlreichen Fehler wären zu verschmerzen, wenn Sie hier nicht immer mit so einem Gestus auftreten würden, als würden Sie die WAHRHEIT und die EINZIGE WAHRHEIT verbreiten.

  682. @Feodor

    Wenn eine per Definition immaterielle Entität (“Geist”) mit materiellen Objekten wechselwirken können soll, dann kann die immaterielle Entität nicht vollständig immateriell sein

    Das entspricht ziemlich exakt der Option 3 im Bieri-Trilemma, “Der Bereich physikalischer Phänomene ist kausal geschlossen”. Sie werden lachen, viele Dualisten verneinen das.

    Das scheint mir doch ein Fehler zu sein.

    Mir scheint der Fehler auf Ihrer Seite zu liegen. Sie nehmen ihre Definitionen und Annahmen als gegeben. Das ist der gleiche Fehler, den Timm Grams macht, um gegen den Naturalismus zu argumentieren.

    “Ideologie” wird sofort mit unhaltbaren Aussagesystemen assoziiert, die zur Ermordung Andersdenkender führten

    Nicht nur die Probleme der Dualisten, auch Ihre Probleme möchte ich nun wahrlich nicht haben. Pejorativ sollte meine Bezeichnung durchaus interpretiert werden, aber “Ermordung Andersdenkender” und assoziiert mit “sofort”, geht es nicht auch eine Spur dezenter?

    Da passt zu meiner Verwendung von ‘Ideologie’ eher die Beschreibung, die ich in dem von Ihnen erwähnten Wikipedia-Artikel gefunden habe.

    „Ideen und Weltbilder, die sich nicht an Evidenz und guten Argumenten orientieren, sondern die darauf abzielen, Machtverhältnisse zu stabilisieren oder zu ändern“

    Dr. habil. Rainer Wolf, als Mitglied des Wissenschaftsrats der GWUP, zielt sicher auch auf Änderungen von Machtverhältnissen ab. Mir liegen hier allerdings insbesondere die guten Argumente am Herzen. (Ich hoffe, bei der Verwendung des Begriffs ‘Argument’ durch mich, assoziieren Sie nicht sofort Totschlag.)

    Der Quatsch (nicht lustig, sondern abwertend gemeint), den Sie da jetzt nochmal von ihm zitiert haben, enthält alles andere als ein gutes Argument.

    Es gibt jedenfalls bisher keine belastbaren Hinweise darauf, dass immaterielle, geistige Entitäten einen kausalen Einfluss haben auf das materielle Geschehen im Gehirn.

    Für einen Dualisten gibt es jede Menge belastbare Hinweise, jeder Entschluss zu einer Handlung, jeder Willensakt ist einer.

    Dr. habil. Rainer Wolf scheint dagegen nur seine eigenen Annahmen, was überhaupt belastbar ist, mit dieser Aussage zu verknüpfen. Wie man das nennt, wenn dann später, ausgehend von den eigenen Annahmen, seine eigenen Annahmen als unwiderlegt identifiziert werden, wissen wir alle.

    Gäbe es sie, läge aus heutiger Sicht ein paranormales Phänomen vor mit dem Potenzial, den Naturalismus zu falsifizieren.

    Das ist der nächste Unfug (wieder abwertend gemeint), und zwar gleich in mehrfacher Hinsicht.

    Erstens, Handlungsentschlüsse und Willensakte werden im üblichen Sprachgebrauch nicht zu den paranormalen Phänomenen gezählt. Im hier verhandelten akademischen Disput der Philosophie, dem zwischen Monisten und Dualisten, geht es aber ausschließlich um solche, um normale Phänomene, nennen Sie sie Denken, Geist oder Bewusstsein. Mit “heutiger Sicht” kann Dr. habil. Rainer Wolf daher weder die Sicht vieler Dualisten meinen noch die aller Naturalisten, er tut aber so.

    Zweitens, der Naturalismus im Allgemeinen ist nicht falsifizierbar, schon gar nicht im Speziellen eines metaphysischen Naturalismus. Wer das meint, hat entweder nicht verstanden, was Metaphysik ist, oder Popper nicht verstanden, auf welche Theorien sich dessen Begriff von Falsifikation überhaupt anwenden lässt. Vielleicht auch beides nicht.

    (Die experimentelle Bestätigung eines paranormalen Phänomens würde nicht einmal zwangsläufig eine physikalische Theorie falsifizieren, das gäbe zunächst nur Anlass neue Theorien aufzustellen. Vergleichbar mit Dunkler Materie und Dunkler Energie, wo ja auch bisher nur eine Wirkung festgestellt wurde.)

    Für den ersten Existenznachweis eines paranormalen Phänomens hat die James Randi Educational Foundation seit Jahrzehnten 1 Million Dollar ausgelobt.

    Wenn jemand diesen Preis gewonnen hätte (ist seit 2015 überhaupt noch jemand auf solche Fähigkeiten getestet worden?) oder noch gewinnen würde, dann wären die meisten Dualisten kaum weniger erstaunt als Naturalisten. Wie kann da ein Argument für die ein oder andere Seite heraus entstehen?

    Mit einem immateriellen Geist führt Popper allerdings ein kritikimmunes Konzept ein

    Sie könnten eventuell bemerkt haben, dass nicht Popper es war, der den immateriellen Geist als Konzept eingeführt hat. Relativ spät in die Diskussion eingeführt wurde hingegen das kritikimmune Konzept, übrigens gegen jede bis dahin gemachte Erfahrung, nur Messbares würde existieren.

    Das bedeutet nun nicht, lieber Joker, dass Sie ihre Position ändern müssten. Das braucht niemand. Es herrscht Meinungsfreiheit.

    Danke für die Klarstellung, ich bin beruhigt.

    Wichtig wäre es aus meiner Sicht heraus, zunächst zu erkennen, bei welchen Positionen, die man so vertritt, es sich um eine Meinung handelt, im Gegensatz zu Verlässlicherem – vielleicht sogar nur um eine persönliche Meinung, die am Ende niemand teilt.

  683. Wolf
    —–demnach gibt es nur eine einzige Welt—-
    und worin ist die eingebettet, die hängt so leer im Raum.
    Sie merken selbst, dass wir an die Grenze der sprachlichen Möglichkeiten stoßen.

    Die Religion überschreitet die Grenze indem sie in Metaphern und Gleichnissen spricht. Nur so ist Transzendentes sprachlich zu erfassen.
    Das dulden die Naturalisten nicht, das darf nicht sein.

  684. @Stegemann: Materialismus

    Wenn ich Schmerz spüre, ist auch das materiell, was sollte es auch sonst sein.

    Das müsste doch das Ergebnis der Diskussion sein und nicht deren Anfang. Wenn ich den Materialismus als wahr annehme, klar, dann kann ich daraus ableiten, dass der Materialismus wahr ist.

    “Was denn sonst?”-Argumente haben keine Überzeugungskraft: Aus einem Mangel an Vorstellungskraft sollte man nicht darauf schließen, wie die Welt wirklich ist.

  685. @Jeckenburger
    Geisteswelten könnten über gezielte Zufälle in der Quantenwelt mit der materiellen Welt interagieren.

    Nein, das können sie nicht. Solche Behauptungen sind nichts als leere Phrasen, ohne irgend eine Vermutung, wie das konkret funktionieren könnte. Es ist mentale Selbstbefriedigung.

    Aber unser Bewusstsein ist eben Teil davon, indem eben eine Synthese dieses kosmisch Geistes mit unserem lokalem Gehirn als Prozess stattfindet.

    Genau dieselbe Phraseologie, eine sinnfreie Aneinanderreihung von Wörtern ohne konkrete Bedeutung.

    In der Philosophie des Geistes hat es immer Entwicklungen und Erklärungen gegeben, die parallel zu den Entwicklungen der Technik liefen, von der Mechanik über Pneumatik, Elektrische Schaltungen, Computer und jetzt die Quantenphysik.

  686. @all: Was ist eine Ideologie?

    Man sollte durchaus mal einen Blick in den Duden werfen, wo man findet:

    Ideologie:

    a) an eine soziale Gruppe, eine Kultur o. Ä. gebundenes System von Weltanschauungen, Grundeinstellungen und Wertungen; b) politische Theorie, in der Ideen der Erreichung politischer und wirtschaftlicher Ziele dienen (besonders in totalitären Systemen); c) weltfremde Theorie.

    Wenn man die Bedeutungen a) und c) mit b) vermischt, dann entsteht nur Chaos. Dass wir hier eine philosophische und keine politische Diskussion führen, wenn wir nach guten Argumenten für und gegen u.a. den Materialismus und Dualismus suchen, das sollte doch klar sein?

  687. @all: Ende der Diskussion

    Die Diskussion scheint sich in keine gute Richtung zu entwickeln. Ich würde sie darum morgen (1. Februar) schließen. Jeder bekommt, wie gehabt, die Gelegenheit zu einem Abschlussstatement.

  688. @Schleim
    Der Monismus oder Materialismus ist eine gut begründete Hypothese, auf der man wissenschaftlich aufbauen kann. Widersprüche oder ebenso gut begründete Alternativen sind nicht in Sicht. Dass es noch viele Lücken gibt, kann Grund zur Kritik sein, aber kein Grund zum Zweifel oder zur Ablehnung.

    Das Geruchsempfinden gehört zu den Qualia oder phänomenalen Inhalten des Bewusstseins. Ein Symptom von Covid19 ist der Verlust des Geruches. Deshalb ist das Geruchssystem des Menschen jetzt in den Fokus der Forschung geraten. Man weiß, dass es etwa 400 verschiedene Rezeptoren für das Geruchsempfinden gibt, also eine riesige Fülle an Gerüchen dadurch ermöglicht wird, weil an Gerüchen viele verschiedene Rezeptoren beteiligt sein können. Durch eine virale Infektion der Geruchszellen kann Geruchsverlust eintreten, oder es kann ein falsches Geruchsempfinden entstehen.

  689. @Balanus / 30.01.2021, 14:44 Uhr

    »Ich kenne keine neurobiologische Bewusstseins-Theorie, die eine maßgebliche Beteiligung des Ich-Bewusstseins […] beinhalten würde.«

    Ich kenne nicht mal eine neurobiolog. Theorie, die imstande wäre, einen Selbstbezug objektsprachlich zu repräsentieren. Selbstbezug lässt sich indessen als observables Phänomen feststellen, schon am Verhalten einer Elster, die sich in einem Spiegel selbst erkennt. Was eine Theorie nicht modellhaft nachbilden kann, das kann sie trivialerweise auch nicht nomologisch erklären.

    In der Tat erscheint es reichlich verfehlt, ich-Bewusstsein zu einem objektsprachlichen Begriff machen zu wollen. Bei Gödel geht es schliesslich auch nicht um Bewusstsein, sondern um Selbstreferenz, und nur darüber kriegt Hofstadter dann auch begrifflich die Kurve zum Bewusstsein.

    Anton Reutlinger hat inzwischen ja noch etwas Licht in die Sache mit Friedrich II. gebracht. Das Ergebnis, dass die Kinder dabei elendig zugrunde gehen, hätte ich auch so erwartet. Selbstorganisation allen reicht nicht hin zur Erklärung, wie geistige Regsamkeit als Ergebnis der Entwicklung eines Kindes schliesslich zustande kommt, den Einfluss seiner Umgebung auf seine Entwicklung kann man dabei nicht einfach ignorieren. Ein Kind ist schwerlich ein “abgeschlossenes System”, das sich selbst Verstand beibringt. Eine Art Pfingstwunder braucht es allerdings nicht.

    Worauf ich Dich mit dem Hinweis auf Prigogine eigentlich bringen wollte, habe ich im folgenden Zitat nochmals fett hervorgehoben:

    To stress the fact that non-equilibrium self-organization in open systems requires the dissipation of energy, Prigogine coined the term ‘dissipative structures’, so as to distinguish them from equilibrium structures [3,10].

    Für Prigogine sind Organismen offene Systeme, wohingegen Du sie anscheinend eher als geschlossene Systeme zu betrachten geneigt bist. Da ist doch ein signifikanter Unterschied.

  690. @Dr. Rainer Wolf 31.01.2021, 03:23 Uhr
    Du schreibst:

    Der von Timm Grams ins Feld geführte angebliche innere Widerspruch des Naturalismus löst sich auf, wenn man anstelle eines eliminativen materialistischen Monismus der Identitätstheorie folgt, und zwar, in Anlehnung an Michael Pauen, in Form eines „monistischen Aspektdualismus“

    Es ist ganz offensichtlich, dass der Aspektdualismus von demselben Einwand getroffen wird wie die Identitätstheorie. Davon war bereits mehrfach die Rede und eine Wiederholung erübrigt sich.

    @Joker 31.01.2021, 09:59 Uhr
    Selbstverständlich stimme ich Ihnen zu, wenn Sie schreiben:

    Der Naturalismus im Allgemeinen ist nicht falsifizierbar, schon gar nicht im Speziellen eines metaphysischen Naturalismus.

    Eine Falsifikation des Naturalismus hatte ich nie im Sinn. Ich habe mir eine bestimmte Version dieser Denkungsart vorgenommen und deren Postulate als Prämissen vorausgesetzt, um daraus einen Widerspruch herzuleiten. Die Wahl dieser Prämissen halten Sie für einen Fehler:

    Sie nehmen ihre Definitionen und Annahmen als gegeben. Das ist der gleiche Fehler, den Timm Grams macht, um gegen den Naturalismus zu argumentieren.

    Aber das, was ich mache, ist Grundhandwerk der Logiker und Mathematiker. Bei mir spielen die Prämissen eine ganz andere Rolle als bei Feodor: Einmal ist es Beweistechnik, ein andermal persönliche Überzeugung.

  691. Zum Schluss, ein sehr guter blog mit einem engagierten Publikum.
    Habe viel dabei gelernt.

  692. Abschlussstatement:
    Bewußtsein ist eine Funktion des Zentralnervensystems und entsteht durch Verknüpfung von Neuronen und verschiedenen Hirnregionen. Es bildet das Ich als eigene Instanz, welche aktiv Denkprozesse orchestrieren kann, und es ist teilautonom (Stichwort freier Wille). Der Mensch ist in der Lage, abstrakte Symbole und Symbolwelten zu schaffen, welche nicht nur Realität rekonstruieren, sondern auch irreale Konstrukte ermöglichen, die keine Verankerung in der Erfahrung (Empirie) haben. Diese reichen von einfachen (Der liebe Gott) bis sehr komplexen Denksystemen (etwa Burkhard Heim).
    Komplexe Systeme wie das Bewußtsein lassen sich nicht „pointilistisch“ darstellen, sondern müssen als System gesehen und erforscht werden, zumal Systeme ihre eigene Dynamik entwickeln, welche auf andere Systeme (z.B. Soma) wirken können (Placeboeffekt).
    Bei der Diskussion muss die Verwechslung der Bezugsebenen vermieden werden: den Begriff ‚Angst‘ als immateriell zu bezeichnen, weil man ihn nicht anfassen kann, ist Unsinn. Reale Angst ist physikalisch und wird individuell erfahren. Die individuelle Erfahrung selbst, etwa das Erleben von Musik, kann nicht in mathematischen Gleichungen ausgedrückt werden, entzieht sich also der Formalisierung durch die Physik.
    Randnotiz zum Zufall: Was im Kleinen als Zufall erscheint, zeigt sich im Großen als Logik.

  693. @Weltmodelle

    Mit oder ohne spirituelle Erfahrungen kommt man zu verschiedenen Arten von Weltmodellen. Eine Verständigung scheint hier offenbar schwierig zu sein. Das Thema Bewusstsein ist hier mitbetroffen.

    Spirituelle Erfahrungen sind draußen in der Welt eher sporadisch anzutreffen, deshalb hat hier auch die Parapsychologie nur sporadische Hinweise auf Effekte, die auf Geisteswelten hindeuten. Beim innere Erleben sieht das ganz anders aus, hier gibt es wesentlich mehr Geistiges zu erleben.

    Aber dieses ist in der Psyche versteckt, und in gewissem Sinne auch selbstwirksam. Wer z.B. an Gott glaubt, geht schon etwas anders an sein Leben heran, und entwickelt u.a. öfter eine gewisse innere Kommunikation mit selbigem, auch wenn hier Nachweise noch ausstehen.

    Aufgrund der vielfältigen internen Wirkungen von verschiedenen Glaubenssystemen vermute ich, dass eventuell die zukünftige Hirnforschung hier etwas mehr Evidenz liefern könnte. Aber wer eben überhaupt keine spirituellen Erfahrungen hat, für den ist dieses dann folgerichtig auch gegenstandslos. Und der Hinweis, dass man dabei im Sinne von geistigen Effekten eventuell fündig wird, kann offenbar schon eine Zumutung sein.

  694. Das Problem ist doch, dass das Gehirn in der Hauptsache eine „Muster“ verarbeitende Maschine ist.

    Ein System dass auf die Regeln der „absoluten Logik“ aufbaut, wäre vermutlich nicht existenzfähig, weil es in der Realität mit den Widersprüchen nicht zurecht kommen würde.

    Absolute „Deckung von Mustern“ kann es vermutlich nur bei genauest definierten „Mustern der Mathematik“ geben. Man sollte sich mit ungefähren Übereinstimmungen zufrieden geben, die Fehlermöglichkeiten müssen einem immer bewusst sein.

    Den Theologen scheint auch dieses Konzept frühzeitig klar geworden sein. Praktisch alle Regeln die für sie bedeutsam scheinen, bauen auf das was sie „Gleichnisse“ nennen auf. Wenn einem dies nicht klar ist, so hält man sie für „blödsinnige Märchen“.

    Angeblich haben spanische Jesuiten nach dem Krieg in kirchlichen höheren Schulen auch im deutschen Sprachraum, besonders in kirchlichen Internaten versucht, das grundlegende Konzept aus „Mustern zu lernen“ umzusetzen. So dass Jugendliche frühzeitig trainiert wurden systematisch nach „passenden Mustern“ zu suchen um geistige Sachverhalte besser oder überhaupt erst, auch noch möglichst „bildhaft und anschaulich“ zu verstehen und sich besser zu merken.

    Das ist heutzutage das grundlegende Konzept der KI.

  695. @Chrys // 31.01.2021, 10:49 Uhr

    Ich habe von einem weitgehend abgeschlossenen System gesprochen und die wichtigsten Grenzflächen zur Außenwelt benannt. Dort findet der selektive Stoffaustausch statt. Insoweit ist das System selbstredend offen, irgendwie müssen die lebenswichtigen Stoffe ja in den Organismus hineingelangen.

    Das (zentrale) Nervensystem, um das es uns hier im Besonderen geht, ist quasi ein Untersystem im Gesamtsystem Organismus. Hier findet kein Austausch von Substanzen direkt mit der Außenwelt statt. Worin könnte also der „Einfluss“ der Umgebung bestehen, wenn in das System Gehirn nichts von außen (außerhalb des Hirns) hereingelangen kann außer modulierte Nervensignale aus der Peripherie?

    Lernen funktioniert bekanntlich nicht nach dem Prinzip des Nürnberger Trichters. Ein neugeborenes Kind ist kein unbeschriebenes Blatt, kein leeres Gefäß, das von Erziehern und Lehrern zu befüllen wäre. Wissen muss das sich entwickelnde (selbst organisierende) Individuum selbst aktiv aneignen, es gibt keinen anderen Weg, und wie wir alle wissen, sind dem Wissenserwerb gewisse Grenzen gesetzt, die insbesondere von den genetisch bedingten Fähigkeiten, sprich der angeborenen Lernfähigkeit des Individuums abhängen.

    Des Weiteren: Aus „Sicht“ eines Gehirns gehört auch der Körper, der das Hirn quasi ernährt und durch die Landschaft trägt, gewissermaßen zur Außenwelt und muss als (eigener) Körper erstmal wahrgenommen und von den anderen Dingen der Außenwelt unterschieden werden. Auch hierbei handelt es sich also in erster Linie um einen selbsttätigen Lernprozess, an dessen Ende, wenn man so will, das Ich-Bewusstsein steht. Dieser selbsttätige Lernprozess, ich wiederhole mich, kann nur dann ordnungsgemäß ablaufen, wenn in der Außenwelt die fürs Lernen notwendigen „Gegenstände“ vorfindlich sind (Menschen, Tiere, Pflanzen, Sprache, Kultur, … )

  696. @Stepahn Schleim

    Ende der Diskussion

    Danke für Deine Geduld.

    @all

    Nicht jeder der auf den Begriff Skeptiker™ Anspruch erhebt, muss so weit gehen, wie uns das Borges von den Bewohnern Tlöns schildert.

    Die philosophischen [Bücher in Tlön] enthalten unfehlbar die These und die Antithese, das strenge Für und Wider einer Lehre. Ein Buch ohne Selbstwiderlegung gilt als unvollständig.

    (Vergleiche die Übersetzung hier)

    Zu Bedenken gilt allerdings zumindest dies: Die Naturwissenschaft ist letztlich so erfolgreich geworden, weil sie ihre eigenen Annahmen und Ergebnisse immer wieder kritisch begutachtet hat. Nicht weil sie Andere, wie vielleicht am deutlichsten, die Religionen, kritisiert hat.

    Das mag als Leitgedanke dem philosophischen Skeptiker hier auf Erden dienen.

    @Timm Grams

    Vorab, offtopic:
    Die Gründe, die sie an anderem Ort angegeben haben, warum Sie sich von der GWUP verabschiedet haben, sind für mich leicht nachzuvollziehen.

    Auf den Strohmann-Vorwurf, der auch von Joker angesprochen wurde, bin ich eingegangen und habe ihn meines Erachtens vollständig entkräftet

    Eingegangen, ja; entkräftet, vielleicht; vollständig, sicher nicht – meines Erachtens nach.

    Danke nochmal für Ihren Beitrag.

    @anton reutlinger

    Dass es noch viele Lücken gibt, kann Grund zur Kritik sein, aber kein Grund zum Zweifel oder zur Ablehnung.

    Mangelnder Zweifel ist ein sicheres Indiz dafür, dass etwas zur Ideologie verkommen ist.

    @Feodor

    Dass ich den Dualismus für mumpitz Halte, sollte klar sein. Zu beachten ist allerdings, was ich da im Fettgedruckten klein und was groß schreibe.

    @Chrys

    Der Bruch mit der modernen Logik ist von Bunge ja erklärtermassen vollzogen worden

    Falls mir dazu noch eine Ausrede ein Argument einfällt oder bekannt wird, ich meine Immunisierung gegen Kritik aufrechterhalten kann, werde ich das bei anderer Gelegenheit einfließen lassen. Ich hoffe auf Unterstützung durch Herrn Mahner.

    @Dr. Rainer Wolf

    Das Standardmodell der heutigen Physik kennt keine „verborgenen Variablen“, mit deren Hilfe man versuchen könnte, die Quantenphysik in eine streng deterministische Theorie umzuwandeln. […] „…und Gott würfelt doch!“

    Vergleiche:

    Die Bohmsche Mechanik ist eine Theorie mit verborgenen Variablen, aber sie ist auch nicht-lokal und mit Bell wunderbar verträglich. […] Ausserdem ist sie deterministisch — Einstein ist also mit seinem Diktum “Der Alte würfelt nicht” noch immer im Rennen.

    (@Chrys, 19.01.2021, 23:52 Uhr)

    Dieser Thread scheint mir damit vollständig zu sein.

    @Stepahn Schleim

    Ende der Diskussion

    Danke für Deine Ungeduld.

    @Axel Krüger

    Die Stufen im von Ihnen verlinkten Video haben mich an die Stufen von Hermann Hesse erinnert, den ich etwas verkürzt hier wiedergebe:

    Es muß das Herz bei jedem Lebensrufe
    Bereit zum Abschied sein und Neubeginne,
    Um sich in Tapferkeit und ohne Trauern
    In andre, neue Bindungen zu geben.

    Wir sollen heiter Raum um Raum durchschreiten,
    An keinem wie an einer Heimat hängen,
    Der Weltgeist will nicht fesseln uns und engen,
    Er will uns Stuf´ um Stufe heben, weiten.

    Wohlan denn, Herz, nimm Abschied und gesunde!

  697. @Joker
    Mangelnder Zweifel ist ein sicheres Indiz dafür, dass etwas zur Ideologie verkommen ist.

    Keineswegs; Zweifel müssen begründet sein. Wenn alles bezweifelt werden darf, dann verfällt man in Nihilismus und in gesellschaftliche Krisen. Die Gegenwart, mit allgegenwärtigen Zweifeln an “der Wissenschaft”, zeigt es ganz deutlich. Ideologie fängt meist mit falschen oder grundlosen Zweifeln an.

  698. Tja, wie soll man auf das oben gesagte eingehen? Einige Statements nehme ich als Aggression gegen meine Person wahr. Hierbei besteht natürlich das Problem der Fehlwahrnehmung: ich bin allerdings nicht die einige Person, die das so rezipiert, daher scheint meine Wahrnehmung prima facie nicht ganz abwegig zu sein. Natürlich kann jeder mal wütend werden. In einem Diskurs, der rational bleiben soll, braucht es deswegen ein emotionales Management. Aus diesem Grund antworte ich öfters nicht sofort, sondern lasse das geschriebene erst ein wenig sacken. Alles, was ich schreibe, betrachte ich als Veröffentlichung unter meinem Namen, daher wähle ich meine Worte sorgfältig.

    Bei diesem Thema ist es zu erwarten, dass Einigkeit nicht erreichbar ist. Es wäre aber wünschenswert, wenn sich die verschiedenen Positionen deutlich herausarbeiten ließen. Leider gelingt das auch meist nicht.

    Ich habe gelernt, dass der Begriff “Naturalismus” schlecht praktikabel ist, da er einen zu großen Bedeutungsumfang hat. Ich werde nur noch von Naturgesetzlichkeit sprechen. Es braucht wohl auch einen sofort verfügbare Erläuterung dieses Begriffs, da dieser wohl auch unterschiedlich aufgefasst wird.

    Interessant ist, wie gering die Rolle des Kritizismus im Diskurs zu sein schien. Als Todo habe ich mir notiert, dass es sinnvoll ist, eine längere Liste von Ereignissen aufzustellen, die alle geeignet wären, das Postulat der Naturgesetzlichkeit des Universums in Frage zu stellen. Mir schien es so, als würde durchaus auch postuliert, dass die Voraussetzung der Naturgesetzlichkeit völlig kritikimmun und daher entbehrlich sei.

    Ein historische Anmerkung: Es war Gilbert Ryle der sich mit dem oben erwähnten Kategoriefehler des Dualismus beschäftigt hat. Von ihm stammt auch der Begriff des „Gespenstes in der Maschine“. Als Kind war “Das kleine Gespenst” von Prof. Otfried Preußler mein Lieblingsbuch, daher ist der Begriff bei mir mit positiven Emotionen verknüpft. In der Neurologie wird das aber nicht verfolgt. Sind das alles schlimme Ideologen? Die Geisteswissenschaft braucht jedenfalls keine Gespenster, sie kann auch ohne diese Entitäten arbeiten.

    Einige Naturwissenschaftler halten die Geisteswisenschaften pauschal für wertlose Spekulation, dagegen argumentiere ich häufig. Einige Geisteswissenschaftler wiederum argumentieren gegen die Naturwissenschaften, dagegen argumentiere ich auch.

    Lieber Joker, welche “Machtverhältnisse zu stabilisieren oder zu ändern” ich und Rainer Wolf angeblich versuchten, bleibt Ihr Geheimnis. Klingt Ihre Einlassung nicht nach Ideologie? Sind wir damit nicht alle Ideologen? Der nicht näher spezifizierte Begriff “Ideologie” kann tatsächlich meist ganz einfach im Sinne einer Retorsion zurück gespielt werden. Daher ist er für mich untauglich. Wenn Ihnen gute Argumente am Herzen liegen, sollten Sie auch die Vokabel “Quatsch” nicht verwenden. Entschlüsse und Willensakte sind leider keine Belege für ein Ryle’sches Gespenst. Das ist Mainstream in der Neurologie. Vor kurzem befragte ich dazu eine Professorin für die Philosophie der Medizinethik: Sie konnte sich gar nicht vorstellen, dass das Ryle’sche Gespenst im wissenschaflichen Diskurs noch als ernsthafte Hypothese verfolgt wird. Diese Frage stelle ich regelmäßig nach einschlägigen Vorträgen. Es erscheint mir daher höchst unplausibel, dass der Dualismus in der Gehirnforschung und der *analytischen* Philosophie noch Mainstream ist. Natürlich werden Dualismus und Idealismus auch weiterhin vertreten. Handlungsentschlüsse und Willensakte sind dann paranormal, wenn sie vom Ryle’schen Gespenst durchgeführt werden. Popper hat übrigens später sein Verdikt, dass die Behauptung des Evolutionsgeschehens nicht falisfizierbar sei zurückgenommen. Er würde wohl auch zugeben, dass die Voraussetzung der Naturgesetzlichkeit empirisch scheitern könnte. Die kantsche Metaphysik ist im Übrigen an der ART und der QM gescheitert. Sie behaupten indirekt, dass Wissenschaft nicht scheitern könne, es lassen sich aber viele Beispiele finden, bei denen eine Wissenschaft, welche die persistente Naturgesetzlichkeit voraussetzt, scheitern kann. Neben Popper gibt es auch noch Hans Albert und Alan Musgrave. Man könnte vermuten, dass diese Autoren des kritischen Rationalismus vielleicht auch höchst unbeliebt sind. Paranormale Behauptungen werden von Rainer Wolf und Martin Mahner regelmäßig getestet.

    Die Unterscheidung immateriell versus materiell wurde oben leider noch nicht stringent herausgearbeitet. “Energie” wird generell als Eigenschaft materieller Objekte betrachtet. Das ist Mainstream. Wenn man nun immateriellen Entitäten ebenfalls Energie zuschreibt, verschwimmt die Unterscheidung zwischen Materiellem und Immateriellen. Generell wird mit dem immateriellen hypostasierten Geist ein höchst vages Konzept postuliert.

    Lieber Herr Schleim, ich bin (1) nicht so minderbemittelt, dass ich nicht auch Berkeley kenne. Bei ihm gibt es nur Geist, alles ist immateriell. Des weiteren habe ich keine Hinweise darauf, dass Popper einen Panpsychismus vertreten hätte. (2) Farbwahrnehmung wird im Mainstream als Eigenschaft materiell realisierter Körperprozesse aufgefasst. Ein Ryle’sches Gespenst wird nicht angenommen. Da kann ich leider keinen Widerspruch entdecken. (3) Eccles ist mir wohlbekannt. Die Beleglage sieht aber so aus, als ob Quantenereignisse rein stochastisch ablaufen. Rein zufällige Entscheidungen helfen aber nicht bei der Willensfreiheit, sie schaffen nur ein Problem bei der Zuordnung einer Entscheidung zu einer Person, was sich in der Fachliteratur ausführlich nachlesen lässt. Menschen sind keine Zufallsgeneratoren. Godehard Brüntrup und Thomas Nagel, beides aus Ihrer Sicht hoffentlich “unverdächtige” Philosophen, geben offen zu, dass sie nichts Drittes zwischen Notwendigkeit und Zufall präsentieren können. Damit ist die *absolute* Willensfreiheit bereits begrifflich fragwürdig.

    Was mache ich jetzt, lieber Herr Schleim, mit ihrer Schlusswendung. Diese ignoriert jedenfalls meinen Prolog. Mir fällt auf, dass Sie Postings mit sehr ähnlichem propositionalem Gehalt höchst unterschiedlich bewerten und sich dabei auf bestimmte Personen einschießen. Das macht ratlos.

    Ich hoffe, lieber Herr Schleim, auf Ihren neuen Artikel. Gerne würde ich z.B. eine luzide Abhandlung lesen, in der stringent dargelegt wird, warum sich die Naturgesetzlichkeit nicht kritisieren lässt.

  699. @Feodors Welt

    Das Wort “Willensfreiheit” taucht auf dieser Seite dreiundvierzig mal auf. Ich habe es kein einziges Mal erwähnt.

    Ich glaube, Sie leben in Ihrer eigenen Welt. Hoffentlich geht es Ihnen da gut!

  700. @Stephan Schleim

    Abschlussstatement

    Dafür, dass Sie meinen Artikel in Ihr Blog übernommen haben, danke ich. Es hat wirklich Spaß gemacht und ich habe in der meist fairen Diskussion einige wertvolle Anregungen erhalten.

  701. »Abschlussstatement«

    Im metaphysischer Naturalismus manifestiert sich aus meiner Sicht das Verlangen, a priori Gewissheiten über die “Welt” aus einem (durchaus falliblen) a posteriori Wissen über die Phänomene zu gewinnen. Das ist rational nicht begründet.

    Und sofern er als ontischer Materialismus daherkommt, ist er inhaltlich eine Neuauflage von Aristotelischem Substanzdenken, dem lediglich neue Etiketten aufgeklebt wurden. Was sowohl mit der modernen Logik wie auch mit modernen Vorstellungen von Materie in der QFT unverträglich ist — und daher auch keinen gerechtfertigten Anspruch darauf erheben kann, besonders wissenschaftlich zu sein.

  702. Schlussbemerkung

    Von allen mir bekannten philosophischen Ismen erscheint mir der Natur-Ismus noch mit am sympathischsten. Aber wie es so ist mit all diesen Ismen, als Gedankengebäude sind sie nur schwer zu fassen. Zumal sich die Philosophen uneins sind, was als allgemeingültige Definition des Naturalismus zu gelten habe.

    Am Ende des Tages bleibt mir nur, mich dem Fazit aus der Einleitung zu dem von Geert Keil und Herbert Schnädelbach herausgegebenen Band über den Naturalismus (Suhrkamp 2000) anzuschließen:

    Es ergibt sich eine merkwürdige Konstellation: Erklärte Naturalisten werfen ihren Kritikern vor, Naturalismus so eng zu definieren, daß dieser zum Strohmann wird. Umgekehrt werfen die Kritiker den Naturalisten vor, den Begriff so weit zu definieren, daß ein jeder als Naturalist gilt, dem es an obskurantistischen Neigungen gebricht. Gleichwohl besteht hier kein begriffspolitisches Patt. […] Wir haben dafür argumentiert, daß vieles, was als Naturalismus firmiert, diesen Namen nicht verdient, während das, was ihn verdient, nicht verdient, vertreten zu werden.

    In diesem Sinne, besten Dank an alle!

  703. Dr. Webbaer schrieb (30.01.2021,10:16 Uhr):
    > Dies hier : -> https://de.wikipedia.org/wiki/Bellsche_Ungleichung …habe ich nie verstanden.

    Falls ein längerer Artikel, wie der verlinkte, jemandem insgesamt unverständlich ist, dann ist es demjenigen i.A. möglich (und es wird entsprechend auch erwartet), jeweils den allerersten Textabschnitt anzugeben, der (an sich und/oder ausgehend von eventuell vorausgehenden verständlich erscheinenden Textabschnitten) ausdrücklich unverständlich ist.

    Meines Erachtens betrifft das im verlinkten Wikipedia-Artikel den Textabschnitt:

    1. […] wenn Messungen nur Eigenschaften ablesen, die unabhängig von der Messung vorliegen, wenn also das Ergebnis jeder denkbaren Messung […] schon feststeht, bevor es durch die Messung bekannt wird.

    Dieser Auffassung ist (verständlicherweise) entgegenzusetzen:

    … dass das Ergebnis einer Messung (d.h. ein Messwert) erst dann feststeht, wenn ein bestimmter Messoperator festgelegt und wenn daraufhin festgestellt wurde, dass die zur Auswertung gegebenen Beobachtungsdaten für die Ermittlung eines Ergebniswertes (als ein bestimmter Eigenwert des festgelegten Messoperators) geeignet und hinreichend sind. (Die ausdrückliche Auswertung mag anschließend separat erfolgen und ergibt jedenfalls zwangsläufig den dann eindeutig feststehenden Messwert.) Den jeweils erhaltenen Messwert bezeichnet man dann auch als “Eigenschaft” der gegebenen Beobachtungsdaten bzw. “der auszuwertenden Versuche des beobachteten Systems”.

    Dies betrifft insbesondere die Ergebnisse des Messens des Orientierungswinkels je zweier Paare von (sich jeweils gegenseitig ausschließenden) Detektoren,
    A ≡ (a, ¬a) und B ≡ (b, ¬b) gegenüber einander, hinsichtlich ihrer (durch Versuchsindex k korrelierten) Beobachtungsdaten Φ_(AB) ≡ { ( A_k, B_k ) }:

    θ_AB[ { (A_k, B_k) } ] :=
    ArcCos[ Sqrt[ Summe_k[ 1 + a_k b_k - (1 - a_k) b_k - a_k (1 - b_k) + (1 - a_k) (1 - b_k) ] / Summe_k[ 2 ] ] ]
    ,

    wobei die gegebenen Beobachtungsdaten Versuch für Versuch als
    entweder a_k = 1 oder a_k = 0, und
    entweder b_k = 1 oder b_k = 0
    codiert wurden;
    und falls in allen Versuchen b_k = 1 zutraf, dann die Zahl Summe_k[ a_k ] / Summe_k[ 1 ] nach Malus als “Intensität” in diesen Versuchen bezeichnet wird.

    Hinsichtlich eventueller dritter Detektorpaare, namentlich z.B. “C ≡ (c, ¬c)“, beinhalten die Beobachtungsdaten Φ_(AB) ≡ { ( A_k, B_k ) } (bzw. die betreffende Menge von Versuchen “des beobachteten Systems”) keine Eigenschaften im o.g. Sinne.
    Die im verlinkten Wikipedia-Artikel offerierte “Ersatzbetrachtung” (von Personen, von denen sich jeweils drei sich paarweise ausschließende Eigenschaften feststellen lassen: “groß, oder nicht”, “blond, oder nicht”, “weiblich, oder nicht”) ist offenbar keine passende Analogie zu den beschriebenen Experimenten zur Detektion von Teilchenpaaren.

  704. Ich sehe alle „-ismen“, aber letztlich auch alle Meinungen, auch meine eigene „Meinung“, noch „psychologischer“ als die Psychologen.

    Sie sind Ausdruck einer bestimmten inneren (teilweise erworbenen) „Einstellung“. (Ich persönlich frage mich höchstens, wo ich mir meine, nicht immer alltäglichen Denkmuster „aufgerissen“ habe.)

    Mit der Realität hat alles höchstens am Rande zu tun. That’s life ….

  705. @hwied

    dieses Programm hat durch das Neue Testament ein update bekommen, wo so etwas “Widersinniges”, wie , Liebe deine Feinde, verlangt wird.

    Das ist und war nicht neu. Ähnliches steht auch im Alten Testament und war auch da schon kein neuer Gedanke mehr.

    z.B.
    “Gib dem Mann, der mit dir streitet, nichts Böses zurück. fordere mit Freundlichkeit deinen Übeltäter auf … lächle deinen Gegner an.”
    (“Räte der Weisheit” Babylon ~1800 BC)
    Gleichlautende Aussagen kennt man auch aus dem alten Ägypten.
    usw.

  706. Frank Wappler – gestern – 14:17

    (Offenbar ist dieser thread doch noch nicht geschlossen).

    Ich bin m.o.w. Laie auf diesem (insb. Formel-) Gebiet. Das mit der lokalen und nicht-lokalen Realität interessiert mich aber. So im Sinne von gibt es nur eine lokale R. (das Wissen von “Gleichzeitigkeit” begrenzt durch die Lichtgeschwindigkeit). Od. gibt es auch eine nicht-lokale “Aktivität” (im Sinne von instantaner Reaktion/”Verschränkung”)? Wozu mir auch die Begriffe zeit-/licht- und raumartig einfallen – und Gegenwartshyperfläche).

    Ich denke halt, dass es absolute Gleichzeitigkeit durchaus gibt, d.h. wenn hier auf der Erde etwas passiert auch irgendwo Milliarden Lichtjahre entfernt etwas im selben Moment passiert. Informationsübertragung (Messung davon) mit Überlichtgeschwindigkeit gibt es (“bisher”) jedoch nicht. Wie ist dann die Verschränkung von Quanten zu erklären. Bzw. was hat Bell nun dazu “bewiesen”/konstatiert? Bzw. kann man das nur in Formeln ausdrücken?

    Das Falls ein längerer Artikel, wie der verlinkte, jemandem insgesamt unverständlich ist, dann ist es demjenigen i.A. möglich (und es wird entsprechend auch erwartet), jeweils den allerersten Textabschnitt anzugeben, der (an sich und/oder ausgehend von eventuell vorausgehenden verständlich erscheinenden Textabschnitten) ausdrücklich unverständlich ist hab´ ich nicht so recht verstanden – aber egal.

    Knackpunkt scheint mir die Messung zu sein. Na klar: Solange ich nicht gemessen habe steht auch kein Messergebnis fest. Trotzdem aber hat das (noch zu messende) Objekt doch aber irgendeine Eigenschaft. “Dinge” (insb. reale) ohne Eigenschaften gibt es nicht; und sei es nur das sie “sind”.

    Einstein soll zu Bohr gesagt haben ** Glauben sie wirklich dass der Mond nicht da ist wenn keiner hinsieht? ** Bohr darauf **Beweisen Sie mir doch das Gegenteil **. Meint: Um sicher zu sein muss man hinschauen/messen. Das ist durchaus OK. Es gibt aber auch Dinge die jenseits unserer (derzeitigen) Wahrnehmungs-/Messfähigkeiten liegen. Od.: Nur weil man etwas nicht weiß heißt das doch noch lange nicht, dass es das nicht gibt.

    Ich bitte um Nachsicht für die (insb. aus Sicht eines Fachmannes) naiven Formulierungen.

  707. @Krüger: Regeln

    Der Mensch ist ein autonomes Wesen, sollte man meinen, und kann sich auch von selbst, ohne technische Lösungen, an Regeln halten.

    Ich bin mit dem Folgeartikel noch nicht fertig und will mir dafür auch die nötige Zeit nehmen.

    Sei’s drum: Mir ist vor allem wichtig, dass sich hier niemand ausgeschlossen fühlt.

  708. Liebe Leserinnen und Leser,

    vielen Dank für den intensiven Gedankenaustausch hier bei MENSCHEN-BILDER!

    Diese Diskussion wurde, wie angekündigt, nun geschlossen. Der Gastautor und ich hatten beide den Eindruck, dass es inzwischen immer häufiger zu Wiederholungen kam.

    Manche haben schon ein Abschlussstatement geschrieben. Wer das noch nachholen möchte, der möge mir bis zum Ende der Woche, also dem 7. Februar schreiben (Richtwert: max. 200 Wörter). Eingegangene E-Mails werden dann zusammen am 8. Februar von mir veröffentlicht: stephan(at)schleim(punkt)email

    Ich lade Sie herzlich dazu ein, die neue Serie zu Bewusstsein, Philosophie, Religion und Naturwissenschaft zu lesen und dort mitzudiskutieren.